Sunteți pe pagina 1din 455

NEUROLOGY (NEU-6)

TRUE-FALSE TYPE QUESTIONS


Put T for true statements and F for false statements!!!

NEU-6.1.
The excitation effect produced by strychnine is due to its blockade of
presynaptic inhibition.
NEU-6.2.
At high doses all barbiturates exhibit an anticonvulsive effect.
NEU-6.3.
The electric stimulation of certain areas of the brain can mimic normal
sleep.
NEU-6.4.
Phenytoin strongly inhibits the post-tetanic increase of excitation.
NEU-6.5.
Local anesthetics inhibit the depolarization and propagation of
physiological stimuli.
NEU-6.6.
D-tubocurarine is a non-specific inhibitor of the motor end-plate.
NEU-6.7.
Phenytoin increases the intracellular sodium level.
NEU-6.8.
impaired liver function affects the metabolism of phenytoin leading
to the subsequent accumulation of this component.
NEU-6.9.
Phenothiazines and Rauwolfia alkaloids induce Parkinson-like
symptoms.
NEU-6.10.
Meprobamate has an anticonvulsive effect.
NEU-6.11.
Trimethadione (Tridione) is effective in "Petit Mal Epilepsy".
NEU-6.12.
It is generally accepted that the dopaminergic pathways of the extrapyramidal
system are damaged in Parkinson's disease.
NEU-6.13.
Trimethadione (Tridione) markedly decreases "petit mal" type seizures
without affecting the normal EEG.
NEU-6.14.
Complications - if any - develop within several hours after an angiography.
NEU-6.15.
Normal relaxed muscle does not exhibit EMG activity.

NEU-6.16.
A short 1-4 cycle/sec burst activity occurs in the EEG of normal
adults.
NEU-6.17.
A higher than 13 cycles/sec activity can occur in the EEG of normal
adults.
NEU-6.18.
Tumors never induce spike activity.
NEU-6.19.
Fast growing tumors alter the EEG more often than the slow growing
tumors.
NEU-6.20
1-4 cycle/sec EEG episodes in infants always indicate brain injury.
NEU-6.21.
Duchenne-type muscular dystrophy never occurs in girls.
NEU-6.22.
Tuberculous meningitis usually starts slowly.
NEU-6.23.
In tuberculous meningitis symptoms other than ophthalmoplegia
are observed.
NEU-6.24.
In tuberculous meningitisthe tuberculin test is usually negative.
NEU-6.25.
In tuberculous meningitis the chest X-ray is usually positive.
NEU-6.26.
In tuberculous meningitis the "guinea-pig" vaccination result is
usually positive.
NEU-6.28.
A brain abscess is a usual complication of a menningococcus infection.
NEU-6.29.
A frontal lobe abscess is a usual complication of chronic otitis.
NEU-6.30.
A cerebellar abscess can occur in chronic mastoididtis.
NEU-6.31.
Cryptococcus is the most common cause of mycotic meningitis.
NEU-6.32.
Cryptococcus meningitis is usually acute and fulminant.
NEU-6.33.
A Cryptococcus infection can only be correctly diagnosed by a CSF
culture.

NEU-6.34.
Meningitis elicited by Actinomyces can be treated by amphotericin B.
NEU-6.35.
Penicillin is the antibiotic of choice in Cryptococcus-induced meningitis.
NEU-6.36.
Mycotic aneurysms and arteritis usually occur together.
NEU-6.37.
Saccular aneurysms frequently occur with polycystic kidney disease.
NEU-6.38.
Saccular aneurysms always develop between 2 points of bifurcation.
NEU-6.39.
Saccular aneurysms are due to damages of the medial lamina.
NEU-6.40.
Deforming muscular dystonia (Dystonia musculorum deformans)
starts at an age between 5-15 years.
NEU-6.41.
The first symptoms of deforming muscular dystonia appear in the
arms.
NEU-6.42.
Half-moon shaped legs are typical of deforming muscular dystonia.
NEU-6.43.
In Wilson's disease the symptoms of liver damage develop prior to
the neurologic ones.
NEU-6.44.
Tremor, dysarthria, rigidity and titubation frequently occur in Wilson's
disease.
NEU-6.45.
A decreased serum ceruloplasmin level is typical of Wilson's disease.
NEU-6.46.
A low-copper diet and BAL are the only therapy of Wilson's disease.
NEU-6.47.
Penicillinamine is of no value in the therapy of Wilson's disease.
NEU-6.48.
A copper analysis of a liver biopsy sample is an unimportant diagnostic
parameter in Wilson's disease.
NEU-6.49.
Sydenham's chorea has an unfavorable prognosis.
NEU-6.50.
Mental disorders are frequently observed in Sydenham's chorea.

NEU-6.51.
CSF pleocytosis occurs in Sydenham's chorea.
NEU-6.52.
In Sydenham's chorea pathologic movements usually disappear 4-6
months after their onset.
NEU-6.53.
Kyphoscoliosis, club-foot, ataxia and damage to the dorsal tracts of
the spinal cord are sufficient findings for the diagnosis of Friedreich's
ataxia.
NEU-6.54.
Cardiac involvement is not typical of Friedreich's ataxia.
NEU-6.55.
Friedreich's ataxia usually develops in newborns.
NEU-6.56.
Clinical symptoms alone are not sufficient to differentiate Friedreich's
ataxia from multiple sclerosis.
NEU-6.57.
Acute cerebellar ataxia resembles phenytoin-intoxication.
NEU-6.58.
Atrophy of the optic nerve helps to differentiate cerebellar ataxia
from phenytoin-intoxication.
NEU-6.59.
Neuritis of the optic nerve helps to differentiate multiple sclerosis
from acute cerebellar ataxia.
NEU-6.60.
Cerebellar symptoms occur in viral encephalitis.
NEU-6.61.
Cerebellar symptoms are typical in infectious neuropathy.
NEU-6.62.
Infectious mononucleosis can be followed by acute infectious
polyneuritis.
NEU-6.63.
Acute infectious polyneuritis is primarily a sensory disorder.
NEU-6.64.
Acute polyneuritis rarely has a fatal outcome.
NEU-6.65.
Pleocytosis of the CSF is a common finding in infectious polyNEUritis.
NEU-6.66.
The protein content of the CSF is rarely elevated in infectious
polyneuritis.

NEU-6.67.
Complete restitution of motor functions is rare in infectious
polyneuritis.
NEU-6.68.
In newborns, injuries to the superior brachial plexus occur more
frequently than those to the inferior plexus.
NEU-6.69.
In fractures of the humerus the median nerve is frequently injured.
NEU-6.70.
Pseudohypertrophy of the triceps surae is a common finding in
Duchenne-type muscular dystrophy.
NEU-6.71.
Facioscapulohumeral dystrophy is compatible with normal life.
NEU-6.72.
A shuffling gait and tip-toeing occur in myotonic dystrophy.
NEU-6.73.
Congenital myotonia is not accompanied by muscle atrophy.
NEU-6.74.
Congenital myotonia is aggravated by cold weather.
NEU-6.75.
Percussion myotonia is not observed in congenital myotonia.
NEU-6.76.
Congenital myotonia is inherited.
NEU-6.77.
Patients with congenital myotonia have a well-built stature.
NEU-6.78.
Occult spina bifida is not common in children.
NEU-6.79.
Meningocele is usually symptom-free.
NEU-6.80.
Meningomyelocele is generally accompanied by paraplegia.
NEU-6.81.
Klippel-Feil anomalies are rarely accompanied by meningomyelocele.
NEU-6.82.
A relationship exists between a closed lumbosacral defect and hydrocephalus.
NEU-6.83.
Mark the prognosis of the following pediatric tumors as favorable (F)
or unfavorable (U):
1. cerebellar astrocytoma

2. cerebral astrocytoma
3. medulloblastoma
4. tumor of the medulla
NEU-6.84.
Mark the following as causing hydrocephalus of a communicating
(C) or non-communicating type (NC):
1. subdural hematoma
2. post-inflammatory hydrocephalus
3. basilar impression
4. thrombosis of the lateral sinus
NEU-6.85.
Mark the following alterations as either of muscular (A) or NEUrogenic
(B) origin:
1. frequent fasciculations
2. predominantly proximal weakness
3. predominantly distal weakness
4. lack of fasciculations
5. fibrillations
6. muscle irritability at percussion
NEU-6.86.
Spine compression is frequently found in idiopathic kyphoscoliosis.
NEU-6.87.
Mark the true (T) and false (F) statements:
1) Low sexual activity is the most common sexual disturbance in
temporal lobe epilepsy.
2) Heterosexual hypersexual behavior is frequently observed in
temporal lobe epilepsy.
3) Surgical treatment has no beneficial effect on sexual behavior
in temporal lobe epilepsy.
NEU-6.88.
The use of dishes covered with a paint composed mainly of lead can
cause lead poisoning.
NEU-6.89.
Arterial constriction and a pale optic papilla are late signs of an
occlusion of the central retinal artery.
NEU-6.90.
The prognosis of an embolization of the central retinal artery is
favorable.
NEU-6.91.
Ophthalmic herpes zoster infection can cause a paralysis of the
oculomotor nerve.
NEU-6.92.
Herpes infection of the geniculate ganglion usually causes a facial
paralysis.
NEU-6.93.
In some cases herpes zoster spares the nuclei and causes paralysis

of only the nerve.


NEU-6.94.
Pleocytosis in the CSF is common in herpes zoster.
NEU-6.95.
Thrombosis of the lateral sinus is a usual sequel of orbital cellulitis.
NEU-6.96.
Thrombosis of the cavernosus sinus is a usual sequel of medial
otitis or mastoiditis.
NEU-6.97.
The glucose content of the CSF decreases in thrombosis of the dural
sinus.
NEU-6.98.
The characteristic symptoms of pontine hemorrhage are:
disconjugated eye movements, coma, quadraplegia, and narrow
pupils.
NEU-6.99.
A family history is always positive in cases of a migraine with aura.
NEU-6.100.
Parenteral histamine is the therapy of choice in migraine fits.
NEU-6.101.
Retroperitoneal fibrosis is a rare complication of a migraine which is
continuously treated by methysergide.
NEU-6.102.
In most cases the site of pain helps to differentiate a migraine from
trigeminal neuralgia.
NEU-6.103.
A Parkinsonian tremor is more pronounced during voluntary motions.
NEU-6.104.
Oculogyric crisis, respiratory tics, spasms, and torticollis are the
typical signs of postencephalitic parkinsonism.
NEU-6.105.
Sensory disturbances can quickly develop in arsenic and alcoholinduced
polyneuritis.
NEU 106.
In infectious polyneuritis there is a quick development of symptoms.
NEU-6.107.
A severe sensory disturbance is the most common symptom of
infectious polyneuritis.
NEU-6.108.
Micturition symptoms develop early in cervical spondylosis.

NEU-6.109.
In cervical spondylosis the reflexes of the lower extremities are
usually sluggish.
NEU-6.110.
Cervical and arm pain occur frequently in cervical spondylosis.
NEU-6.111.
Radiculopathy and myelopathy are two symptoms of cervical
spondylosis.
NEU-6.112.
The clinical picture of Dejerine-Sotta's disease corresponds to that of
a slowly developing polyneuritis.
NEU-6.113.
Peroneal muscle atrophy can always occur in hereditary-familial
diseases.
NEU-6.114.
The nerve conduction velocity is usually normal in Charcot-MarieTooth disease.
NEU-6.115.
Foot deformities occur in about 1/3 of the cases in Charcot-MarieTooth disease.
NEU-6.116.
Pigeons are one of the reservoirs of Cryptococcus.
NEU-6.117.
Cryptococcosis due to animal vaccine can be diagnosed in the CSF.
NEU-6.118.
In case of a well localized cerebral abscess the body temperature is
normal or subnormal.
NEU-6.119.
1) Creutzfeldt-Jakob's disease can be transferred to chimpanzees
with suspension prepared from the brain of infected patients.
2) Subacute spongiform viral encephalopathies are: Kuru,
Scrapie and Mink-encephalopathy.
NEU-6.120
Abscesses can be accurately localized with the EEG.
NEU-6.121.
In primary atrophy of the optic nerve many arteries cross the optic
papilla.
NEU-6.122.
In most cases, a defect of the visual field due to optic nerve damage
is localized in the inferior quadrant.
NEU-6.123.
Leptomeningeal damage frequently occurs in Sarcoidosis.

NEU-6.124.
Sarcoidosis can cause chronic adhesive arachnoiditis.
NEU-6.125.
Sarcoidosis can cause peripheral neuropathy.
SINGLE CHOICE QUESTIONS
Select the single best response to each of the following
questions!!!
NEU-6.126.
The cells of the fasciculus cuneatus are localized in the:
A) gelatinous substance
B) ganglia of the dorsal horns
C) nucleus proprius
D) all of the above.
E) none of the above
NEU-6.127.
Fibers enter the gelatinous substance from:
A) the lateral spinothalamic tract
B) the dorsal roots associated with sensation of pain and heat
C) the Clarke nucleus
D) all of the above
E) none of the above
NEU-6.128.
The angular vein:
A) is localized at the level of Monro's foramen
B) septal veins drain into the angular vein
C) both of the above
D) none of the above
NEU-6.129.
The following two lobes constitute the inferior part of the parietal
lobe:
A) triangular and opercular lobes
B) supermarginal and angular lobes
C) triangular and angular lobes
D) all of the above
E) none of the above
NEU-6.130.
Pigment granules can be observed in:
A) the cells of the substantia nigra
B) the cells of the locus ceruleus
C) the dorsal efferent nuclei of the vagus
D) all of the above
E) none of the above
NEU-6.131.
Which of the following is a parasympathetic nucleus?
A) Edinger-Westphal nucleus
B) Inferior salivatory nucleus

C) Dorsal efferent nucleus of the vagus


D) all of the above
E) none of the above
NEU-6.132.
The putamen and the caudate nucleus are termed as:
A) archistriatum
B) paleostriatum
C) neostriatum
D) all of the above
E) none of the above
NEU-6.133.
Which types of cells can be found in the brain cortex?
A) pyramidal cells
B) stellate cells
C) Cajal's cells
D) all of the above
E) none of the above
NEU-6.134.
The recurrent artery (Heubneri) originates from:
A) the internal carotid artery
B) the anterior communicating artery
C) the anterior choroid artery
D) all of the above
E) none of the above
NEU-6.135.
The three parts of the hippocampus are as follows:
A) praesubiculum, subiculum, prosubiculum
B) gyrus dentatus, cornu Ammonis, subiculum
C) cornu Ammonis, alveus, enterorhinal cortex
D) all of the above
E) none of the above
NEU-6.136.
What percent of the total cortical surface is comprised by the frontal
lobe?
A) 20%
B) 40%
C) 60%
D) 70%
E) 90%
NEU-6.137.
Which areas of the body represented in Penfield's homunculus are
disproportionally large?
A) hand, leg
B) face, trunk
C) face, hand
D) all of the above
E) none of the above
NEU-6.138.
The cells of the optic nerve fibers are:

A) retinal cones and rods


B) retinal bipolar cells
C) retinal ganglion cells
D) all of the above
E) none of the above
NEU-6.139.
Efferent neurons of the cerebellar cortex are:
A) Purkinje fibers
B) climbing fibers
C) mossy fibers
D) all of the above
E) none of the above
NEU-6.140
Cerebral posterior arteries:
A) originate from the basilar artery
B) can join with the superior cerebellar artery
C) supply the occipital lobe
D) all of the above
E) none of the above
NEU-6.141.
The "artery of cerebral hemorrhage" or the "Charcot's artery" is:
A) a branch of the lenticulo-striate system
B) a branch of the chorioid artery
C) a branch of the thalamus-geniculatum system
D) all of the above
E) none of the above
NEU-6.142.
The "genu" of the internal capsule carries the following fibers:
A) fibers from the anterior thalamic nuclei
B) frontoparietal fibers
C) corticobulbar fibers
D) all of the above
E) none of the above
NEU-6.143.
The limbic system contains:
A) the anterior olfactory nucleus
B) the hippocampus
C) Broca's diagonal fasciculus
D) all of the above
E) none of the above
NEU-6.144.
Select the right projection of the fibers originating from the inferior
part of the retina (superior visual field):
A) inferior part of the fissura calcarina
B) superior part of the fissura calcarina
C) anterior part of the fissura calcarina
D) posterior part of the fissura calcarina
E) none of the above
NEU-6.145.

Following the occlusion of the saggital sinus:


A) the ventricles dilate
B) the CSF pressure increases
C) both of the above
D) none of the above
NEU-6.146.
In multiple sclerosis:
A) Waller's degeneration is a frequent outcome
B) there is a decomposition of axons
C) oligodendroglia remain unchanged
D) all of the above
E) none of the above
NEU-6.147.
Hematogenous abscesses occur most frequently in:
A) the thalamus
B) the cerebellum
C) at the border between the grey and white matter
D) none of the above
NEU-6.148. The most common brain tumor is a:
A) metastatic tumor
B) glioma
C) meningioma
D) none of the above
NEU-6.149.
The blood-brain barrier is composed of.
A) capillary endothelium
B) basal membrane
C) podocytes of the astrocytes
D) all of the above
NEU-6.150.
Central chromatolysis is frequently observed in Betz's cells:
A) true, because the axons are too long
B) false
C) true, because the axons have many branches
D) true, but both (A) and (C) are true
NEU-6.151.
Which of the following are glial elements?
A) astrocytes
B) ependymocytes
C) both of the above
D) none of the above
NEU-6.152.
Senile plaques:
A) contain amyloid
B) occur in Alzheimer's disease
C) both of the above
D) none of the above
NEU-6.153.

If the embryonic neural tube does not close:


A) anencephaly can occur
B) dysraphia can occur
C) spina bifida can occur
D) all of the above
E) none of the above
NEU-6.154.
Epidural hematoma:
A) can be of venous or arterial origin
B) can develop following cranial trauma without unconsciousness
C) both of the above
D) none of the above
NEU-6.155.
Cortical contusion differs from hemorrhagic infarction in that:
A) contusion affects all the layers of the cortex
B) contusion usually affects one lobe
C) both of the above
D) none of the above
NEU-6.156.
Pseudolamellar necrosis:
A) develops only after strangulation
B) usually develops during paresis
C) is rather observed in anoxic encephalopathy
D) none of the above
NEU-6.157.
Hypoxic encephalopathy of newborns:
A) causes alterations in the medullary nuclei
B) causes alterations in the cortex
C) causes alterations in the striatum
D) none of the above
NEU-6.158.
Alterations in Wernicke's encephalopathy occur:
A) in the medial group of the thalamic nuclei and in the
mammilary body
B) in the thalamic pulvinar and and periaqueductal grey matter
C) both of the above
D) none of the above
NEU-6.159.
Polyneuropathy causes:
A) Waller's degeneration
B) segmental demyelinization
C) distal axonal fragmentation
D) all of the above
NEU-6.160.
Saccular aneurysms:
A) 85% develop in the circle of Willis
B) 15% are multiple
C) both of the above
D) none of the above

NEU-6.161.
Perivenous demyelinization occurs:
A) in experimental allergic encephalomyelitis
B) after rabies' vaccination
C) after a varicella infection
D) after a measles infection
E) all of the above
NEU-6.162.
The following nerves and nerve roots contain parasympathetic fibers:
A) cranial and lumbar nerves
B) cranial and sacral nerves
C) cervical and lumbar nerves
D) cervical and sacral nerves
E) cranial and thoracic nerves
NEU-6.163.
Which of the following is used for the differential diagnosis of
myasthenic weakness and cholinergic crisis?
A) atropine iv.
B) edrophonium iv.
C) neostigmine im.
D) edrophonium po.
E) neostigmine po.
NEU-6.164.
Which of the following is a neurotransmitter in the parasympathetic
postganglionic synapses?
A) muscarine
B) norepinephrine
C) epinephrine
D) acetylcholine
E) buterylcholine
NEU-6.165.
Which of the following is not an intermediate product of norepinephrine
synthesis?
A) tyrosine .
B) epinephrine
C) dopamine
D) DOPA
NEU-6.166.
Which of the following is not a side-effect of antihistamine drugs?
A) supression of sea-sickness
B) alleviation of some symptoms of Parkinson's disease
C) alleviation of symptoms of tertiary syphilis
D) suppression of petit mal epilepsy
NEU-6.167.
The typical memory loss occurring in Korsakoffs syndrome is due to
a lesion of which of the following structures?
A) parietal associative cortex
B) frontal associative cortex
C) mammilary body, limbic cortex

D) habenula
NEU-6.168.
Which fibers convey nociceptive stimuli?
A) unmyelinated C fibers
B) myelinated fibers of small diameter
C) both of the above
D) none of the above
NEU-6.169.
A bilateral lesion of the feeding center in the lateral hypothalamus
causes:
A) aphasia
B) aphagia
C) hyperphagia
D) polyuria
NEU-6.170.
Mark the false statement - the pyramidal tract:
A) a part of it arises from Brodmann's area 4
B) most of it originates from neocortical areas
C) 80-90% of the pathways cross in the inferior part of medulla
NEU-6.171.
The following was found in a motor unit of a patient with myasthenia
gravis with prolonged muscle fatigue:
A) a decreased amplitude of spikes
B) a decreased frequency of spikes
C) both of the above
D) none of the above
NEU-6.172.
Select the most typical feature of synaptic transmission:
A) conduction is not unidirectional
B) it is a combination of electrical and chemical transmission
C) norepinephine is always involved
D) synaptic transmission cannot be blocked
NEU-6.173.
The conduction velocity of an axon:
A) increases as the diameter increases
B) is less in axons with lower threshold values
C) both of the above
D) none of the above
NEU-6.174.
The motor unit:
A) consists of the alpha-motor neurons of a given muscle
B) consists of the alpha and gamma-motor neurons of a given
muscle
C) consists of an alpha-motor neuron and the muscle fibres
supplied by this neuron.
D) consists of the muscle fibers supplied by one motor neuron
and a nerve.
NEU-6.175.

The retinal representation of the eye is observed in:


A) the lateral geniculate body
B) the primary visual cortex
C) both of the above
D) none of the above
NEU-6.176.
The membrane potential is created by a:
A) concentration difference on both sides of the membrane and
different permeabilities for individual ions
B) Na+-pump
C) resting potential
D) reduced activity of the ions in the cytoplasm
NEU-6.177.
Typical electric signs of a paradox sleeping behavior are:
A) low amplitude, high frequency
B) high amplitude, high frequency
C) low amplitude, low frequency
D) high amplitude, low frequency
NEU-6.178.
After stimulation of the sciatic nerve the alpha and beta components
of a complex action potential represent the following:
A) two independent and different conduction velocity values of
two fibers with different diameters
B) depolarization after a complete action potential
C) repetitive firing
D) orthodromic and antidromic conduction in long fibers
NEU-6.179.
After transection of the, spinal cord in humans:
A) flexion reflexes reappear earlier than the extension reflexes
B) extension reflexes reappear earlier than the flexor reflexes
C) flexor and extensor reflexes reappear at the same time
D) after trauma sweating is a common response to strong
stimuli
NEU-6.180.
The typical firing rate in a classic petit mal fit is:
A) 3 cycles/sec
B) 6 cycles/sec
C) 9 cycles/sec
D) in the alpha range of 8-13 cycles/sec
NEU-6.181.
Loss of which of the following functions develops in spinal shock
caused by transection of the spinal cord?
A) voluntary movement
B) sensory function
C) spinal reflexes
D) all of the above
NEU-6.182.
A patient in hypoglycemic coma will regain consciousness after an
intravenous injection of which of the following:

A) fructose
B) lactic acid
C) ethanol
D) none of the above
NEU-6.183.
The following substrate is mainly metabolized by the human
brain:
A) glutamate
B) albumin
C) glucose
D) none of the above
NEU-6.184.
During generalized seizures brain metabolism:
A) increases
B) decreases
C) remains unchanged
NEU-6.185.
During hypoxia how does the brain alter its lactate metabolism?
A) lactate metabolism decreases
B) lactate metabolism increases
C) lactate metabolism remains unchanged
NEU-6.186.
Which of the following amino acids occurs in the nervous system?
A) glutamine
B) asparagine
C) glycine
D) valine
E) all of the above
F) none of the above
NEU-6.187.
Cyanide:
A) has no effect on the brain
B) affects the cytochrome system
C) is a metabolized fatty acid
D) none of the above
NEU-6.188.
Phosphorylase:
A) does not occur in the brain
B) occurs in the brain in low concentrations
C) occurs in the brain in high concentrations
NEU-6.189.
The most common tumor of the 3rd ventricle is:
A) ependymoma
B) pinealoma
C) colloid cyst
D) medulloblastoma
E) astrocytoma
NEU-6.190.

The EEG in subacute sclerosing panencephalitis:


A) shows periodicity
B) is similar to that of Creuzfeldt-Jakob's disease
C) is synchronous with myoclonus
D) all of the above
E) none of the above
NEU-6.191.
Alpha rhythm recorded on the EEG:
A) is usually found in the occipital leads
B) disappears if the subject opens his eyes
C) both of the above
D) none of the above
NEU-6.192.
Which diagnosis is assumed froM the following X-ray findings separation of the sutura and increased density of the long bone
ends:
A) hyperparathyroidism
B) fibrotic dysplasia
C) lead intoxication
D) rachitis
E) arsenic intoxication
NEU-6.193.
The most common tumor which causes sella enlargement is:
A) eosinophilic pituitary adenoma
B) basophilic pituitary adenoma
C) chromophobic pituitary adenoma
D) dorsurri sellae meningioma
E) craniopharyngioma
NEU-6:194.
After cerebral infarction:
A) a slow-wave focus can develop
B) in a persisting hemiparesis the EEG can normalize
C) both of the above
D) none of the above
NEU-6.195.
The frequency of the alpha rhythm is:
A) 4-6 cycles/sec
B) 6-8 cycles/sec
C) 8-12 cycles/sec
D) none of the above
NEU-6.196.
In a chronic subdural hematoma:
A) the EEG can be normal
B) the EEG exhibits a slow-wave focus
C) the EEG has decreased amplitude
D) all of the above
E) none of the above
NEU-6.197.
The conducting velocity of a normal motor nerve measured with

electromyography is:
A) 10-20 m/sec
B) 20-30 m/sec
C) 40-60 m/sec
D) none of the above
NEU-6.198.
A cloudy homogenous calcification on the skull X-ray suggests:
A) oligodendroglioma
B) meningioma
C) aneurysm
D) subdural hematoma
E) Sturge-Weber's syndrome
NEU-6.199.
Hyperostosis, increased vascularization, and calcification on the
skull X-ray imply the following diagnosis:
A) glioblastoma
B) oligodendrogloma
C) dermoid cyst
D) meningioma
E) fibrous dysplasia
F) frontal internal hyperostosis
NEU-6.200.
Positive radionuclide brain scanning occurs in:
A) fibrous dysplasia
B) eosinophilic granuloma
C) cranial metastasis
D) cranial osteomyelitis
E) all of the above
F) none of the above
NEU-6.201.
The tumor which most frequently causes supersellar calcification is:
A) chromophobic adenoma
B) optic nerve glioma
C) craniopharyngioma
D) aneurysm
E) clivus chordoma
NEU-6.202.
Of all gliomas calcification occurs in:
A) ependymoma
B) medulloblastoma
C) oligodendroglioma
D) astrocytoma
E) ganglioglioma
NEU-6.203.
The EEG in hepatic encephalopathy:
A) can show a generalized slowing down
B) can show triphasic waves
C) can show impairement of the alpha rhythm
D) all of the above
E) none of the above

NEU-6.204.
Blood in a CSF sample gained with lumbar puncture quickly disappears:
A) if it was caused by subarachnoid bleeding
B) if bleeding was caused by the puncture itself
C) if it was caused by thrombosis of the cerebral vessels
D) if it was caused by an embolism of the cerebral vessels
E) suggests rupture of an intracranial aneurysm
NEU-6.205.
An electrode is considered to be positioned over an epileptic focus if:
A) alpha-activity is observed
B) if the so-called "inversed phase" phenomenon is observed
C) both (A) and (B) are true
D) the rest activity is recorded
E) none of the above
NEU-6.206.
Case Study:
Progressive gait disorders, a lack of tendon reflexes, a high protein
level in the cerebrospinal fluid, and granules yielding brown staining
with toluidine-blue were found in a 2-year-old child. Select one of
the following diagnoses:
A) Schilder's disease
B) metachromatic leukodystrophy
C) spongious degeneration
D) diffuse sclerosis
E) cerebral paralysis
NEU-6.207.
Eczema, blond hair, blue eyes and mental retardation are typical of:
A) phenylketonuria
B) cretinism
C) tuberous sclerosis
D) Down's syndrome
E) toxoplasmosis
NEU-6.208.
In acute purulent meningitis, the CSF is NEVER:
A) opalescent
B) clear
C) turbid ("cloudy")
D) purulent
E) flowing at increased pressure
NEU-6.209.
Select a CSF finding typical of an acute purulent meningitis:
A) 25 granulocytes/mm3, protein 45 mg%, sugar 50 mg%
B) 250 granulocytes/mm3, protein 45 mg%, sugar 50 mg%
C) 250 mononuclear cells/mm3, protein 450 mg%, sugar 50 mg%
D) 2500 granulocytes/mm3, protein 450 mg%, sugar 50 mg%
E) 2500 granulocytes/mm3, protein 450 mg%, sugar 10 mg%

NEU-6.210.
CSF gained with a lumbar puncture is opalescent, flows at increased

pressure, the sugar level is low, and fibrin precipitation is observed.


The most probable diagnosis is:
A) subarachnoid hemorrhage
B) bleeding due to the intervention
C) tuberculous meningitis
D) normal (but the patient is at strain)
E) brain tumor
NEU-6.211.
The infant has no appetite, vomits, has fever and seizures. What
should be done?
A) the cultivation of any nasopharyngeal discharge
B) a lumbar puncture
C) a urine sample gained by catheter
D) check for any emotional disorders
E) an intravenous pyelography
NEU-6.212.
A 6-year-old child with a fever and stiff neck has a sore throat,
headache, and arthralgia. What should be done?
A) th cultivation of any nasopharyngeal discharge
B) check the RBC sedimentation rate
C) a lumbar puncture
D) an X-ray of the elbows and knee joints
E) an ECG
NEU-6.213.
Which disease is accompanied by the following symptoms: fever,
confusion, exophthalmus, chemosis and paralysis of the extraocular
muscles?
A) a tumor of the minor wing of the sphenoid bone
B) a craniopharyngioma
C) septic thrombosis of the cavernous sinus
D) an abscess of the temporal lobe
E) an infraclinoid aneurysm
NEU-6.214.
Which disease is accompanied by the following symptoms: fever,
lack of occipital stiffness, unilateral-later bilateral neurologic signs,
epileptic seizures:
A) parasaggital meningioma
B) cerebral abscess
C) cerebral embolism
D) thrombosis of the sagittal sinus
E) mucormycosis
NEU-6.215.
How many days after smallpox vaccination does postvaccination
encephalomyelitis develop?
A) 2-3 days
B) 4-6 days
C) 10-12 days
D) 20-25 days
E) 30-40 days

NEU-6.216.
Seizures, chorioretinitis, hydrocephalus, and calcification of the
cranial bones in infants suggest:
A) Sturge-Weber disease
B) toxoplasmosis
C) connatal neu rosyphilis
D) tuberous sclerosis
E) tuberculotic meningitis
NEU-6.217.
Which symptom occurs only rarely in infants with a subdural
hematoma?
A) fever
B) fontanelle protrusion
C) hemiparesis
D) papilledema
E) retinal hemorrhage
NEU-6.218.
The time elapsed between trauma and the clinical symptoms of an
epidural hematoma in children is:
A) seconds
B) minutes
C) hours
D) days
E) weeks
NEU-6.219.
Which of the following are the most common symptoms of an epidural
hematoma in a child?
A) papilledema and stupor
B) retinal bleeding and coma
C) ataxia and hemiparesis
D) hemiparesis and contralateral pupil dilation
E) hyperreflexia and contralateral paresis of cranial nerve VI.
NEU-6.220.
If the case history of an infant mentions trauma, vomiting, seizures,
and an enlarged head. What should be done?
A) an EEG
B) a lumbar puncture
C) an X-ray of the skull
D) a CT of the skull
E) a radionuclide scan of the brain
NEU-6.221.
The most common symptom of increased intracranial pressure in
children is:
A) a loss of visual acuity
B) stupor
C) vomiting
D) ataxia
E) diplopia

NEU-6.222.
Headache is usually a bad sign in pediatric brain tumors if it:
A) starts after school
B) starts after dinner
C) starts after going to bed
D) starts after getting up
E) starts after lunch
NEU-6.223.
What should be suspected if the following symptoms occur: visual
disorders, diabetes insipidus, growth retartdation?
A) pituitary adenoma
B) craniopharyngioma
C) subdural hematoma
D) medulloblastoma
E) retinoblastoma
NEU-6.224.
What should be suspected if the following symptoms occur: vomiting,
headache, ataxia, nystagmus, limited neck movement?
A) craniopharyngioma
B) medulloblastoma
C) glioma of the optic nerve
D) subdural hematoma
E) astrocytoma of the frontal lobe
NEU-6.225.
Medulloblastoma usually:
A) develops in the thalamus
B) develops in the basal ganglia
C) develops in the cerebellar hemispheres
D) develops on the top of the fourth ventricle
E) develops in the base of the third ventricle
NEU-6.226.
A rare pediatric brain tumor is:
A) glioblastoma
B) pontine glioma
C) medulloblastoma
D) glioma of the optic nerve
E) cerebellar astrocytoma
NEU-6.227.
The most common cause of an intracranial hemorrhage in children is:
A) an intracranial aneurysm
B) trauma
D) glomerulonephritis
E) hepatic disease
NEU-6.228.
Wilson's disease:
A) a Kayser-Fleischer ring is observed in nearly all cases,
B) it is usually not inherited
C) it is resistant to all kinds of therapy
D) has only neurologic symptoms
E) can only be diagnosed by a cortex biopsy

NEU-6.229.
Case Study:
Retinitis pigmentosa, cerebellar ataxia, pes causs, and
acanthocytosis were found in a 6-year-old child. What is the most
probable diagnosis?
A) Ataxia telangiectasia
B) Friedreich's ataxia
C) Refsum's disease
D) Juvenile lipoidosis
E) none of the above
NEU-6.230.
The best diagnostic method in pediatric head trauma is:
A) X-ray of the skull
B) pneumoencephalography (PEG)
C) skull CT
D) EEG
E) radionuclide brain scan.
NEU-6.231.
Seizures in pediatric patients:
A) occur most frequently in tumors
B) occur most frequently with high fever
C) rarely occur after the introduction of "Sabin's vaccination
D) rarely occur in subdural hematoma
E) rarely occur in hypocalcemia
NEU-6.232.
Case Study:
Shortly after a penetrating foot injury a pediatric patient developed
trismus, risus sardonicus, opisthotonus, abdominal rigidity. The
most probable diagnosis is:
A) drug allergy
B) hysteria
C) tetanus
D) gas gangrene
E) meningitis
NEU-6.233.
Subdural hematomas in infants are:
A) bilateral and occipital
B) unilateral and frontoparietal
C) bilateral and frontoparietal
D) unilateral and basal
E) unilateral and temporal
NEU-6.234
Which of the following CSF findings occur in acute purulent meningitis?
A) normal pressure, clear and xanthochromic
B) normal pressure, clear and colorless
C) increased pressure, hemorrhagic
D) increased pressure, turbid
E) increased pressure, clear

NEU-6.235.
Which of the following pediatric tumors spreads (is disseminated) in
the CSF prior to surgery?
A) cerebellar astrocytoma
B) medulloblastoma
C) craniopharyngioma
D) glioma of the optic nerve
E) teratoma
NEU-6.236.
Epicanthus, mongolism, short phalanges, and mental retardation
are typical of:
A) cretinism
B) gargoylism
C) Down's syndrome
D) prenatal rubeola
E) Hartnup disease
NEU-6.237.
The following maternal infection affects the fetus:
A) syphilis
B) rubeola
C) toxoplasmosis
D) all of the above
E) none of the above
NEU-6.238.
"Hypsarrhythmia" or infantile spasm:
A) is frequently observed in infants
B) is frequently related to cerebral malformations
C) is frequently accompanied with mental retardation
D) shows no improvement to anticonvulsive drugs
E) all of the above
NEU-6.239.
Petit mal therapy (trimethadione) should be discontinued if.
A) cutaneous symptoms develop
B) blood dyscrasia occurs
C) photophobia develops
D) nephrosis occurs
E) all of the above
NEU-6.240.
The following finding(s) are similar at the age of three months and in
adults:
A) CSF findings
B) EEG results
C) nerve conduction velocities
D) all of the above
E) none of the above
NEU-6.241.
In normal infants:
A) a Babinski reflex can be elicited in the lower extremities
B) alpha activity is predominant in the EEG
C) the anterior fontanelle closes in newborns

D) closing of the sutures has occurred


E) all of the above
NEU-6.242.
Myoclonic seizures are observed:
A) in hereditary photosensitive epilepsy
B) in maple syrup disease
C) in subacute sclerotizing panencephalitis
D) in juvenile amaurotic idiots
E) all of the above
NEU-6.243.
Hydrocephalus can occur:
A) in Arnold-Chiari's malformation
B) in spina bifida and meningomyelocele
C) in a closed foramen of Magendie
D) in atresia of the aqueduct
E) all of the above
NEU-6.244.
Which of the following is typical of tuberous sclerosis?
A) adenoma sebaceum, Scagreen's (non-pigmented) patches, nail
fibroma
B) cortical gliosis and calcification
C) mental retardation and seizures
D) nodular tumors of the kidney, heart, and brain
E) all of the above
NEU-6.245.
One of the following is typical of neurofibromatosis:
A) cafe-au-lait spots; neurofibromas on the skin
B) seizures and symptoms of a space occupying process
C) it affects the optic nerve and retina
D) a sand-glass tumor
E) all of the above
NEU-6.246.
Intracranial calcification is typical of.
A) toxoplasmosis
B) encephalofacial angiomatosis
C) tuberous sclerosis
D) all of the above
E) none of the above
NEU-6.247.
Ectodermal alterations are observed. in:
A) tuberous sclerosis
B) multiple sclerosis
C) Krabbe's diffuse sclerosis
D) Schilder's diffuse sclerosis
E) none of the above
NEU-6.248.
The CSF contains 180 mononuclear cells/mm3, its sugar content is
low. Select one of the following diagnoses:
A) tuberous meningitis

B) sarcoid meningitis
C) cryptococcus meningitis
D) all of the above
E) none of the above
NEU-6.249.
Complications of pediatric meningitis include:
A) seizures, mental retardation
B) hydrocephalus
C) hemiparesis, ataxia
D) blindness, deafness
E) all of the above
NEU-6.250.
Subacute sclerosing panencephalitis can be caused by the:
A) ECHO virus
B) Measles virus
C) Herpes simplex virus
D) Coxsackie virus
E) none of the above
NEU-6.251.
The most frequently occurring pediatric brain tumor is:
A) meningioma
B) metastasis
C) glioma
D) adenoma
E) none of the above
NEU-6.252.
The first symptoms of an intracranial tumor in children are:
A) behavioral disturbances
B) hemiparesis and hyperreflexia
C) seizures and coma
D) headaches and vomiting
E) none of the above
NEU-6.253.
Sydenham's chorea:
A) usually develops in infants
B) is usually accompanied by rheumatic fever
C) mainly occurs in boys
D) all of the above
E) none of the above
NEU-6.254.
In Friedreich's ataxia:
A) only bone deformations occur in siblings
B) spinal and leg deformations frequently occur
C) the cells of the ventral horn are usually unaffected
D) there is a lack of tendon reflexes with extension-type plantar
reflexes
E) impaired sensation of posture and vibration occurs
F) all of the above

NEU-6.255.
Which of the following is typical of Charcot-Marie-Tooth disease?
A) it is dominantly inherited
B) predominant atrophy of the small muscles of the hands and
leg muscles
C) it is sometimes accompanied by Friedreich's ataxia
D) the nerve conduction velocity is markedly decreased
E) all of the above
NEU-6.256.
If a child has a repeated painless hand injury the following is assumed:
A) a congenital lack of feeling pain
B) familial dysautonomia
C) syringomyelia
D) all of the above
E) none of the above
NEU-6.257.
After an intramuscular penicillin injection an immediate sciatic
nerve neuropathy develops. What is suspected?
A) an allergic reaction
B) direct injury to the sciatic nerve
C) a toxic effect of penicillin on the nerve tissue
D) none of the above
NEU-6.258.
The X-ray study is a valuable tool in the diagnoses of lead intoxication
because it reveals:
A) lead deposits along the axis of tubular bones
B) abdominal images reveal lead in the gastrointestinal tract
C) suture disruption on skull images
D) all of the above
E) none of the above
NEU-6.259.
Case Study:
A deep injury of the foot has been treated with tetanus-antitoxin;
several days later the patient reported weakness and pain in his
arms. What is suspected:
A) tetanus
B) septicemia
C) botulism
D) brachial serum-neuritis
E) none of the above
NEU-6.260.
In children peripheral neuropathy occurs in:
A) diabetes
B) lupus erythematosus
C) sarcoidosis
D) lymphoma
E) all of the above
NEU-6.261.
Acute infective polyneuritis should be differentiated from:
A) poliomyelitis

B) acute transverse myelitis


C) diphteritic polyneuritis
D) all of the above
E) none of the above
NEU-6.262.
Pseudohypertrophic muscular dystrophy (Duchenne's):
A) is transferred by the X chromosome and is recessivelyinherited
B) only boys are affected, girls are carriers
C) by the age of 12, the patients cannot walk
D) the serum creatine kihase activity is elevated
E) a lack of the dystropin protein in the muscle
F) all of the above
NEU-6.263.
Myotonic dystrophy (myotonia atrophica):
A) is dominantly inherited
B) is accompanied by endocrinopathies
C) occurs in both men and women
D) all of the above
E) none of the above
NEU-6.264.
Duchenne's dystrophy:
A) the symptoms develop at the age of 3-5
B) sometimes mental retardation occurs
C) can be best diagnosed with muscle biopsy
D) the serum creatine-phosphokinase is elevated
E) all of the above
NEU-6.265.
In healthy newborns:
A) Moro's reflex can be elicited
B) the patella and Babinski reflexes can be elicited
C) optokinetic nystagmus can be elicited
D) all of the above
E) none of the above
NEU-6.266.
Which of the following etiologic types of mental retardation can be
successfully treated by a specific therapy?
A) idiopathic hypoglycemia
B) galactosemia
C) phenylketonuria
D) hypothyroidism
E) all of the above
NEU-6.267.
Case Study:
After a first-second degree burn injury, a 14-year-old child has a
fluctuating consciousness with parallel changes in the EEG. What
can be suspected?
A) an acute subdural hematoma
B) an electrolyte disturbance and imbalance
C) brain anoxia
D) burn encephalopathy

E) none of the above


NEU-6.268.
Which of the following neurologic complications can occur in bacterial
endocarditis?
A) cerebral embolism
B) acute cerebral syndrome
C) meningitis and brain abscess
D) subarachnoid hemorrhage from a mycotic aneurysm
E) all of the above
NEU-6.269.
Case Study:
3 weeks after a myocardial infarction a 65-year-old male patient complains
of pain and stiffness in his left arm. What can be suspected?
A) a brachial plexus injury
B) a cerebral ischemic episode
C) a cerebral embolism
D) the shoulder-hand syndrome
E) none of the above
NEU-6.270.
Which of the following is accompanied by cardiac symptoms?
A) myotonic dystrophy
B) Duchenne's dystrophy.
C) Refsum's syndrome
D) Freidreich's ataxia
E) tuberous sclerosis
F) all of the above
NEU-6.271.
Case Study:
A 45-year-old male patient complains of lumbar pain radiating to his
leg.Ipsilateral Achilles areflexia and hypesthesia in the small toe were
found.
At which spinal root level does the patient have a disc hernia
compression?
A) L2 root
B) L3 root
C) L4 root
D) L5 root
E) S1 root
NEU-6.272.
The following are typical of retrobulbar neuritis - central scotoma and:
A) hyperemia of the papilla
B) papilledema
C) slightly opaque vitreous body
D) all of the above
E) none of the above
NEU-6.273.
When do neural symptoms develop in mumps?
A) prior to the development of parotitis
B) simultaneously with parotitis
C) 5 days following the onset of parotitis

D) 7-14 days following the onset of parotitis


E) 14-21 days following the onset of parotitis
NEU-6.274.
Which of the following should be primarily performed in "Juvenile"
Parkinson's disease?
A) cerebral angiography
B) slit lamp test
C) muscle biopsy
D) all of the above
E) none of the above
NEU-6.275.
Herpes zoster:
A) is caused by the herpes simplex virus
B) always has sensory involvement
C) has no etiologic relationship with varicella
D) is usually restricted to spinal nerves
E) none of the above
NEU-6.276.
Central scotoma occurs primarily in:
A) pituitary tumors
B) papilledema
C) inflammation of the optic nerve
D) lead neuropathy
E) optic nerve gliomas
NEU-6.277.
A faded edge of the papilla, visual impairment, eye pain and tenderness
occur in:
A) papilledema
B) occlusion of the central retinal veins
C) papillitis
D) occlusion of the central retinal artery
E) chiasma tumor
NEU-6.278.
Hypotensive syncope usually occurs in:
A) decreased resistance of the peripheral arterial resistance
B) decreased cardiac output
C) decreased circulating blood volume
D) vertebral diseases of the aorta or carotid artery
E) none of the above
NEU-6.279.
The clinical relevance of a Drusen:
A) occurs in brain tumors
B) occurs in visual field defects
C) occurs in degenerative diseases
D) can be mixed with papilledema
E) is of geriatric origin
NEU-6.280.
Papillitis is accompanied by central scotoma and:
A) papilla hyperemia

B) papilledema
C) slightly opaque color of the vitreous body
D) all of the above
E) none of the above
NEU-6.281.
The most common visual field defect in multiple sclerosis is:
A) central scotoma
B) quadrant anopia
C) hemianopia
D) full blindness
E) none of the above
NEU-6.282.
The most common cause of optic nerve neuritis is:
A) smoking
B) multiple sclerosis
C) alcohol
D) encephalitis
E) vasculitis
NEU-6.283.
Occlusion of the main trunk of the middle cerebral artery results in:
A) hemiplegia and hemianesthesia
B) hemiparesis
C) hemiplegia, hemianesthesia and homonymous hemianopsia
D) the thalamus syndrome and choreoathetosis
NEU-6.284.
Thrombosis of which artery is the most common vascular lesion of
the brain stem?
A) the anterior chorioid artery
B) the paramedian pontine artery
C) the anterior inferior cerebellar artery
D) the posterior inferior cerebellar artery
E) the superior cerebellar artery
NEU-6.285.
Define the following: emotional lability, dysarthria, dysphagia, hyperactive
mandibular reflex:
A) Wallenberg's syndrome
B) Weber's syndrome
C) Pseudobulbar syndrome
D) Millard-Gubler's syndrome
E) Benedict's syndrome
NEU-6.286.
A sudden development of hemiplegia and contralateral ophthalmoplegia
is most probably due to alterations in the:
A) frontal lobe
B) lateral medulla
C) mesencephalon
D) pons
E) paracentrallobe
NEU-6.287.

A sudden development of dysphagia, dysarthria, ipsilateral cerebellar


symptoms, Horner's syndrome, and crossed disturbances of the pain sensation
(face-body dissociation) most probably suggest alterations in the:
A) anterior inferior cerebellar artery
B) superior cerebellar artery
C) paramedian pons artery
D) inferior posterior cerebellar artery
E) vertebro-basilar artery
NEU-6.288.
The most common site of congenital aneurysms is:
A) between the middle cerebral artery- internal carotid artery
B) between the anterior cerebral artery - anterior communicant artery
C) between the posterior cerebral artery - posterior communicant
artery
D) between the basilar artery - vertebral artery
E) the ophthalmic artery
NEU-6.289.
The symptoms of a left-sided epidural hematoma in comatous patients
include:
A) contralateral hemiplegia
B) ipsilateral hemiplegia
C) contralateral hemiplegia and ipsilateral paralysis of CN III
D) contralateral hemiplegia and paralysis of CN III
E) ipsilateral hemiplegia and paralysis of CN III
NEU-6.290.
Case Study:
A male patient complains of diplopia and numbness of one side of his body
and face. Later he became comatous, developed quadriplegia and narrow
pupils not sensitive to light. The most probable site of thrombosis is
the:
A) anterior cerebral artery
B) basilar artery
C) superior cerebellar artery
D) inferior cerebellar artery
E) lenticulostriatal artery
NEU-6.291.
Case Study:
A 64-year-old male patient has mental disorders, abnormal gait and
incontinence. The CT study revealed expanded ventricles and compression
of the subarachnoid space. The most probable diagnosis is:
A) Alzheimer's disease
B) Pick's disease
C) a normal pressure hydrocephalus
D) Jakob-Creutzfeldt disease
E) Quentin's disease
NEU-6.292.
The worst prognosis of the functional restitution of the bladder is expected
in:
A) a thoracic spinal injury
B) a cervical spinal injury
C) a lumbar spinal injury

D) an injury of the sacral plexus


E) a cervical plexus injury
NEU-6.293.
The best diagnostic approach in familial periodic paralysis is:
A) a muscle biopsy
B) an EMG
C) the serum K+ concentration
D) an oral glucose tolerance test
E) the nerve conductance velocity
NEU-6.294.
Nasal discharge of cerebrospinal fluid occurs in:
A) cranial fractures
B) brain tumors
C) congenital defects
D) pseudotumor
E) all of the above
NEU-6.295.
A sudden development of hemiplegia with contralateral facial paresis
and inward turning of the eye suggests thrombosis of the:
A) short paramedian pontine artery
B) posterior inferior cerebellar artery
C) anterior inferior cerebellar artery
D) internal auditory artery
E) posterior chorioid artery
NEU-6.296.
Which structure remains unaffected in multiple sclerosis?
A) the cerebellar tract
B) the pyramidal tract
C) the motoneurons of the ventral horn
D) the spinal dorsal horn
E) the visual tract
NEU-6.297.
Lumbar and low extremity pain, peripheral low extremity paresis
and urination disorders occur in:
A) lumbar spondylosis
B) lumbar disc hernia
C) a tumor of the cauda equina
D) all of the above
NEU-6.298.
Cataplexy occurs in:
A) grand mal epilepsy
B) Kleine-Levin's syndrome
C) narcolepsy
D) psychomotor epilepsy
E) "status epilepticus"
NEU-6.299.
Which diseases are accompanied by paralysis of the external ocular
muscles?
A) myasthenia gravis

B) multiple sclerosis
C) Tolosa-Hunt's syndrome
D) Wernicke's encephalopathy
E) migraine headache
F) ischemic injury of the brain stem
G) all of the above
NEU-6.300.
The therapy of choice in trigeminal neuralgia is:
A) clonezepam (Rivotril)
B) alcohol infiltration
C) carbamazepine
D) retroganglionic neurotomy
E) none of the above
NEU-6.301.
Occlusion of the anterior cerebral artery has the following symptoms:
A) contralateral homonymous hemianopsia
B) contralateral hemihypesthesia
C) Gerstmann's syndrome
D) ipsilateral paresis and sensory disturbances of the lower
extremity.
E) contralateral paresis and sensory disturbances of the lower
extremity
NEU-6.302.
Atrophy of the optic nerve can be caused by:
A) glioma of the optic nerve
B) severe congestion of the fundus
C) Kennedy-Gowers's syndrome
D) optochiasmatic arachnoiditis
E) all of the above
F) none of the above
NEU-6.303.
Atrophy of the optic nerve can occur:
A) as a sequel of glaucoma
B) after rupture of an intracranial aneurysm
C) after papillitis
D) after cerebral commotion
E) none of the above
NEU-6.304.
Which of the following can occur after an ipsilateral occlusion of the
internal carotid artery?
A) no neurologic signs
B) ipsilateral amaurosis and contralateral hemiparesis
C) contralateral hemiparesis and hemihypesthesia
D) contralateral hemiparesis, hemihypesthesia, and homonymous
hemianopsia
E) all of the above
NEU-6.305.
A sudden visual disorder can occur in:
A) temporal arteritis
B) disease of the ipsilateral internal carotid artery

C) ipsilateral embolism of the central retinal artery


D) migraine headaches
E) all of the above
NEU-6.306.
Which of the following is typical of the retrobulbar NEUritis occurring
in multiple sclerosis?
A) it is usually unilateral
B) marked visual disorders
C) later it is accompanied by temporal pallor
D) all of the above
E) none of the above
NEU-6.307.
Embolization of the cerebral vessels occurs:
A) as a sequel of mitral valve prolapse
B) as a seqel of atrial fibrillation
C) if a lateral thrombus develops
D) as a sequel of subacute bacterial endocarditis
E) after cardiac surgery
F) all of the above
NEU-6.308.
Which of thefollowing should be considered in the differential diagnosis
of papilledema?
A) pseudoedema of the papilla
B) papillitis
C) thrombosis of the central vein
D) all of the above
E) none of the above
NEU-6.309.
Which of the following diseases is accompanied by papilledema?
A) Gullain-Barr syndrome
B) lung emphysema
C) anemia
D) hypoparathyroidism in children
E) hypervitaminosis -A
F) all of the above
NEU-6.310.
The most common cause of cerebral vascular thrombosis is:
A) hypertension
B) arteriosclerosis
C) diabetes
D) syphilis
E) collagen disease
NEU-6.311.
When do the symptoms of parainfectional encephalomyelitis develop?
A) 2 weeks before the appearance of exanthemas
B) 1 week before the appearance of exanthemas
C) simultaneously with the exanthemas
D) 2 weeks after the appearance of exanthemas
E) none of the above

NEU-6.312.
Which of the following can accompany infectious mononucleosis?
A) generalized polyneuritis
B) facial paralysis
C) encephalitis
D) a lesion of the oculomotor nerve
E) all of the above
NEU-6.313.
Which of the following is typical of rabies?
A) it develops following different incubation periods
B) it develops within a short time period following a bite from an
infected animal
C) bites of different animals can elicit the disease
D) it causes difficulties in swallowing
E) all of the above
NEU-6.314.
A brain abscess:
A) is a sequel of bronchiectasis
B) is a sequel of frontal sinusitis
C) is similar to other intracranial space occupying processes as it
causes neurologic symptoms
D) is fatal if left untreated
E) all of the above
NEU-6.315.
A spinal epidural abscess:
A) usually affects the mid-thoracic area
B) is caused by Staphylococcus aureus
C) is a granulomatous process caused by the low-virulent microorganisms
D) is fatal if left untreated
E) all of the above
NEU-6.316.
The primary site of the infection in pneumococcus meningitis is,either unknown, or is localized in the:
A) upper airways
B) lung
C) ear
D) all of the above
E) none of the above
NEU-6.317.
If meningitis symptoms are accompanied by petechias on the skin,
the most probable pathogenic microorganism is:
A) Staphylococcus
B) Streptococcus
C) Pneumococcus
D) Meningococcus
E) Hemophilus influenzae
NEU-6.318.
Nneurologic symptoms accompanying cardiac diseases are:
A) thrombosis and embolism
B) abscess and aneurysm
C) seizures and syncope

D) all of the above


NEU-6.319.
Which type of seizure does not occur following cranial injury.?
A) classic grand mal
B) classic petit mal
C) psychomotor seizures
D) focal motor seizures
E) Jacksonian seizure
NEU-6.320.
Which symptoms occur before an aneurysm rupture?
A) symptoms of a lesion of the oculomotor nerve
B) a defect in the visual field
C) ipsilateral periodic headache
D) ipsilateral facial pain
E) all of the above
NEU-6.321.
Hemiplegia developing after a subarachnoid hemorrhage can be
caused by:
A) softening due to vascular spasm
B) an intracerebral hematoma
C) both of the above
D) none of the above
NEU-6.322.
Case Study:
During physical work a 50-year-old male patient had a sudden
headache, neck rigidity, seizures, paralysis of the 3rd cranial nerve
and a rapidly developing coma The most probable diagnosis is:
A) bacterial meningitis spontaneous subarachnoid hemorrhage due to an aneurysm
C) rupture of a brain abscess
D) glioma hemorrhage
E) brain metastasis
NEU-6.323.
Case Study:
A male patient develops a sudden paraplegia and dies of pneumonia
several weeks later. Post mortem a thrombosis of the anterior spinal
artery was found. Which neurologic function was least involved?
A) the function of the corticospinal tract
B) the function of the ventral root cells
C) the function of the dorsal root cells
D) the function of the spinothalamic tract
E) the function of the spinocerebellar tract
NEU-6.324.
Which process is usually followed by hematomyelitis?
A) a blood dyscrasia
B) congenital aneurysm
C) trauma
D) a demyelinizating process
E) all of the above

NEU-6.325.
Case Study:
A 55-year-old male patient has frequent syncopes, urine incontinence
and impotence. The clinical examination revealed orthostatic
hypotension, corticospinal, extrapyramidal and cerebellar symptoms.
The most probable diagnosis is:
A) multiple sclerosis
B) postencephalitic parkinsonism
C) Shy-Drager's syndrome
D) aneurysm of the basilar artery
E) L-Dopa intoxication
NEU-6.326.
Which pathological process is usually accompanied by an epidural
hematoma?
A) brain contusion
B) rupture of the medial meningeal artery
C) rupture of the lateral sinus
D) rupture of the pontine veins
E) none of the above
NEU-6.327.
If a head injury is followed by unconsciousness. Which pathological
process is assumed?
A) brain contusion
B) brain commotion
C) an epidural hematoma
D) a subdural hematoma
E) abrain abscess
NEU-6.328
The most common neurologic symptoms in amyotrophic lateral
sclerosis are:
A) organic mental syndromes
B) urological symptoms
C) aphasia; agnosia
D) spinal muscle atrophy, spasticity, pyramidal symptoms
E) visual field disturbances
NEU-6.329.
Case Study:
A 10-year-old girl with normal gait complains of proximal muscle weakness.
This, as well as atrophy of the proximal muscles of the lower extremities
has been verified during the examination. A weak patellar reflex
and pseudohyperlrophy of the leg muscles are also observed. Muscle
biopsy indicated neurogenic atrophy. The most probable diagnosis is:
A) myasthenia gravis
B) hereditary amyotrophic lateral sclerosis
C) Wolfhart-Kugelberg-Welander's disease
D) Duchenne's dystrophy
E) polymyositis
NEU-6.330.
The most common sequel of a cerebrospinal fluid fistula is:
A) headache
B) meningitis or brain abscess

C) a decrease of the CSF pressure


D) cortical atrophy
E) development of a hygroma
NEU-6.331.
The diagnosis of syringomyelia is primarily based on:
A) myelography
B) the CSF findings
C) localized muscle atrophy, sensory disorders, and impaired
development of the bones
D) an EMG
E) any remissions in the course of the disease
NEU-6.332.
Which of the following is typical of Charcot-Marie-Tooth disease?
A) proximal muscle atrophy
B) trisomy of chromosome 21
C) an unchanged nerve conductance velocity
D) a nerve biopsy is of no value
E) distal muscle atrophy, leg deformities
NEU-6.333.
Which test is of no (less) value in the diagnosis of progressive muscle
dystrophy?
A) a muscle biopsy
B) the serum creatinine phosphokinase level
C) the EMG
D) the muscle histochemistry
E) the EEG
NEU-6.334.
How can myotonic muscle dystrophy be differentiated from
Duchenne's dystrophy?
A) CSF protein content
B) EEG
C) EMG
D) X-rav of the skull
NEU-6.335.
A less typical symptom of myotonic dystrophy is:
A) pseudohypertrophy
B) cataract
C) baldness
D) testis atrophy
E) mental disorders
F) atrophy of the facial muscles
NEU-6.336.
One of the following is not characteristic of Parkinson's syndrome:
A) degeneration of the substantia nigra
B) dopamine deficiency
C) cholinergic substances provoke parkinsonism
D) decreased muscle tone
E) mental symptoms

NEU-6.337.
Which of the following structures is the most frequently involved in
periarteritis nodosa?
A) the cerebellum
B) the peripheral nerves
C) the brain cortex
D) the basal ganglia
E) the spinal cord
NEU-6.338.
One of the following is not typical of temporal arteritis:
A) steroid therapyis initiated only after a diagnostic biopsy
B) only 50-year-olds and over are affected
C) it is one of the forms of Giant-cell arteritis
D) a headache and an elevated RBC sedimentation rate
NEU-6.339.
Which of the following is characteristic of polymyositis?
A) a normal muscle biopsy
B) dementia
C) pain in the proximal muscles and joints and possibly fever
D) a hereditary background
E) normal values of RBC sedimentation rates and creatine kinase
levels
NEU-6.340.
What is the most probable diagnosis of the following: polyneuropathy,
bilateral abducent paralysis, ataxia, alcoholism, and internuclear
ophthalmoplegia:
A) arsenic intoxication
B) lead intoxication
C) Wernicke's encephalopathy
D) diabetes
E) hyperthyroidism
NEU-6.341.
Which of the following is typical of an alcoholic polyneuropathy?
A) a piercing pain is always present
B) sensory disorders in-the lower extremities
C) absence of autonomic symptoms
D) normal conducting velocity in the peroneal nerve
E) hyperthermia
NEU-6.342.
The rarest symptom in multiple sclerosis is:
A) impaired visual acuity
B) ataxia
C) vertigo
D) paresthesia
E) weakness
F) seizures
G) urinary disorders
NEU-6.343.
The most common CSF finding in multiple sclerosis is:
A) increased pressure

B) a cell count of over 100/mm3


C) an elevated gamma-globulin level
D) a decreased protein level
E) a decreased glucose level
NEU-6.344.
One of the following is less typical of multiple sclerosis:
A) spastic paraparalysis
B) internuclear ophthalmoplegia
C) nystagmus
D) a concomitantpregnancy should be interrupted
E) retrobulbar neuritis
NEU-6.345.
One of the following is not characteristic of migraine:
A) it is a unilateral headache in more than half of the cases
B) it is usually accompanied by autonomic symptoms
C) the EEG is always normal
D) the symptoms of ophthalmoplegic migraine last for several days
E) aspirin usually helps
NEU-6.346.
Which of the following is not true for epilepsy?
A) the diagnosis of epilepsy does not solely depends on the EEG
B) the CT or MRI can be helpful if focal EEG alterations are observed
C) after cessation of a seizure the medication used for its therapy
can be discontinued immediately
D) pathogenic causes can be ruled out in symptomatic epilepsy
E) the dose of the effective drug should be continuously increased
NEU-6.347.
Where do 75% of pediatric tumors develop?
A) in the temporal lobe
B) in the cerebellum
C) in the frontal lobe
D) in the optic nerve
E) in the parietal lobe
NEU-6.348.
Case Study:
A 45-year-old male patient is being examined for epilepsy. A mild left
facial and left-sided hemiparesis were found. A circumscribed bone
thinning and bone spicules on the right side were also visualized on
the X-ray. What is the most probable diagnosis?
A) occlusion of the right carotid artery
B) medulloblastoma
C) cerebellar astrocytoma
D) withdrawal symptoms
E) meningioma
NEU-6.349.
The most frequent cause of retrobulbar neuritis is:
A) a paranasal sinus infection
B) temporal arteritis
C) multiple sclerosis
D) aneurysm

E) alcohol; smoking
NEU-6.350.
Case Study:
A 30-year-old woman complains of diplopia, disturbances of speech
and swallowing. The symptoms show daily variations. Which of the
following tests is the most relevant?
A) an oral glucose tolerance test
B) the EEG
C) an Edrophonium (Tensilon) test
D) an oligoclonal gamma test
E) the VEP
NEU-6.351.
Select one disease which is not always accompanied by dementia:
A) Alzheimer's disease
B) Huntington's chorea
C) Marchiafava-Bignami's disease
D) Parkinson's disease
E) Pick's syndrome
NEU-6.352.
Case Study:
After sustaining a skull injury a 50-year-old male patient complains
of progressive pulsating exophthalmus and diplopia What could
have happened?
A) a subtemporal hematoma
B) an epidural hematoma
C) a carotid-cavernous fistula
D) an intracerebral hematoma
E) an aneurysm of the basilar artery
NEU-6.353.
Select a lumbar CSF finding typically occurring in viral meningitis:
A) 2 mononuclear cells/mm3, protein 20 mg%, glucose 20 mg%
B) 2 mononuclear cells/mm3, protein 50 mg%, glucose 20 mg%
C) 20 mononuclear cells/mm3, protein 100 mg%, glucose 20 mg%
D) 200 mononuclear cells/mm3, protein 100 mg%, glucose 20 mg%
E) 100-1000 mononuclear cells/mm3, protein 60 mg%, glucose 50 mg%
NEU-6.354.
A suddenly developing nystagmus, vertigo, Horner's syndrome,
ataxia without paralysis, and alternating sensory disorder suggest
occlusion of which artery?
A) anterior chorioidal artery
B) posterior chorioidal artery
C) basilar artery
D) inferior posterior cerebellar artery
E) superior cerebellar artery
NEU-6.355.
Case Study:
An 80-year-old alcoholic male patient complains of a headache
probably due to skull injury. He has had left-sided hemiparesis for a
month. The right carotidangiography showed detachment of the
cerebral vessels from the internal surface of the skull. The most

probable diagnosis is:


A) cerebral contusion
B) right-sided glioblastoma of the frontal lobe
C) right-sided subdural hematoma
D) right-sided epidural hematoma
E) occlusion of the right middle cerebral artery
NEU-6.356.
Which visual disturbance is typical of a right-sided temporal focus?
A) hemianopsia of the left superior temporal quadrant
B) hemianopsia of the left inferior temporal quadrant
C) hemianopsia of the left superior nasal quadrant
D) hemianopsia of the left inferior nasal quadrant
E) none of the above
NEU-6.357.
One of the following is not characteristic of the cauda syndrome:
A) it is commonly caused by traumatic alterations in Ll-2
B) it can impair all sensory modalities
C) sphincter paralysis
D) no pyramidal symptoms
E) spastic paralysis
NEU-6.358.
Select the least probable cause of motor disturbances of cranial
nerves X, XI, XII
A) syringobulbia
B) amyotrophic lateral sclerosis
C) vascular disease of the brain stem
D) acute anterior poliomyelitis
E) acoustic neuroma
NEU-6.359.
Exophthalmic ophthalmoplegia can occur in:
A) hyperthyroidism
B) euthyroid state after a thyroidectomy
C) euthyroid patients
D) all of the above
E) none of the above
NEU-6.360.
In thyrotoxic myopathy:
A) other signs off' thyrotoxicosis also occur
B) weakness and atrophy of the iliac region develop
C) there are normal or hyperreflexes
D) there are no EMG signs of fibrillation
E) all of the above
NEU-6.361.
The least valuable diagnostic finding in the diagnosis of combined
subacute spinal degeneration is:
A) the mean corpuscular volume (MCV)
B) a gastric examination
C) a CSF examination
D) Schilling's test
E) a bone marrow examination

NEU-6.362.
Case Study:
A 40-year-old male patient develops a sudden vertigo, vomiting, and
imbalance. Which disease is the least probable?
A) Menire's syndrome
B) acute labyrinthitis
C) acoustic neuroma
D) multiple sclerosis
E) vascular disease of the brain stem
NEU-6.363.
At what age does syringomyelia usually develop?
A) in infants
B) between 10-20 years-old
C) between 30-40 years-old
D) between 50-60 years-old
NEU-6.364.
The neurologic complications of myxedema are:
A) headache and paralysis of the cranial nerves
B) neuropathy and myopathy
C) psychosis and coma
D) all of the above
NEU-6.365.
The least valuable laboratory finding in myxedema is:
A) the concentration of iodine bound to serum protein
B) the radio-iodine uptake
C) the CSF protein findings
D) the EEG
E) a chest X-ray
NEU-6.366.
In which disease is facial nerve paralysis the least characteristic?
A) Guillain-Barr's syndrome
B) uveoparotid fever
C) mumps
D) meningococcus meningitis
E) leprosy
NEU-6.367.
Dystonic movements occur in:
A) tumors of the basal ganglia
B) Wilson's disease
C) postencephalitis parkinsonism
D) deforming muscular dystonia
E) all of the above
NEU-6.368.
Which part of the nervous system is the least affected in multiple
sclerosis?
A) the spinal cord
B) the cerebral cortex
C) the cerebellar tracts
D) the brain stem

E) the visual tract


NEU-6.369.
Case Study:
A week after consuming pork meat 10 patients had diarrhea, eyelid
edema, and muscle weakness. Several days later some of these
patients were confused, while others developed focal neurologic
symptoms. What is suspected?
A) schistosomiasis
B) echinococcosis
C) trichinosis
D) cysticercosis
E) toxoplasmosis
NEU-6.370.
Sudden pain, dysuria, ataxia, pupil alterations, areflexia, and proprioceptive
disorders suggest one of the following diseases:
A) mixed systemic disease
B) tabes dorsalis
C) multiple sclerosis
D) diabetic pseudotabes
E) lumbar syringomyelia
NEU-6.371.
In optic atrophy which of the following alterations are accompanied
by a bitemporal visual field defect?
A) parachiasmallesion
B) retinal vascular disease
C) chiasmatic lesion
D) tumor of the optic nerve
E) none of the above
NEU-6.372.
The most common intracranial tumor is:
A) meningoma
B) acoustic neuroma
C) glioma
D) pituitary adenoma
E) angioma
F) metastasis
NEU-6.373.
Pain, cramps, weakness, and parasthesia in the lower extremity of a
patient with aortic coarctation suggest one of the following:
A) arterial insufficiency of the muscles
B) spinal arterial insufficiency
C) dilation of the collateral arteries thereby compressing the spine
D) occlusion of the anterior spinal artery
E) all of the above
NEU-6.374.
Which disease does not accompany alcoholism?
A) central pontine myelinolysis
B) Pick's psychosis
C) primary degeneration of the corpus callosum
D) Wernicke's syndrome

E) Korsakoffs psychosis
F) parenchymal cerebellar degeneration
NEU-6.375.
Which of the following diseases shows the greatest coincidence with
myasthenia gravis?
A) thymus tumor
B) thymus hyperplasia
C) hyperthyroidism
D) hypothyroidism
E) all of the above
NEU-6.376.
Select the main anastomoses of the cerebral circulation:
A) The circle of Willis
B) supracortical anastomoses of the main cerebral arteries
C) anastomosis between internal and external carotid arteries
D) all of the above
E) none of the above
NEU-6.377.
Internuclear ophthalmoplegia is a symptom of.
A) multiple sclerosis
B) vascular disease of the brain stem
C) brain stem tumor
D) all of the above
E) none of the above
NEU-6.378.
Continous therapy with phenytoin can cause:
A) peripheral neuropathy
B) visual impairment, ataxia
C) gingival hyperplasia
D) all of the above
E) none of the above
NEU-6.379.
A side-effect of vincristine therapy is:
A) cortex atrophy
B) myelopathy
C) polyneuropathy
D) all of the above
E) none of the above
NEU-6.380.
Case Study:
A 45-year-old female patient developed subacute progressive dementia,
pyramidal and extrapyramidal symptoms and myoclonic
convulsions. The EEG revealed periodic anomalies. Which disease is
suspected?
A) Alzheimer's disease
B) Creutzfeldt-Jakob's disease
C) Huntington's chorea
D) Kuru
E) none of the above

NEU-6.381.
The cause of alcoholic myopathy is:
A) vitamin-E deficiency
B) toxic alterations in mitochondria and muscle membranes
C) inhibition of glycolytic enzymes
D) chronic ischemia of the muscles
E) all of the above
NEU-6.382.
Coma frequently occurs in:
A) thrombosis
B) embolism
C) hemorrhage
NEU-6.383.
Headache frequently occurs in:
A) thrombosis
B) embolism
C) hemorrhage
NEU-6.384.
Blood pressure is usually normal in:
A) thrombosis
B) embolism
C) hemorrhage
NEU-6.385.
Atrial fibrillation frequently occurs in:
A) thrombosis
B) embolism
C) hemorrhage
NEU-6.386.
Anisocoria is frequently observed in:
A) thrombosis
B) embolism
C) hemorrhage
NEU-6.387.
Case Study:
A 54-year-old hypertensive male patient suddenly develops vertigo,
headache, and vomiting. At examination the patient is conscious
and has unilateral ataxia without weakness. Conjugated eye movements
are disturbed. What is suspected?
A) pontine hemorrhage
B) capsule hemorrhage
C) cerebellar hemorrhage
D) subarachnoid hemorrhage
E) aneurysm of the basilar artery
NEU-6.388.
Which symptom is not typical of tabes dorsalis?
A) Argyll-Robertson pupils
B) ataxia
C) hypertonic muscles
D) lack of vibration sensation

E) root pain
NEU-6.389.
The typical course of herpes zoster is:
A) pain-blisters-pigmentation
B) blisters-pigmentation-pain
C) pain-pigmentation-blisters
D) pigmentation-blisters-pain
E) blisters-pain-pigmentation
NEU-6.390.
Herpes zoster usually affects the:
A) geniculate ganglion
B) maxillary branch of the trigeminal nerve
C) neck region
D) chest region
E) lumbar region
NEU-6.391.
The most common cause of acute meningitis in adults is:
A) PNEUmococcus
B) Streptococcus
C) Hemophilus influenza
D) Meningococcus
E) none of the above
NEU-6.392.
Cases of meningitis:
A) rarely occur at present
B) are due to only a few pathogenic microorganisms
C) are usually localized in the subarachnoid space
D) are rarely due to Listeria infections in newborns
E) none of the above
NEU-6.393.
The onset of bacterial meningitis can be best diagnosed by the
following:
A) the general symptoms of the patient
B) a murmur over the skull
C) central scotoma
D) bitemporal hemianopsia
E) fasciculations
NEU-6.394.
The early symptoms of bacterial meningitis include:
A) occipital stiffness and hypertension
B) Kernig and Brudzinsky's signs
C) photophobia and papilledema
D) bradycardia
E) hypothermia
NEU-6.395.
The CSF in case of meningococcal meningitis:
A) is clear and colorless
B) has a normal protein content
C) has a normal pressure

D) has a normal cell count


E) none of the above
NEU-6.396.
Influenza-meningitis differs from other forms of meningitis as:
A) it occurs predominantly in infants and children
B) pathogenic microorganism can scarcely be cultured
C) it has no nervous system sequels
D) it has different CSF changes
E) it has a different clinical picture
NEU-6.397.
The most common site of a spinal epidural abscess is:
A) the cervical region
B) the thoracic region
C) the lumbar region
D) the sacral region
E) all of the above
NEU-6.398.
The postmortem findings in bacterial meningitis:
A) edema with temporal and cerebellar impaction; an altered
volume of subarachnoid exudate
B) parenchymal inflammation is frequently observed
C) in untreated cases the roots of the cauda equina are damaged
D) in untreated cases myelitis usually occurs
E) none of the above
NEU-6.399.
Acute porphyria:
A) frequently causes abdominal pain and gastric dilatation
B) opaque urine is a frequent finding
C) is systematically accompanied by vast necrotic lesions of the
central nervous system
D) frequently causes seizures
E) elicits weakness with typical proximal localization
NEU-6.400.
Which of the following does not comprise the neurologic damages
occurring in hypoparathyroidism?
A) calcification of the basal ganglia
B) a congested papilla
C) 2-5 Hz burst waves on the EEG
D) an elevated CSF protein level
E) a decreased nerve excitation threshold
NEU-6.401.
In boys with precocious puberty there is:
A) hyperplasia of the adrenal cortex
B) a hypothalamic hamartoma
C) a normal hypothalamus
D) decreased excretion of 17-ketosteroids
E) an occurance as frequent as feminization caused by estrogenproducing
tumors.

NEU-6.402.
The nervous system in Hodgkin's disease:
A) the brain is usually involved
B) the spine is usually involved
C) brain involvement is more common than spinal involvement
D) the meninges are usually involved
E) the cauda equina is usually involved
NEU-6.403.
Adrenal cortical tumors can cause:
A) recurrent fits of weakness resembling familial periodic paralysis
B) hypertension with hypokalemia, hyponatremia and alkalosis
C) hypertension with hyponatremia but without hypokalemia
D) hyperetension with hypokalemia and acidosis
E) none of the above
NEU-6.404.
Clinically platybasia does not mimic:
A) multiple sclerosis
B) syringomyelia
C) a tumor of the scala tympani
D) Arnold-Chiari's malformation
E) none of the above
NEU-6.405.
Spina bifida means a congenital anomaly of which of the following
structures?
A) the spine and brain stem
B) the cerebellum, but not of the cerebrum
C) the cerebrum but not of the cerebellum
D) the spine but not of the brain stem
E) the brain stem but not of the spine
NEU-6.406.
The typical onset of syringomyelia is:
A) a focal atrophy of the upper extremities, weakness and a lack
of pain sensation to burning
B) pain in the upper extremity
C) sphincter disorders
D) fasciculations
E) none of the above
NEU-6.407.
Amyotrophic lateral sclerosis:
A) usually starts in the proximal muscles
B) usually causes dysphagia and dysarthria
C) rarely causes diffuse hyperreflexia
D) causes fasciculation, the intensity of which correlates with the
severity of the disease
E) none of the above
NEU-6.408.
Familial periodic paralysis:
A) is treated by electrolyte replacement therapy
B) causes spastic paralysis
C) the cranial nerves are usually involved

D) commonly causes frequent micturition or incontinence


E) none of the above
NEU-6.409.
Eaton-Lambert syndrome:
A) the amplitude of the action potential elicited in a muscle by
electric stimulation is usually markedly enhanced
B) repetitive stimulation decreases the action potential
C) accompanies oat-cell carcinoma and other tumors and diseases
sometimes without any visible cause
D) the defect is not related to acetylcholine release
E) there is a cranial muscle weakness
NEU-6.410.
Therapy of patients with dementia:
A) oxygen therapy is useful in Alzheimer's disease
B) hyperbaric oxygen therapy is useful in Alzheimer's disease
C) the oxygen consumption of the brain decreases in Alzheimer's
disease
D) catastrophic reactions are not a serious problem in this disease
E) none of the above
NEU-6.411.
Drug-induced parkinsonism:
A) develops primarily in adults, and occurs with akinesia, rigidity,
and tremor
B) a similar clinical picture develops in children but dyskinetic
movements are less pronounced
C) anticholinergic drugs with central action are usually ineffective
D) akathisia occurs rather rarely
E) in some cases symptoms persist for years after. discontinuation
of therapy
NEU-6.412.
Chorea classification does not depend on:
A) the age of onset
B) any other intercurrent diseases
C) the presence of other neurologic disorders
D) the familial background
E) none of the above
NEU-6.413.
The clinical picture of athetosis:
A) muscles innervated by the cranial nerves are rarely Involved
B) an abnormal position of the extremities and intellectual damage
C) hypotonia is nearly always present, weakness, however, is not
typical
D) normal reflexes are present
E) the proximal extremities are mainly involved
NEU-6.414.
Collagen disease:
A) affects the central nervous system, but the muscles are not involved
B) affects the peripheral nervous system
C) affects the the muscles but the central nervous system is not
involved

D) is becoming less common


E) does not affect the spine
NEU-6.415.
In periarteritis nodosa:
A) hemorrhage and bleeding can occur
B) multiple neuropathy is a rare outcome
C) the autonomic nervous system is not involved
D) keratitis and deafness can occur if the "Wasserman's reaction"
of the CSF is positive
E) differentiation from the multiple sclerosis is based on the lack
of remissions
NEU-6.416.
The recommended therapy of "paralysis agitans" (a form of parkinsonism)
is:
A) L-dopa therapy combined with a peripheral decarboxylase inhibitor
B) surgical treatment in elderly patients with bilateral disease
C) trihexyphenidyl, 20-25 mg/day in split doses
D) atropine and prochlorperazine
E) diphenhydramine, 50 mg daily
NEU-6.417.
Alzheimer's and Pick's disease:
A) are degenerative diseases which lead to dementia in puberty,
middle aged and elderly patients
B) can be differentiated on the basis of a circumscribed frontotemporal
atrophy in occurring Alzheimer's disease and not
occurring in Pick's disease
C) can be differentiated on the basis of the following: their onset,
progression, duration, and clinical symptoms
D) the incidence of both diseases is similar
E) none of the above
NEU-6.418.
In multiple sclerosis:
A) seizures occur in about 20% of the cases
B) visual damage is usually unilateral
C) headache and aphasia are not rare
D) a neurogenic atrophy is not rare
E) internuclear ophthalmoplegia, impaired hearing and tinnitus
are common
NEU-6.419.
Valproic acid in epilepsy:
A) is poorly absorbed from the gastrointestinal tract
B) might affect the GABA metabolism
C) if administered together with phenobarbital, decreases the
serum level of the latter drug
D) can cause severe side-effects which necessitates the discontinuation
of drug therapy
NEU-6.420.
Syncope can occur:
A) after defecation, urination or cough
B) after compression of the carotid sinus

C) after sudden cold exposure


D) the (B) and (C) answers are true, but (A) is false
E) the (A) and (B) answers are true, but (C) is false
NEU-6.421.
Which of the following does not occur in migraine?
A) transient dysphasia
B) paresthesia
C) visual impairement without headache
D) the (A) and (C) answers are true, but (B) is false
E) none of the above
NEU-6.422.
Which of the following is typical of a motoneuron disease?
A) fasciculation
B) urinary incontinence
C) early disappearance of the abdominal wall reflex
D) nystagmus
E) intellectual deterioration
NEU-6.423.
The clinical symptoms of Guillain-Barr's syndrome (infectious
polyneuritis) include:
A) pathological alterations affecting the distal parts of the peripheral
nerves
B) incontinene develops
C) the cranial nerves are not involved
D) ACTH therapy is beneficial
E) hyperpyrexia
NEU-6.424.
In multiple sclerosis, the cerebrospinal fluid findings include:
A) a positive serology for syphilis
B) electrophoresis shows elevated levels of oligoclonal
immunoglobulins
C) a total protein count in the CSF of 1.2 g/1
D) an increased WBC (especially NEUtrophil granulocytes) count
E) xanthochromia
NEU-6.425.
Which of the following complications of spinal anesthesia are due to
a direct effect of the local anesthetic on the nerve?
A) chronic progressive adhesive arachnoiditis
B) headaches
C) the cauda equina syndrome
D) all of the above
NEU-6.426.
Congenital sacular (berry) aneurysm:
A) such unruptured aneurysms occur in 2% of postmortem
findings
B) its wall is comprised only of intima and adventitia
C) both of the above
D) none of the above
NEU-6.427.

In purulent meningitis:
A) the cortex is massively involved
B) the subarachnoid arteries are frequently occluded
C) the cortex is only slightly involved
D) the (A) and (B) answers are onlytrue
NEU-6.428.
Post-traumatic epilepsy:
A) rarely develops following car accidents
B) usually develops 6-15 months following injury
C) usually develops 1 week after injury
D) none of the above
NEU-6.429.
Case Study:
A hemorrhagic CSF which rapidly cleared up was obtained after a lumbar
puncture. The first portion of the CSF contained 2000 RBC/mm3. The
RBC content of the third portion was only 10 RBC/ mm3. The most probable
diagnosis is:
A) cerebral embolization
B) rupture of an intracranial aneurysm
C) subarachnoid hemorrhage
D) artificial CSF hemorrhage
E) cerebral thrombosis
NEU-6.430.
Case Study:
A lumbar puncture revealed an opalescent CSF with increased
pressure. The most probable diagnosis is:
A) the patient is healthy but at strain
B) a brain tumor
C) the alterations are due to trauma caused by the puncture
D) subarachnoid hemorrhage
E) tuberculous meningitis
NEU-6.431.
Case Study:
An infant has lost his appetite, vomits, has seizures and fever. What
should be done?
A) a cultivation of nasal and pharyngeal discharge
B) try to find emotional problems in the family
C) obtaining a urine sample with a catheter
D) a lumbar puncture
E) intravenous pyelography
NEU-6.432.
Case Study:
A 7-year-old child complains of a sore throat, joint pain, and headache.
He has fever and occipital stiffness. What should be done?
A) an ECG
B) an X-ray of the wrists and knees
C) a lumbar puncture
D) a cultivation of nasal and pharyngeal discharge
E) the RBC sedimentation rate

NEU-6.433.
After smallpox vacination any postvaccination encephalomyelitis
ocurs within:
A) 30-40 days
B) 20-25 days
C) 10-12 days
D) 4-6 days
E) 2-3 days
NEU-6.434.
One of the following occurs in a subdural hematoma.
A) an increased intracranial pressure
B) a xanthochromic CSF
C) an elevated CSF protein level
D) all of the above
E) none of the above
NEU-6.435.
Which of the following rarely occurs in infants in case of a subdural
hematoma?
A) papilledema
B) fever
C) protrusion of the fontanelle
D) a hemorrhage in the retina
E) hemiparesis
NEU-6.436.
When do the symptoms of an epidural hemorrhage in children
develop?
A) weeks after trauma
B) days after trauma
C) seconds after trauma
D) hours after trauma
E) minutes after trauma
NEU-6.437.
Common symptoms of an epidural hemorrhage in a child include:
A) ataxia and hemiparesis
B) papilledema and stupor
C) hyperreflexia and contralateral paralysis of the abducent nerve
D) retinal hemorrhage and coma
E) hemiparesis and contralateral pupil dilatation
NEU-6.438.
The most common symptom of increased intracranial pressure in
children is:
A) ataxia
B) diplopia
C) stupor
D) vomiting
E) impaired visual acuity
NEU-6.439.
A common cause of intracranial hemorrhage in children is:
A) hepatic disease
B) a blood dyscrasia

C) an intracranial aneurysm
D) glomerulonephritis
E) trauma
NEU-6.440.
Case Study:
3 days after a tibial fracture the child became confused, had fever, blood
in the sputum, and developed hemiparesis. The most probable diagnosis is:
A) cerebral fat embolism
B) subdural hematoma
C) metastatic cerebral abscess
D) cortical contusion
E) traumatic cerebral thrombosis
NEU-6.441.
Case Study:
2 weeks after a penetrating hand injury a child developed abdominal
muscle rigidity, trismus, risus sardonicus, and opisthotonus. Diagnosis:
A) meningitis
B) hysteria
C) drug allergy
D) intercurrent peritonsillar abscess
E) tetanus
NEU-6.442.
The CSF in acute purulent meningitis:
A) clear; colorless; and has normal pressure
B) increased pressure; clear
C) increased pressure; opaque
D) increased pressure; hemorrhagic
E) normal pressure; clear; xanthochromic
NEU-6.443.
Which of the following parameters are identical in a 3-month-old
infant and in an adult?
A) the nerve conduction velocity
B) the EEG findings
C) the cerebrospinal fluid findings
D) all of the above
E) none of the above
NEU-6.444.
If a child has recurrent otitis the following is assumed:
A) mastoiditis
B) deficient alimentation
C) a dermal sinus duct
D) all of the above
E) none of the above
NEU-6.445.
Examination of the following is important in the diagnosis of viral
meningoencephalitis:
A) the blood and CSF
B) any throat discharge and the saliva
C) the feces
D) the serum in the acute and convalescent period

E) all of the above


NEU-6.446.
A postinfection encephalomyelitis can be due to the following:
A) measles
B) varicella
C) mumps
D) rubella
E) all of the above
NEU-6.447.
Which of the following is frequently observed in children after
head injuries?
A) vertigo
B) seizures
C) headaches
D) all of the above
E) none of the above
NEU-6.448.
Which of the following occur in a subdural hematoma in infants?
A) lethargy and stupor
B) vomiting and growth retardation
C) irritability and seizures
D) all of the above
E) none of the above
NEU-6.449.
The first symptoms of a brain tumor in children are:
A) seizures and coma
B) behavior disturbances
C) headaches and vomiting
D) hemiparesis and hyperreflexia
E) none of the above
NEU-6.450.
Following an intramuscular administration of penicillin, neuropathy
of the sciatic nerve developed. What is suggested?
A) a toxic effect of penicillin on the nerve tissue
B) an allergic reaction
C) direct damage of the sciatic nerve
D) all of the above
E) none of the above
NEU-6.451.
An X-ray study is a valuable tool in the diagnosis of lead intoxication
because:
A) it reveals lead lines along the long bones
B) it reveals suture separation
C) an abdominal image shows lead incorporation
D) all of the above
E) none of the above
NEU-6.452.
A Rubeola infection during pregnancy can cause:
A) microcephalia and mental retardation

B) cataract and microphthalmus


C) congenital malformations and deafness
D) all of the above
E) none of the above
NEU-6.453.
A lucid period following a head injury indicates a:
A) subdural hematoma
B) cerebral laceration
C) cerebral contusion
D) cerebral commotion
E) epidural hematoma
NEU-6.454.
Secondary Parkinsonism suggests one of the following intoxications:
A) manganese
B) phenothiazine
C) carbon monoxide
D) all of the above
E) none of the above
NEU-6.455.
Case Study:
A 31-year-old male patient complains of a bifrontal headache and
impaired visual acuity lasting for about 4 weeks. During the previous 4 months the patient had a mild, intermittant headache. At
present the patient is irritable and difficult to live with. In the previous
months he felt sleepy, sometimes sleeping for 20-30 hours. 8-9
months earlier the patient had an accident, when he had fallen out
from a moving car causing a laceration of the skull skin.
Examination revealed papilledema, a dilated right pupil and leftsided
hemiparesis. The assumed diagnosis is:
A) paralytic dementia
B) a chronic subdural hematoma
C) bromide intoxication
D) a space-occupying process in the brain
NEU-6.456.
Case Study:
A 38-year-old female patient complains of intermitting episodes of
tinnitus, vertigo, and feelings that the auditory passage on the rightside
was closed, which have been persisting for about a year. These
episodes usually last for 3-5 hours. Recently the patient complained
of a hearing impairment on the right side which always got worse
during the fits. Examination performed during one of these fits
revealed right nystagmus while looking to the right and a right-sided
perception hearing impairment. The patient was instable in Romberg's
position. The assumed diagnosis is:
A) petrositis
B) a tumor of the cerebellar-pontine angle
C) Mnire's disease
D) vertebrobasilar insufficiency
NEU-6.457.
Which of the following can be observed in a healthy new-born?
A) patellar clonus and Babinski reflex

B) optikinetic nystagmus
C) suckling and Moro's reflex
D) all of the above
E) none of the above
NEU-6.458.
Patients who have a negative angiogram after a subarachnoid
hemorrhage:
A) have a poor prognosis
B) have a prognosis similar to those with a ruptured aneurysm
C) have a prognosis similar to those with a rupture of an arteriovenous
malformation
D) have a good prognosis
E) will probably have a brain tumor
NEU-6.459.
Arterio-venous malformations:
A) in 20-30% of the cases occur together with intracranial aneurysms
B) can undergo extensive growth without eliciting clinical
deterioration
C) rarely occur without subarachnoid hemorrhages .
D) can be easily revealed with angiography studies and scarcely
visualized with a CT
E) none of the above
NEU-6.460.
Case Study:
A patient has a sudden vertigo, swallowing disturbances and subsequently
vomits. Examination reveals Horner's syndrome, analgesia,
thermanesthesia on one side of his face, and sensory disturbances
on the contralateral side. The most likely cause is:
A) thrombosis of the middle or anterior cerebral artery
B) thrombosis of the inferior posterior cerebellar or vertebral
artery
C) hemorrhage affecting the internal capsule
D) embolization of the posterior cerebral artery
E) occlusion of the carotid artery
NEU-6.461.
Acute paralysis of the left lower extremity with only a mild involvement
of the upper extremity suggests:
A) occlusion of the anterior cerebral artery
B) occlusion of the middle cerebral artery
C) occlusion of the posterior cerebral artery
D) occlusion of the inferior posterior cerebellar artery
E) occlusion of the anterior spinal artery
NEU-6.462.
Occlusion of which artery is accompanied by a typical symptom of
homonymous hemianopsia?
A) lenticulostriatal artery
B) Heubner's artery
C) posterior cerebral artery
D) superior cerebellar artery
E) none of the above

NEU-6.463.
Transient unilateral amblyopia with unilateral motor and sensory
deficits is most characteristic of which of the following?
A) internal carotid artery disease
B) vertebrobasilar artery disease
C) middle cerebral artery disease
D) posterior chorioid artery disease
E) the (A) and (C) answers are true
NEU-6.464.
The most common cause of a subarachnoid hemorrhage of nontraumatic
origin is:
A) aneurysm
B) arterio-venous malformation
C) intracranial tumor
D) blood dyscrasia
E) none of the above
NEU-6.465.
The most important factor in the development of cerebral aneurysms is:
A) trauma
B) congenital anomaly
C) syphilis
D) septic embolism
E) none of the above
NEU-6.466.
The most common site of an intracranial aneurysm in adults is
between:
A) the anterior communicans artery and the internal carotid
artery
B) the basilar artery and the internal carotid artery
C) the middle cerebral artery and the vertebrobasilar artery
D) the posterior cerebral artery and the basilar artery
E) none of the above
NEU-6.467.
An arteriovenous malformation of Galen's vein:
A) can be successfully surgically treated in nearly all cases
B) is rarely accompanied by cardiac insufficiency
C) a subdural hematoma is a rare complication after surgery to
correct the malformation
D) is rarely congenital
E) none of the above
NEU-6.468.
The most common site of an intracerebral hemorrhage is:
A) the lobes of the hemispheres
B) the brain stem
C) the basal ganglia
D) the cerebellum
E) none of the above

NEU-6.469.

A cerebral infarction is typically caused by:


A) polyarteritis nodosa
B) arteriosclerosis
C) lupus erythematosus
D) polycythemia
E) none of the above
NEU-6.470.
A common cause of cerebral embolization in adults is:
A) cardiac disease
B) air embolism
C) tumor thrombus
D) fat embolism due to fracture
E) septic lung disease
NEU-6.471.
Occlusion of the internal carotid artery:
A) if the internal carotid artery is affected, a murmur is auscultated
on the neck; in case of external carotid artery disease
this finding is absent
B) a hypersensitive carotid sinus reflex is nearly always absent
C) transient unilateral amblyopia usually develops; homonymous
hemianopsia occurs in about 50% of the cases
D) a transient ischemic attack lasting for 5-30 min. can occur
E) pathological ophthalmic findings are as common as in
vertebrobasilar disease
NEU-6.472.
Cardiac disturbances accompanying cerebrovascular disease:
A) are rare
B) are unknown
C) sometimes occur
D) are common
E) occur in a ratio of 1:1
NEU-6.473.
The anterior spinal artery thrombosis syndrome:
A) occurs in Hodgkin's disease
B) is accompanied by paralysis and dissociated sensory function
C) is usually a sequel of syphilis or tuberculosis
D) the (A) and (B) answers are true, but (C) is false
E) the (A) and (C) answers are true, but (B) is false
NEU-6.474.
The rupture of an intracranial aneurysm usually does not occur
together with the following:
A) severe headache
B) nausea and vomiting
C) unconsciousness
D) occipital stiffness
E). none of the above
NEU-6.475.
In patients with a recognized intracranial aneurysm the occurrance
of subsequent multiple aneurysms is:
A) 10-25%

B) 100%
C) 50-60%
D) 2-5%
E) none of the above
NEU-6.476.
Raynaud's disease:
A) arterial pulsation is usually absent
B) occurs with a higher incidence in male patients
C) has no neurologic symptoms
D) an ischemic fit cannot be elicited following a sympathectomy
E) none of the above
NEU-6.477.
The cerebral collateral circulation is not comprised of which of the
following?
A) the arteries of the base of the skull: the circle of Willis and its
anterior and posterior communicating arteries
B) the arteries on the surface of the brain: the pial branches
connecting the anterior, middle, and posterior cerebral arteries
C) ophthalmic communications between the internal and external
carotid arteries, and the cervical muscular communicating
arteries connecting the vertebral and the external carotid
artery
D) the Adamkiewicz's artery
E) it is not comprised of any of these
NEU-6.478.
Cerebral venous thrombosis:
A) can cause infarction if a larger sinus is involved
B) if only the cortical veins are involved infarction will not occur
C) venous thrombosis of the superficial veins is usually not
accompanied by a subdural abscess
D) does not occur following head injury or dehydration
E) does not occur in polycythemia vera or leukemia
NEU-6.479.
The role of hypotension in the development of cerebral infarction:
A) both hypotension and hypertension play an etiological role in
the development of cerebral ischemia and infarction
B) obstruction and the consequent distal decrease of the blood
pressure and blood flow play an important role
C) hypotension due to a cardiac disease is a rare cause
D) in healthy individuals hypotension does not affect cerebral
blood flow unless it gets below 40 mmHg
E) none of the above
NEU-6.480.
Which of the following changes of blood properties can contribute to
cerebral infarction?
A) blood coagulation is of importance, since increased coagulation
is observed prior to the infarction
B) thrombus formation is an important step, since infarction
always occurs at the site of vascular thrombosis
C) polycythemia vera plays an important role in the development
of cerebral infarction, since blood viscosity greatly increases at

hematocrit values over 0.48 and causes a transient ischernic


attack.
D) anemia enhances cerebral ischemia, especially if the
hematocrit value is below 0.30
E) none of the above
NEU-6.481.
The course of a progressive stroke DOES NOT:
A) show stepwise development
B) show a constant development
C) show similarity to a subdural hematoma
D) show similarity to a brain tumor
E) none of the above
NEU-6.482.
Stroking the sole of a foot with a pointed object is a technique for
eliciting:
A) Hoffman's sign
B) Babinski's sign
C) Chaddock's sign
D) Romberg's sign
E) Gordon's sign
NEU-6.483.
All of the following are true for the Agyll-Robertson pupil, EXCEPT:
A) tabes dorsalis
B) small pupils
C) irregular, nonuniform pupils
D) the pupils do not dilate adequately
E) the pupils show a proper light reaction
F) the pupils narrow at accomodation
NEU-6.484.
Delirium or confusion occur in all of the following EXCEPT:
A) pneumonia
B) thyrotoxicosis
C) congested cardiac failure
D) thiamine deficiency
E) homocystinuria
NEU-6.485.
Peripheral neuropathy occurs in all of the following EXCEPT:
A) carbon monoxide poisoning
B) macroglobulinemia
C) sarcoidosis
D) infectious mononucleosis
E) diabetes mellitus
F) urticaria pigmentosa
NEU-6.486.
All of the following can cause cerebral embolism EXCEPT:
A) myocardial valve prosthesis
B) Trichinosis
C) tumor
D) atrial fibrillation
E) air

F) pulmonary alveolar proteinosis


NEU-6.487.
The following symptoms - headache, anemia, rheumatic polymyalgia,
increased RBC sedimentation rate, fever, and leukocytosis - are
typical of.
A) tuberculous meningitis
B) lupus erythematosus
C) temporal arteritis
NEU-6.488.
All of the following are true for amyotrophic lateral sclerosis EXCEPT:
A) it starts at the age of 50-60-years-old
B) it is symmetric
C) it has symptoms of impaired sensory function
D) fasciculations in the upper extremities
E) it should be differentiated from a cervical spine compression
F) there are no signs of a neurogenic bladder or incontinence
NEU-6.489.
Case Study:
A 40-year-old male patient, - occupation: technician. His motorcycle
smashed into a car and the patient was attended to at a traumatology
unit. Commotio cerebri and twisted neck were diagnosed. The
patient was discharged after a 5 day observation period. After a two
week rest period he returned to work. Subsequently 2 months later,
at home, the patient had a left-sided headache, vertigo, transient
impairment of speech, and numbness on the right side of his face
and tongue. His family doctor failed to find any external injures on
his body. A two-dimensional cranial and cervical X-ray did not reveal
any traumatic alterations. Blood pressure 140/90, heart rate 82/min.
Examination: the left pupil was somewhat larger than the right one.
The grip in his right hand was slightly weaker than the left (the
patient is right-handed). Ophthalmic examination: 0.5D congestion
in the left fundus.
What is the suggested diagnosis?
A) hypertensive disease
B) brain tumor
C) post-commotion syndrome
D) chronic subdural hematoma
E) vertebrobasilar insufficiency
NEU-6.490.
Complete stroke:
A) in 80-90% of the cases seizure fits will develop in patients with
both embolic or atheromatous infarction
B) a sudden onset is typical of an embolic infarction, while the
atheromatous form is characterized by a gradual onset
C) alternating symptoms suggest supratentorial ischemia
D) anosognosia usually develops in a right-sided paralysis
E) none of the above
NEU-6.491.
In cases of occlusion of the internal carotid artery:
A) Weber's or Raymond's syndrome can develop
B) unilateral cervical murmur or elevation of the retinal arterial

pressure strongly suggests occlusion of the internal carotid


artery
C) if the occlusion develops slowly the patient can be symptomfree
D) Doppler flow measurement in the ophthalmic artery reveals
the reduction of the blood flow
E) none of the above
NEU-6.492.
The circulatory disturbances of the occipital cerebral regions:
A) comprise about 80% of the cases of cerebral ischemia
B) do not affect head movements (passive moving)
C) a bilateral occipital infarction can be followed by cortical
blindness
D) occipital infarction is always accompanied by the Weber's
syndrome
E) none of the above
NEU-6.493.
Which of the following is typical of a midbrain infarction?
A) a vigil coma and cessation of a sleep-waking cycle
B) occlusion of the basilar artery does not cause confusion
C) hemiataxia, intentional tremor, and impaired sensory function
D) hemiballism which is rarely accompanied by hemiparesis
E) none of the above
NEU-6.494.
In case of a pontine infarction:
A) damage of the posterior tegmentum can cause the MillardGubler's syndrome, but not the Foville's syndrome
B) paramedian arterial occlusion does not cause internuclear
ophthalmoplegia
C) a lateral lesion causes tinnitus, deafness, and oculomotor
paralysis
D) hemiballism does not occur
E) none of the above
NEU-6.495.
In case of a medullary infarction:
A) paramedian softening causes Wallenberg's syndrome
B) occlusion of the inferior posterior cerebellar artery causes vertigo,
nausea, vomiting, dysarthria, dysphagia, nystagmus,
contralateral Horner's syndrome, unilateral paralysis of the vocal
cords, but does not elicit impairment of the sensory functions
C) Wallenberg's syndrome occurs very rarely in medullary circulatory
disturbances
D) softening of the lateral parts of the medulla does not elicit
paralysis of the vocal cords
E) none of the above
NEU-6.496.
Occlusion of the basilar artery:
A) a visual and sensory impairment without any type of paralysis
B) unconsciousness is rare
C) cortical blindness never occurs
D) has a good prognosis
E) none of the above

NEU-6.497.
Diagnosis of cerebral embolism:
A) a complex examination (EEG, carotid and heart ultrasound
study, complete blood count including hemostasis) is useful
B) an examination of the cerebrospinal fluid is of crucial importance
C) a normal sinus rhythm excludes embolism
D) MRI is the method of choice
E) amnesia is typical
NEU-6.498.
In cerebral vein thrombosis:
A) epileptic fits rarely occur
B) it is usually of a septic or focal origin
C) headache and delirium are common, but confusion and seizures
are not typical
D) headache and vomiting never develop
E) focal neurologic symptoms are rare
NEU-6.499.
If a murmur is detected during the examination of a possible
extracranial vascular obstruction:
A) a murmur below the mandibular angle suggests a disease of
the carotid artery
B) a murmur directly under the clavicle suggests a disease of the
vertebral artery
C) the murmur always accurately reflects the severity of stenosis
D) a loud murmur indicates obstruction
E) none of the above
NEU-6.500.
In a patient with cerebral infarction:
A) the CT study has fully replaced the pneumoencephalography
in radiologicexaminations
B) a CT study does not substitute static brain scintigraphy in the
evaluation
C) a CT study does not substitute echoencephalography in the
evaluation
D) during the first 1-2 hours, a CT study can differentiate between
the intact and infarction area
E) In 50% of the cases a CT study can help to detect any complications
such as hemorrhage into the brain matter
NEU-6.501.
An ECG is not required in cerebral infarction because:
A) a CT can determine the diagnosis and the cause of this disease
B) this is a false statement becausethe ECG can reveal arrhythmia,
coronary disease, and infarction
C) an isoenzyme study is more important
D) only the ECG under a physical stress test is informative
E) none of the above
NEU-6.502.
Which of the following special examinations should be done in case
of cerebral infarction?
A) angiography, which reveals different vascular changes and

hemodynamic disturbances
B) a brain radionuclide study, which shows a marked isotope
accumulation during the first day followed by a subsequent
decrease
C) echoencephalography, which can differentiate between cerebral
edema and tumor, as well as between tumor and subdural
hematoma
D) angiography because mortality due to this intervention at
present is only 10%
E) none of the above
NEU-6.503.
In the differential diagnosis of a cerebral infarction:
A) a hemorrhagic CSF excludes a cerebral infarction
B) an angiographic study visualizes a brain abscess as an
avascular region; and lymphocytes are frequently found in
the CSF
C) if the state of the patient worsens, angiography should be
postponed until an exploration bore is made
D) the CT study helps in the differential diagnosis of infarction,
and subdural or epidural hematoma
E) none of the above
NEU-6.504.
In the prognosis of cerebral infarction:
A) the age of the patient, but not the extent of the nervous system
damage is an important predictive factor of early mortality
B) after the development of a stroke, hypertension and diabetes
do not affect the outcome
C) the severity of a concomitant cardiovascular disease is an
important factor of the survival
D) about 3/4 of the patients die during the first attack
E) About one-half of the patients surviving the first ischemic
infarction will have a hemorrhage within 1-7 years.
NEU-6.505.
Antiedematous therapy in cerebral infarction:
A) hypertonic urea is not effective
B) mannitol is effective and has no rebound effect
C) glycerine is not effective
D) dexamethasone is mainly used for the therapy of cytotoxic
edema
E) none of the above
NEU-6.506.
In cerebral infarction the cerebral blood flow can be increased with
the use of:
A) aminophylline, tolazoline, histamine
B) papaverine and nicotinic acid
C) a blockade of the stellate ganglion
D) carbon dioxide
E) none of the above
NEU-6.507.
In repeated Transient Ischemic Attacks (TIAs), the following therapy
is recommended:

A) lithium carbonate if neutropenia develops


B) aspirin if an ulcer develops
C) aspirin, 2-3 g/day
D) anticoagulants if cardiogenic stroke develops
E) anticoagulants if endocarditis underlies the ischemia
NEU-6.508.
The therapy of progressive stroke:
A) anticoagulant therapy is of no value
B) multifactorial therapy by itself or if combined with anticoagulant
therapy is of no value
C) surgical correction of arterial obstruction has good results
D) inhibition of platelet aggregation is never effective
E) none of the above
NEU-6.509.
The general treatment of completed stroke, including long-term therapy:
A) multifactorial therapy is of no value
B) surgical intervention in patients with pronounced neurologic
defects alleviates the symptoms
C) long-term anticoagulant therapy is beneficial and has no
significant complications
D) carotid surgery in patients with pronounced neurologic defects
decreases the morbidity in cases of fresh softening
E) none of the above
NEU-6.510.
The therapy of a cerebral embolism of cardiac origin:
A) anticoagulants decrease morbidity and mortality due to the
recurrent embolization
B) anticoagulants are given for 6-12 months after valve prosthesis
surgery
C) cardioversion is always performed in atrial fibrillation to restore
the normal sinus rhythm; this intervention reduces the
danger of re-embolization
D) the incidence of myocardial ischemia is not increased after
myocardial infarction
E) none of the above
NEU-6.511.
In considering contraindications to anticoagulant therapy the following
is NEGLIGIBLE:
A) an inadequate laboratory background
B) unsatisfactory patient compliance
C) peptic ulcer, hemorrhagic diathesis, hypertension, sever liver
or renal disease
D) hemorrhagic CSF
E) none of the above
NEU-6.512.
Anticoagulant therapy:
A) can be started even in case of hemorrhagic softening
B) as a rule does not cause bleeding, unless the prothrombin
time is 4-5x longer compared to the control
C) can be started in cases of severe ischemia (vast area of softening)
D) can be started in endocarditis lenta (subacute infectious

endocarditis)
E) can be very effective in cases of severe ischemic insult if initiated
early
NEU-6.513.
In hypertensive encephalopathy:
A) hypotensive therapy does not markedly affect the outcome
B) visual impairment is rare
C) confusion, seizures and papilledema are typical
D) papilledema is a rare finding
E) the CSF fmding is normal
NEU-6.514.
Hematomyelia:
A) is due to an abnormal blood composition (blood dyscrasia)
B) is always painless
C) is always due to a rupture of an arteriosclerotic vessel
D) never causes sensory disturbances
E) usually develops after trauma
NEU-6.515.
Which of the following is typical of an aneurysm?
A) usually develops at bifurcations; about 50% of the cases occur
in the anterior communicating artery
B) all aneurysms will rupture sooner or later
C) the rupture is due to an atheroma plaque and the coagulated
blood covering this plaque
D) the rupture of an infraclinoid aneurysm leads to a subarachnoid
hemorrhage
E) most aneurysms are of 4-5 cm in diameter
NEU-6.516.
Which of the following is typical of the rupture of an aneurysm?
A) a sudden headache, vomiting, sometimes confusion
B) it typically starts in the morning
C) a CSF sample has no diagnostic value
D) it has no typical neurologic symptoms
E) the CSF obtained immediately after the hemorrhage is xanthochromic
NEU-6.517.
Which of the following is typical of brain tumors?
A) usually progressive symptoms, frequent headaches, increased
intracranial pressure
B) the CSF finding is always abnormal
C) metastases are the most common finding
D) multiform glioblastoma is a malignant, well circumscribed
tumor
E) brain tumors comprise about 1/3 of all occurring tumors
NEU-6.518.
Which of the following possibilities should not be considered in the
differential diagnosis of an intracranial hemorrhage?
A) an overdose of anticoagulants
B) the hemorrhage fully destroying a minor malformation
C) a hemorrhagic infarction with marked softening, hence differing
from a primary hemorrhage

D) leukemia or thrombocytopenia
E) none of the above
NEU-6.519.
Blood originating from a subarachnoid hemorrhage DOES NOT:
A) cause meningeal exudation, scarring, CSF absorption, and
finally a communicating hydrocephalus
B) irritate the vessels, the meninxes and the brain
C) cause cardiac arrhythmia by affecting the descending autonomic
impulses
D) cause hypertension by affecting the descending autonomic
impulses
E) none of, the above
NEU-6.520.
Ischemia and infarction occurring after a subarachnoid hemorrhage:
A) both can be prevented by drug therapy
B) very rarely occur in the brain areas perfused by the artery with
the ruptured aneurysm
C) posthemorrhagic irritative vasospasm is caused by the vascular
damage, furthermore, surgical manipulations can also elicit
severe vasospasm.
D) do not cause a postoperative vegetative state
E) none of the above
NEU-6.521.
Which of the following is typical of an intracerebral hemorrhage?
A) the CT visualizes hyperdense regions even after several
months
B) the CSF is always hemorrhagic
C) it usually. occurs due to a hypertensive vascular disease or a
minor a-v malformation
D) a fresh hemorrhage is not always detectable
E) these patients always have hypnoid confusion
NEU-6.522.
After subarachnoid hemorrhage:
A) seizures and headache occur at the onset
B) hypertonic hemiplegia or hemiparesis with Babinski's sign is
the most common neurologic symptom
C) papilledema is the most common symptom
D) no signs of meningeal excitement are observed
E) a CT is of no diagnostic value
NEU-6.523.
If an aneurysm hemorrhage occurs in the:
A) middle cerebral artery: hemimotor and
hemisensory disturbances are expected
B) infraclinoid part of the internal carotid artery: death will
always occur
C) anterior cerebral artery - anterior communicating artery: a
bilateral paresis of the extremities can develop
D) vertebral artery: it is expected as hemorrhages frequently
occur in this area
E) none of the above

NEU-6.524.
Which of the following is important in the diagnosis of hemorrhages
caused by the rupture of arteriovenous malformations?
A) focal seizures in the history, focal neurologic symptoms, hemilateral
headaches, nausea
B) clinical symptoms markedly differ from those occurring in a
ruptured aneurysm
C) murmurs equally occur in aneurysms and arteriovenous
malformations
D) seizures developing at the onset of hemorrhage rather suggest
a traumatic origin
E) the CT is of no value
NEU-6.525.
In the diagnosis of an intracerebral hemorrhage:
A) a lumbar puncture nearly always helps to differentiate it from
a hemorrhagic infarction
B) the onset is usually gradual
C) unconsciousness and hypertension are common after the
onset
D) headache is as frequent as in cerebral infarction
E) a CT can differentiate the hemorrhagic area from the intact
tissue only several hours after hemorrhage
NEU-6.526.
An artificial CSF hemorrhage after a lumbar puncture:
A) after immediate centrifugation the supernatant can still be of
pink color due to the presence of oxyhemoglobin
B) after centrifugation the supernatant can be yellow
C) the cell count shows 100 WBC corresponding to every 1000
RBC
D) the CSF sample should be taken in several fractions, which
will clear up; after centrifugation the sample is water-clear
E) gives a negative benzidine reaction
NEU-6.527.
The treatment of a subarachnoid hemorrhage caused by arteriovenous
malformations:
A) is determined by the symptoms, localization; age, and the
collaterals
B) carotid ligature or a ligature of a supplying vessel is the
method of choice
C) amputation of the brain tissue containing the malformation is
the method of choice
D) surgery should be done even in cases of extensive and diffuse
malformations
E) the incidence of repeated hemorrhage is about 100%
NEU-6.528.
In cases of intracerebral bleeding:
A) surgery is the method of choice
B) independently of the applied therapy a high ratio of patients in
deep coma will die
C) surgical removal of intracerebral hematoma due to a ruptured
aneurysm has better results compared to those, in which
hemorrhage was due to a hypertensive vascular event

D) in most cases drug therapy gives satisfactory results


E) patients in coma usually survive if treated by calcium-channel
blockers
NEU-6.529.
Pseudotumor cerebri:
A) causes headache and papilla congestion, but does not cause
diplopia and disturbances of visual function
B) sometimes the ventricles are narrowed
C) the CSF is always abnormal
D) accompanies hyperthyrosis, but not hypothyrosis
E) none of the above
NEU-6.530.
Metstatic intracranial tumors:
A) comprise about 80% of all intracranial tumors
B) primarily originate from the lung or breast
C) a supratentorial localization has poor prognosis
D) are common in the cerebrum, but rare in the cerebellum
NEU-6.531.
All the following are typical of a pinealoma EXCEPT:
A) it is common in young male patients
B) it is common in young female patients
C) it can cause paralysis of upward gaze
D) the pupils react to convergence, but not to light
E) hydrocephalus is a common sequel
NEU-6.532.
The ependymomas:
A) mainly develop in elderly patients
B) mainly develop in the vicinity of the ventricles
C) the survival period is very short
D) the formation of perivascular pseudorosettes is not typical
E) none of the above
NEU-6.533.
The two main causes of cerebral metastases are the:
A) breast and lung
B) lung and colon
C) colon and rectum
D) colon and nasal sinuses
E) uterus and ovaries
NEU-6.534.
Oligodendroglioma:
A) has an extremely fast growth
B) hemispheral, frontal or basal ganglionic localization is extremely
rare
C) practically never causes epileptic seizures
D) relapses are rare
E) none of the above
NEU-6.535.
Pseudotumor cerebri:
A) rarely occurs during pregnancy

B) rarely accompanies endocrinopathy or drug therapy


C) the ventricles and subarachnoid space are distended
D) visual acuity is severely impaired
E) headaches are rare
NEU-6.536.
The most common pituitary tumor is:
A) chromophobic adenoma
B) chromophilic, adenoma
C) colloid cyst
D) carcinoma
E) none of the above
NEU-6.537.
A long period between the onset of the symptoms and the diagnosis
of an intracranial tumor occurs in cases of.
A) melanoma metstasis
B) medulloblastoma
C) multiforme glioblastoma
D) pituitary adenoma
E) melanosarcoma
NEU-6.538.
The most frequent intradural extramedullary spinal tumor is:
A) glioma and angioma
B) meningioma and neurofibroma
C) sarcoma and lipoma
D) glioma
E) cold abscess
NEU-6.539.
Ependymoma:
A) mainly occurs in elderly patients
B) surgery is the only possible therapy
C) is extramedullary
D) radiotherapy is the only possible treatment
E) combined therapy (surgery + radiotherapy) gives the best
results
NEU-6.540.
Medulloblastoma:
A) mainly occurs in elderly patients
B) 90% are localized in the cerebellum
C) the tumor is not radiosensitive
D) never has relapses
E) all of the above
F) none of the above

NEU-6.541.
If an acoustic neurinoma is suspected:
A) the CSF protein is increased in about 3/4 of the cases
B) the enlargement of the internal acoustic meatus is of minor
importance
C) tinnitus and a hearing impairment are late symptoms
D) a congested papilla is an early symptom

E) the trigeminal and facial nerves are rarely involved


NEU-6.542.
Which of the following is typical of a tumor or a disc-compression of
the cauda equina?
A) the processes are above the L1-2 level
B) pain; flaccid paralysis of the lower extremities
C) spastic paralysis
D) Babinski's sign; hyperreflexia
E) none of the above
NEU-6.543.
After cranial injury:
A) an epileptic fit indicates at least a cerebral contusion
B) brain commotion is always accompanied by morphological
symptoms
C) headaches following brain commotion last only for several
hours
D) a CSF sample helps to confirm the diagnosis
E) lucid intervals always occur in epidural hematoma
NEU-6.544.
The carpal-tunnel syndrome:
A) usually affects the musculocutanous nerve
B) usually the median nerve is not involved
C) occurs in myxedema, diabetes, acromegaly or during pregnancy
D) does not cause thenar atrophy
E) affects the radial nerve
NEU-6.545.
The most frequent cause of a bilateral lower motoneuron hypoglossus
paralysis is:
A) poliomyelitis
B) progressive bulbar paralysis
C) a metastatic tumor
D) diabetes
E) all of the above
F) none of the above
NEU-6.546.
In the differential diagnosis of a peroneal nerve lesion the following
should be considered:
A) an L4-5 disc disease
B) a late stage of Charcot-Marie-Tooth's disease
C) polyneuropathy
D) all of the above
E) none of the above
NEU-6.547.
Which of the following is typical of diabetic polyneuropathy?
A) it develops only in severe diabetes
B) the CSF is always normal
C) a determination of the nerve conduction velocity is not relevant
in the diagnosis
D) a symmetric, distal sensomotor diabetic polyneuropathy is the
most common disorder

E) all of the above


F) none of the above
NEU-6.548.
A subdural hematoma:
A) is not accompanied by epileptic symptoms
B) a xanthochromic CSF is not typical
C) the CSF is always Clear
D) is never isodense on the CT
E) all of the above
F) none of the above
NEU-6.549.
Bell's palsy:
A) Bell's palsy (idiopathic form) comprises about 75% of all cases
of facial palsy
B) rarely occurs in diabetes
C) only 5-10% of the patients will recover
D) hyperacusis does not occur
E) all of the above
F) none of the above
NEU-6.550.
Delivery paralysis of the brachial plexus:
A) is an upper plexus type injury
B) is a lower plexus type injury
C) affects the whole plexus
NEU-6.551.
The most common causes of a cauda equina lesion include:
A) trauma, tumor, inflammation, disc damage
B) vascular malformation and inflammation
C) metabolic and toxic diseases
D) congenital malformation and occlusive vascular disease
E) trauma and bone development anomalies
NEU-6.552.
The injury of the obturator nerve during delivery would cause one of
the following:
A) hyperalgesia on the anterolateral surface of the thigh without
motor disorders
B) incontinence
C) a symptom free state
D) severe gait disorders
E) adduction disorders in the legs
NEU-6.553.
Full recovery after Bell's palsy:
A) occurs in 20% of the cases
B) does not occur in elderly patients, hyperacusis, and cases of
severe muscle weakness at the onset of the disease
C) develops within 10-14 days
D) does not depend on the severity of the lesion
E) surgery should be immediately performed
NEU-6.554.

Neuralgia paresthetica:
A) treatment usually means splitting of the fascia lata
B) pain develops at the medial part of the thigh
C) is usually caused by the compression of the inguinal ligament
D) female patients are more frequently affected
E) none of the above
NEU-6.555.
Injury of the femoral nerve:
A) originates from the S2-4 root
B) the patient can walk with extended extremities on a horizontal
surface
C) does not affect an upward walk
D) does not affect stair climbing
E) all of the above
F) none of the above
NEU-6.556.
Epilepsy after head injury:
A) is always accompanied by the same type of seizures
B) usually occurs following closed skull injuries
C) develops within several minutes or hours following the injury
D) the correct therapy indicates a good prognosis
E) all of the above
F) none of the above
NEU-6.557. Single Choice Question
The vagus nerve:
A) nerve stimulation elicits dyspnea, vomiting, bradycardia,
B) damage of its sensory fibers does not cause any neurologic
deficits
C) damage of the nerve does not cause hoarseness
D) bilateral damage of the nerve does not cause significant symptoms
E) all of the above
F) none of the above
NEU-6.558.
An extradural hematoma:
A) affects the middle cerebral artery
B) is usually bilateral
C) the patient always has a lucid interval which is the essential
sign for the correct diagnosis
D) usually accompanies fractures of the ethmoid bone
E) all of the above
F) none of the above
NEU-6.559.
An epidural hemorrhage:
A) is usually localized in the scala media over the hemispheral
convexity
B) the CSF is usually hemorrhagic
C) is usually accompanied by a lucid interval of several days
D) usually elicits a contralateral pupil dilation
E) usually causes ipsilateral hemiplegia
NEU-6.560.

In Creutzfeld-Jakob's disease:
A) motor symptoms and myoclonus rarely occur
B) dementia does not occur
C) the patient is bedridden and shows a decerebrate rigidity for
several months
D) myoclonus is not a typical sign
E) all of the above
F) none of the above
NEU-6.561.
In Creutzfeld-Jakob's disease:
A) vaccination of a laboratory animal with brain tissue obtained
from a patient does not cause any pathological changes in the
animal
B) vaccination of a laboratory animal with brain tissue obtained
from a patient induces similar symptoms in the animal
C) the disease has a good prognosis
D) typical EEG changes develop early in the disease
E) none of the above
NEU-6.562.
In Cryptococcus meningoencephalitis:
A) there is a decreased glucose level, but it can be modified as in
a diabetic patients
B) usually the polymorphonuclear leukocytes are prevalent
C) as a rule the pathogenic microorganism cannot be identified
D) the infection is sexually transmitted
E) the disease has a good prognosis
NEU-6.563.
In tuberculous meningitis:
A) the upper cranial nerves are damaged
B) the CSF glucose level is usually normal
C) the WBC count in the CSF usually exceeds several thousand
D) the CSF glucose level is elevated
E) the CSF usually contains several hundred lymphocytes
NEU-6.564.
In the acute phase of bacterial meningitis:
A) the glucose level is normal
B) the cell count is between 50-100
C) the cell count (mainly polymorphonuclear cells) is between
1000-10,000
D) therapy can be started only after evaluation of the antibiogram
E) the CSF pressure is low
NEU-6.565.
In acute bacterial meningitis the CSF shows:
A) lymphocytes
B) polymorphonuclear leukocytes comprising 80-90% of the cells
C) a low protein content
D) positive bacteriologic cultivation results
E) all of the above
F) none of the above
NEU-6.566.

In acute bacterial meningitis:


A) clinical symptoms develop over several weeks
B) occipital stiffness and Brudzinsky's sign are not always
present (like in infants, elderly patients)
C) therapy cannot be started before evaluation of the antibiogram
D) a daily dose of 10,000 U of penicillin should be administered
E) all of the above
F) none of the above
NEU-6.567.
The most frequent cause of meningitis:
A) Leptospira; Brucella
B) Salmonella; Listeria
C) E.Coli; Shigella
D) Cocci; Hemophilus influenzae
E) none of the above
NEU-6.568.
Sarcoidosis:
A) mainly affects the vessels of the brain convexity
B) involvement of the hypothalamus and optic chiasma can cause
diabetes insipidus and amblyopia
C) does not cause myelitis
D) does not cause brain stem symptoms
E) causes neuropathy, but not myopathy
NEU-6.569.
Leukemic meningiosis:
A) there is a high cell count and an elevated glucose content in
the CSF
B) is not accompanied with any meningeal signs
C) the CSF glucose level shows minor if any changes
D) cranial nerve symptoms never occur
E) none of the above
NEU-6.570.
Viral meningitis:
A) causes a typical CSF pressure elevation
B) the cell count is usually several thousand
C) polymorphonuclear cells are found in the CSF
D) the pleocytosis in mumps is milder
E) all of the above
F) none of the above
NEU-6.571.
Viral encephalitis:
A) usually accompanies viral meningitis
B) rarely causes headache
C) usually does not cause focal symptoms, seizures
D) the EEG changes are not typical
E) the CT is always normal
NEU-6.572.
In a brain abscess:
A) the CSF cell count is always elevated
B) in more than a half of the cases the process is localized only in

the brain stem


C) an abscess and a tumor can be only differentiated with
neuroradiology studies
D) there is always fever
E) the temporal lobe is rarely involved
NEU-6.573.
Which of the following do not cause any meningeal signs?
A) a high cell count in the CSF
B) a low glucose content in the CSF
C) an increased intracranial blood pressure
D) dehydration
NEU-6.574.
Which symptom is not a sign of meningeal excitement?
A) occipital stiffness
B) "foxhound" posture
C) Kernig's sign
D) Brudzinsky's sign
E) Chaddock's sign
NEU-6.575.
The main cause in the development of meningeal signs is:
A) leptomeninx excitement
B) spinal dorsal root excitement
C) spinal neuron excitement
D) deterioration of the spinal neurons
E) all of the above
F) only (A) and (B) are true
G) only (C) and (D) are true
NEU-6.576.
Which pathological process does not elicit any meningeal symptoms?
A) a rupture of an intracranial aneurysm
B) an increase of the intracranial pressure
C) any circumscribed softening of brain tissue
D) leptomeningeal inflammation
NEU-6.577.
An olfactory meningioma starts from the:
A) hippocampal uncal gyrus
B) arachnoidea covering the lamina cribrosa
C) olfactory bulb
D) olfactory tract
NEU-6.578.
Which symptom is not typical of multiple sclerosis?
A) a scanning speech
B) a lack of the abdominal cutaneous reflex
C) flaccid paraparalysis
D) optic atrophy - temporal pallor
E) pyramidal signs
NEU-6.579.
Which of the following does not cause a congested papilla?
A) an elevation of intracranial pressure
B) intracranial space occupying processes

C) myopia
D) brain edema
NEU-6.580.
Which of the following does not cause a congested papilla?
A) pseudotumor cerebri
B) toxic damage; any direct compression; damage to the optic
nerve
C) cerebral tumor
D) hypertensive encephalopathy
NEU-6.581.
Which of the following is not typical of a congested papilla?
A) an indistinct border
B) hyperemia
C) a "china-white" papilla
D) the papilla itself protruding from the surface
NEU-6.582.
Which of the following is not typical of retrobulbar neuritis?
A) temporal pallor
B) deterioration of the central vision
C) a decreased central fusion frequency
D) contralateral homonymous hemianopsia
NEU-6.583.
In which of the following can retrobulbar neuritis occur?
A) a tumor of the optic nerve
B) multiple sclerosis
C) a tumor of the occipital lobe
D) Foster-Kennedy's syndrome
NEU-6.584.
Which of the following pathological processes cannot be diagnosed
on the basis of the fundus finding?
A) the presence of an intracranial space occupying process
B) whether the patient had an intracranial space occupying
process at an earlier time
C) an alteration of the vascular system
D) any brain atrophy
NEU-6.585.
Which of the following is typical of multiple sclerosis?
A) it mainly develops in elderly patients
B) it is related to an infection caused by ticks
C) any oligoclonal y-antibidies in the CSF is of diagnostic value
D) a relapsing course with accumulating disability
E) it improves after steroid or cytostatic therapy
F) only (A), (B), and (C) are true
G) only (A), (B), (C), and (D) are true
H) only (C), (D), and (E) are true
NEU-6.586.
Which of the following is typical of the optical light reflex after an
ipsilateral optic nerve lesion?
A) the ipsilateral direct reflex is absent, while the indirect one is

present
B) both direct and indirect reflexes are absent on the ipsilateral
side
C) both direct and indirect reflexes remain intact on the contralateral
side
D) no direct reflex on the ipsilateral side while the indirect reflex
is present on the contralateral side
NEU-6.587.
Which of the following comprises the efferent part of the pupillary
light-reflex?
A) the sympathetic nerve
B) the oculomotor nerve
C) the facial nerve
NEU-6.588.
Which symptom does not belong to Adie's syndrome?
A) a tonic pupil
B) a lack of deep reflexes in the lower extremities
C) a lack of the cornea reflex
NEU-6.589.
Reflex stiff pupil:
A) lack of the light reaction while accomodation and convergence
remain intact
B) intact light reaction and an impaired accomodation and convergence
reaction
C) intact light reaction and accomodation; impaired convergence
reaction
D) intact convergence reaction; impaired light reaction and
accomodation
NEU-6.590.
The primary cause of optic nerve atrophy:
A) toxic and other types of damage (not common), direct compression
B) papilledema
C) papillitis
D) papilledema and papillitis
NEU-6.591.
lpsilateral amaurosis and contralateral loss of the temporal visual
field are caused by:
A) ipsilateral chiasma damage
B) bilateral damage of the optic tract
C) damage to the medial part of the chiasma
D) bilateral damage to the external part of the chiasma
NEU-6.592.
Bitemporal anopia of the lower quadrant is caused by:
A) pituitary tumors
B) tumors originating from the tuberculum sellae region
C) tumors of the olfactory gyrus
NEU-6.593.
Which of the following causes ipsilateral blindness?
A) ipsilateral damage of the optic tract

B) ipsilateral damage of the optic nerve


C) ipsilateral damage of the chiasma
D) contralateral damage of the optic radiation
NEU-6.594
Which of the following is typical of ipsilateral damage (till midline) of
the chiasma?
A) ipsilateral temporal hemianopsia
B) contralateral nasal hemianopsia
C) ipsilateral blindness and contralateral nasal hemianopsia
D) ipsilateral blindness and contralateral temporal hemianopsia
NEU-6.595.
Heteronymous hemianopsia is caused by:
A) damage to the medial part of the chiasma
B) damage to the optic tract
C) damage to the optic nerve
NEU-6.596.
Binasal heteronymous hemianopsia is caused by:
A) a pituitary tumor
B) bilateral damage of the external part of the chiasma
C) damage of the optic tract
D) damage of the optic nerve
NEU-6.597.
When does Wernicke's hemianopsic pupil reaction fail to occur?
A) if the optic tract is damaged
B) if the lateral geniculate body is damaged
C) if the optic radiation is damaged
D) if the primary visual area is damaged
NEU-6.598.
When does psychic blindness develop?
A) if there is damage to the primary visual field
B) if there is damage to the secondary visual field
C) if there is damage to the optic radiation
D) after a psychotrauma
NEU-6.599.
When does cortical blindness develop?
A) in damage of the secondary visual field
B) after a psychotrauma
C) in damage of the primary visual field
D) in damage of the optic tract
NEU-6.600.
Which Brodmann's area does not belong to the visual cortex?
A) Brodmann's area 17
B) Brodmann's area 8
C) Brodmann's area 18
D) Brodmann's area 19
NEU-6.601.
Contralateral homonymous hemianopsia is caused by:
A) damage to the optic nerve

B) damage to the medial part of the chiasma


C) ipsilateral damage to the chiasma
D) damage to the optic tract
NEU-6.602.
The symptoms of an ipsilateral lesion of the oculomotor nerve are:
A) ptosis; the bulb deviates outward-downward; dilated, fixed
pupils
B) ptosis; the bulb deviates outward-upward; mydriasis
C) lagophthalmus; ptosis; the bulb deviates outward-downward;
miosis
D) ptosis; the bulb deviates outward-downward; miosis
NEU-6.603.
Which muscle is innervated by the abducent nerve?
A) the medial rectus muscle
B) the inferior rectus muscle
C) the lateral rectus muscle
D) the inferior oblique muscle
NEU-6.604.
Which muscle is innervated by the trochlear nerve?
A) the inferior oblique muscle
B) the inferior rectus muscle
C) the superior oblique muscle
D) the superior rectus muscle
NEU-6.605.
How should the convergence reaction be examined?
A) approach the near point with a finger
B) move the finger beyond the near point
C) fix the finger at 30 cm before the patient's eyes
NEU-6.606.
The accomodation reaction:
A) accompanies horizontal vision
B) is important for acute vision
C) is important for vertical vision
NEU-6.607.
Which symptom cannot be due to damage of the medial longitudinal
fasciculus?
A) Pick's vision
B) internuclear ophthalmoplegia
C) Hertwig-Magendie's eye position
D) impaired convergence
E) doll's eye phenomenon
NEU-6.608.
Directed eye movements occur:
A) on fixation of any moving objects
B) if the patient fixes on lateral moving objects only
C) on fixation of approaching objects only
D) if the patient looks without fixating on any moving object
NEU-6.609.

Which of the following is typical of searching eye movement?


A) the patient gazes in all directions without any type of fixation
B) the patient always fixes on a moving object
C) the patient looks in all directions in order to fix on an object
NEU-6.610.
After a central type of damage to eye movements one of the following
symptoms develop:
A) conjugated deviation
B) diplopia
C) indistinct images
NEU-6.611.
Which of the following is true for a peripheral visual impairment?
A) no diplopia occurs
B) diplopia develops
C) conjugated deviation develops
NEU-6.612.
Which nerve innervates the pupil dilator muscle?
A) the oculomotor nerve
B) the first branch of the trigeminal nerve (ophthalmic nerve)
C) a cervical sympathetic nerve
NEU-6.613.
Which damage does not elicit anisocoria?
A) ipsilateral Horner's syndrome
B) ipsilateral damage to the ciliary ganglion
C) an ipsilateral lesion of the oculomotor nexve
D) an ipsilateral lesion of Brodmann's area 19
NEU-6.614.
Hertwig-Magendie's eye position:
A) the axis of the eyes are different in the horizontal plane
B) the axis of the eyes are different in the vertical plane
C) the axis of the eyes are different in the saggital plane
NEU-6.615.
Which of the following is true if the eyes deviate upward in Bell's test
and voluntary upward gaze is inhibited?
A) there is peripheral damage
B) there is central damage
C) it is impossible to decide whether the damage is central or
peripheral
NEU-6.616.
Which of the following does not cause impairment of gaze?
A) damage to the interstitial nucleus
B) damage to the posterior comissural nucleus
C) damage to the nucleus ambiguus
D) damage to Brodmann's area 8

NEU-6.617.
Which of the following causes impairment of gaze?
A) damage to the oculomotor nuclei

B) damage to the oculomotor nerves


C) damage to the precentral region of the frontal lobe
NEU-6.618.
The cause of internuclear oculomotor damage is:
A) damage to the reticular formation
B) a lesion of the gaze centers
C) damage to the medial longitudinal fasciculus
D) a bilateral lesion of the oculomotor nerve nucleus
NEU-6.619.
Which of the following is not a symptom of a 5th cranial nerve
lesion?
A) impaired masseter function
B) an impaired sensation of flavor
C) a lack of the corneal reflex
D) facial sensory disorders
NEU-6.620.
Which of the following is not a symptom of a 5th cranial nerve
lesion?
A) paralysis of the facial muscles
B) paralysis of the masseter muscle
C) a lack of the corneal reflex
D) neuroparalytic keratitis
NEU-6.621.
Which muscle is not innervated by the trigeminal nerve?
A) the masseter muscle
B) the platysma muscle
C) the temporalis muscle
D) the pterygoid muscle
NEU-6.622.
Which of the following types of trigeminal neuralgia does not exist?
A) symptomatic
B) psychogenic
C) genuine
NEU-6.623.
Which of the following nerves should be intact for a corneal reflex to
occur?
A) only the trigeminal nerve
B) only the optic nerve
C) only the facial nerve
D) both the trigeminal and facial nerves
NEU-6.624.
The symptoms of a peripheral facial nerve lesion include:
A) the angle of the mouth on the ipsilateral side is localized
deeper, and ptosis develops
B) the angle of the mouth on the ipsilateral side is localized
deeper, the patient has lagophthalmus and cannot wrinkle his
forehead
C) the angle of the mouth on the contralateral side is localized
deeper.

D) the angle of the mouth on the contralateral side is localized


deeper, the patient has lagophthalmus and cannot wrinkle his
forehead
NEU-6.625.
Which of the following is typical of a peripheral facial nerve lesion?
A) ptosis on the ipsilateral side
B) ptosis on the contralateral side
C) lagophthalmus on the ipsilateral side
D) lagophthalmus on the contralateral side
NEU-6.626.
Which of the following is true for a peripheral facial nerve paralysis?
A) steroid therapy is recommended
B) cytostatic therapy is recommended
C) surgery depends on the EMG diagnostic findings
D) the patient usually recovers in 5-10 days
E) all of the above
F) only (A) and (B) are'true
G) only (B) and (C) are true
H) only (C) and (D) are true
I) only (A) and (C) are true
NEU-6.627.
Supranuclear projections run towards the motor facial nucleus from:
A) the hippocampus
B) the thalamus
C) the locus ceruleus
D) the nucleus intermedius
NEU-6.628.
Which of the following does not cause a peripheral facial nerve lesion?
A) a space occupying process in the pontocerebellar angle
B) any type of draft
C) otitis media
D) a process at the pontomedullary junction
E) any processes occurring in the medial part of the pons
NEU-6.629.
When do lacrimation disturbances occur?
A) if the facial nerve is damaged after its exit from the stylomastoid
foramen
B) if the facial nerve is damaged after the separation of the stapedius
nerve
C) if the facial nerve is damaged after its exit from the internal
acoustic meatus
NEU-6.630.
Which of the following are symptoms of a peripheral facial nerve
paralysis?
A) ipsilateral paralysis of the facial muscles of the eye, mouth,
and forehead
B) ipsilateral tinnitus
C) ipsilateral lacrimation disorders; hyperacusis
D) flavor sensation disorders on the whole surface of the tongue
E) all of the above

F) only (A), (B), and (C) are true


G) only (B), (C) and (D) are true
H) only (A), (B), and (D) are true.
NEU-6.631.
Damage of which cranial nerve elicits disorders of flavor sensation in
the posterior one-third of the tongue?
A) the facial nerve
B) the vagus nerve
C) the glossopharyngeal nerve
D) the trigeminal nerve
NEU-6.632.
What does the term "third-grade nystagmus" mean?
A) that the direction of the nystagmus corresponds to the direction
of gaze
B) that the nystagmus appears when the patient looks ahead
C) that the direction of the nystagmus opposes the direction of
gaze
D) that nystagmus develops when the patient looks at either
Side
NEU-6.633.
Damage to which area is assumed if optokinetic nystagmus cannot
be elicited?
A) Brodmann's area 17
B) Brodmann's area 18
C) Brodmann's area 19
D) Brodmann's area 8
NEU-6.634.
Vestibular nystagmus:
A) in peripheral vestibular disorders nystagmus is directed opposite
to the side of the lesion
B) in central vestibular disorders nystagmus is directed towards
the side of the lesion
C) in peripheral vestibular disorders nystagmus is directed towards
the side of the lesion
D) in central vestibular disorders nystagmus is directed opposite
to the side of the lesion
E) only (A) and (B) are true
F) only (C) and (D) are true
NEU-6.635.
Which nerve is not localized in the cavernous sinus?
A) the trochlear nerve
B) the oculomotor nerve
C) the abducent nerve
D) the ophthalmic nerve
E) the maxillary nerve
NEU-6.636.
Which symptom does not belong to those compromising a vestibular
lesion?
A) ipsilateral tilting of the body
B) nystagmus directed towards the lesion

C) nystagmus directed away from the lesion


D) ipsilaterallateralization
NEU-6.637.
The direction of nystagmus is defined:
A) according to its slow component
B) according to its fast component
C) according to the side of the vestibular lesion
NEU-6.638.
Tinnitus is usually caused by:
A) arteriosclerosis
B) hypertension
C) medication
D) a tumor of the cerebello-pontine angle
E) multiple sclerosis
F) all of the above
G) only (A), (B), (C), and (D) are true
H) only (B), (C), (D), and (E) are true
NEU-6.639.
Stimulation of the horizontal semicircular canal with cold water
elicits nystagmus with a direction opposite to that of stimulation,
because:
A) cold water causes ampullo-fugal endolymph flow
B) warm water, and not cold water, causes ampullo-fugal endolymph
flow
C) cold water causes ampullo-petal endolymph flow
NEU-6.640.
Which of the following is not typical of Mnire's disease?
A) confusion
B) rotation vertigo
C) a feeling of dissolution
D) vegetative symptoms
NEU-6.641.
Which of the following areas has no connection with the vestibular
nuclei?
A) the oculomotor nerve nuclei
B) the cerebellum
C) the spinal cord
D) the reticular formation
E) the hippocampus
NEU-6.642.
The result of the tuning fork test in a perception-type hearing impairment
is:
A) Weber test: lateralization to the left
B) Rinn test: negative on the right side
C) Weber test: lateralization to the right
NEU-6.643.
The result of the tuning fork test in a right-sided hearing impairment
due to a disease in the middle ear is:
A) Weber test: lateralization to the right

B) Weber test: lateralization to the left


C) Rinn test: positive on the right side
NEU-6.644.
Which of the following can cause full deafness?
A) meningitis
B) trauma
C) any ipsilateral lesion of the temporal lobe
D) any ipsilateral medullary lesion
E) all of the above
F) only (A) and (B) are true
G) only (C) and (D) are true
H) only (B) and (C) are true
NEU-6.645.
Which of the following does not belong to the auditory pathways?
A) the superior colliculus
B) the lateral lemniscus
C) the inferior colliculus
D) the medial geniculate body
NEU-6.646.
Psychic deafness is caused by:
A) psychotrauma
B) damage to the cochlear nerve
C) damage to the primary auditory area
D) damage to the secondary auditory area
NEU-6.647.
Which of the semicircular canals is stimulated while the body rotates?
A) the anterior canal
B) the posterior canal
C) the lateral canal
NEU-6.648.
Which of the following develops if the horizontal semicircular canal
is stimulated by instilling warm water?
A) nystagmus directed towards the side of stimulation
B) nystagmus directed to the unstimulated side
C) nystagmus directed to both sides
D) vertical nystagmus
NEU-6.649.
Which of the following is not a symptom of a bilateral lesion of the
glossopharyngeal nerve?
A) impaired swallowing
B) impaired articulation
C) an impaired sensation of taste in the proximal two-thirds of
the tongue
NEU-6.650.
Which of the following is not a symptom of a glossopharyngeal nerve
lesion?
A) an impaired taste sensation in the posterior one-third of the
tongue
B) impaired swallowing

C) an impaired taste sensation in the proximal two-thirds of the


tongue
D) a lack of palatinal and pharyngeal reflexes
NEU-6.651.
Which of the following is not a symptom of vagal stimulation?
A) attacks of dyspnea
B) vomiting
C) bradycardia or tachycardia
D) tinnitus
NEU-6.652.
Which of the following statements is false for a central lesion of the
hypoglossal nerve?
A) the tongue deviates towards the side of the cerebral lesion
B) the tongue deviates to the side opposite to that of the cerebral
lesion
C) no fasciculations develop in the tongue
D) no atrophy is observed in the tongue
NEU-6.653.
Which of the following is not typical of accessory nerve function?
A) innervation that allows the head to bend to either side
B) innervation that allows the head to bend to the same side
C) innervation that allows the head to bend to the opposite side
D) it innervates the first cervical dermatome
NEU-6.654.
Which of the following statements is false in a lesion of the hypoglossal
nerve?
A) ipsilateral tongue atrophy
B) fibrillation developing 10-14 days later
C) Jackson's symptom develops if the spinal artery is occluded
also
D) a bilateral peripheral lesion does not cause any definite complaints.
NEU-6.655.
Damage to the vagus nerve:
A) causes tongue deviation
B) causes hoarseness or aphonia
C) causes vomiting and bradycardia
D) all of the above
E) only (A) and (C) are true
F) only (B) and (C) are true
NEU-6.656.
Which of the following statements about the anatomy of the hypoglossal
nerve is FALSE?
A) it's nucleus is localized in the medulla along the midline
B) it's supranuclear innervation is fully crossed
C) the root fibers leave the brain stem between the pyramidal
decussation and the inferior olive
D) the fibers are localized in the genu of the internal capsule
NEU-6.657.
An ipsilateral lesion of the hypoglossal nerve:

A) the tongue deviates to the side opposite to that of the lesion


B) the tongue deviates towards the side of the lesion
C) the tongue does not deviate at all
D) the patient cannot move the tongue
NEU-6.658.
Hyperthyroidism is accompanied by:
A) myopathy
B) myasthenia
C) tremor
D) dementia
E) all of the above
F) only (A), (B), and (C) are true
G) only (A) and (D) are true
H) only (A), (C), and (D) are true
NEU-6.659.
Which of the following is false in the bulbar syndrome?
A) it is caused by a bilateral lesion of the lower cranial nerves
B) it is caused by a bilateral lesion of the nuclei of the lower
cranial nerves
C) it is caused by a lesion of the nuclei of the oculomotor nerve
D) there is impaired swallowing
E) all of the above
F) only (A) and (B) are true
G) only (C) and (D) are true
NEU-6.660.
Which of the following symptoms does not belong to the "syndrome
of the ponto-cerebellar angle"?
A) a decreased corneal reflex
B) masseter atrophy
C) paralysis of the facial muscles
D) tinnitus
E) conjugated deviation
NEU-6.661.
What does paraneoplasia mean?
A) cerebellar atrophy
B) polyneuropathy
C) encephalitis
D) abscess
E) polymyositis
F) all of the above
G) only (A), (B), and (D) are true
H) only (A), (B), and (E) are true
NEU-6.662.
Which of the following is not true for the masseter reflex?
A) it is a proprioreflex of the 5th cranial nerve
B) it is one of the cranial nerve reflexes
C) it is one of the signs of mental deterioration
D) it is a skin reflex
NEU-6.663.
Which of the following symptoms is not caused by an inflammatory

process of the pyramid (petrous) bone?


A) an ipsilateral lesion of the abducent nerve
B) ipsilateral facial pain
C) contralateral hemiparesis
NEU-6.664.
Which of the following symptoms is not caused by damage to the
ciliospinal center?
A) pupillotonia
B) miosis
C) enophthalmus
NEU-6.665.
In peripheral vestibular damage:
A) nystagmus and vertigo occur together
B) vertigo does not accompany nystagmus
C) only vertigo occurs
D) the body has tilted in the direction opposite to that of the fast
component of the nystagmus
E) only (A) and (B) are correct
F) only (A) and (D) are correct
NEU-6.666.
Which of the following is typical of internuclear paralysis of the
concerned ocular muscle?
A) the medial rectus muscles do not function if a patient turns
his eyes, but the muscles do function during convergence
B) the medial rectus muscles do not function when a patient
turns his eyes, nor during convergence
C) the medial rectus muscles function when a patient turns his
eyes, but do not function during convergence
NEU-6.667.
Tremor at rest is typical of:
A) multiple sclerosis
B) epilepsy
C) parkinsonism
D) neurasthenia
NEU-6.668
Which of the following is not characteristic of the Edinger-Westphal
nucleus?
A) it is one of the nuclei of the oculomotor nerve
B) it is a paired nucleus
C) it innervates the pupillary sphincter muscle
D) it innervates the ciliary muscle
NEU-6.669.
Which cranial nerve exits the medulla at its dorsal surface?
A) the oculomotor nerve
B) the abducent nerve
C) the trochlear nerve
D) the facial nerve
E) the trigeminal nerve
NEU-6.670.

The examination of headaches includes:


A) CT and skull X-ray studies
B) focus examination and psychological studies
C) provocative tests
D) a CSF sample is always taken
E) WBC count, RBC sedimentation rate
F) all of the above
G) only (A), (B), (C), and (D) are correct
H) only (A), (B), (C), and (E) are correct
I ) only (B), (C), (D), and (E) are correct
NEU-6.671.
A patient is considered as epileptic if:
A) an epileptic seizure can be proved
B) a patient had the seizure following a head trauma
C) a patient had a seizure and epilepsy was noted in the family
history
D) a patient has systematic and EEG verified seizures
E) all of the above
F) only (A) and (B) are correct
G) only (C) and (D) are correct
NEU-6.672.
Which speech disorder cannot develop if the nuclei of the lower
cranial nerves are damaged?
A) aphonia
B) anarthria
C) aphasia
D) dysarthria
NEU-6.673.
Which spinal segment should remain intact for the patellar reflex?
A) S 1
B) L4
C) S2
D) L5 and S1
NEU-6.674.
Pseudobulbar syndrome is caused by:
A) bilateral damage of the nuclei of the lower cranial nerves
B) bilateral damage of the lower cranial nerves
C) bilateral damage of the pyramidal tracts projecting towards the
nuclei of the lower cranial nerves
D) ipsilateral damage of the pyramidal tracts projecting towards
the nuclei of the lower cranial nerves
NEU-6.675.
Damage of which structure does not cause muscle hypotonia?
A) a peripheral nerve
B) the cerebellum
C) the thalamus
D) the spinal ventral horns
NEU-6.676.
The distribution of spastic hypertonia includes:
A) flexors of the upper extremities and extensors of the lower

extremities
B) flexors of the upper and lower extremities
C) extensors of the upper and lower extremities
D) all groups of muscles
NEU-6.677.
Which type of a disturbance of speech occurs in cerebellar damage?
A) dysarthria
B) scanning speech
C) aphasia
D) anarthria
E) aphonia
NEU-6.678.
Which of the following symptoms does not occur in a cerebellar
lesion?
A) nystagmus
B) ataxia
C) intention tremor
D) tremor at rest
E) muscle hypotonia
F) a rebound effect
NEU-6.679.
Which of the following symptoms does not occur in cerebellar lesion?
A) intention tremor
B) scanning speech
C) aphasia
D) dysmetria
NEU-6.680.
The cause of intention tremor is a:
A) cerebellar lesion
B) extrapyramidal lesion
C) spinal lesion
D) psychic strain
NEU-6.681.
Which of the following symptoms does not occur in a lesion of the
pyramidal tract?
A) spastic hypertonia
B) hyperreflexia of the deep reflexes
C) pyramidal symptoms
D) fasciculation
NEU-6.682.
Which of the following symptoms does not belong to those of a
transverse lesion?
A) all sensory modalities are altered beneath the level of injury
B) spasticity or paraparesis of the lower extremities
C) urinary disorders
D) segmental deficiencies (lack or alteration of certain reflexes,
individual atrophy)
E) dissociated sensory disorders
NEU-6.683.

Which of the following does not occur in central motor neuron


damage?
A) global atrophy
B) hyperreflexia of the deep reflexes
C) individual atrophy
D) pyramidal signs
E) decreased surface reflexes
NEU-6.684.
Select a pyramidal sign for the lower extremities:
A) Mayer's reflex of the proximal joints
B) Juster's sign
C) Oppenheim's sign
D) Wartenberg's sign
NEU-6.685.
Select a pyramidal sign for the upper extremities:
A) Chaddock's sign
B) Rossolimo's sign
C) Tr6mner's sign
D) Schaefer's sign
NEU-6.686.
Which of the following is not a pyramidal sign?
A) Babinski's sign
B) Brudzinsky's sign
C) Rossolimo's sign
D) Chaddock's sign
E) Gordon's sign
F) Trmner's sign
NEU-6.687.
Select the definition of an increased deep reflex:
A) hyperreflexia
B) an enlarged reflexogenic zone
C) hyperreflexia accompanied with pyramidal signs
D) hyperreflexia accompanied with a grasping reflex
NEU-6.688.
Which of the following does not control the muscle tone?
A) the muscle spindle
B) the motor neuron of the ventral horn
C) the cerebellum
D) the spinothalamic tract
NEU-6.689.
Paralysis means:
A) cessation of muscle strength
B) decreased muscle strength
C) flaccid muscles
D) coordination disorders
E) decreased muscular mass
F) complete paraplegia
NEU-6.690.
Which of the following structures does the pyramidal tract NOT

CROSS?
A) the corona radiata
B) the genu and posterior area of the internal capsule
C) the cerebral peduncule
D) the tegment of the pons
E) the medulla oblongata
NEU-6.691.
The primary motor cortex:
A) is localized in the parietal lobe
B) is localized behind the central sulcus
C) is localized just before the central sulcus
D) is localized in the temporal lobe
NEU-6.692.
The primary motor cortex corresponds to:
A) Brodmann's area 44
B) Brodmann's area 3
C) Brodmann's area 4
D) Brodmann's area 41
NEU-6.693.
Which of the following does not belong to the conus syndrome?
A) pyramidal signs
B) autonomic disorders
C) sensory disorders
D) the motor function which usually remains intact
E) the conus is at the level of L1
NEU-6.694.
If a patient with a disc hernia suddenly develops paralysis and
autonomic symptom's, then:
A) the patient should be examined and operated within 6 weeks
B) the patient should be examined and operated within 1 month
C) bed rest, with the patient being examined and operated within
1 week
D) the patient should be examined and operated immediately
E) bed rest, with the patient being examined and operated within
6 weeks
NEU-6.695.
Which symptom reflects damage of the cauda?
A) pyramidal signs in the lower extremities
B) spastic paraparesis in the lower extremities
C) autonomic symptoms
D) clonus of the foot
NEU-6.696.
Which symptom does not belong to the cauda syndrome?
A) flaccid paralysis
B) hyporeflexia
C) pyramidal signs
D) assymmetric symptoms
NEU-6.697.
Which of the following does not cause paraparesis of the lower

extremities?
A) parasagittal processes
B) a pontine focus
C) any spinal damage
D) a lesion of the midbrain tegmentum
NEU-6.698.
A typical representative of hypertonic hypokinetic disorders includes:
A) Alzheimer's disease
B) Pick's lobe atrophy
C) Korsakoffs syndrome
D) parkinsonism
NEU-6.699.
A typical representative of hypotonic hyperkinetic disorders includes:
A) Wernicke's encephalopathy
B) spastic torticollis
C) Huntington's chorea
D) Foster-Kennedy's syndrome
NEU-6.700.
Which of the following is not typical of chorea?
A) decreased muscle tone
B) involuntary movements in the distal parts of the lower extremities
C) involuntary movements of the tongue
D) pyramidal signs
NEU-6.701.
Which of the following is not typical of polyneuropathy?
A) decreased deep reflexes
B) autonomic symptoms
C) pyramidal signs
D) sensory disorders
E) paralysis is common
NEU-6.702.
Cerebellar hemorrhage:
A) is always fatal
B) surgery can be performed depending on the size of the
hemorrhage
C) should always be treated conservatively
D) is easy to diagnose because the CSF is always hemorrhagic in
these cases
NEU-6.703.
Which area is damaged if the patient has left hemiparesis and weakness
while looking to the right?
A) the right frontal lobe
B) the right side of the pons
C) the left side of the pons
D) the left frontal lobe
NEU-6.704.
Which of the following is not a symptom of cerebellar damage?
A) adiadochokinesis
B) muscle hypotonia

C) nystagmus
D) scanning speech
E) apraxia
NEU-6.705.
Damage of which area does not cause hemiparesis?
A) damage of the frontal lobe
B) damage of the parietal cortex
C) damage of the internal capsule
D) damage of the cervical spinal cord
NEU-6.706.
Which of the following is not an exteroreflex?
A) the plantar reflex
B) the abdominal cutaneous reflex
C) the conjunctival reflex
D) the cremaster reflex
E) the masseter reflex
NEU-6.707.
Which of the following does not belong to the cerebellum?
A) declive
B) folium
C) operculum
D) uvula
E) flocculus
NEU-6.708.
Which statement is not true for the therapy of epilepsy?
A) the importance of carbamazepine therapy has grown
B) monotherapy is a method of choice
C) therapeutic surgery should be considered in drug-resistant
epilepsy
D) the drug blood level evaluation is of growing importance
E) the EEG changes are the most important signs for therapy
F) certain antiepileptic drugs interact, decreasing each other's
efficiency and increasing toxicity
NEU-6.709.
The incidence of epilepsy is:
A) 0.04-0.05
B) 0.1-0.2%
C) 0.5-1%
D) 5-6%
E) 10-12%
F) 8-10%
NEU-6.710.
Which of the following cannot develop after a lesion of the extrapyramidal
tract?
A) ballismus
B) athetosis
C) Huntington's chorea
D) trismus
E) torsion dystonia

NEU-6.711.
Which of the following is not typical of a peripheral motor neuronal
lesion?
A) individual paralysis
B) individual atrophy
C) decreased deep reflexes
D) clonus of the foot
E) fasciculation
NEU-6.712.
Which of the following is not a deep reflex?
A) the conjunctival reflex
B) the masseter reflex
C) the ulnar reflex
D) the biceps reflex
NEU-6.713.
Which reflex has its receptors in the corresponding muscle?
A) the abdominal cutaneous reflex
B) the grasping reflex
C) the mucosal reflexes
D) the plantar reflex
E) the cremaster reflex
NEU-6.714.
Which of the following is not considered as hyperkinesis?
A) ballism
B) intention tremor
C) chorea
D) athetosis
E) myoclonus
NEU-6.715.
Which of the following is not a cerebellar nucleus?
A) the dentate nucleus
B) the emboliform nucleus
C) the fastigial nucleus
D) the nucleus ambiguous
NEU-6.716.
Which of the following is not a cerebellar symptom?
A) dysdiadochokinesis
B) muscle hypotonia
C) a positive Romberg's test
D) anosognosia
NEU-6.717.
Which of the following is not a sign of psychic deterioration?
A) oral exploration
B) a tendency to pronate the distal extremities
C) a grasping reflex
D) a loss of initiation
NEU-6.718.
Which of the following is the cause of Huntington's chorea?
A) a damage of the pallidum

B) a damage of the amygdala


C) a damage of the subthalamic nucleus
D) a damage of the neostriatum
NEU-6.719.
Where should the damaging focus be localized in order to elicit a
central-type paralysis in the upper extremities?
A) a parasaggital localization
B) the precentral gyrus is damaged over the Sylvian fissure
C) the postcentral gyrus is damaged over the Sylvian fissure
D) a damage of the premotor cortex
NEU-6.720.
Which of the following is not a coordination disorder?
A) ataxia
B) rebound phenomenon
C) adiadochokinesis
D) intention tremor
E) nystagmus
NEU-6.721.
Damage of which structure causes hemiballism?
A) the striatum
B) the cerebellum
C) the subthalamic nucleus
D) the putamen
NEU-6.722.
In which of the following do the deep reflexes remain unchanged?
A) damage of a peripheral nerve
B) damage of the motor neuron of the ventral horn
C) damage of the parietal lobe
D) cerebellar damage
E) damage of the occipital lobe
NEU-6.723.
Which of the following is not typical of dysdiadochokinesis?
A) impaired alternative movements
B) it is one of the forms of coordination disorders
C) it is one of the symptoms of a cerebellar lesion
D) decreased muscle force
NEU-6.724.
In which muscle groups is hemiparesis of the upper extremeties
more pronounced?
A) the flexors of the elbow
B) the extensors of the elbow
C) the abductors
NEU-6.725.
Assign Parkinson's disease to one of the following:
A) a hypertonic hypokinetic disorder
B) a hypotonic hyperkinetic disorder
C) a hypotonic hypokinetic disorder
D) a hypertonic hyperkinetic disorder

NEU-6.726.
Which of the following does not cause tetraparesis?
A) extensive damage to the basis of the pons
B) parasaggital processes
C) a transverse spinal lesion
NEU-6.727.
Which of the following is not characteristic of a pyramidal tract lesion?
A) hyporeflexia of the surface reflexes
B) spastic hypertonia
C) Achilles clonus
D) patellar clonus
E) myoclonus in the muscles of the upper extremities
NEU-6.728.
Which spinal segment is required for the Achilles reflex?
A) I
B) L3
C) S 1
D) L2
NEU 6.729.
Which of the following is not a pyramidal sign of the upper extremities?
A) Hoffman's sign
B) Oppenheim's sign
C) Tr6mner's sign
D) Juster's sign
NEU-6.730.
A lesion of which of the following structures leads to the development
of a rebound phenomenon?
A) the cerebellum
B) the putamen
C) the caudate nucleus
D) the striatum
NEU-6.731.
In which of the following does cogwheel rigidity develop?
A) Foster-Kennedy's syndrome
B) Parkinson's disease
C) Wernicke's encephalopathy
D) Marchiafava-Bignami's, disease
NEU-6.732.
In which of the following are the abdominal skin reflexes not decreased?
A) a lesion of the pyramidal tract
B) a lesion of the spinal Th7-Th12 segments
C) in damage to the lumbar spinal cord
D) in obesity
NEU-6.733.
Which of the following is not a central reflex?
A) the sucking reflex
B) the mediopubian reflex
C) the glabellar reflex
D) the masseter reflex

NEU-6.734.
At what level does the spinal cord end?
A) L2
B) L5
C) S 1
D) S2
NEU-6.735.
Which symptom is not typical of a radial nerve lesion?
A) atrophy of the thenar muscles
B) a "wrist drop"
C) decreased synergic dorsal flexion of the wrist
D) pronation of the forearm
NEU-6.736.
A "cerebellar fit" is caused by:
A) an elevated intracranial pressure
B) a congenital cerebellar anomaly
C) alcohol-induced cerebellar atrophy
D) vertobrobasilar insufficiency
NEU-6.737.
Which of the following symptoms does not occur in a thalamic lesion?
A) thalamic dementia
B) thalamic fever
C) thalamic hand
D) thalamic hyperkinesis
E) thalamic pain
NEU-6.738.
Which of the following is not a symptom of a cerebellar lesion?
A) muscle hypotonia
B) dysdiadochokinesis
C) nystagmus
D) agraphia
NEU-6.739.
Which of the following symptoms is not typical of a hypertonichypokinetic
disorder?
A) rigor
B) tremor
C) akinesis
D) chorea
NEU-6.740.
Which of the following is not characteristic of a lesion of the upper brachial
plexus?
A) Horner's triad
B) abduction weakness in the forearm
C) outward rotation weakness in the forearm
D) the forearm hangs if rotated inward
NEU-6.741.
How is Oppenheim's reflex elicited?
A) by pressing the Achilles tendon

B) by pressing the triceps muscle of the calf


C) by drawing a forger along the edge of the tibia
D) by drawing a pointless object along the lateral edge of the
upper surface of the foot
NEU-6.742.
Which of the following is not typical of Lrfs reflex?
A) an asymmetric reflex indicates a lesion of the pyramidal tract
B) bilateral hyperreflexia indicates a lesion of the pyramidal tract
C) it is a negative supporting reflex
NEU-6.743.
The phases of disorientation are:
A) somnolence-stupor-coma
B) stupor-somnolence-coma
C) coma-stupor-somnolence
NEU-6.744.
Which type of sensory disorders develop in a lesion of the spinotlialamic
tract?
A) surface sensory disorders
B) paresthesia
C) deep sensory disorders
D) graphesthesia disorders
NEU-6.745.
Which of the following is typical of multiple sclerosis?
A) the vision is usually involved
B) diplopia is common
C) a lack of abdominal skin reflexes
D) intention tremor
E) rigid hypertonia
F) only (A), (B), (C), and (D) are true
G) only (A), (B), and (E) are true
H) only (B), (C), (D), and (E) are true
NEU-6.746.
Which of the following is not characteristic of the anatomy of the
sensory system?
A) it is comprised of three neurons
B) the sensory tracts cross in the pons
C) stimuli are projected to the thalamus
D) stimuli are projected to the sensory cortex
NEU-6.747.
The cause of deep sensation disorders is:
A) a lesion of the spinal ventral horn
B) a lesion of the spinal posterior column
C) a lesion of the area around the central canal
D) a lesion of the ventral root
NEU-6.748.
A lesion of which of the following nerves is typical of a "stepper's
gait"?
A) the tibial nerve
B) the peroneal nerve

C) the femoral nerve


D) the sural nerve
NEU-6.749.
Where is the primary sensory cortex located?
A) in the frontal lobe
B) in the temporal lobe
C) in the parietal lobe
D) in the prefrontal area
NEU-6.750.
Damage to which area does not cause impaired two-point touch
discrimination?
A) a lesion of the spinal dorsal fascicle
B) damage to the dorsal root
C) damage to the medial lemniscus
D) damage to the spinothalamic tract
NEU-6.751.
Where is the primary sensory cortex localized?
A) just behind the central sulcus
B) just before the central sulcus
C) at the opercular area
D) in the temporal lobe
NEU-6.752.
Which of the following is not a subjective sensory disorder?
A) paresthesia
B) spontaneous pain
C) numbness
D) an impaired vibration sensation
NEU-6.753.
Where does the spinothalamic tract cross?
A) in the spinal cord
B) in the medulla
C) in the pons
D) in the mesencephalon
NEU-6.754.
Which of the following is not a supporting tissue?
A) Schwann-cells
B) astrocytes
C) oligodendrocytes
D) microglia
NEU-6.755.
Which of the following is typical of cervical spondylotic headaches?
A) they are usually ipsilateral
B) they are usually occipital and/or the pain radiates forward
C) there are pathological findings on the X-ray of the neck
D) they can be caused by abnormal (pathological) positions of the
head
E) all of the above
F) none of the above

NEU-6.756.
Which of the following symptoms does not occur in lesions of the
medial lemniscus?
A) an impaired deep sensation of the trunk
B) an impaired deep sensation of the extremities
C) an impaired deep sensation of the face
NEU-6.757.
Which of the following can be used for migraine therapy?
A) hydergine (Imigran)
B) Aspirin
C) dihydroergotamine
D) all of the above
NEU-6.758.
A dissociated sensory disorder of the syringomyelic-type is caused by:
A) a damage of the spinal dorsal fascicle
B) damaging foci located in the central part of the spinal cord
C) ipsilateral spinal damage
D) occlusion of the posterior spinal arteries
NEU-6.759.
Impaired deep sensation is caused by:
A) damage to the ventral root
B) damage to the area around the central canal
C) damage to the ventral horn
D) damage to the dorsal column
NEU-6.760.
How is Chaddock's reflex elicited?
A) by pressing the Achilles tendon
B) by pressing the triceps surae muscle of the calf
C) by drawing a finger along the edge of the tibia
D) by drawing a pointless object along the lateral edge of the
upper surface of foot
NEU-6.761.
How is Gordon's reflex elicited?
A) by pressing the Achilles tendon
B) by pressing the triceps surae muscle of the calf
C) by drawing a finger along the edge of the tibia
D) by drawing a pointless object along the lateral edge of the
upper surface of foot
NEU-6.762.
Side effects of phenytoin therapy include:
A) gingival hyperplasia
B) anemia
C) alterations in the bones
D) enlarged lymph nodes
E) sleep disorders
F) all of the above
G) only (A), (B), and (E) are true
H) only (A), (C), and (E) are true
I) only (A), (B), (C) and (D) are true

NEU-6.763.
The daily dose of phenytoin is:
A) 0.03-0.07 mg/kg
B) 3-7 mg/kg
C) 30-70 mg/kg
D) 100-150 mg/kg
NEU-6.764.
Side-effects of carbamazepine therapy include:
A) allergy
B) leukopenia
C) polycythemia
D) liver disorders
E) all of the above
F) only (A), (B), and (D) are true
G) only (A), (B), and (C) are true
H) only (B), (C), and (D) are true
NEU-6.765.
Which of the following is not a pyramidal sign?
A) hard-palate reflex
B) clonus of the foot
C) tonic dorsal flexion of the big toe
D) corneal hyperreflexia
NEU-6.766.
The daily dose of carbamazepine is:
A) 0.01-0.02 mg/kg
B) 0.1-0.2 mg/kg
C) 10-20 mg/kg
D) 100-200 mg/kg
E) 500-1000 mg/kg
NEU-6.767.
Which of the following is not a complication of chronic alcoholism?
A) superior hemorrhagic polyencephalitis
B) a lesion of the optic nerves
C) central pontine myelinolysis
D) syringomyelia
NEU-6.768.
The therapy of status epilepticus is:
A) phenytoin (Diphedan) iv.
B) diazepam iv.
C) clonazepam iv.
D) disulfiram
E) all of the above
F) only (B), (C), and (D) are true
G) only (C) and (D) are true
H) only (A), (B), and (C) are true
NEU-6.769.
The medication of primary choice in a grand mal attack is:
A) phenytoin (Diphedan)
B) carbamazepine
C) dipropylacetate
D) nitrazepam

E) only (A), (B), and (C) are true


F) only (A) and (D) are true
G) only (C) and (D) are true
H) all of the above
NEU-6.770.
Ophthalmoplegia does not belong to one of the following syndromes:
A) Gradenigo's syndrome
B) Nothnagel's syndrome
C) Millard-Gubler's syndrome
D) Benedict's syndrome
NEU-6.771.
Which of the following does not belong to the visual system?
A) the lateral geniculate body
B) the striate area
C) the medial geniculate body
D) Brodmann's area 17
NEU-6.772.
Where is the nucleus of the trochlear nerve localized?
A) in the mesencephalon (in the periaqueuctal grey matter)
B) in the pontine tegmentum
C) in the basis of the mesencephalon
D) in the basis of the pons
NEU-6.773.
Which symptom does not occur in a lesion of the frontal lobe?
A) epilepsy
B) motor aphasia
C) sensory aphasia
D) a low level of motivation
E) disorders of gaze
F) signs of psychic deterioration
NEU-6.774.
Which of the following is not a frontal lobe symptom?
A) motor paralysis
B) a dreamy state
C) apathy
D) signs of psychic deterioration
NEU-6.775.
Which symptom does not occur in a lesion of the parietal lobe?
A) contralateral hemianopsia in the lower quadrant
B) apraxia
C) motor aphasia
D) acalculia
E) amnestic aphasia
NEU-6.776.
Which symptom does not occur in a lesion of the frontal lobe?
A) amnestic aphasia
B) a sensory disorder
C) finger agnosia
D) astereognosia

E) optic agnosia
NEU-6.777.
Which symptom does not belong to Gerstmann's syndrome?
A) finger agnosia
B) acalculia
C) right-left disorientation
D) agraphia
E) astereognosia
NEU-6.778.
Which of the following cannot develop as a consequence of alcoholism?
A), polyneuropathy
B) cerebral atrophy
C) Wernicke's encephalopathy
D) central pontine myelinolysis
E) Foster-Kennedy's syndrome
F) Marchiafava-Bignami's syndrome
NEU-6.779.
Which symptom does not occur in a lesion of the temporal lobe?
A) psychomotor attacks
B) hemianopsia in the contralateral quadrant
C) uncinate attacks
D) amnestic aphasia
E) sensory aphasia
NEU-6.780.
Which symptom does not occur in a lesion of the temporal lobe?
A) astereognosia
B) olfactory sensations
C) vestibular sensations
D) gustatory sensations
NEU-6.781.
Aphasia develops in:
A) a lesion of the supranuclear motor neurons
B) disorders of peripheral innervation
C) a lesion of Brodmann's area 44
D) a lesion of Brodmann's area 1
NEU-6.782.
The causes of aphasia include:
A) damage to the nuclei of the lower cranial nerves
B) damage to the lower cranial nerves
C) a cerebellar lesion
D) a lesion of the dominant hemisphere
NEU-6.783.
Which of the following does not enhance cerebral blood flow under
normal conditions?
A) an increased systemic pressure
B) an increased PCO2
C) a decreased P02
D) an increased level of EDRF (endothelium derived relaxing
factor)

NEU-6.784.
Which symptom does not occur in a lesion of the premotor
cortex?
A) a gaze disorder
B) a release effect
C) a grasping reflex
D) a sucking effect
E) moria (a morbid tendency to joke)
NEU-6.785.
Define the term agnosia:
A) the rejection of paralysis
B) a disorder of speech
C) psychic deterioration
D) a disorder of the recognition of objects and symbols
E) forgetting new information
F) a failure of the aquisition of new information
NEU-6.786.
Conduction aphasia occurs in:
A) a lesion of the insula
B) a lesion of the operculum
C) a lesion of Wernicke's area
D) a lesion of the parietal lobe
NEU-6.787.
Damage to which area causes "moria" (a morbid tendency to joke)?
A) the basal area of the prefrontal lobe
B) the dorsal area of the prefrontal lobe
C) the basal area of the premotor lobe
D) the dorsal area of the premotor lobe
NEU-6.788.
Which of the following does not increase cerebral blood flow?
A) a decrease of pH
B) an increase of PCO2
C) a decrease of p02
D) a high blood glucose level
NEU-6.789.
Which of the following does not belong to the pathological
photostimulation response?
A) increased synchronization
B) photomyoclonus
C) photoconvulsions
D) desynchronization
NEU-6.790.
The normal action potential of muscle is:
A) monophasic
B) bi- or triphasic
C) always biphasic
D) always triphasic
E) usually polyphasic

NEU-6.791.
Which of the following does not occur in atherosclerotic encephalopathy?
A) pseudoneurasthenic symptoms
B) pseudobulbar paralysis
C) parkinsonian symptoms
D) congested papillae
NEU-6.792.
Which of the following 'is not characteristic of multi-infarct dementia?
A) continous slow psychic deterioration
B) a lack of neurologic symptoms
C) gnostic disorders
D) aphasia
NEU-6.793.
The blood-brain barrier is not functioning if the serum albumin/
CSF albumin ratio is:
A) less than 100
B) higher than 100
C) higher than 1000
D) higher than 5000
E) higher than 10000
NEU-6.794.
Hemiparalysis alternans (Avellis syndrome) can develop in:
A) a medullary lesion
B) a lesion at the pontomesencephalic border
C) a lesion of the medial part of the pons
D) a lesion of the mesencephalon
NEU-6.795.
Which syndrome develops in a pontine lesion?
A) an ipsilateral oculomotor lesion and contralateral
hemiparalysis
B) an ipsilateral oculomotor lesion and contralateral hyperkinesis
C) an ipsilateral gaze disorder with vertical nystagmus
D) an upward gaze disorder with vertical nystagmus
NEU-6.796.
Which pathological processes cannot be visualized by a CT?
A) identification of an infarction during the early hours
B) identification of any demyelinization disorders
C) identification of abscesses
D) identification of hemorrhages
NEU-6.797.
Which of the following is not applicable in myelography?
A) the intravenous administration of contrast substance
B) the intrathecal administration of contrast substance
C) the intrathecal administration of air
NEU-6.798.
A simple spinal X-ray image cannot reveal:
A) any degenerative alteration of the spine
B) any osteolytic vertebral metastases
C) any osteoplastic vertebral metastases

D) any intramedullary processes


NEU-6.799.
A Doppler study can be used for the identification of.
A) the cause of cerebral circulatory disorders
B) the diagnosis of any space-occupying process in the brain
C) Marchiafava-Bignami's disease
D) Foster-Kennedy's syndrome
NEU-6.800.
Lhermitte's visions occur in:
A) peduncular injuries
B) injuries of the longitudinal medial fasciculus
C) emotional strain
D) injuries of the occipital lobe
NEU-6.801.
Which of the following is not characteristic of an uncinate attack?
A) Duffs symptom
B) a dreamy state
C) dja vu sensation
D) an olfactory sensation
E) seizures in the extremities
F) oral automatism
NEU-6.802.
Occlusion of which artery elicits ipsilateral damage of the hypoglossal
and contralateral damage of the pyramidal tract?
A) the basilar artery
B) the anterior spinal artery
C) the vertebral artery
D) the superior cerebellar artery
NEU-6.803.
Which of the following is not included in the caudal pontine syndrome?
A) alternating oculomotor hemiparalysis
B) alternating abducent hemiparalysis
C) alternating facial hemiparalysis
NEU-6.804.
Which of the following is commonly used in myasthenia gravis?
A) domperidone (Motilium)
B) papaverine (Meristin)
C) pyridostigmine (Mestinon)
D) mexiletine (Mexitil)
E) moroxydine hydrochloride (Morgalin)

NEU-6.805.
Which structure does not secrete CSF?
A) Pacchioni's granulation
B) the chorioid plexus
C) the ependyma
NEU-6.806.
A congested fundus:

A) always indicates a brain tumor


B) is frequently caused by inflammation
C) is common in multiple sclerosis
D) frequently occurs in brain tumors, but can also accompany
other processes (for example obscure encephalopathies or
lymphostatic encephalopathies)
E) is always accompanied by severely impaired vision
NEU-6.807.
The normal cell count in the lumbar CSF is:
A) 20-30/mm3
B) 30-40/mm3
C) 40-50/mm3
D) maximum 8-10/mm3
NEU-6.808.
Which pathologic process is not detectable with a simple skull
X-ray?
A) progressive staging of intrasellar space-occupying processes
B) tumors showing calcification
C) a cranial fracture
D) cerebral atrophy
NEU-6.809.
Cerebral angiography cannot be used for the detection of:
A) vascular occlusions
B) malformations
C) the cause of cranial polyneuropathy
D) a subdural hematoma
NEU-6.810.
Which frequency range corresponds to the theta-waves?
A) 5-7 cycles/sec
B) 1-4 cycles/sec
C) 8-12 cycles/sec
D) 15-20 cycles/sec
NEU-6.811.
The EMG is used for the following purpose:
A) a study of the electric activity of the brain
B) evoked potentials due to sensory stimuli
C) the state of the muscles, activity of the motor neurons and
peripheral nerves
D) only intracellular leads are used in clinical practice
NEU-6.812.
The membrane potential of normal healthy muscle fibers is:
A) 80-100 mV
B) not detectable
C) 8-10 mV
D) 0.8-1 mV
NEU-6.813.
Which of the following CT images shows no hyperdensity?
A) a subdural hematoma
B) Alzheimer's disease

C) an intracerebral hemorrhage
D) an epidural hemorrhage
E) melanoma
NEU-6.814.
What does a hyperdense ring surrounding a hyperdense area mean
on a CT image?
A) a fresh insult
B) circumscribed cerebral atrophy
C) an abscess
D) a primary brain tumor
NEU-6.815.
Which of the following is the most common complication of chronic
alcoholism?
A) polyneuropathy
B) Marchiafava-Bignami disease
C) central pontine myelinolysis
D) cerebellar vermis atrophy
E) alcoholic hallucinosis
NEU-6.816.
Which structure is primarily damaged in Alzheimer's disease?
A) the subcortical white matter
B) cortical neurons
C) the white matter of the brain stem
D) the mamillary body
E) Alzheimer's fibrils
NEU-6.817.
Which symptom is not typical of delirium tremens?
A) tremor
B) increased vegetative symptoms
C) visual and tactile hallucinations
D) olfactory hallucinations
NEU-6.818.
Compression-derived CSF is found in:
A) intracranial space-occupying processes
B) spinal space-occupying processes
C) intrasellar processes
D) Wernicke's encephalopathy
E) herpes zoster
NEU-6.819.
An epileptic patient can drive a car if:
A) the drug level is properly set
B) if the patient is not a professional driver and has attacks only
at night
C) if the patient experienced no attacks for 6 weeks and the
epilepsy is not due to a progressive disease
D) if the patient does not take drugs and is attack-free for at least
2 years and epilepsy is not a sign of a progressive disease
E) if the patient takes drugs and has no attacks
F) if the patients takes drugs and has fits without unconsciousness

NEU-6.820.
Which vessels are primarily damaged in hypertension?
A) the arteries of the circle of Willis
B) arteries of middle caliber
C) capillaries
D) arterioles
E) the collaterals
NEU-6.821.
During which period of the day is thrombosis of the cerebral arteries
manifested for the first time?
A) any time
B) at day time during physical exercise
C) at dawn
D) when the patient goes to bed
NEU-6.822.
Damage of which lobe causes astereognosia?
A) the frontal lobe
B) the temporal lobe
C) the parietal lobe
D) the occipital lobe
NEU-6.823.
Damage of which lobe causes sensory aphasia?
A) the frontal lobe
B) the temporal lobe
C) the parietal lobe
D) the occipital lobe
NEU-6.824.
Which of the following is typical of borreliosis?
A) it can be accompanied by arthralgia and cutaneous symptoms
B) it can be followed by a bilateral paralysis of the peripheral
facial nerve
C) the CSF content is always normal
D) it is caused by a virus
E) only (A), (B), and (D) are true
F) only (A) and (B) are true
NEU-6.825.
How is cerebral hemorrhage differentiated from "brain softening"?
A) by the clinical symptoms
B) by the course of the disease
C) with the EEG
D) by the case history
E) with a CT study
NEU-6.826.
What is the relevance of calcium ions in ischemic brain softening?
A) calcium protects the parenchymal cells
B) calcium protects the filial cells
C) an enhanced influx of extracellular calcium activates catabolic
processes
D) the outflow of calcium from the cell impedes normal propagation
of the stimuli

E) calcium plays no role in this alteration


NEU-6.827.
Which of the following plays no role in the diagnosis of cerebral
hemorrhage?
A) the taking of blood pressure
B) an EEG study
C) a percussion of cardiac dullness
D) the case history
E) the determination of the immunoglobulin level
NEU-6.828.
Which of the following causes brain atrophy?
A) perinatal damage
B) alcoholism
C) arteriosclerosis
D) all of the above
E) none of the above
NEU-6.829.
Which of the following does not cause polyneuropathy?
A) alcoholism
B) diabetes mellitus
C) uremia
D) impaired absorption
E) parkinsonism
NEU-6.830.
When is the CSF protein content not increased?
A) in an epileptiform seizure
B) in a brain tumor
C) in a spinal tumor
D) in damage to the spinal roots
E) in multiple sclerosis
NEU-6.831.
Which of the following studies should be primarily performed if a
brain tumor is suspected?
A) CT
B) angiography
C) myelography
D) electromyography
E) transcranial Doppler
NEU-6.832.
Posttraumatic epilepsy frequently occurs in:
A) damage to the parietal lobe
B) damage to the occipital lobe
C) cerebral commotion
D) damage to the frontal lobe
E) a contusion of the temporal lobe
NEU-6.833.
Damage of which of the following lobes is accompanied by a dreamy
state?
A) the frontal lobe

B) the temporal lobe


C) the parietal lobe
D) the occipital lobe
NEU-6.834.
From the following select the typical EEG findings of a non-sleeping
subject at rest lying with his eyes closed?
A) occipital and parietal delta activity
B) occipital and parietal alpha activity
C) occipital and parietal theta activity
D) occipital and parietal gamma activity
NEU-6.835.
Which of the following is used to study the electrical activity of the
brain?
A) electromyography
B) electromyelography
C) electroencephalography
D) electroneuronography
NEU-6.836.
In which disease is the CSF measles antibody titer always high?
A) Hurst encephalitis
B) subacute sclerotizing panencephalitis
C) diffuse periaxial encephalitis
D) concentric periaxial encephalitis
NEU-6.837.
Which study is unnecessary in dementia?
A) EEG
B) EMG
C) determination of the serum fat level
D) IQ test
E) CT
NEU-6.838.
Where is the CSF localized?
A) the subdural space
B) the subarachnoid space .
C) under the pia mater
D) above the arachnoid
NEU-6.839.
Which of the following studies is used to check medicated epileptic patients?
A) the CSF and a blood smear
B) a blood smear and CT (systematically)
C) the blood level of the drug and a systematic CT
D) the blood level of the drug and a blood smear
E) the blood level of the drug and urinalysis
NEU-6.840.
In which disease is the CSF gamma globulin level not elevated?
A) myasthenia gravis
B) Guillain-Barr's syndrome
C) subacute sclerotizing panencephalitis
D) multiple sclerosis

NEU-6.841.
Which symptom does not occur in a lesion of the prefrontal lobe?
A) slow psychomotility
B) indifference
C) left-right disorientation
D) frontal akinesia
E) hypokinesia
NEU-6.842.
Select the true statement:
A) Broca's aphasia means misunderstanding of the pronounced
words and the structure of the sentence
B) Broca's aphasia develops after a lesion of Brodmann's area 8
C) Broca's aphasia develops after a lesion of Brodmann's area 41
D) patients with Broca's aphasia talk excessively
E) Broca's aphasia develops after a lesion of Brodmann's area 40
and an accompanying damage of the motor component of
speech has occured
NEU-6.843.
Which of thefollowing is typical of amnestic aphasia?
A) in right-handed patients amnestic aphasia is induced by a
focus in the parietal lobe
B) amnestic aphasia is due to an insular focus
C) the patient can name objects properly
D) amnestic aphasia is caused by a focus in Brodmann's area 44
E) amnestic aphasia is caused by a focus in Brodmann's area 41
NEU-6.844.
In the acute phase of purulent meningitis:
A) the lymphocyte count in the CSF is high
B) the leukocyte count in the CSF is high
C) the eosinophil count in the CSF is high
NEU-6.845.
A transcranial Doppler study cannot be used:
A) for the identification of intracranial collaterals
B) for the identification of stenosis of the intracranial arteries
C) for the identification of intracranial tumors
D) for the identification of any occlusions in the intracranial
arteries
NEU-6.846.
Motor aphasia is caused by:
A) a lesion of the anterior part of the third frontal gyrus
B) a lesion of the posterior part of the third frontal gyrus
C) a lesion of the anterior part of the first frontal gyrus
D) a lesion of the posterior part of the first frontal gyrus
E) a lesion of the posterior part of the second frontal gyrus
NEU-6.847.
Sensory aphasia is caused by:
A) a lesion of Brodmann's area 40
B) a lesion of Brodmann s area 39
C) a lesion of Brodmann's area 41

D) a lesion of Brodmann's area 44


NEU-6.848.
Which of the following is not typical of a compression CSF syndrome?
A) a water clear CSF in several findings
B) a marked increase in the cell count
C) a marked elevation of the total protein content
D) a low glucose concentration
NEU-6.849.
Which area is damaged in Wallenberg's syndrome?
A) the dorsolateral region of the medulla
B) the dorsolateral region of the pontomedullary border
C) the tegmentum of the pons
NEU-6.850.
Which of the following is not typical of alcoholic polyneuropathy?
A) the symptoms are usually symmetric
B) the symptoms are more pronounced in the distal parts of the
limbs
C) the symptoms first occur in the lower limbs
D) micturition disorders develop
NEU-6.851.
The author(s) who first described aphasia was:
A) Lhermitte
B) Wernicke
C) Broca
D) Ramon and Cajal
NEU-6.852.
The alternating syndrome develops in:
A) a cerebellar lesion
B) an extrapyramidal lesion
C) damage to the brain stem
D) a lesion of the frontal lobe
NEU-6.853.
Disturbance of upward gaze accompanied by vertical nystagmus
suggests a pathological process of the following localization:
A) a cerebellar lesion
B) damage of the corpora quadrigemina (sup.and inf. colliculus)
C) a lesion of the corpus callosum
D) a lesion of the medulla
NEU-6.854.
Conjugated deviation is a:
A) excitation symptom
B) functional loss
C) all of the above
D) none of the above
NEU-6.855.
Which of the following is caused by the excitation of the sensory
cortex?
A) paresthesia

B) hyperesthesia
C) hyperalgesia
NEU-6.856.
Which of the following describes the prognosis of Parkinsonism?
A) Parkinsonism usually has a good prognosis
B) the disease has a progressive character even despite effective
medication
C) a good prognosis of patients susceptible to therapy can be
maintained for decades
D) only the prognosis of patients resistant to therapy is poor
NEU-6.857.
In which disease is the corpus callosum damaged?
A) Encephalitis Lethargica
B) Wernicke's encephalopathy
C) Marchiafava-Bignami disease
D) Korsakoffs syndrome
NEU-6.858.
Which drug is used for the treatment of parkinsonism?
A) amantadine
B) aminopyrine
C) allopurinol
D) phenytoin
E) buformin
NEU-6.859.
The starting Madopar (L-Dopa + benserazide) dose in parkinsonism
is:
A) 0.06-0.1 mg
B) 0.6-1 mg
C) 60-100 mg
D) 0.6-1 g
E) 1-6 g
NEU-6.860.
In which pathological process do the spinal dorsal funiculi still
remain intact?
A) in occlusion of the anterior spinal artery
B) in occlusion of the posterior spinal artery
C) in tabes dorsalis
D) in funicular myelosis
NEU-6.861.
Hemiparalysis with primary lower limb involvement and pyramidal
symptoms develop after:
A) an occlusion of the anterior cerebral artery
B) an occlusion of the middle cerebral artery
C) an occlusion of the posterior cerebral artery
NEU-6.862.
Pathological processes occurring around the foramen lacerum do
not damage the:
A) glossopharyngeal nerve
B) facial nerve

C) vagus nerve
D) accessory nerve
NEU-6.863.
Which tumor can cause Parinaud's syndrome?
A) an olfactory meningioma
B) a meningioma of the wings of the sphenoid bone
C) a pinealoma
D) a neuroma of the cerebello-pontine angle
NEU-6.864.
Where do the major part of the cerebellar efferent fibers end?
A) in the red nucleus
B) in the subthalamic nucleus
C) in the caudate nucleus
D) in the striatum
NEU-6.865.
Which of the following is not a part of the limbic system?
A) the cingulate gyrus
B) the mamillary bodies
C) the putamen
D) the amygdala
NEU-6.866.
Which of the following is not an excitation sign?
A) epilepsy
B) micropsia
C) a dreamy state
D) miosis
E) ansognosia
NEU-6.867.
Define the term anosognosia:
A) a body recognition disorder
B) a failure to recognize left-sided hemiparalysis
C) a spatial recognition disorder
D) a temporal recognition disorder
NEU-6.868.
Which area is damaged in Parkinson's disease?
A) the dentate nucleus
B) the substantia nigra
C) the subthalamic nucleus
D) the putamen
NEU-6.869.
Which of the following is typical of a cholinergic crisis?
A) sweating and salivation
B) enhanced intestinal function
C) both of the above
D) none of the above
NEU-6.870.
The inferior olive is localized:
A) in the medulla

B) in the pons --.. .


C) in the mesencephalon
D) at the pontomedullary border
MULTIPLE CHOICE QUESTIONS / TYPE I
Select the correct answers to the following questions!!!
...each qestion may have more than one correct answer.
NEU-6.871. Multiple Choice Question
The two tests best in revealing changes developing in
postvaccination encephalomyelitis are:
A) the peripheral blood cell count
B) the RBC sedimentation rate
C) a study of the acute and convalescent serum
D) the EEG
E) a CSF study
NEU-6.872.
Leukemic involvement of the nervous system means:
A) involvement of the meninges and the roots
B) the presence of leukemic cells in the CSF
C) the CSF glucose level is normal or elevated
D) that CSF pleocytosis is rare
E) none of the above
NEU-6.873.
The typical signs of a basal lesion of the frontal lobe include:
A) astereognosis
B) a positive grasping reflex
C) homonymous superior quadrantanopia
D) ipsilateral optic atrophy
E) anosmia
NEU-6.874.
Factors related to the development of a migraine headache include:
A) an age of over 70 years old
B) taking oral contraceptives
C) abdominal attacks in childhood
D) an intracerebral angioma
E) hypertension
NEU-6.875.
The cranial X-ray reveals pathologic calcification in:
A) vitamin B12 deficiency
B) a craniopharyngioma
C) hypoparathyroidism
D) Cushing's disease
E) a chronic subdural hematoma
NEU-6.876.
Which of the following is typical of postencephalitic parkinsonism?
A) oculogyric crises
B) signs of a hypothalamic lesion
C) somnolence
D) it develops earlier than idiopathic parkinsonism
E) a tremor is markedly expressed

NEU-6.877.
The typical causes of a predominantly motor peripheral neuropathy
include:
A) lead intoxication
B) diphteria
C) vitamin B12 deficiency
D) the leprosy which makes the skin scaly
E) Guillain-Barr's syndrome
NEU-6.878.
Ptosis occurs in:
A) Friedreich's ataxia
B) Wilson's disease
C) myotonic dystrophy
D) occlusion of the posterior inferior cerebellar artery
E) salbutamol therapy
NEU-6.879.
Symptoms characteristic of temporal epilepsy include:
A) 3/s spikes on the EEG
B) that it is more prone to medication than the other forms of
epilepsy
C) olfactory hallucinations
D) antisocial behavior and explosive reactions
E) that the seizures always start locally, affecting the same
groups of muscles and later spread to other muscles
NEU-6.880.
Cerebellar ataxia is caused by:
A) a motor neuron disease
B) chronic alcoholism
C) toxic doses of phenytoin therapy
D) a vitamin B12 deficiency
E) bronchial carcinoma
NEU-6.881.
Typical sequels of a cauda equina lesion include:
A) hyperreflexia in the lower limbs
B) impotence
C) a lack of abdominal reflexes
D) a sensory loss in the perineal area
E) a loss of the anal sphincter tone
NEU-6.882.
A lesion of the lateral medulla causes the following symptoms:
A) ipsilateral Horner's syndrome
B) a loss of contralateral reflexes
C) nystagmus
D) paralysis of the ipsilateral hypoglossal nerve
E) dysphagia
NEU-6.883.
Which of the following symptoms are typical of myasthenia gravis?
A) asymmetric weakness
B) it is accompanied by thyrotoxicosis

C) dysphagia
D) a loss of reflexes
E) spontaneous remission
NEU-6.884.
Which of the following is typical of facio-scapulo-humeral muscle
dystrophy?
A) it is inherited in an autosomal dominant fashion
B) the patients usually need a wheel chair by the age of 20
C) there is an early disappearance of the tendon reflexes
D) there are normal creatine phosphokinase values
E) typically winged shoulder-blades
NEU-6.885.
An elevated CSF protein and a decreased CSF glucose level occur in:
A) sarcoidosis
B) ECHO-viral meningitis
C) carcinomatous meningeal metastases
D) tuberculous meningitis
E) amoebal meningoencephalitis
MULTIPLE CHOICE QUESTIONS WITH KEY ANSWERS / TYPE II
Every question or incomplete statement has only one answer in the
following combinations:
if answers 1, 2, and 3 are true
B) if answers 1 and 3 are true
C) if answers 2 and 4 are true
D) if only answer 4 is true
E) if all the four answers are true
Select one of these key combinations!!!
NEU-6.886.
In the acute phase of a suspected cerebrovascular disease:
1) the CT shows no immediate changes if a patient has
ischemia
2) the CT shows no changes for about 48 hours if a patient has
had an infarction
3) the CT reveals immediate pathological signs in case of
hemorrhage
4) a radiolucent CT image indicates hemorrhagic areas
NEU-6.887.
If a cerebrovascular disease is suspected:
1) the CT is a method of choice in cases of an intracranial
hemorrhage
2) nearly all aneurysms can be visualized with a CT
3) the same CT image can usually visualize an old and a fresh
infarction
4) a CT-guided lumbar puncture is not obligatory in the diagnosis
of an intracranial hemorrhage
NEU-6.888.
Neurofibromatosis:
1 is a congenital disease with multiple spinal and cranial nerve
tumors and pigmentation of the skin
2) peripheral, intracranial, or intraspinal nerve tumors can

develop
3) tumors can develop at the sites of terminal branching of nerves
(plexiform neuroma)
4) a fully developed form is rather rare
NEU-6.889.
Causalgia:
1) usually is a common complication of all nerve lesions
2) usually elicits a burning sensation
3) usually develops after radial nerve lesions
4) usually affects the median and peroneal nerves
NEU-6.890.
In which disease does vertigo occur?
1) Foster-Kennedy's syndrome
2) trigeminal neuralgia
3) amyotrophic lateral sclerosis
4) Menire's disease
NEU-6.891.
Disorders of vertical gaze are caused by:
1) hippocampal lesions
2) lesions of the nucleus ambiguous
3) lesions of the subthalamic nucleus
4) lesions of the interstitial Cajal's nucleus
NEU-6.892.
With which structures is the medial longitudinal fascicle connected?
1) oculomotor nuclei
2) vestibular nuclei
3) cervical spinal segments
4) pontine and mesencephalic centers regulating gaze
NEU-6.893.
In Which diseases is the cochlear nerve damaged?
1) trauma
2) herpes zoster
3) salicylate-induced damages
4) uremia
NEU-6.894.
Which afferent tract is relayed in the thalamus?
1) the visual system
2) the spinothalamic tract
3) the lemniscus medialis
4) the olfactory system
NEU-6.895.
Which of the following can cause ptosis or proptosis?
1) Horner's syndrome
2) thrombosis of the cavernous sinus
3) myasthenia gravis
4) lesions of the midbrain tegmentum
NEU-6.896.
Which cranial nerves contain parasympathetic fibers?

1) the vagus nerve


2) the facial nerve
3) the glossopharyngeal nerve
4) the oculomotor nerve
NEU-6.897.
Where do the fibers of the lateral olfactory tract project?
1) to the amygdala
2) to the ventrolateral thalamic nucleus
3) to the hippocampal uncal gyrus
4) to the posterior thalamic nucleus
NEU-6.898.
Excitation of the olfactory system comprises:
1) anosmia
2) parosmia
3) cacosmia
4) olfactory hallucinations
NEU-6.899.
Factors contributing to the development of congested papilla include:
1) that an elevation of the CSF pressure is directly transmitted to
the CSF of the subarachnoid space surrounding the optic nerve
2) that an elevated CSF pressure disturbs venous circulation
3) edema of the optic fibers
4) a direct compression of the optic nerve
NEU-6.900.
The afferent fibers of the pupillary light reflex:
1) do not form synapses in the Edinger-Westphal nucleus
2) follow other optic fibers from the level of the lateral geniculate
body
3) do not cross in the posterior commissure
4) form synapses in the pretectal region
NEU-6.901.
Arteries supplying different parts of the optic system are the:
1) anterior choriodal artery
2) middle cerebral artery
3) posterior cerebral artery
4) pericallosal artery
NEU-6.902.
In which diseases does paralysis of the eye muscles develop?
1) a sinus thrombosis
2) in multiple sclerosis
3) in Wernicke's encephalopathy
4) an aneurysm
NEU-6.903.
Which of the following structures receive bilateral supranuclear
innervation?
1) the motor nucleus of the trigeminal nerve
2) a part of the motor nucleus of the facial nerve which innervates
the frontal and periorbital muscles
3) the nucleus of the accessory nerve

4) a part of the motor nucleus of the facial nerve which innervates


the perioral muscles
NEU-6.904.
Which of the following is typical of a Brodmann's area 19 lesion?
1) acoustic agnosia
2) visual field disorders
3) Pick's vision
4) a lack of optokinetic nystagmus
NEU-6.905.
Which of the following is typical of a Brodmann's area 8 lesion?
1) a lack of reflex eye movements
2) an intact reflex eye movement
3) an intact voluntary eye movement
4) a lack of voluntary eye movements
NEU-6.906.
Unilateral excitation of Brodmann's area 8 includes:
1) ipsilateral conjugated deviation
2) contralateral conjugated deviation
3) that the head is turned to the same side
4) that the head is turned to the contralateral side
NEU-6.907.
A unilateral lesion of Brodmann's area 8 includes:
1) ipsilateral conjugated deviation
2) contralateral conjugated deviation
3) the head is turned to the same side
4) the head is turned to the contralateral side
NEU-6.908.
Which of the following is typical of accomodation?
1) contraction of the ciliary muscle
2) relaxation of the ciliary muscle
3) an increased curvature of the lens
4) a flattened lens
NEU-6.909.
Sympathetic innervation of the eye muscles:
1) the nucleus is localized in the ciliospinal center
2) the ciliospinal center is localized in the lateral horn of the 8th
cervical to the 2nd thoracic segments
3) innervation includes the dilator pupillae, superior tarsal and
orbital muscles.
4) its lesion leads to the development of Horner's triad.
NEU-6.910.
Trismus can develop in:
1) myasthenia gravis
2) trigeminal neuralgia
3) hypertonic encephalopathy
4) tetanus
NEU-6.911
A lesion of the facial nucleus occurs in:

1) syringobulbia
2) bulbar paralysis
3) polioencephalitis
4) pontine tumors
NEU-6.912.
The acoustic system includes:
1) the otic ganglion
2) the geniculate ganglion
3) the stellate ganglion
4) the spiral ganglion
NEU-6.913.
Vestibular nuclei include:
1) the nucleus lateralis of Deiters
2) the nucleus medialis of Schwalbe
3) the nucleus superior of Bechterew
4) the spinal nucleus descendens of Roller
NEU-6.914.
The primary acoustic center is located in:
1) Brodmann's area 38
2) Brodmann's area 39
3) Brodmann's area 40
4) Brodmann's area 41
NEU-6.915.
Which symptoms are typical of circulatory disorders in the brain
stem?
1) they are usually provoked by head movement
2) visual disorders
3) swallowing disorder; hoarseness
4) typical pronounced rotatory nystagmus, vomiting
NEU-6.916.
What are the typical features of Wallenberg's syndrome?
1) it is caused by an occlusion of the posterior cerebral artery
2) it is caused by an occlusion of the anterior inferior cerebellar
artery
3) it is caused by an occlusion of the superior cerebellar artery
4) is caused by an occlusion of the posterior inferior cerebellar
artery
NEU-6.917.
Which nuclei are damaged in Wallenberg's syndrome?
1) the inferior vestibular nucleus
2) the dorsal nucleus of the vagus nerve
3) the glossopharyngeal nucleus
4) the nucleus ambiguous
NEU-6.918.
Which tracts and structures are damaged in Wallenberg's syndrome?
1) the anterior spinocerebellar tract
2) the central tegmental tract
3) the spinothalamic tract
4) the inferior cerebellar peduncule

NEU-6.919.
Which symptoms develop in Wallenberg's syndrome?
1) Horner's triad
2) ipsilateral nystagmus
3) ipsilateral ataxia
4) contralateral body sensory disorders
NEU-6.920.
Which arteries supply the medulla?
1) the posterior inferior cerebellar artery
2) the anterior inferior cerebellar artery
3) the anterior spinal artery
4) the posterior cerebral artery
NEU-6.921.
Which arteries supply the pons?
1) the posterior cerebral artery
2) the anterior spinal artery
3) the posterior choroidal artery
4) the b basilar artery
NEU-6.922.
Which arteries supply the mesencephalon?
1) the superior cerebellar artery
2) the posterior cerebral artery
3), the interpeduncular branches
4) the posterior communicating branches
NEU-6.923.
Which of the following is typical of Parinaud's syndrome?
1) vertical nystagmus
2) weakness of upward gaze
3) it can be caused by a tumor of the pineal gland
4) Bell's effect is present
NEU-6.924.
The nuclei of the cerebellum include:
1) the emboliform nucleus
2) the globose nucleus
3) the dentate nucleus
4) the fastigial nucleus
NEU-6.925.
The cerebellum is involved in the following functions:
1) vision
2) gnostic performance
3) hearing
4) coordination of muscle activity
NEU-6.926.
Which of the following is typical of the thalamus?
1) it is here that subcortical processing of the environmental and
internal stimuli occurs
2) it is connected with the cerebellum
3) it is a relay station for specific and aspecific afferent pathways

4) non-specific cortical stimulation occurs via the thalamus


NEU-6.927.
Which of the following structures are part of the limbic system?
1) the cingulate gyrus
2) the hippocampus
3) the fornix
4) the mammilary bodies
NEU-6.928.
Which nerves terminate in the nucleus of the solitary tract?
1) the intermediate nerve
2) glossopharyngeal nerve
3) the vagus, nerve
4) the hypoglossal nerve
NEU-6.929.
The components of the vestibular system include:
1) the utricule
2) the saccule
3) the semicircular canals
4) Corti's organ
NEU-6.930.
Vertigo occurs in:
1) Meniere's disease
2) it can be of a psychogenic origin
3) intoxication
4) vestibular neuritis
NEU-6.931.
In which diseases does a lesion of the abducent nerve occur?
1) in Benedikt's syndrome
2) in Jackson's syndrome
3) in Weber's syndrome
4) in Gradenigo's syndrome
NEU-6.932.
In which syndrome does a lesion of both oculomotor nerves occur?
1) in Foville's syndrome
2) in Gradenigo's syndrome
3) in Millard-Gubler's syndrome
4) in Tolosa-Hunt's syndrome
NEU-6.933.
In which syndrome does the oculomotor nerve remain UNDAMAGED?
1) in Nothnagel's syndrome
2) in Benedikt's syndrome
3) in Weber's syndrome
4) in Fovill's syndrome
NEU-6.934.
In which syndrome does the facial nerve remain undamaged?
1) in Melkersson-Rosenthal's syndrome
2) in Millard-Gubler's syndrome
3) in Foville's 'syndrome

4) in Avellis' syndrome
NEU 6.935.
In wich diseases does a lesion of the facial nerve occur?
1) in otitis media
2) inflammation of the parotid gland
3) a viral infection
4) an acoustic neuroma
NEU-6.936.
Which of the following is typical of basilar meningitis?
1) the CSF mainly contains granulocytes
2) the CSF glucose level is increased
3) the CSF cell count is 20 000/mm3
4) the CSF glucose level is low
NEU-6.937.
Which symptoms can be caused by circulatory disorders of the
vertebrobasilar area?
1) vertigo
2) confusion
3) drop attack
4) imbalance
NEU-6.938.
Which of the following is typical of sclerozing panencephalitis?
1) there is a typical onset in childhood
2) the child's school performance gets worse
3) there are high gamma globulin values
4) epileptic grand mal seizures and myoclonus
NEU-6.939.
Medication used,in multiple sclerosis includes:
1) levodopa + benserazide (Madopar)
2) pyridostigmine (Mestinon)
3) methyldopa (Dopegyt)
4) steroids
NEU-6.940.
Select the diseases primarily affecting muscles:
1) Werdnig-Hoffman's disease
2) Aran-Duchenne's syndrome
3) lateral amyotrophic sclerosis
4) progressive muscle dystrophy
NEU-6.941.
Which of the following is damaged in poliomyelitis?
1) the spinal dorsal horn
2) the pontine tegmentum .
3) the thalamus
4) the spinal ventral horn
NEU-6.942.
In which diseases does the peripheral motor neuron remain undamaged?
1) Aran-Duchenne's syndrome
2) Heine-Medin's disease

3) Werdnig-Hoffman's disease
4) pseudobulbar paralysis
NEU-6.943.
Which of the following does not cause a peroneal lesion?
1) a fibular fracture
2) a dislocation of the knee joint
3) a compression-induced peroneal lesion
4) an occlusion of the posterior spinal artery
NEU-6.944.
In which cases of severe carotid stenosis is a carotid endarterectomy
NOT PERFORMED?
1) a transient ischemic attack
2) in symptom-free carotid stenosis
3) a crescendo transient ischemic attack (TIA)
4) a hemorrhage in the brain matter
NEU-6.945.
Which diseases are usually accompanied by epileptic fits?
1) intracranial tumors
2) alcoholism
3) head injuries
4) multiple sclerosis
NEU-6.946.
Which pathological processes can cause unconsciousness?
1) a thrombosis of the basilar artery
2) "grand mal" epilepsy
3) an intracranial hemorrhage
4) syncope
NEU-6.947.
Which of the following medications is used in cerebral infarction?
1) pentoxifylline (Trental )
2) vinpocetine (Cavinton)
3) mannitol (Mannisol)
4) xantinol (Xavin)
NEU-6.948.
The following is typical of Eaton-Lambert's syndrome:
1) it usually accompanies malignant processes
2) a bronchosarcoma is usually identified
3) pathological fatigue is mainly manifested in the proximal
muscles of the hind limbs
4) it is resistant to cholinesterase inhibitors
NEU-6.949.
Plasmapheresis is used in:
1) myasthenia gravis
2) polmyositis
3) Guillain-Barr's syndrome
4) Foster-Kennedy's syndrome
NEU-6.950.
The most typical symptoms of encephalitis include:

1) confusion
2) sleep disorders
3) epileptic fits
4) a high level of CSF protein
NEU-6.951.
Which disease(s) is (are) suspected if a patient complains of a slowly
developing left spastic hemiparalysis?
1) a primary tumor of the right hemisphere
2) a right hemisphere hemorrhage
3) a right hemisphere metastasis
4) hernia of a cervical disc
NEU-6.952.
Which disease(s) is (are) suspected if a young woman complains of
occasional diplopia?
1) myasthenia gravis
2) a tumor of the medulla
3) multiple sclerosis
4) a cerebellar hemorrhage
NEU-6.953.
What are the sequellae of AIDS?
1) an occurrance of opportunistic infections
2) dementia
3) malignant intracranial tumors can develop
4) polyneuritis can develop
NEU-6.954.
Which are the most important collaterals between the internal and
external carotid arteries?
1) leptomeningeal anastomoses
2) the occipital artery
3) the posterior communicant artery
4) the ophthalmic artery
NEU-6.955.
What are the typical features of borreliosis?
1) it develops after a tick bite
2) it is a viral disease
3) arthralgia and polyneuropathy can develop
4) the CSF finding is always normal
NEU-6.956.
Which of the following is typical of Guillain-Barr's syndrome?
1) there is a dissociation of cell proteins
2) it has a poor prognosis
3) flaccid para- and tetraparalysis, and a loss of reflexes
4) a sudden onset
NEU-6.957.
Which of the following medications is used in parkinsonism?
1) levodopa + benserazide (Madopar)
2) bromocriptine
3) amantadine (Viregyt K)
4) pyridostigmine (Mestinon)

NEU-6.958.
Which of the following is typical of parkinsonism?
1) hypo- or akinesia
2) no mental symptoms
3) rigidity and tremor
4) spasticity
NEU-6.959.
Which of the following is typical of an intracranial hemorrhage?
1) a lobular hemorrhage is the most common form
2) the hemorrhage usually occurs in the area of the basal ganglia
3) it usually occurs in the basin of the posterior cerebral artery
4) it usually occurs in the basin of the lenticulostriate aartery
NEU-6.960.
Which of the following is typical of an intracranial aneurysm?
1) it can cause compression symptoms
2) it usually develops in the vertebrobasilar region
3) a subarachnoid hemorrhage can be the first sign
4) the first signs usually appear in children
NEU-6.961.
Common complications of subarachnoid hemorrhage include:
1) a vasospasm-induced softening
2) a relapse
3) a communicating hydrocephalus
3) secondary suppuration
NEU-6.962.
The criteria of brain death are:
1) coma
2) a loss of brain stem reflexes
3) a lack of spontaneous respiration
4) the exclusion of the effects of drugs and other toxic agents
NEU-6.963.
Which of the following is typical of Binswanger's encephalopathy?
1) hypertension in the history of 90% of the patients
2) pseudobulbar symptoms
3) dementia is typical
4) the CT finding: thinning of the white matter, multiple
infarctions
NEU-6.964.
Which of the following is typical of a communicating hydrocephalus?
1) dementia
2) gait disorders
3) micturition disorders
4) a lumbar puncture has a beneficial effect
NEU-6.965.
Which of the following is typical of Wilson's disease?
1) a Kayser-Fleischer's ring on the cornea
2) autosomal recessive inheritance
3) it leads to dementia

4) extrapyramidal symptoms
NEU-6.966.
The therapy of Wilson's disease includes:
1) D-penicillinamine
2) a copper-deprived diet
3) the possible administration of ZnSO4
4) potassium sulphide
NEU-6.967.
The local administration of botullinum toxin decreases:
1) athetosis
2) blepharospasm
3) chorea
4) spastic torticollis
NEU-6.968.
The administration of beta-blocking agents beneficially affects:
1) hypotension
2) epilepsy
3) Guillain-Barr's disease
4) any essential tremor
NEU-6.969.
Which drug reduces increased tone spasticity?
1) Baclofen
2) clozapine (Leponex)
3) tolperisone (Mydeton)
4) levodopa + benserazide (Madopar)
NEU-6.970.
Structures responsible for voluntary eye movement include:
1) the superior longitudinal fasciculus
2) Brodmann's area 8
3) Brodmann's area 19
4) the thalamus
NEU-6.971.
The medication of choice in a cholinergic crisis is:
1) neostigmine (Stigmosan)
2) pyridostigmine (Mestinon)
3) edrophonium (Tensilon)
4) atropine
NEU-6.972.
Which disease develops after a tick-bite?
1) tick-encephalitis
2) borreliosis
3) tick-meningoencephalitis
4) subacute sclerozing panencephalitis
NEU-6.973.
Malignant tumors can be accompanied by:
1) cerebellar atrophy
2) progressive multifocal leukoencephalopathy
3) polyneuropathy

4) dermatomyositis
MULTIPLE CHOICE QUESTIONS WITH KEY ANSWERS/TYPE II
Every question or incomplete statement has only one answer in the
following combinations:
A) if the answers 1, 2, and 3 are true
B) if the answers 2, 3, and 4 are true
C) if the answers 1, 3, and 5 are true
D) if the answers 1 and 5 are true
E) if the answers 2 and 5 are true
F) if the answers 3 and 4 are true
Select one of these key combinations!!!
NEU-6.974.
Cerebellar atrophy:
1) can be caused by alcoholism
2) occurs as a preneoplastic sign in case of malignancy
3) can be due to trauma
4) can be caused by Guillain-Barr's syndrome
NEU-6.975.
Which of the following factors contribute to the development of a
cerebral venous thrombosis?
1) ovulation inhibitors
2) the postpuerperal period
3) an infectional, septic focus
4) thrombocytopenia
NEU-6.976.
Which of the following is typical of a subdural hematoma?
1) a cranial fracture is an important prerequisiste
2) the CSF finding is of diagnostic value
3) it more frequently develops in alcoholics
4) it can be typically diagnosed with a CT
NEU-6.977.
Guillain-Barr's syndrome:
1) has a good prognosis
2) can be treated in an out-patient clinic
3) in this syndrome the CSF protein levels are extremely high
4) spastic paralysis and hyperreflexia are typical
5) sometimes requires admission to an, intensive care unit
NEU-6.978.
Which of the following frequently causes spastic paralysis of the
lower limbs?
1) multiple sclerosis
2) a parasagittal meningioma
3) the non-acute phase of an anterior spinal artery occlusion
4) Guillain-Barr's syndrome
5) spastic paralysis of the lower limbs is never caused by any
spinal space-occupying processes
NEU-6.979.
Which of the following structures is usually damaged in amyotrophic
lateral sclerosis?

1) the pyramidal system


2) the spinothalamic tract
3) the spinal ventral motor neurons
4) the thalamus
5) the visual system
NEU-6.980.
In subarachnoid hemorrhage:
1) surgery is the method of choice
2) surgery should be optimally performed between days 2-10
3) a dull, not severe headache can be present
4) conservative therapy is the method of choice
5) meningeal excitation signs develop
ASSOCIATION QUESTIONS
Associate the following terms/statements marked by the letters
A, B, C... with the corresponding statements/terms marked by and in
the order given by the figures 1, 2, 3...
...for example: 1-C, 2-B, 3-A, 4-D. Put the answer as C, B, A, D!
(Note: Different statements can be associated with the same terms!!!)
NEU-6.981.
Associate the following term(s) with their corresponding statement(s)!
A) Brodmann's area 39, 40
B) Brodmann's area 18, 19
C) Brodmann's area 4
D) Brodmann's area 3, 2, 1
E) Brodmann's area 6, 8
F) Brodmann's area 17
G) Brodmann's area 41, 42
1) somatic sensory cortex
2) speech association area
3) perception of vision
4) visual association
5) acoustic cortex
6) primary motor cortex
7) supplementary motor cortex
NEU-6.982.
Associate the following term(s) with their corresponding statement(s)!
A) Acetylcholine
B) Norepinephrine
C) Epinephrine
D) Dopamine
1) a precursor of noradrenaline
2) it is synthesized by the spinal motor neurons
3) the main amine produced by the adrenal chromaffin cells
4) the main amine produced by postganglionic sympathetic
neurons
NEU-6.983.
Associate the following term(s) with their corresponding statement(s)!
A) Extradural tumor
B) Intramedullary tumor

C) Extramedullary and Intramedullary tumor


1) a partial obstruction of contrast substance flow with a spindle
shaped widening of the spinal cord
2) a clear demarcation of the tumor edges with a shift in the
spinal opacity
3) a cone-like narrowed area marked by contrast substance in
the spinal canal
NEU-6.984.
Associate the following statements(s) with their corresponding term(s)!
A) enlarged optic foramen
B) suprasellar calcification
C) forward protrusion of the aqueduct
D) backward protrusion of the aqueduct
1) Medulloblastoma
2) Glioma of the optic nerve
3) Craniopharyngioma
4) Brain stem glioma
NEU-6.985.
Associate the following statements(s) with their corresponding term(s)!
A) rarely occurs in children but occurs frequently in adults
B) frequently occured in the past but is rare nowadays
C) surgical treatment
D) a paralysis of forward gaze
E) the most frequent cerebral glioma
1) Tuberculoma
2) Meningioma
3) Pinealoma
4) Ependymoma
5) Cerebellar astrocytoma
NEU-6.986.
Associate the following term(s) with their corresponding statement(s)!
A) Malformation of the Great Galenic vein
B) Saccular aneurysm
C) Arterio-venous malformation
D) Intracranial hemorrhage
1) it is a rare cause of subarachnoid hemorrhage in pediatric
patients
2) it is a common cause of subarachnoid hemorrhage and can
cause migraine as a complication
3) it is a common cause of hydrocephalus in infants and elicits
congested symptoms
4) it is common in infantile hemiplegia
NEU-6.987.
Associate the following term(s) with their corresponding statement(s)!
A) Ramsey-Hunt's syndrome
B) Bell's paralysis
C) Guillain-Barr's syndrome
D) Forceps paralysis

E) Melkerson-Rosenthal-Mischer's syndrome
1) idiopathic facial paralysis with a sudden onset in pediatric
patients
2) facial nerve paralysis due to herpes of the auditory canal
3) paralysis of the facial nerve in infants
4) facial paralysis with swollen lips
5) bilateral facial paralysis
NEU-6.988.
Associate the following term(s) with their corresponding statement(s)!
A) Tuberous sclerosis
B) Phenylketonuria
C) Galactosemia
D) Down's syndrome
E) Kernicterus
F) Hartnup's disease
G) Toxoplasmosis
H) Laurence-Moon-Biedle's syndrome
1) a hanging tongue
2) cataract
3) blue eyes
4) Shagreen's patches
5) pellagra
6) greenish teeth
7) testicular hypoplasia
8) chorioretinitis
NEU-6.989.
Associate the following term(s) with their corresponding statement(s)!
A) Tabes dorsalis
B) Amyotrophic lateral sclerosis
C) Hematomyelia
D) Multiple sclerosis
E) Spinal tumor
1) an intermittant course
2) subarachnoid block
3) non-reactive pupils
4) a loss of sensory signs
5) a history of trauma
NEU-6.990.
Associate the following term(s) with their corresponding statement(s)!
A) Ethosuccimide (Suxilep)
B) Ergotamine
C) Diphenylhydantoin (Diphedan)
D) Carbamazepine
E) Dramamine (Daedalon)
1) Cluster headache
2) migraine with aura
3) "grand mal" seizures
4) "petit mal" seizures
5) neuralgia

6) Menire's syndrome
NEU-6.991.
Associate the following statement(s) with their corresponding
term(s)!
A) 40-50-years-old
B) 20-30-years-old
C) 10-20-years-old
D) 50-70-years-old
E) 0-10-years-old
1) multiple sclerosis
2) Alzheimer's disease
3) amyotrophic lateral sclerosis
4) Friedreich's ataxia
5) tuberous sclerosis
NEU-6.992.
Group the structures) which correspond to the typical symptom(s) of
Wallenberg's syndrome (lateral medullary syndrome)!
A) vestibular nucleus
B) spinal lemniscus
C) restiform body
D) nucleus ambiguous
E) descending tract of the trigeminal nerve
1) dysarthria
2) cerebellar dysfunction
3) facial hemihypalgesia
4) body hemihypalgesia
5) nystagmus
NEU-6.993.
Associate the following statement(s) with their corresponding term(s)!
A) brain stem tumor
B) pediatric cerebellar hemisphere tumors
C) tumor of the IV ventricle
D) medial line cerebellar tumor
E) the most common primary cerebellar tumor in adults
1) Astrocytoma
2) Polar spongioblastoma
3) Hemangioblastoma
4) Medulloblastoma
5) Ependydoma
NEU-6.994.
Associate the following term(s) with their corresponding statement(s)!
A) Median nerve
B) Peroneal nerve
C) Facial nerve
D) Ulnar nerve
1) it is the most frequently injured nerve
2) causalgia
3) damge elicits the so-called "Froment" symptom

4) it is involved in Ramsey-Hunt's syndrome


NEU-6.995.
Associate the following term(s) with their corresponding statement(s)!
A) Radial nerve
B) Median nerve
C) Sciatic nerve
D) Femoral nerve
E) Axillary nerve
1) this upper limb nerve is rarely injured
2) this lower limb nerve is rarely injured
3) "saturday night" palsy
4) is damaged after an intramuscular injection
5) is affected in the carpal tunnel syndrome
NEU-6.996.
Associate the following term(s) with their corresponding statement(s)!
A) Normal pressure hydrocephalus
B) Alzheimer's disease
C) Both (A) and (B)
D) None of the above
1) . a history of subarachnoid hemorrhage
2) a history of cranial injury
3) a history of meningitis
4) marked postoperative deterioration
5) in most cases enlarged ventricles and basal cisterns on PEG
(pneumoencephalography) and CT images with less pronounced
cortical atrophy
6) dementia; gait disorder; urinary incontinence
7) pathological findings on radionucleide cisternography
8) spontaneous improvement without shunt is reported
9) in some cases a ventriculolateral shunt has a beneficial
effect
10) in some cases a ventriculoperitoneal shunt has a beneficial
effect
NEU-6.997.
Associate the following statement(s) with their corresponding term(s)!
A) a loss of voluntary movements without paralysis
B) a cortical or subcortical lesion
C) an ipsilateral loss of vibration and position sensation, and a
contralateral loss of pain and temperature sensation
D) damage to the pyramidal system and involvement of the oculomotor
nerve
1) Aphasia
2) Apraxia
3) Brown-Sequard's syndrome
4) Weber's syndrome
NEU-6.998.
Associate the following statement(s) with their corresponding term(s)!
A) primarily develops in pediatric patients; calcification usually
occurs
B) paralysis of upward gaze

C) tinnitus; vertigo
D) bitemporal hemianopsia
E) crossed motor and sensory disorders
1) Pinealoma
2) Craniopharyngioma
3) Acoustic neuroma
4) Pituitary adenoma
5) Brain stem tumor
NEU-6.999.
Associate the following term(s) with their corresponding statement(s)!
A) Cerebral hemorrhage
B) Cerebral ischemia due to embolism
C) Cerebral ischemia due to occlusive vascular disease
D) Subarachnoid hemorrhage
1) the average age of patients at the onset of the disease is the
highest from the noted list
2) two-thirds are localized in the basal ganglia
3) it is the most common ischemic disease
4) it usually occurs with physical strain and a sudden onset is
typical
5) even in fresh cases the CT finding is 100% positive
6) a hemorrhagic CSF is found only if the process is directed
towards the subarachnoid space
7) three-fourths of the cases affect the ventricle and about onehalf
of the patients develop confusion in the early phase of the
disease
8) a headache at the onset is rare
9) the process is usually accompanied by endocarditis
NEU-6.1000.
Associate the following statement(s) with their corresponding term(s)!
A) it is the most malignant
B) it has a slow growth and usually contains calcium
C) it causes Parinaud's syndrome
D) in pediatric patients this posterior scala tympani tumor is
relatively susceptible to X-ray therapy
E) the formation of perivascular pseudorosettes is typical
1) Pinealoma
2) Multiform glioblastoma
3) Medulloblastoma
4) Oligodendroglioma
5) Ependymoma
NEU-6.1001.
Associate the following term(s) with their corresponding statement(s)!
A) Glioblastoma multiforme
B) Oligodendroglioma
C) Both (A) and (B)
D) None of the above
1) a rare form of gliomas
2) it is usually localized in the cerebral hemispheres

3) cystic degeneration areas are rare


4) hemorrhagic areas are rare
5) these are the predominant glioma forms in adults
6) it is available for total resection
7) a native skull X-ray reveals marked calcification spots
8) it rapidly leads to death which usually occurs within a year
9) it consists of many cells of the same size and shape
10) it frequently spreads to the other hemisphere via the corpus
callosum
NEU-6.1002.
Associate the following term(s) with their corresponding statement(s)!
A) Median nerve
B) Radial nerve
C) Ulnar nerve
D) Sciatic nerve
E) Femoral nerve
F) Lateral femoral cutaneous nerve
1) it is susceptible to traumatic injury
2) affliction typically elicits causalgia
3) affliction causes a "drop wrist"
4) affliction causes a "claw-hand"
5) traumatic injury is rare
6) "Froment's symptom" (involuntary flexion of the distal part of
the thumb while holding a sheet of paper between the fingers)
7) the carpal tunnel syndrome
8) it can be compressed by crutches
9) it is the largest nerve of the body
10) it is sometimes injured after an intramuscular injection
11) paresthetic meralgia (pain in the thigh)
NEU-6.1003.
Associate the following term(s) with their corresponding statement(s)!
A) Electrical injury
B) X-ray irradiation
C) Both (A) and (B)
D) None of the above
1) seizures
2) white matter necrosis and myelopathy-induced NEUrologic
symptoms are the most severe complications
3) death is due to respiratory paralysis or ventricular fibrillation
4) Marcus-Gunn's phenomenon
5) Gunn's pupil
NEU-6.1004.
Associate the following term(s) with their corresponding statement(s)!
A) Sensory cortex
B) Motor cortex
C) Both (A) and (B)
D) None of the above
1) it is localized in the frontal lobe
2) it is localized in the temporal lobe
3) it is localized adjacent to the central sulcus

4) it is localized in the parietal lobe


5) Brodmann's area 1, 2, and 3
6) Brodmann's area 4
7) it has connections with the thalamus
8) it is supplied by the basilar artery
9) excitation leads to epilepsy
NEU-6.1005.
Associate the following term(s) with their corresponding statement(s)!
A) Anterior spinal artery
B) Middle cerebral artery
C) Anterior communicant artery
D) Anterior cerebral artery
E) Ophthalmic artery
F) Posterior cerebral artery
G) Posterior inferior cerebellar artery
1) the most common site of aneurysms
2) occlusion causes Wallenberg's syndrome
3) the most important collateral vessels between the internal and
external carotid artery
4) occlusion is accompanied by contralateral homonymous hemianopsia
5) circulatory disorders cause hemiparalysis primarily affecting
the upper limbs
6) circulatory disorders cause hemiparalysis primarily affecting
the lower limbs with micturition disorders
7) occlusion causes Jackson's symptom
NEU-6.1006.
Associate the following term(s) with their corresponding statement(s)!
A) Motor aphasia
B) Sensory aphasia
C) Conduction aphasia
D) Amnestic aphasia
E) None of the above
1) the center is localized in the left lower parietal lobe
2) the center is localized in the distal part of the third frontal
gyrus
3) an intact insula is important
4) the center is localized in the left upper temporal gyrus
5) the center is localized in the occipital lobe
NEU-6.1007. Association Question
Associate the following statement(s) with their corresponding term(s)!
A) it is accompanied by damage of the pyramidal tract
B) it is accompanied by damage of the ventral root motor neurons
C) both (A) and (B)
D) none of the above
1) Myasthenia gravis
2) Amyotrophic lateral sclerosis
3) Multiple sclerosis
4) Acute anterior poliomyelitis
5) Spastic spinal paralysis

NEU-6.1008.
Associate the following term(s) with their corresponding statement(s)!
A) Guillain-Barr syndrome
B) Multiple sclerosis
C) Both (A) and (B)
D) None of the above
1) it has a good prognosis
2) it has a poor prognosis
3) it is accompanied by spastic paraparalysis
4) it is accompanied by flaccid paraparalysis, hypo- or areflexia
5) it is accompanied with signs of meningeal excitation
6) usually spontaneous improvement occurs
7) usually requires hospital treatment
8) there is damage of the pyramidal tract
9) usually cerebellar symptoms develop
NEU-6.1009.
Associate the following term(s) with their corresponding statement(s)!
A) Cisternal puncture
B) Lumbar puncture
C) Both (A) and (B)
D) None of the above
1) it is less dangerous than the other noted procedure
2) it does not (or scarcely) causes any postpuncture symptoms
3) if applied the CSF flows spontaneously
4) the normal CSF sample contains 8- l0mm3 cells
5) the CSF sample contains 0.1-0.2 g/L protein
6) the CSF sample contains 1-2 g/L protein
7) in optimal cases the puncture needle should be introduced
between cervical vertebra C1 - C2
8) the normal CSF sample is colorless, odorless and water-clear
9) the CSF is obtained from the interpeduncular cistern
NEU-6.1010.
Associate the following term(s) with their corresponding statement(s)!
A) Mannisol + Vasodilators
B) Corticosteroids + Cytostatics
C) Surgery
D) Myorelaxant
E) B complex vitamins
F) Carbamazepine (Tegretol + Stazepin)
G) None of the above
1) . brain softening
2) subarachnoid hemorrhage
3) disc hernia with foot paralysis and urine incontinence
4) alcoholic polyneuropathy
5) cerebellar hemorrhage
6) epilepsy (idiopathic)
7) basilar meningitis
8) polymyositis
CASE STUDIES
Answer the multiple task questions (simple choice and multiple choice

with/without key answers; relation analysis etc.) as they are related to


each case study!!!
NEU-6.1011.
A 39-year-old male patient was treated for renal disease in the past.
Since that time his blood pressure has been about 160 mmHg and he
has complained of regular unilateral headaches. Two days ago he had
suffered a severe occipital headache after trying to lift a heavy object.
Further the patient experienced sweating, turned pale and complained
of vertigo; he later vomited several times, but had no nausea before
vomiting. He also complained of impaired vision and that light disturbs
him. Present status: blood presure: 185/85 mmHg; heart rate
60/min; temperature: 37.3 C. A systolic murmur at the apex can be
recognized. The liver is enlarged by two fingers. Mild occipital stiffness.
Positive Brudzinsky's and Kernig's signs. Latent paralysis of the
right limbs. A positive Babinski's reflex.
6.1011/1. Single Choice Question
The most probable diagnosis is:
A) uremia
B) meningitis
C) subarachnoid hemorrhage
D) myalgia due to cervical diskopathy
E) migraine headache
F) cerebral tumor
6.1011/2. Single Choice Question
The most important immediate intervention is:
A) suboccipital lidocaine infiltration plus analgetics
B) immediate admission to a NEUrologic ward
C) an X-ray of the patient's spine
D) further, more intensive history taking
E) a complete ophthalmologic study
F) a renal function study
6.1011/3. Single Choice Question
If a CSF study is performed which CSF findings would you expect?
A) a water clear CSF
B) a greenish-yellow CSF
C) a xanthochromic CSF
D) an opaque CSF
E) a hemorrhagic CSF
NEU-6.1012.
A 21-year-old male patient had left-sided otologic surgery performed
at the age of six. At the age of eleven he had mumps and hepatitis. A
month before admission the patient complained of malaise and vertigo.
2-3 weeks later he had a pulsating headache, fever (38C) and
nausea. At admission: no internal symptoms were found. Occipital
stiffness was present but no focal neurologic symptoms could be observed.
A lumbar puncture revealed 760/mm3 cells (predominantly
lymphocytes).
6.1012/1. Single Choice Question
The most probable diagnosis is:
A) cerebral tumor

B) basilar tuberculous meningitis


C) purulent meningitis
D) lymphocytic meningitis
6.1012/2. Single Choice Question
The typical symptom of this disease is:
A) a lack of neurologic focal symptoms
B) confusion
C) a pulsating headache
D) an increased lymphocyte count
E) meningeal symptoms and an increased lymphocyte count
6.1012/3. Single Choice Question
The most important diagnostic examination is:
A) CT
B) carotid angiography
C) pneumoencephalography
D) a detailed CSF study
E) electroencephalography
6.1012/4. Single Choice Question
Characteristic dynamics of this disease include:
A) ictal onset
B) a steady progression for many years
C) an intermittant progression
D) it can develop within several days or weeks
E) remissions can occur
NEU-6.1013.
A 58-year-old doctor suffering for many years from hypertension
collapses at work. At admission: confusion, aphasia and right-sided
weakness are found. BP: 230/160 mmHg. A lumbar puncture
reveals a hemorrhagic CSF with a xanthochromic supernatant.
6.1013/ l. Single Choice Question
The most probable etiology is:
A) a hypertensive intracerebral hemorrhage
B) a cerebral embolism
C) a cerebral thrombosis
6.1013/2. Single Choice Question
Which other pathological changes can develop in the course of this
disease?
A) gastric erosion
B) myocardial infarction
C) fat atrophy of the liver
6.1013/3. Single Choice Question
Which therapy would you suggest?
A) steroids
B) antihypertensive therapy
C) hydantoin iv
D) all of the above
E) none of the above
NEU-6.1014.

A 40-year-old female patient presented with severe back pain. It began


at her work place when she bent down and suddenly had a piercing
back pain, so that she could not stand upright. The pain radiated
along the dorsal surface of her lower limbs down to her ankle. Coughing
and sneezing increased the pain. At examination: no cranial nerve
symptoms, decreased Achilles reflex and hypesthesia of the S1 dermatome
on the right side. Paralumbar defense, a flattening of the normal
lumbar lordosis, forced posture, and a positive Lasegue's sign were
also found. Lumbosacral X-ray: flattened L5 with elongated ventral
and dorsal edges of the vertebra. Lumbar CSF: cell count: 2 mm3,
total protein: 0.95 g/L.
6.1014/ 1. Single Choice Question
The most probable diagnosis is:
A) spondylolisthesis
B) a lumbosacral herniated disc
C) a compressive fracture of the L5 vertebra
D) an intramedullary tumor at the lumbar level
E) lumbar spondylosis
6.1014/2. Single Choice Question
The least typical symptom is:
A) paravertebral defense
B) segmental sensory disorders
C) a flattening of the normal lumbar lordosis
D) elongated vertebral edges
E) an increase of the acute root pain when coughing or sneezing
6.1014/3. Select One Of The Key Combinations
Which of the following is not important in this diagnosis?
1) myelography
2) spinal X-ray
3) a CSF study
4) EMG
5) a muscle biopsy
A) only (1) and (3) are true
B) only (3) and (4) are true
C) only (2) and (4) are true
D) only (2) and (5) are true
E) only (4) and (5) are true
6.1014/4. Single Choice Question
Immediate surgery is indicated because of:
A) foot paralysis
B) a further increase of pain
C) relapses
D) a loss of the Achilles reflex
E) restricted movements of the lumbar vertebra
NEU-6.1015.
Emergency admission by ambulance: a 48-year-old female patient
and her husband left their apartment in the morning. On the way to
work the patient's husband observed jerks in the left hand and the left
corner of the mouth of the patient. The patient became unconscious,
urinated, and the jerks continued for about a minute in all four limbs.So

her husband immediately called for an ambulance. In the ambulance


the patient regained consciousness and could only talk slowly and
with difficulty. Weakness in the left hand developed.
At examination: left sided hemiparesis with a slight facio-brachial prevalence.
Hyperreflexia of the tendon reflexes.
BP: 160/ 100 mmHg; heart rate 58/min; no fever. History: Occupation:
administrator, consumes 4 cups of coffee and smokes 10 cigarettes
daily. The patient complains of a periodic occipital headache that has
been present for about 5 years. A year ago hypertension (160/100
mmHg) was diagnosed. Since then the patient has been on 3x1 tabl. of
methyldopa daily. Three weeks ago a periodic headache turned into a
severe permanent one and the patient regularly vomited in the morning.
Her blood pressure increased to 180/120 mmHg and therefore
additional dihydroergotoxin (Redergam) therapy was prescribed (3x20
drops). The patient complains of fatigue, memory disorders and a loss
of concentration at her work. Sometimes she has vision disorders (darkness
for several seconds) and feels that she has no control over her left
hand.
6.1015/ 1. Single Choice Question
The most probable diagnosis is:
A) a destructive pontine hemorrhage
B) epilepsy
C) meningitis
D) cerebral tumor
E) thrombosis of the right internal carotid artery
6.1015/2. Single Choice Question
The most relevant study for this diagnosis is:
A) a CSF sample
B) an EEG study
C) consultation with an internist
D) right-.sided carotid angiography
E) lumbar pneumoencephalography
6.1015/3. Single Choice Question
What should be done?
A) immediate admission to an internal ward
B) after examination, a neurologic follow-up on an outpatient basis
C) recommend antiepileptic and antihypertensive therapy which
should be controlled by the family doctor
D) immediate admission to a neurologic ward
E) bed rest at home
6.1015/4. Single Choice Question
The ideal therapy would be:
A) intravenous vasodilation therapy
B) dehydration
C) antiepileptic medication
D) antihypertensive drugs
E) anticoagulation therapy
6.1015/5. Single Choice Question
Some recommendations on how to alter her way of life are to:
A) avoid coffee, cigarettes, and stress
B) same as above plus antihypertensive medication

C) ask the patient for informed consent for surgical therapy


D) prepare documents which would allow the patient to retire
NEU-6.1016.
A 39-year-old male patient was treated for renal disease in the past.
Since that time his blood pressure has been about 160 mmHg and he
has complained of regular unilateral headaches. Two days ago he had
suffered a severe occipital headache after trying to lift a heavy object.
Further the patient experienced sweating, turned pale and complained
of vertigo; he later vomited several times, but had no nausea before
vomiting. He also complained of impaired vision and that light disturbs
him. Present status: blood presure: 185/85 mmHg; heart rate
60/min; temperature: 37.3 C. A systolic murmur at the apex can be
recognized. The liver is enlarged by two fingers. Mild occipital stiffness.
Positive Brudzinsky's and Kernig's signs. Latent paralysis of the
right limbs. A positive Babinski's's reflex.
6.1016/ 1. Single Choice Question
The most probable diagnosis is:
A) uremia
B) meningitis
C) subarachnoid hemorrhage
D) myalgia due to cervical diskopathy
E) migraine
F) a cerebral tumor
6.1016/2. Single Choice Question
The most important immediate intervention is:
A) suboccipital lidocaine infiltration plus analgetics .
B) immediate admission to a neurologic ward
C) an X-ray of the patient's spine
D) further, more intensive history taking
E) a complete ophthalmologic study
F) a renal function study
6.1016/3. Single Choice Question
The most important diagnostic study is:
A) angiography
B) hemostatic study
C) X-ray
D) a study of the eye fundus and a lumber puncture
E) EEG
F) echo-encephalography
6.1016/4. Single Choice Question
If a CSF study is performed which CSF findings would you expect?
A) a water clear CSF
B) a greenish-yellow CSF
C) a xanthochromic CSF
D) a opaque CSF
E) a hemorrhagic CSF
NEU-6.1017.
A 42-year-old male patient who has never had any systematic disease
in the past. Ten years ago the patient had a head injury and became
unconscious. An X-ray at that time study revealed a cranial fracture

line and the CSF was hemorrhagic. For five years the patient has been
having regular (every 2-3 months) attacks of unconsciousness, limb
jerks, biting of the tongue and urination. He never remembers his attacks.
At examination: no neurologic symptoms were found. Routine laboratory
findings were normal. EEG: bilateral irritation signs without
lateralizationerve pneumoencephalography (PEG): symmetric wide
ventricles and marked subarachnoid air filling.
6.1017/1. Single Choice Question
The most probable diagnosis is:
A) proneness to orthostatic collapse
B) hypoglycemic fits
C) a cerebral tumor
D) posttraumatic epilepsy
E) Adams-Stokes syncope
6.1017/2. Single Choice Question
The most typical symptom of this disease is:
A) disorientation
B) memory disorders
C) "grand mal" type seizure attacks
D) dementia
E) changes of personality
6.1017/3. Single Choice Question
The typical CSF finding is:
A) a normal CSF
B) an elevated total protein level
C) pleocytosis
D) cell protein dissociation
E) hemorrhagic CSF
NEU-6.1018.
A 59-year-old female patient has been hypertensive for years. She had
a tonsillectomy and an appendectomy during her childhood. The patient
regulary has swollen legs. One month before admission the patient's
behaviour had changed. She became silent. unmotivated, could
not do her job and neglected herself. The patient became absent-minded
and several days before admission urinated and defecated in bed.
At examination: BP: 140/80 mmHg; heart rate: 64/min. Bilateral mild
anasarca and induration of the limbs. Neurologic status: fundus:
blurred left papilla. The left pupil is slightly narrowed. A right-sided
central facial paralysis is present. A latent paresis in the right
extremities.
Babinski's sign, hyperreflexia of the deep reflexes on the right
side. Bilateral oral reflexes and an increased grasping reflex. Psychic
condition: temporal and spatial disorientation; slow cognition; pronounced
dysarthria; impaired memory. Routine laboratory studies:
within normal limits. Skull X-ray: no pathological alterations. EEG:
slow temporal activity on the left side. Left carotid angiography: the
anterior cerebral artery is displaced 2 cm to the right.
6.1018/ 1. Single Choice Question
The most probable diagnosis is:
A) hypertonic encephalopathy
B) a cerebral tumor

C) presenile dementia
D) cerebral atrophy
E) encephalomalacia of the left hemisphere
6.1018/2. Single Choice Question
The typical clinical symptoms:
A) absent-mindness
B) incontinence
C) central paralysis
D) signs of psychic deterioration
E) early signs of fundal congestion
6.1018/3. Single Choice Question
Typical dynamics of this disease includes:
A) a steady progression
B) slow improvement
C) a fast onset followed by slow deterioration
D) deterioration caused by acute attacks
E) transient remissions 1
6.1018/4. Single Choice Question
Which studies play no role in this diagnosis?
A) angiography
B) EEG
C) computer tomography
D) cisternal puncture
E) lumbar puncture
NEU-6.1019.
A 2-year-old male child with no history of any previous diseases was
brought to the hospital by his friends. While running uphill the patient
had experienced severe frontal and occipital headaches and collapsed.
His friend had observed a transient disorder of speech and the
patient had vomited several times in the ambulance.
At admission: occipital stiffness; Kernig's sign positive; multiple poollike
hemorrhages in the fundus; a central facial paralysis on the right
side; right hand tends to move back when lifted; dorsal flexion tendency
on the right side; dysarthria; somnolence. Lumbar puncture:
CSF with massive uniform hemorrhage flowing at increased pressure.
6.1019/ 1. Single Choice Question
The most probable diagnosis is:
A) cerebral hemorrhage
B) hemorrhagic encephalitis
C) hemorrhagic infarction
D) subarachnoid hemorrhage
E) cerebral contusion
6.1019/2. Single Choice Question
The pathomechanisms of the disease include:
A) rupture of the vascular wall due to a sudden increase of blood
pressure
B) secondary hemorrhage in a hypoxic area
C) angioma or aneurysm rupture
D) rupture of a pontine vein
E) capillary hemorrhage

NEU-6.1020. Case Study


45-year-old miner with no history of any previous diseases presented
to the hospital. About a year ago he started complaining of frontal
headaches, though did not consider those a serious problemuscle
Recently the headaches appeared more frequently. Before admission
the patient had collapsed while watching TV. His family told that his
whole body became rigid, his eyes became wide open and his limbs
were jerking. The attack lasted for 3-4 minutes, during which the patient
bit the left side of his mouth and urinated. After the attack he
was very tired and slept. At admission: a central facial and hypoglossal
paralysis on the right side. Slight hyperreflexia of the
proprioreflexes in the right upper limb. The latent paralysis test showed pronation of
the right upper limb. EEG: local temporal cortical disorder of organic
character. CSF: cell count: 0/mm3; total protein: 95 mg%; CSF protein
level (Pndy): ++; benzoic: normal.
6.1020/ 1. Single Choice Question
Define the character of the attack:
A) a functional fit
B) a traumatic ischemic attack
C) an epileptic attack
D) hyperesthesia of the carotid sinus
6.1020/2. Single Choice Question
Assign the above disease to one of the following groups:
A) psychogenic background - neurosis, exhaustion
B) disorders of cerebral circulation, hypoxia
C) a space occupying process
D) chronic toxicity (for example alcoholism) concealed by the
patient
6.1020/3. Single Choice Question
What should be done?
A) observation at home
B) immediate hospitalization
C) delayed hospitalization
NEU-6.1021. Case Study
A 56-year-old male patient, (occupation: unskilled construction worker)
complains of back pain that has persisted for 3 years and of gastric
pain that has lasted for about 2 years. The patient had no other diseases
in his history. His wife said that for several weeks her husband
has had a bad disposition and that he had complained of dull headaches,
and fatigue. Three weeks before admission the patient was on a
sick-allowance due to general malaise. At home he became more and
more silent, inactive, stayed in bed all day long and lost his usual
interests. Two weeks ago his gait became unsteady;. the patient drags
his left leg and uses his left hand less than his right hand because he
drops objects gripped by the left hand. During the last 3-4 days he has
complained of nausea and vomiting often. He has become somnolent
and could walk only with help. He had no fever. The patient has smoked
10-15 cigarettes daily for about 30 years; he has also consumed 50 ml
liquor twice a day for 4-5 years. Once or twice a week he consumes
even more and gets drunk. The patient never had surgery. Two months
ago when the patient was drunk, his neighbour had seen him collapse

and hit his head against a stone For about half a minute he remained
motionless, then regained his consciousness and went home. He
vomitted twice but thought it was due to alcohol. Since he scarcely
had any injures he did not consult a doctor, but during the weekend
the patient stayed in bed complaining of headache and nausea. On
Monday he went to work and remained complaint-free till the present
pre-admission complaints.
At admission: heart rate 52/min.; BP: 170/90 mmHg; temperature:
36.6oC; dry, coated tongue/ fetor. Slight epigastric hypersensitivity to
compression. No other internal disorders. The patient is slightly somnolent,
and cannot maintain his attention. The patient has a degree of
temporal disorientation, is indifferent, and has no interest in his surroundings.
His speech and movement are slow. He does not cooperate
well during the examination. Neurologic status: mild occipital stifness;
the right pupil is wider than the left one. He barely feels a needle prick
or a touch of cotton wool on the left side of his face and body. When
showing his teeth the left corner of his mouth does not follow the right
one. Hyperreflexia of the proprioreflexes of the left limbs is present.
The grip of his left hand is weaker. The patient's elevated extremities
tend to fall down. The left extremities are more clumsy in performing
pointing tests. Instability in the Romberg's position.
6.1021 / 1. Select One Of The Key Combinations
Which of the following pathological processes is the most probable?
1) superior hemorrhagic polioencephalitis
2) chronic subdural hematoma
3) multiform glioblastoma
4) Cerebral arteriosclerosis
5) Alzheimer's or Pick's disease
A) only (1) and (2) are true
B) only (4) and (5) are true
C) only (2) and (3) are true
D) only (1), 2) and (4) are true
E) only (3) and (4) are true
6.1021/2. Select One Of The Key Combinations
Which of the following processes are the least probable?
1) cerebral embolism
2) metastatic tumor
3) medulloblastoma
4) astrocytoma
5) encephalitis virus
A) only (1), (2) and (3) are true
B) only (2), (4) and (5) are true
C) only (1), (3) and (4) are true
D) only (1), (3) and (5) are true
E) only (2) and (4) are true
6.1021/3. Single Choice Question
Correct measures taken by the family doctor includes:
A) close observation at home: prescription of analgetics,
antiemetics, sedatives, vasodilators; (The following steps will
depend on the course of the disease).
B) admission to find any suspected primary tumor

C) emergency admission to an internal medicine ward


D) emergency admission to a neurologic -neurosurgery ward
E) the patient should first be attended by the family doctor and
should be examined (fundus study) by an ophthalmologist the
next day; then the patient should be sent to a neurologist for
further study.
6.1021/4. Single Choice Question
The possible localization of this pathological process is:
A) supratentorial
B) the posterior cranial fossa
C) the craniospinal area
D) the spinal cord
E) the extracranial part of a large artery supplying the brain
6.1021/5. Single Choice Question
If the process is due to a malignant glioma, which outcome would
you expect after a successful operation?
A) death within several weeks
B) relapse after 1/2 - 1 year
C) a 10-12 year remission
D) a full recovery is expected
E) extrecerbral metastases can develop
NEU-6.1022. Case Study
A 41-year-old female patient, (occupation - usher), with no history of
previous illnesses was admitted to the hospital. The day before admission
she complained of blurred vision, nausea, and a severe, pulsating
tempo-occipital headache on the right side which lasted for about 4
hours, then ceased. The patient felt that the left extremity became
"clumsy", but it lasted only till the next morning. The patient reported
that she had similar severe, pulsating, left- or right-sided headaches
with nausea during menses for about 20 years. During two pregnancies
the headaches never occurred. At admission: her neurologic status
was negative, blood pressure 120/80 mmHg.
6.1022/ 1. Single Choice Question
Which disease would you suspect?
A) a focal epileptic attack
B) a multiple sclerosis attack
C) migraine with complications
D) glaucoma
E) malaise due to increased intracranial pressure
6.1022/2. Single Choice Question
What should be done?
A) examination and medication
B) ophthalmologic consultation
C) immediate admission to a neurosurgical ward
D) refer the patient to a neurologic ward
NEU-6.1023. Case Study
A 74-year-old male patient, (occupation - tailor) has a 20 year history
of hypertension. For 15 years he systematically takes reserpine
(Rausedyl) - occasionally. with thiazide diuretics. Three years ago the
patient noticed that his movement had slowed down, he could only

walk in small steps, had difficulties in initiating movement or turning


around. He has also aquired a stooping posture. He felt that his extremities
became rigid and he could talk in a low voice. During several
months his gait became uncertain and he reported falling down when
turning around.
At admission: slight occipital rigor, stone face, monotonous
speech, low voice. Cranial nerves normal. Inertia, uniform
proprioreflexes, loss of extroreflexes, no pathological reflexes. All extremities
exhibit a medium rigorous tone increase with predominance
in the lower limbs and on the right side. Positive pulsion tests;
semiflexion posture. Small-paced, shuffling, hypokinetic gait with difficulties
at starting and turning. Postural instability.
6.1023/1. Single Choice Question
Which disease would you suspect?
A) cerebral arteriosclerosis
B) depression
C) myasthenia gravis
D) Parkinson's syndrome
NEU-6.1024.
A 57-year-old metal worker has been treated for gastric disorders for
15 years. Eight days before admission the patient was digging, when
he observed numbness and weakness of his toes and both feet. His
family doctor told him to have a rest and consult him again if the
symptoms persisted. The next day the weakness had spread up to his
knees; the upper limbs also became numb and clumsy and small objects
slipped from the patient's hands.At admission: neurologic status:
intact cranial nerves; hyporeflexia of the proprioreflexes in both
upper limbs; no pathological reflexes in the upper limbs. Proprioreflexes
of the lower limbs cannot be elicited. No pathological reflexes in the
lower limbs. Hypotonia of all the muscle groups of the body. A latent
paralysis test reveals difficulties in keeping his upper limbs in position;
pronation occurs on the left side. Severe paralysis of the lower
limbs increasing toward the distal parts, with left predominance. Sensibility:
hypesthesia to all modalities in all the extremities with left
and lower limb predominance increasing in the distal directionerve
The patient can stand with help but cannot walk. Lumbar puncture:
CSF: cell count: 1 /mm3; total protein: 120 mg; CSF protein level (Pndy):
+++; benzoic: normal. EEG: negative.
6.1024/ 1. Single Choice Question
Which disease would you suspect?:
A) multiple sclerosis
B) a spinal tumor
C) Guillain-Barr's syndrome
D) leukoencephalitis
6.1024/2. Single Choice Question
What therapy would you recommend?
A) bed rest, observation at home
B) physical therapy
C) rheumatological consultation
D) emergency admission to a neurologic ward, where a respirator
is available.

NEU-6.1025.
A 23-year-old female with no history of any major illnesses other than a
tonsillectomy and appendectomy. Two months ago she delivered a healthy
child. Two weeks ago the patient experienced severe psychological trauma,
when she learned from her neighbour that her husband had been unfaithful.
A week ago she complained of increasing numbness and a "dull"
feeling in the lower limbs. Though the patient does not complain of vertigo,
her gait became unsteady. Two days ago she also felt numbness on
the right side of her face and complains of diplopia while looking to the
right. Furthermore, she remembered, that these visual disorders had
started about 1.5 year ago when she had blurred vision for about one
month. When she finally decided to consult a doctor the complaints
ceased. Her general condition is normal, she never complained of headaches
or nausea. The patient is afebrile.
At examination: blood pressure 120/70 mmHg; no internal alterations;
vision: right: 0.5, left: 0.8. At the fundus clear papillary borders, with
more pale temporal areas. The patient complains of diplopia while looking
to the right. During this performance the right bulbus scarcely
turns outward. At gazing upward and to the left a nystagmus with a
large wave rotatory component, corresponding to the direction of the
gaze develops. When showing her teeth the left corner of her mouth
does not quite follow the right one. The rest of the cranial nerves are
normal. Marked hyperreflexia of the proprioreflexes in the upper extremities,
and sustained increased proprioreflexes in the lower limbs.
Bilateral Babinski's and Chaddock's signs. Knee and ankle clonus on
the right side can be visualized. Abdominal cutaneous reflexes cannot
be elicited. Bilateral tactile and painful hypesthesia of spotty, uncertain
character in both lower limbs. Decreased sense of position and
movement in the tendons of the toes. Fatigue of the lower limbs with a
right side prevalence; weakness of dorsal flexion. Sustained bilateral
increase of the extension tone in the lower limbs. Uncertainty, ataxia
in the knee-heel test. Broad-based, clumsy gait. Psychic condition:
hypothymic, emotionally sensitive, several times cried during the examination
- she is convinced that her husband has "ruined her".

6.1025/ 1. Single Choice Question


Select the most probable diagnosis:
A) polyneuropathy
B) a depressive hysteric reaction
C) spastic spinal paralysis
D) multiple sclerosis
E) funicular myelosis
6.1025/2. Select One Of The Key Combinations
Select two most relevant studies:
1) carotid and vertebral angiography
2) pneumoencephalography
3) blood smear and Schilling's test
4) EEG
5) CSF electrophoresis
5) myelography
7) ophthalmological study (object- and color visual field, CFF)
A) only (1), (3), and (7) are true
B) only ( 2), (4), and (5) are true

C) only (1), (3), and (4) are true


D) only (5) and (7) are true
E) only (1), (2), (3), and 4) are true
6.1025/3. Single answer question:
The most probable prognosis is:
A) fatal outcome in several months
B) complete recovery
C) a slow steady progression for many years
D) improvement, but a new attack-like relapse is expected
E) complete recovery with residual symptoms, relapses are not
expected.
NEU-6.1026.
A 37-year-old female patient (occupation: administrator) complains
that the day before, she had a transient right-sided weakness which
lasted for about 15 min. She further complained that for several minutes
she couldn't talk, although she was in touch with her surroundings.
The only other time she had felt similar complaints was three
weeks ago when she had felt numbness in her right limbs which had
lasted for several minutes - she did not consider it so serious illness.
Apart from an appendectomy the patient has not had any serious ilness.
She has taken oral contraceptives for 10 years. At admission: her neurologic
status was normal, blood pressure 130/90 mmHg. Ophthalmologic
and CSF studies gave negative results. A marked slow-wave
activity was observed on the EEG in the left hemisphere.
6.1026/ 1. Single Choice Question
Select the most probable diagnosis:
A) myoclonic seizure
B) functional disorder
C) TIA (transient ischemic attack)
D) "grand mal" seizure
6.1026/2. Single Choice Question
Select the most important diagnostic study:
A) native skull X-ray
B) a detailed psychiatric evaluation
C) ultrasound and CT
D) carotid angiography
ANSWER KEY (NEU-6)
-

1.F
2.T
3.T
4.T
5.T
6.F
7.F
8.T
9.T

101.T
102.T
103.F
104.T
105.T
106.T
107.F
108.F
109.F

201.C
202.C
203.D
204.B
205.B
206.B
207.A
208.B
209.E

301.E
302.E
303.C
304.E
305.E
306.D
307.F
308.D
309.F

401.B
402.D
403.A
404.E
405.A
406.A
407.B
408.A
409.C

501.B
502.A
503.D
504.C
505.E
506.E
507.D
508.E
509.E

601.D
602.D
603.C
604.C
605.B
606.B
607.D
608.A
609.A

701.C
702.B
703.B
704.E
705.B
706.E
707.C
708.E
709.C

801.E
802.B
803.A
804.C
805.A
806.D
807.D
808.D
809.C

10.T
11.T
12.T
13.T
14.T
15.T
16.F
17.T
18.F
19.T
20.F
21.T
22.T
23.T
24.F
25.F
26.T
27.T
28.F
29.F
30.T
31.T
32.F
33.F
34.F
35.F
36.T
37.T
38.F
39.T
40.T
41.F
42.T
43.T
44.T
45.T
46.F
47.F
48.F
49.F
50.F
51.F
52.T
53.T
54.F
55.F
56.F
57.T
58.F

110.T
111.T
112.T
113.T
114.F
115.F
116.T
117.F
118.T
119.TT
120.F
121.F
122.F
123.T
124.T
125.T
126.B
127.B
128.129.B
130.D
131.D
132.C
133.134.E
135.136.B
137.C
138.C
139.A
140.D
141.A
142.C
143.D
144.A
146.C
146.E
147.C
148.B
149.D
150.B
151.C
152.C
153.D
154.C
155.C
156.C
157.A
158.A

210.C
211.B
212.C
213.C
214.D
215.C
216.B
217.D
218.C
219.D
220.D
221.C
222.D
223.B
224.B
225.D
226.227.228.A
229.230.231.B
232.C
233.C
234.D
235.B
236.C
237.D
238.E
239.E
240.A
241.A
242.E
243.E
244.E
245.E
246.D
247.A
248.D
249.E
250.B
251.C
252.D
253.B
254.F
255.E
256.D
257.B
258.D

310.B
311.D
312.E
313.E
314.E
315.E
316.D
317.D
318.D
319.B
320.E
321.C
322.B
323.C
324.C
325.C
326.B
327.328.D
329.C
330.B
331.332.E
333.E
334.C
335.A
336.D
337.B
338.A
339.C
340.C
341.B
342.F
343.C
344.D
345.C
346.C
347.B
348.E
349.C
350.C
351.D
352.C
353.E
354.D
355.C
356.A
357.E
358.E

410.C
411.A
412.E
413.B
414.B
415.A
416.A
417.E
418.B
419.B
420.E
421.E
422.A
423.A
424.B
425.C
426.C
427.428.B
429.D
430.E
431.D
432.C
433.434.D
435.A
436.D
437.E
438-.D
439.E
440.A
441.E
442.C
443.C
444.A
445.E
446.E
447.D
448.D
449.C
450.D
451.D
452.D
453.E
454.D
455.B
456.C
457.D
458.D

510.A
511.E
512.B
513.C
514.E
515.A
516.A
517.A
518.E
519.E
520.C
521.C
522.A
523.A
524.A
525.C
526.D
527.A
528.B
529.530.531.532.B
533.A
534.E
535.536.A
537.D
538.B
539.E
540.B
541.A
542.B
543.A
544.C
545.B
546.D
547.D
548.F
549.A
550.A
551.A
552.E
553.B
554.C
555.B
556.D
557.A
558.F

610.A
611.B
612.C
613.D
614.B
615.B
616.C
617.C
618.C
619.B
620.A
621.B
622.B
623.D
624.B
625.C
626.627.628.E
629.C
630.631.C
632.C
633.C
634.635.E
636.B
637.B
638.G
639.A
640.A
641.E
642.A
643.A
644.F
645.A
646.D
647.C
648.A
649.C
650.C
651.D
652.A
653.D
654.D
655.E
656.B
657.A
658.F

710.D
711.D
712.A
713.B
714.B
715.D
716.D
717.B
718.D
719.B
720.E
721.C
722.E
723.D
724.B
725.A
726.B
727.E
728.C
729.B
730.A
731.B
732.C
733.734.A
735.A
736.A
737.B
738.D
739.D
740.A
741.C
742.B
743.A
744.A
745.F
746.B
747.B
748.B
749.C
750.D
751.A
752.D
753.A
754.A
755.E
756.C
757.D
758.B

810.A
811.C
812.B
813.B
814.C
815.A
816.B
817.D
818.B
819.D
820.D
821.C
822.C
823.B
824.F
825.E
826.C
827.828.D
829.E
830.A
831.A
832.E
833.B
834.B
835.C
836.B
837.B
838.B
839.D
840.A
841.C
842.A
843.A
844.B
845.C
846.B
847.C
848.B
849.A
850.D
851.C
852.C
853.B
854.C
855.A
856.B
857.C
858.A

59.T
60.T
61.T
62.T
63.F
64.F
65.F
66.F
67.F
68.T
69.F
70.T
71.T
72.F
73.T
74.T
75.F
76.T
77.T
78.F
79.F
80.T
81.F
82.T
83.FUUU
84.CCCC
85.NMNMNN
86.F
87.TFT
88.T
89.T
90.F
91.T
92.T
93.T
94.T
95.F
96.F
97.F
98.T
99.F
100.F

159.D
160.C
161.E
162.B
163.C
164.D
165.B
166.C
167.C
168.C
169.B
170.B
171.A
172.B
173.A
174.D
175.C
176.A
177.A
178.A
179.A
180.A
181.D
182.D
183.C
184.A
185.B
186.E
187.B
188.C
189.B
190.D
191.C
192.C
193.C
194.C
195.C
196.D
197.C
198.B
199.D
200.E

259.D
260.E
261.D
262.F
263.D
264.E
265.D
266.E
267.D
268.E
269.D
270.F
271.E
272.E
273.D
274.E
275.E
276.C
277.C
278.E
279.D
280.D
281.A
282.B
283.C
284.D
285.C
286.C
287.D
288.A
289.C
290.B
291.C
292.D
293.C
294.A
295.A
296.C
297.D
298.C
299.G
300.C

359.D
360.E
361.C
362.C
363.C
364.D
365.E
366.C
367.E
368.B
369.C
370.B
371.C
372.C
373.E
374.B
375.E
376.D
377.D
378.D
379.C
380.B
381.E
382.C
383.C
384.B
385.B
386.C
387.C
388.C
389.A
390.D
391.D
392.E
393.A
394.B
395.E
396.A
397.B
398.A
399.A
400.D

459.B
460.B
461.A
462.C
463.A
464.A
465.B
466.A
467.E
468.C
469.B
470.A
471.D
472.D
473.D
474.E
475.A
476.E
477.D
479.A
479.A
480.E
481.E
482.B
483.F
484.B
485.C
486.A
487.C
488.E
489.D
490.B
491.C
492.C
493.C
494.D
495.E
496.E
497.A
498.B
499.A
500.-

559.A
560.C
561.B
562.A
563.E
564.C
565.B
566.B
567.D
568.B
569.A
570.F
571.A
572.C
573.B
574.E
575.F
576.C
577.B
578.C
579.C
580.B
581.C
582.D
583.B
584.D
585.H
586.A
587.B
588.C
589.A
590.A
591.A
592.B
593.B
594.D
595.A
596.B
597.A
598.B
599.C
600.B

659.C
660.E
661.H
662.D
663.C
664.A
665.F
666.A
667.C
668.B
669.C
670.H
671.D
672.C
673.B
674.C
675.C
676.A
677.B
678.D
679.C
680.A
681.D
682.E
683.C
684.C
685.C
686.B
687.C
688.D
689.B
690.D
691.C
692.C
693.A
694.D
695.C
696.C
697.D
698.D
699.C
700.D

901.A
902.E
903.A
904.D
905.C
906.C

915.E
916.D
917.E
918.E
919.E
920.A

929.A
930.E
931.D
932.D
933.D
934.D

943.944.945.946.E
947.E
948.E

957.A
958.B
959.C
960.B
961.A
962.-

971.D
972.A
973.974.975.A
976.F

985.BADEC
986.BCAD
987.BADEC
988.DCBA
FEHG
989.DEABC

759.D
760.D
761.B
762.I
763.B
764.F
765.D
766.C
767.D
768.H
769.E
770.C
771.C
772.A
773.C
774.B
775.C
776.E
777.E
778.E
779.D
780.A
781.C
782.D
783.A
784.E
785.D
786.A
787.A
788.D
789.D
790.B
791.D
792.B
793.A
794.A
795.C
796.A
797.A
798.D
799.A
800.A

859.C
860.A
861.A
862.B
863.C
864.A
865.C
866.E
867.B
868.B
869.C
870.A
871.DE
872.AB
873.BDE
874.BCD
875.BCE
876.ABD
877.ABE
878.CD
879.CD
880.BC
881.BDE
882.ACE
883.ABCE
884.ADE
885.ACDE
886.887.B
888.A
889.C
890.D
891.D
892.E
893.E
894.A
895.E
896.E
897.B
898.D
899.A
900.D

907.B
908.B
909.E
910.D
911.E
912.D
913.914.D

921.D
922.E
923.E
924.E
925.D
926.E
927.E
928.A

997.BACD
998.BACDE
999.CACDA
AACB
1000.CADBE
1001.BCBBA
DBABA
1002.CABCEC
ABDDF
1003.1004.BDCAA
BCDC
1005.CGEFBDA
1006.DACBE
1007.DCABA
1008.ABBAD
ACBB
1009.BABBA
DDCD
1010.ACCE
CFGB

1011.1012.1013.1014.BDEA
1015.1016.CBDE
1017.1018.1019.DC
1020.CCB
1021.1022.CA
1023.D
1024.CD
1025.1026.CC

935.E
936.D
937.E
938.E
939.D
940.D
941.D
942.D

949.A
950.B
951.B
952.B
953.E
954.D
955.B
956.B

963.964.965.E
966.E
967.C
968.D
969.B
970.A

977.C
978.A
979.C
980.D
981.982.DACB
983.984.CABD

INTERNAL MEDICINE (INT-7)


True-False Type Questions
Put T for true statements and F for false statements!!!
INT-7.1. True-False Type Question
Distension is the only natural stimulus which can increase gastric
emptying.
INT-7.2. True-False Type Questions
Fats are not effective in slowing down gastric emptying.
INT-7.3. True-False Type Question
Proximal stomach innervation by the vagus nerve regulates fluid
emptying.
INT-7.4. True-False Type Question
Antrum innervation by the vagus nerve regulates the emptying of
solid contents.

990.BBCADE
991.BDACE
992.993.BAEDC
994.BADC
995.EDACB
996.AAABA
CAAAA

INT-7.5. True-False Type Question


A vagotomy always increases "late-phase" fluid emptying.
INT-7.6. True-False Type Question
In patients with duodenal ulcers the rate of gastric emptying is
higher than that ofnormal individuals
INT-7.7. True-False Type Question
Gastric emptying in patients with gastric ulcers does not differ from
that of normal individuals.
SINGLE CHOICE QUESTIONS
Select the single best response to each of the following
questions!!!
INT-7.11.
The following statements are all true for Exanthematous Typhus,
EXCEPT:
A) a severe drug-resistant headache typically occurs following the
7- to l4-day incubation period
B) hypotension occurs in the most seriously ill patients
C) exanthema develops at the end of the second week
D) splenomegaly occurs in some cases
E) small pink macules appear on the 4th to 6th day
F) severe diarrhea occurs at the onset of the disease
INT-7.12.
All of the following are true for coccidioidomycosis, EXCEPT:
A) acute bronchitis
B) erythema nodosum
C) leukopenia
D) arthritis
E) a positive complement-binding test and a negative skin-test in
cases of a disseminated infection
INT-7.15.
In which of the following diseases does myoglobin appear in the
urine?
A) muscle infarction
B) polymyositis
C) Meyer-Betz disease
D) alcoholic myopathy
E) McArdle syndrome
F) all of the above
G) none of the above
INT-7.16.
All of the following can cause osteoporosis, EXCEPT:
A) a calcium deficiency
B) a phosphate deficiency
C) hypercortisonism
D) hypogonadism
INT-17.
Which of the following can cause demineralization of the bones?

A) tumors
B) bone marrow diseases
C) hyperparathyroidism
D) a vitamin D deficiency
E) all of the above
INT 7.18.
All of the following are used to treat osteoporosis, EXCEPT:
A) calcium
B) vitamin D
C) sodium fluoride
D) calcitonin when estrogen therapy is contraindicated
E) potassium
INT-7.19.
Which class of immunoglobulines primarily occurs in secreted fluids?
A) IgG
B) IgE
C) Igm
D) IgD
E) IgA
INT-7.20.
All of the following are typical of Wiskott-Aldrich syndrome EXCEPT:
A) chronic eczema
B) thrombocytopenic purpura
C) a sex-dependent recessive inheritance pattern
D) anemia
E) thymus aplasia
INT-7.21.
Rheumatoid factor usually occurs together with the following symptoms
EXCEPT:
A) subcutaneous nodules
B) splenomegaly
C) vasculitis
D) neuropathy
E) juvenile rheumatoid arthritis
F) Sjorgen's syndrome
INT-7.23.
Rheumatoid factor can be serologically determined in all of the following,
EXCEPT:
A) systemic lupus erythematosus (SLE)
B) liver disease
C) bacterial endocarditis
D) sarcoidosis
E) Fanconi's syndrome
INT-7.24.
Chondrocalcinosis occurs together with:
A) gout
B) hyperparathyroidism
C) hemochromatosis
D) diabetes mellitus
E) ochronosis

F) all of the above


INT-7.25.
Which of the following can trigger systemic lupus erythematosus
(SLE)?
A) suplight
B) infection
C) medication
D) severe stress
E) all of the above
INT-7.26.
Select the most important restricting side-effect of chloroquine,
when used for systemic lupus erythematosus (SLE) therapy:
A) cardiotoxicity
B) hepatotoxicity
C) ocular toxicity
D) peripheral neuropathy
E) drug fever
INT-7.27.
All of the following are typical cardiopulmonary alterations in
sarcoidosis EXCEPT:
A) hydrothorax
B) pulmonary fibrosis
C) cor pulmonarye
D) hilus lymphadenopathy
INT-7.28.
All of the following are true for osteogenesis imperfecta, EXCEPT:
A) progressive deafness
B) blue sclerae
C) a soft skull with "wormian" bones
D) multiple fractures
E) cataracts
INT-7.29.
Which of the following makes the patient more susceptible to herpes
zoster?
A) a lumbar puncture
B) an arsenic injection
C) lymphoid leukosis
D) a spinal tumor
E) all of the above
INT-7.30.
Impetigo is a superficial infection caused by one of the following bacteria:
A) hemolytic staphylococcus
B) hemolytic streptococcus group A
C) both of the above
D) none of the above
INT-7.31.
Bites can cause necrotic lesions in humans if infected by one of the
following:
A) aerobic microorganisms

B) anaerobic microorganisms
C) both of the above
D) none of the above
INT-7.32.
Streptococcus infections can cause the following diseases EXCEPT:
A) lymphangitis
B) puerperal infections
C) erysipelas
D) pyelonephritis
INT-7.33.
Which site (excluding the lung) is mainly affected by childhood pulmonary
tuberculosis?
A) the mediastinal lymph nodes
B) the kidney
C) the spine
D) the pericardium
E) the peritoneum
INT-7.34.
All of the following alterations occur in syphilis, EXCEPT:
A) condyloma latum
B) mucocutaneous pustular lesions
C) gumma
D) meningitis
E) uveitis
F) condyloma acuminatum
INT-7.35.
All of the following changes are typical of tabes dorsalis, EXCEPT:
A) piercing pain
B) ataxia
C) neurogenic bladder
D) Argyll-Robertson's pupil
E) most of the serological reactions of the serum and CSF are
positive
INT-7.36.
Fetal infection by Treponema pallidum occurs:
A) after the first month of pregnancy
B) after the third month of pregnancy
C) after the fifth month of pregnancy
D) after the eighth month of pregnancy
INT-7.37.
What is the disadvantage of inactivated viral vaccines used against
poliomyelitis?
A) they do not decrease the incidence of non-paralytic poliomyelitis
B) they have no effect on the intestinal phase of the disease
C) they do not prevent spread of infection from patient to patient
D) repeated administration is necessary
E) all of the above
INT-7.38.
All of the following are typical of endogenous pyrogens, EXCEPT:

A) they are low molecular weight proteins


B) they are similar to lysozyme
C) they are heat labile
D) they elicit fever
E) they are inhibited by an endotoxin
INT-7.39.
Which of the following can elicit renal insufficiency?
A) mercury
B) phosphorus
C) carbon-tetrachloride
D) bromate
E) all of the above
F) none of the above
INT-7.40.
All of the following occur in opiate intoxication, EXCEPT:
A) pinpoint pupils
B) hypotension
C) drawsiness
D) deep breathing
E) none of the above
INT-7.41.
All of the following are opiate withdrawal symptoms, EXCEPT:
A) yawning and rhinorrhea
B) sweating and lacrimation
C) dilated pupils
D) piloerection
E) myalgia
F) hypothermia
INT-7.42.
All of the following typically occur in the course of adaptation to
heat, EXCEPT:
A) dilation of the peripheral vessels
B) a decreased circulating blood voume
C) an increased renal blood flow
D) higher levels of serum ADH
E) at the beginning cardiac output increases
INT-7.43.
Case Study:
A 36-year-old male patient had an appendectomy at the age of 20 and
an anal fistula 4 weeks before admission. The patient presently complains
of right-sided chest pain and sometimes has bloody expectoration.
A physical examination revealed a soft pleural rub on the right
side corresponding to the site of pain. The chest X-ray revealed no
pathological changes. Consequently pulmonary embolism was suggested.
Select one of the following examinations to verify or cancel this
diagnosis:
A) chest tomography
B) serum enzyme levels (SGOT, SGPT)
C) perfusion lung scintigraphy
D) inhalation lung scintigraphy
E) only (C) and (D) are true

F) ECG
G) respiratory function test
INT-7.44.
Case Study:
A 60-year-old male patient has suffered for years from chronic
bronchitis and emphysema. Respiratory function tests indicate severe
obstruction. The patient was admitted due to severe respiratory
insuffciency. The ECG showed a 160/min tachycardia (AV-nodal). The
patient has never taken digitalis. The respiratory insufficiency was
treated with oxygen, aminophylline, antibiotics and steroids.
Which of the following should be used for treatment of the arrhythmia?
A) no other therapy is required
B) digitalis
C) beta-blockers (for example pindolol)
D) beta-stimulators (for example albuterol)
E) quinidine
INT-7.45.
Which of the following ECG findings does not occur in right ventricular
hypertrophy?
A) ST depression in II, III, aVF
B) qR complex in V1
C) SI, QIII
D) ST depression in V5-6
E) right axis deviation
INT-7.46.
Case Study:
A 26-year-old primipara patient delivered a 3000 g at-term newborn 4
weeks before admission. 10 days ago she had fever (38,2-38,80C). The
family doctor prescribed oxytetracycline. After 5 days of therapy the
fever still persisted. The patient does not cough, has no sore throat,
but during the last three days observed a decreased urine output.
Moreover the last day before admission she consumed a substantial
amount of fluid but excreted only some drops of urine.
Admission status: pale, yellow face with yellowish scleras. Poor general
condition, weakness. Breasts: normal, without tenderness; no lactation.
At compression a discharge is excreted. Soft palpable abdomen.
Temperature: 38.80C. Heart rate: 100/min; Blood pressure: 110/
60 mmHg; WBC 16000; RBC sedimentation rate: 56 mm/h; Hb: 6.24;
Ht:0.38; SeK+ 7.1 mmol/L; urea-N: 40.0 mmol/L. Liver: enlarged, protruding
from under the costal arch by 2 fingers. Palpable spleen; free
renal region.
Gynecological study: a well-developed cervical opening sensitive to
touch, closed cervical canal; a subinvolved uterus of the size of a man's
fist, tender on palpation.Parametrium of the uterus is bilaterally enlarged
and tender. Adnexes, Douglas space - empty. Sanguinous, odorous
discharge. Coloscopy: smooth surface of the cervical canal.
The diagnosis based on the above findings and symptoms is puerperal
sepsis with oligo or anuria. Select the correct therapy:
A) bed-rest; hepatoprotective diet; vitamins
B) transfusion; cultivation of the lochia; aimed antibiotic therapy
C) immediate surgery (extirpation of the uterus)
D) dialysis. followed by surgery (mentioned in (C) above)
E) bed rest; wide-spectrum antibiotics; Heparin therapy

INT-7.47.
If the mother has measles during the first trimester of pregnancy all
of the following can occur in the newborn, EXCEPT.
A) cardiac disorders
B) cataracts
C) thrombocytopenic purpura
D) deafness
E) the measles virus cannot be identified in newborns
INT-7.48.
All of the following are contraindications of the viable measles vaccine,
EXCEPT:
A) pregnancy
B) leukemia
C) active tuberculosis
D) simultaneous steroid therapy
E) polio vaccination performed 4 weeks before
INT-7.49.
All of the following are useful symptoms which help to differentiated
acute pancreatitis from a perforated peptic ulcer, EXCEPT:
A) hypertension develops in acute pancreatitis
B) a perforated ulcer is more pressure sensitive
C) intestinal sounds are typical of acute pancreatitis
D) in perforated peptic ulcer air is visible under the diaphragm,
while in acute pancreatitis pleural effusion is typical
E) an elevated serum amylase level
INT-7.50.
Which of the following findings observed during a prostate examination
best indicates prostatic cancer?
A) pressure sensitivity
B) an elastic, enlarged prostate
C) a smoothed medial sulcus
D) a prostate protruding into the rectal lumen
E) prostatic nodular induration
INT-7.53.
Which of the following can cause postoperative hypokalemic,
hypochloremic alkalosis?
A) dehydration due to artificial nutrition
B) acute renal insufficiency
C) unrecognized diarrhea
D) nasogastric catheter
E) overnutrition
INT-7.54.
Select the pCO2 value characteristic of pure metabolic alkalosis if
the serum bicarbonate value is 34 mmol/L?
A) 20 mmHg
B) 30 mmHg
C) 40 mmHg
D) 60 mmHg
E) 70 mmHg

INT-7.55.
In gastric fluid loss, all of the following can occur, EXCEPT:
A) an increased urine pH
B) a decreased urine potassium level
C) an increased urine bicarbonate level
D) a decreased urine chloride level
E) increased renin secretion
INT 7.56.
Metabolic alkalosis occurs in all of the following, EXCEPT:
A) diabetes mellitus
B) pyloric stenosis
C) Bartter's syndrome
D) after administration of calcium carbonate
E) as a side-effect of ethacrynic acid therapy
INT-7.61.
Case Study:
A patient with chronic metabolic acidosis treated with alkalic
equivalents to correct the acidosis kept hyperventilating for several
days. Which of the following is the most typical finding in this case?
A) pH 7.4; pCO2 40; HCO3 24
B) pH 7.5; pCO2 40; HCO3 30
C) pH 7.5; pCO2 20; HCO3 16
D) pH 7.3; pCO2 30; HCO3 14
E) pH 7.5; pCO2 50; HCO3 36
INT-7.62.
Which of the following can occur if the pCO2 is 70 mmHg and the
bicarbonate level is 33 mmol/L?
A) chronic respiratory acidosis
B) acute respiratory acidosis
C) respiratory and metabolic acute acidosis
D) respiratory acidosis and metabolic alkalosis
E) none of the above
INT-7.63.
The anion gap can be increased in:
A) ammonium chloride intoxication
B) pyloric stenosis
C) bromide intoxication
D) multiple myeloma
E) liver cirrhosis
INT-7.65.
The kidney excretes nitrogen in correlation with the following compounds,
EXCEPT:
A) uric acid
B) creatinine
C) urea
D) amino acids
E) ketone bodies
INT-7.66.
Urea is formed when one of the following molecules is split:
A) ornithine

B) arginine
C) citrulline
D) succinic acid
E) fumaric acid
INT-7.67.
Which of the following statements about renal urea excretion is FALSE?
A) a low-protein diet improves renal function and the kiney's concentrating
ability
B) the medullary part of the collecting duct is moderately permeable
to urea and ADH increases urea permeability
C) the cortical part of the collecting duct is impermeable to urea
D) the distal convoluted tubule is impermeable to urea
E) the thin ascending part of the loop of Henle has sustained permeability
to urea
INT-7.68.
Which of the following statements about urea excretion is FALSE?
A) at a low fluid intake a large portion of urea is reabsorbed
B) urea is reabsorbed from the medullary part of the collecting ducts
C) reabsorbed urea is re-uptaken by the descending and thin
ascending part of the loop of Henle
D) ADH does not affect urea transport
E) during diuresis urea recirculation is low
INT-7.69.
Which of the following statements about urea excretion is FALSE?
A) during diuresis urea clearance increases to 3 ml/min
B) the lower the urine plasma creatine ratio the lower the urea
clearance
C) at normal urine flow about 50% of the filtered urea is reabsorbed
D) if diuresis is inhibited more urea is reabsorbed from the collecting
duct
E) the filtration pressure of urea in the distal convoluted tubules
is higher than in the loop of Henle
INT-7.70.
Which of the following statements about uric acid transport is FALSE?
A) uric acid is reabsorbed and secreted by the renal tubules
B) transport mainly occurs in the proximal part of the nephron
C) changes in glomerular filtration do not significantly regulate
the urate clearance
D) urate clearance increases with the increase of urine flow
E) urate is mainly transported by diffusion
INT-7.71.
According to data obtained in many hospital patients, hyperuricemia
is primarily caused by:
A) chronic renal insufficiency
B) poorly treated hypertension
C) chronic diuretic therapy
D) obesity and an increased consumption of purines
E) diabetes mellitus in adults
INT-7.72.
Which of the following increases uric acid clearance?

A) sulfinpyrazone
B) angiotensin
C) noradrenaline
D) long-acting thiazide diuretics
E) respiratory acidosis
INT-7.73. Single Choice Question
Which of the following statements about uric acid transport is FALSE?
A) interstitial uric acid concentration is highest in the
juxtamedullary region
B) in proximal tubuh the uric acid transport occurs in both
directions
C) hyperthyroidosis increases the uric acid clearance
D) pyrazinoate mainly decreases the uric acid secretion
E) acetazolamide slightly increases the uric acid excretion
INT-7.74. Single Choice Question
In which of the following cases is the serum uric acid level the
LOWEST?
A) hypertriglyceridemia
B) obesity
C) high-dose aspirin therapy
D) chronic thiazide therapy
E) hyperparathyroidism
INT-7.75.
In which of the following does the damaged kidney cause
hyperuricemia?
A) psoriasis
B) lead intoxication
C) polycythemia vera
D) sickle cell anemia
E) infectious mononucleosis
INT-7.76.
Which of the following cases is the least probable cause of an extremely
high serum uric acid level?
A) nephropathy due to analgetics
B) polycystic kidney disease
C) toxemia of pregnancy
D) hypertension-nephrosclerosis
E) lead-nephropathy
INT-7.77.
Which of the following is the least probable cause of gout?
A) sickle cell anemia
B) chronic lead intoxication
C) Lesch-Nyhan's syndrome
D) chronic myeloid leukemia
E) chronic renal insufficiency
INT-7.78.
Which of the following is the most probable cause of a high uric acid
level?
A) generalized atherosclerosis
B) sarcoidosis

C) hypoparathyroidism
D) anaplastic lung carcinoma
E) aseptic necrosis of the femur
INT-7.79.
Which of the following statements does not refer to the LeschNyhan syndrome?
A) the uric acid level usually exceeds 10 mg/ 100 ml
B) it is dominantly inherited
C) renal concrements and renal insufficiency are common
D) mental retardation, choreoathetosis and self-mutilation are
typical
E) a deficiency of hypoxanthine-guanine-phosphoribosyltransferase
is typical
INT-7.80.
Which of the following statements does not refer to the LeschNyhan syndrome?
A) the renal function is impaired
B) it is transmitted as a sex-linked recessive trait
C) gouty arthritis is typical
D) athetosis is typical
E) it is transmitted as a sex-linked dominant trait
INT-7.81.
Which of the following is accompanied by a low uric acid level?
A) diabetes without glucosuria
B) diabetic ketoacidosis
C) diabetic nephropathy
D) Hodgkin's disease
E) none of the above
INT-7.82.
The most probable cause of hyperuricemia due to chronic thiazide
therapy is:
A) a decreased GFR
B) a decreased extracellular fluid
C) an increased uric acid production
D) hypertension and a decreased renal plasma flow (RPF)
E) an increased production of lactic acid
INT-7.83.
The urate clearance is low after the administration of.
A) aspirin
B) beta-hydroxybutyric acid
C) estrogen
D) probenecid
E) mannitol
INT-7.84.
Which of the following hyperuricemic states is not due to a high
lactic acid level?
A) starvation
B) toxemia of pregnancy
C) Down's syndrome
D) glycogen storage disorders

E) diabetic ketoacidosis
INT-7.85.
Which of the following statements about renal glucose secretion is
true?
A) it depends only on the glomerular filtration rate
B) the secretion occurs in the proximal tubules
C) the Tin glucose value is about 300 mg/min
D) glucose does not appear in the urine until the filtration pressure
is 50% higher than the reabsorption capacity
E) at low serum glucose values, the appearance of glucose in the
urine depends on the ratio between filtration and reabsorption
INT-7.86.
Which of the following statements about glucose reabsorption is FALSE?
A) glucose is reabsorbed with limited active transport
B) the glucose carrier is unknown
C) the Tin glucose value is about 300 mg/min
D) phlorhizin decreases the glucose/insulin ratio
E) under normal conditions only trace amounts of glucose appear
in the urine
INT-7.87.
In diabetic ketoacidosis, the normalization of dehydration affects
glucosuria in the following way:
A) it increases glucosuria
B) it decreases glucosuria
C) the glucosuria remains unchanged
D) the glucosuria decreases if the blood glucose level decreases
E) the glucosuria disappears
INT-7.88.
Parathyroid hormone acting on the tubules causes:
A) the renal tubular reabsorption of calcium
B) an inhibition of adenylate cyclase
C) hypophosphaturia
D) hypomagnesiuria
E) none of the above
INT-7.89.
Which of the following physiological sequences indicates the effect of
a diuretic on the proximal tubule?
A) a significant phosphaturia
B) adecreased free water clearance
C) hyperkalemia
D) the fractional bicarbonate excretion is under 0.2
E) a metabolic alkalosis
INT-7.90.
Which of the following statements about osmotic diuresis is FALSE?
A) urine flow and sodium excretion are proportional to the osmotic
pressure
B) the water withdrawal shifts the urine concentration toward
isotonic values
C) early alterations probably impair the reabsorption of sodium
D) a significant sodium loss occurs in the loop of Henle

E) the medullary concentration gradient decreases


INT-7.91.
Which of the following does not play a role in diuretic-induced
hyponatremia?
A) a decreased glomerular filtration and a consequent increase of
sodium reabsorption in the proximal tubules
B) a decreased effective plasma volume and a consequent stimulation
of ADH secretion
C) a decreased aldosterone level and a consequent increase of
potassium uptake by the cells
D) a decreased chloride and sodium reabsorption in the distal
convoluted ducts
E) a decreased flow of the filtrated fluid to the distal diluting segment
INT-7.92.
Which of the following statements corresponds to the effect of hypertensive
salt or mannitol solution?
A) the free water clearance (Tc H2O) starts to decrease
B) the maximum possible Tc H2O value is 15 ml/min
C) hydrochlorothiazide (Hypothiazid) will stop the Tc H2O
D) the determined Tc H2O value underestimates the true water
reabsorption from the the collecting duct
E) the low sodium transport in the ascending part of the loop of
Henle enhances the Tc H2O
INT-7.93.
Which of the following statements about the antihypertensive effect
of diuretics is FALSE?
A) a decreased plasma renin level
B) . a decreased cardiac output
C) decreased extracellular fluid
D) decreased peripheral resistance
E) a decreased sensitivity to vasoconstrictor agents
INT-7.94.
Acetazolamide usually causes all of the following EXCEPT:
A) an alkaline urine
B) bicarbonate excretion is increased
C) potassium excretion is increased
D) chloride excretion is increased
E) carbon dioxide reabsorption is decreased
INT-7.95.
Which of the following statements about the effect of thiazide diuretics
on potassium excretion is FALSE?
A) they inhibit potassium reabsorption in the proximal tubules
B) potassium diffuses into the distal tubules, the process being
regulated by the transtubular potential of sodium and potassium
C) a higher transtubular fluid flow increases distal. potassium secretion
D) potassium loss can be replaced by giving potassium salt supplementation
E) potassium administration is more efficient when not administered
together with diuretics
INT-7.96.
All of the following are side-effects of thiazide diuretics EXCEPT:

A) transient myopia
B) hearing loss
C) central visual disorders
D) agranulocytosis
E) thrombocytopenia
INT-7.97.
Which of the following statements about the effect of thiazide diuretics
on glucose metabolism is FALSE?
A) a latent diabetes is aggravated
B) there is a detorioration of glucose tolerance
C) a decreased circulating insulin level can be a sequel of potassium
depletion
D) glucose utilization is increased in the peripheral tissues
E) the normalization of hyperglycemia is usually important
INT-7.98.
Which of the following is true if the kidney's diluting capacity is impaired,
but the concentrating ability remains intact?
A) this is due to thiazide diuretics
B) this is due to furosemide
C) this is due to mercury-containing diuretics
D) it is because the medullary part of the loop of Henle is affected
E) it is because the proximal tubule is affected
INT-7.99.
The main mechanism of action of the thiazide diuretics is on:
A) the thin segment of the loop of Henle
B) the pyramids
C) the collecting duct
D) the whole nephron
E) the early portion of the distal tubule
INT-7.100.
Which of the following statements about diuretic-induced metabolic
alkalosis is true?
A) it can be compensated with stimulation of the respiratory
center
B) an increase of the plasma bicarbonate level by 1 mmol/L increases
the arterial pCO2 by 0.9 mmHg
C) metabolic alkalosis is well tolerated by patients with respiratory
insufficiency
D) an efficient therapy means chloride restriction and the administration
of adequate amounts of potassium
E) chloride restriction decreases the pCO2 and improves cyanosis
and the other associated symptoms
INT 7.103.
Which of the following agents is not contraindicated in severe renal
insufficiency?
A) furosemide
B) spironolactone
C) triamterene
D) amiloride
E) potassium tablets (with a long-term effect)

INT-7.106.
Chronic thiazide therapy can cause all of the following symptoms,
EXCEPT:
A) an increased excretion of bicarbonate
B) a decreased excretion of calcium
C) a decreased excretion of uric acid
D) a decreased excretion of potassium
E) an increased excretion of chloride
INT-7.108.
Which of the following statements about furosemide is FALSE?
A) in a maximum dose furosemide is more effective than the
thiazides
B) furosemide is efficient even at very low filtration rates
C) it increases RBF and GFR if severe hypovolemia is not present
D) it maintains the concentrating capacity of the kidney
E) other diuretics potentiate it's effect
INT 7.110.
The chronic administration of furosemide can cause all of the following
symptoms, EXCEPT:
A) hypokalemia
B) hypochloremia
C) hypomagnesiemia
D) hypocalcemia
E) hyperchloremia
INT-7.111.
Furosemide inhibits:
A) chloride transport in the ascending loop of Henle
B) potassium transport in the distal tubules
C) sodium transport in the collecting ducts
D) potassium transport in the proximal tubules
E) potassium transport in the collecting duct
INT-7.112.
Furosemide:
A) decreases renal blood flow
B) increases renal blood flow without increasing the GRF
C) can produce a metabolic acidosis
D) promotes chloride reabsorption in the loop of Henle
E) large doses promote uric acid reabsorption
INT-7.114.
Spironolactone-induced full-scale sodium excretion occurs:
A) within 10-30 min.
B) within 2-4 hours
C) within 12-20 hours
D) within several days
E) none of the above
INT-7.115.
Which of the following statements about the effects of spironolactone
is FALSE?
A) it decreases the excretion of potassium

B) it increases the activity of bicarbonate


C) it may potentiate the effects of antihypertensives
D) it may cause a metabolic alkalosis
E) it causes gynecomastia and impotence
INT-7.116.
Case Study:
A 35-year-old male patient complains of a resistant limb edema which
has persisted for many years. He spent most of his life in tropical
countries. His urine is milk-like. Which of the following explains the
pathological changes of his urine?
A) lipiduria
B) a urinary tract infection
C) gonorrhea
D) chyluria
E) an increased consumption of milk
INT-7.117.
Case Study:
A 25-year-old unconscious female patient was admitted to the hospital.
No friends or witnesses could give any further information on what
had happened Centrifugation of the urine obtained by catheter revealed
fat drops in the sample. Which of the following diagnoses is
possible?
A) eclampsia
B) bone fracture
C) CO intoxication
D) all of the above
E) none of the above
INT-7.118.
Case Study:
Nephrosis was diagnosed in a 15-year-old boy. A renal biopsy revealed
a "minimal change" process. The creatinine clearance determined from
a timed urine sample while the patient remained recumbent was 150
ml/min. Select the possible cause of the high creatinine clearance rate:
A) an increased permeability of glomeruo-capillary surface
B) inaccurate urine sampling
C) a decreased plasma osmotic pressure in the glomeruli
D) an increased renal blood flow (RBF)
E) an increased transcapillary hydrostatic pressure
INT-7.119.
An iso-osmotic urine usually occurs:
A) in acute tubular necrosis
B) in the hepatorenal syndrome
C) in acute glomerulonephritis
D) after a large loss of body fluids
E) after intravenous urography
INT-7.120.
A marked deterioration of the concentrating ability of the kidney can
occur in:
A) hypercalcemic nephropathy
B) acute glomerulonephritis
C) chronic glomerulonephritis

D) after the constant consumption of high-protein food


E) an embolism of the renal artery
INT-7.121.
Case Study:
A 26-year-old female patient is admitted due to a suspected nephrotic
syndrome. The determination of the different molecular mass protein
clearance indicated an albumin predominance. High molecular-mass
globulins were not found in the urine. Select a possible diagnosis:
A) membranous glomerulopathy
B) proliferative glomerulonephritis
C) a "minimal change" process
D) membranoproliferative glomerulonephritis
E) focal glomerulonephritis
INT-7.122.
In which of the following cases is the occurrance of anADH
(Pitressin) -resistant hyposthenuria least probable?
A) obstructive uropathy
B) focal nephritis
C) sarcoidosis
D) amyloidosis
E) hypokalemic nephropathy
INT-7.123.
An impaired concentrating ability with an intact diluting ability occurs
in all of the following cases, EXCEPT:
A) sickle cell anemia
B) hypokalemia
C) adrenal cortex insufficiency
D) hyperparathyroidism,
E) cystic renal disease
INT-7.124.
Which of the of the following is not true in respect to a concentrating
test if the urea nitrogen value is 70 mg%:
A) this test has little clinical value
B) this situation can cause dehydration and is therefore dangerous
C) this will show that the lack of concentrating capability is due
to an osmotic diuresis occurring in the functioning nephrons
D) the test will show a specific density of at least 1.003
E) the administration of ADH can normalize the kidney's concentrating
capability
INT-7.125.
Case Study:
A routine test has revealed proteinuria in a 35-year-old male patient.
Electrophoresis of the urine proteins showed low molecular proteins
at high concentration and only sustained albuminuria. RIA
(radioimmunoassay) identifiedthe predominance of beta 2microglobulins. Which of the following caused the proteinuria?
A) cadmium intoxication
B) mercury intoxication
C) penicillin intoxication
D) glomerulonephritis
E) none of the above

INT-7.126.
The GFR can be accurately evaluated by the creatinine clearance despite
this compound's secretion in the proximal tubules. Normally
spectrophotometry yields higher values, compared to the true creatinine
level, because non-creatinine chromogenic molecules are also measured.
In mild renal insufficiency however, the GFR value determined
via the creatinine clearance value is also higher than than the true
GFR value. Select one of the following explanations:
A) increased creatinine secretion with a lesser elevation of noncreatinine
chromogens .
B) increased creatinine secretion while the non-creatinine
chromogenic level remains at high normal values
C) decreased creatinine secretion and a low level of the noncreatinine
chromogens
D) decreased creatinine secretion while the non-creatinine
chromogen level remains at high normal value
E) none of the above
INT-7.127.
What is the most typical error in the determination of creatinine
clearance in clinical practice?
A) creatinine secretion is not considered
B) the inaccurate collection of urine samples
C) non-creatinine chromogens are not considered
D) changes of endogenous creatinine production are not considered
E) none of the above
INT-7.128.
Though the urea nitrogen level is frequently used for the evaluation of
GFR changes, this parameter is less accurate than the determination
of the plasma creatinine level. All of the following contribute to the
inaccuracy of the former parameter, EXCEPT:
A) any urea production changes depend on the liver function and diet
B) urea absorption depends on the hydration state
C) in renal insufficiency tubular urea secretion is a predominant
process
D) in renal insufficiency the protein content of food detremines
the urea nitrogen level
E) intestinal bleeding increases the serum urea-nitrogen level
INT-7.129.
Case Study:
A 47-year-old diabetic patient developed oliguria after intravenous
urography. The urine osmolarity was 480 mOsm. The most probable
diagnosis is:
A) acute renal insufficiency
B) prerenal azotemia
C) papillary necrosis
D) diabetic nephropathy
E) obstructive uropathy
INT-7.130.
Inulin is used to determine the GFR because:
A) it is excreted in a similar way to creatinine
B) it is totally excreted by the kidney

C) once filtered, inulin is not reabsorbed or secreted


D) it is filtered and secreted but not reabsorbed
E) it is filtered and reabsorbed but not secreted
INT-7.131.
In which of the following diseases do enlarged or normal-size kidneys
never occur?
A) diabetes
B) amyloidosis
C) scleroderma
D) acute tubular necrosis
E) progressive chronic pyelonephritis
INT-7.132.
The maximum urine concentrating ability is determined by fluid deprivation.
Maximum urine concentration occurs in all of the following, EXCEPT:
A) if the body mass decreases by at least 3%
B) if there is no further increase in osmolarity in three subsequent
urine samples
C) after a 16 hour-long fluid withdrawal
D) a shorter fluid withdrawal in patients uncapable of fluid retention
E) if the body mass decreases by at least 1%
INT-7.133.
The maximum diluting ability of the kidney is evaluated after an immediate
consumption of 1200 ml of water. Which of the following determinations
best reflects the maximum diluting ability of the kidney?
A) osmolarity lower than 80 mOsm in at least one sample collected
for 3 hours
B) the total volume of urine collected for 3 hours exceeds 900 ml
C) the maximum urine flow occurs during the 2nd hour
D) all of the above
E) none of the above
INT-7.134.
All of the following can decrease both the maximum diluting and
maximum concentrating ability of the kidney, EXCEPT:
A) furosemide
B) ethacrynic acid
C) glucosuria
D) starvation
E) mannitol
INT-7.135.
The urine of healthy individuals is usually more acidic than the body
fluids due to the production and renal excretion of endogenous acids.
Which of the following is not important in the process of acid excretion?
A) the ammonia concentration
B) the free hydrogen ion concentration
C) the titratable acidity
D) all of the above
E) none of the above
INT-7.136.
Case Study:
In a patient with severe hypertension a normal creatinine level and a

4 g/24h proteinuria were found. Urography revealed an enlarged (16


cm) right kidney and a smaller (7 cm) left kidney with a blurred border.
Results of arteriography suggest stenosis of the right renal artery.
Which of the following should be the next adequate study?
A) a bilateral retrograde urography
B) a right-sided percutaneous renal biopsy
C) left-sided percutaneous renal biopsy
D) a right-side biopsy with surgical approach
E) the determination of the renin concentration in the renal veins
INT-7.137
The serum complement level is low in all of the following, EXCEPT:
A) membranous glomerulonephritis
B) mesangiocapillary glomerulonephritis
C) bacterial endocarditis
D) shunt nephritis
E) systemic lupus erythematosus (SLE)
INT-7.138 Single Choice Question
What do the WBC casts appearing in the urine of a patient with
glomerulonephritis due to an acute Streptococcus infection indicate?
A) proof of the Streptococcus infection
B) a relapsing pyelonephritis
C) nothing but elements originating from the lower urinary tract
D) an acute exudative glomerulonephritis
E) an overdose of aspirin
INT-7.139.
Case Study:
An elderly stuporous, hypertensive, dehydrated patient is admitted to
the hospital from a home for the elderly. The urine sodium level is
under 10 mmol/L. Select one of the following diagnosis:
A) Addison s disease
B) hyperaldosteronism
C) sodium-losing kidney
D) extrarenal sodium loss
E) inadequate ADH secretion
INT-7.140.
Case Study:
A 24-year-old male patient with oliguria is admitted to the hospital.
The urine sodium level is 40 mmol/L. This finding can occur in all of
the following, EXCEPT:
A) acute renal insufficiency
B) incomplete obstruction of the urinary tract
C) prerenal azotemia
D) furosemide therapy
E) therapy with ethacrynic acid
INT-7.141.
Case Study:
A 25-year-old female patient is admitted with metabolic alkalosis. The
chloride concentration in the urine exceeds 110 mmol/L. This fording
can occur in all of the following, EXCEPT:
A) severe vomiting
B) diuretic therapy

C) severe potassium loss


D) Bartter's syndrome
E) Cushing's syndrome
INT-7.142.
In which of the following cases does the test-paper, containing
tetrabromphenol-blue, show a pseudo-positive result?
A) the urine is collected after urography
B) after high-dose penicillin therapy
C) if the urine is too opaque
D) all of the above
E) none of the above
INT-7.143.
Case Study:
Oliguria develops in a 72-year-old female patient after surgery due to an
aortic aneurysm. Both renal arteries are involved. After a fluid load and
furosemide therapy diuresis increased to 100 ml/hour. Select the best answer:
A) a transient acute tubular necrosis has occurred
B) prior to furosemide treatment the urine sodium concentration
was 60 mmol/L
C) the ratio of the urine urea to the plasma urea is 20:1
D) furosemide treatment was a wrong decision
E) anticoagulants could have elicited this pathological state
INT-7.144.
Case Study:
A 49-year-old male patient was examined due to proteinuria. After
intravenous urography the renal function had deteriorated. Which of
the following cannot cause this renal disorder?
A) diabetes mellitus
B) a low uric acid level
C) multiple myeloma
D) a decreased fluid volume
E) a pre-existing azotemia
INT-7.145.
Case Study:
A 45-year-old patient's blood pressure is 140/95 (stage 4); the patient
has no other risk factors. Which of the following should be performed
after a 6-month observation period?
A) propranolol therapy should be started
B) potassium chloride therapy should be started
C) diuretic therapy combined with a reduced sodium uptake
should be started
D) the stage-5 diastolic pressure should be checked
E) a larger cuff should be used and the patient's ECG should be taken
INT-7.146.
Which of the following statements is true for the taking of blood
pressure?
A) place the arm at the level of the heart; if the arm is not supported,
the diastolic pressure can be 10% lower
B) if the blood pressure values taken in both arms is different,
the lower value is the correct one
C) in mild hypertension the diastolic pressure taken in an

orthostatic patient is lower than in the recumbent position


D) the first pressure value is usually lower than those obtained in
subsequent determinations
E) the pressure difference between the two arms indicates the
"subclavian steal" syndrome
INT-7.147.
The most probable cause of a fluctuating blood pressure reading is:
A) that the patient does not take diuretics
B) insufficient drug absorption
C) an anxious patient
D) that a non-reliable manometer was used
E) all of the above
INT-7.148.
Which of the following statements about hypertension is FALSE?
A) if the diastolic pressure is over 90 mmHg, life expectancy is
always reduced in all patients
B) the official WHO definition of normotension is: systolic pressure
140-160 mmHg; diastolic pressure 90-95 mmHg
C) the rate of pressure increase is higher in pre-existing hypertension
D) females tolerate hypertension better than males
E) 15-20% of the adult population in the USA is hypertensive
INT-7.149.
Which of the following statements is true for a "borderline" hypertension?
A) the cardiac output is usually increased
B) the blood volume is usually increased
C) the peripheral vascular resistance is usually increased
D) there is an increased role of the alpha-adrenergic nervous system
E) these patients are usually resistant to beta-blockers
INT-7.150.
Most epidemiologic studies agree on all of the following EXCEPT:
A) renovascular diseases occur in 0.05% of hypertensive patients
B) the incidence of pheochromocytoma is 0,2%
C) essential hypertension comprises 89-95% of all cases of hypertension
D) the incidence of chronic renal diseases is 2-5%
E) none of the above
INT 7.151.
Which of the following pressure ranges is considered "borderline hypertension"
in young patients?
A) 140/90-160/95 mmHg
B) 120/80-140/90 mmHg
C) 120/80-165/90 mmHg
D) 130/85-140/95 mmHg
E) 130/85-139/89 mmHg
INT-7.152.
The blood pressure can increase:
A) in obesity
B) after consumption of substantial amounts of alcohol
C) due to smoking
D) in gout
E) all of the above

INT-7.154.
Which of the following factors is the least important in the direct
regulation of blood pressure according to Guyton's theory?
A) the arteriolar peripheral resistance
B) the cardiac output
C) the state of the venous system
D) the baroreceptors
E) the regulation of the renal volume
INT-7.157.
All of the following transport disorders occur in the red blood cells of
hypertensive patients, EXCEPT:
A) an increased intracellular sodium content
B) an increased potassium-sodium exchange with enhanced potassium
outflow
C) an increased passive sodium influx
D) an increased sodium/potassium exchange with enhanced sodium
outflow
E) an increased sodium/lithium exchange
INT-7.159.
In essential hypertension:
A) the serum sodium level is high
B) the serum sodium level is low
C) the urine sodium level is high
D) the urine sodium level is low
E) there is an increased sodium content in the vessel wall
INT-7.161.
In essential hypertension the intracellular concentration of:
A) calcium is elevated
B) magnesium is elevated
C) potassium is elevated
D) calcium is decreased
E) sodium is decreased
INT-7.162.
Which of the following statements about the baroreceptors of the
carotid sinus and the aortic arch is FALSE?
A) in animals full denervation causes constant hypertension
B) the baroreceptor sensitivity slightly decreases in essential hypertension
C) the baroreceptor activity can increase in young individuals
with "borderline hypertension"
D) at decreased baroreceptor activity, the peripheral resistance
and diastolic pressure can markedly increase in the
orthostatic position
E) a higher pressure is necessary to activate baroreceptors in hypertension
INT-7.163.
All of the following hormones participate in the regulation of blood
pressure in humans, EXCEPT:
A) adrenaline
B) noradrenaline
C) renin
D) vasopressin

E) aldosterone
INT-7.164.
Catecholamines exhibit all of the following effects, EXCEPT:
A) anxiety followed by hyperventilation, palpitation and tremor
B) a long-term elevation of blood pressure
C) an increased cardiac output
D) an increased muscle blood flow
E) an increased renal blood flow
INT-7.165.
Which of the following statements about prostacyclin (PGI2) is
not true?
A) a dilation of the peripheral arterioles occurs
B) it is metabolized in the lung
C) it is a potent stimulus for the excretion of sodium
D) it stimulates the release of renin
E) it exhibits a negative chronotropic effect
INT-7.166.
In which of the following diseases does kallikrein secretion decrease?
A) essential hypertension
B) pheochromocytoma
C) primary aldosteronism
D) all of the above
E) none of the above
INT-7.167.
Angiotensin infusion administered to normotensive patients can
cause all of the following symptoms, EXCEPT:
A) an increased glomerular filtration rate (GFR)
B) a decreased renal salt excretion
C) a decreased renal blood flow (RBF)
D) antidiuresis
E) an increased prostaglandin production
INT-7.168.
Which of the following statements is true for angiotensin I?
A) it is the most potent vasoconstrictor known
B) it is activated by the liver converting enzymes
C) it consists of 10 aminoacids
D) it stimulates aldosterone release
E) it is formed from the "big renin"
INT-7.169.
Which of the following statements about angiotensin II is FALSE?
A) it is an octopeptide
B) it constricts the arterioles
C) under certain conditions causes Na retention
D) under certain conditions causes Na excretion
E) in hypovolemia it acts on the adrenal cortex decreasing
aldosterone secretion
INT-7.170.
Which of the following statements about angiotensinogen (renin
substrate) is FALSE?

A) it is mainly produced in the liver


B) its concentration is markedly increased after bilateral
adrenalectomy
C) its concentration remains unchanged after bilateral adrenalectomy
D) its concentration decreases in liver cirrhosis
E) estrogens increase its level
INT-7.171.
Which of the following does not stimulate renin release?
A) venous occlusion
B) prolonged orthostatic position
C) furosemide iv.
D) indomethacin
E) metolazone
INT-7.172.
Which of the following mechanisms inhibits renin release?
A) a decreased pressure in the afferent arteriole
B) an elevated Na or Cl concentration in the macula densa
C) stimulation of the sympathetic renal nerves
D) hyperkalemia
E) adrenaline
INT-7.173.
Which of the following is not an angiotensin II effect?
A) constriction of vascular smooth muscle
B) stimulation of aldosterone secretion
C) decreased thirst
D) increased catecholamine secretion
E) stimulation of the vasomotor center
INT-7.174.
In which of the following is increased plasma renin activity expected?
A) congenital adrenal hyperplasia
B) vegetative dysfunction
C) androgenic therapy
D) Cushing's disease
E) estrogen therapy
INT-7.175.
In which of the following is the angiotensinogen (renin substrate)
level abnormally high?
A) during estrogen therapy
B) Hodgkin's disease
C) colitis
D) essential hypertension
E) leukemia
INT-7.176.
The aldosterone level is always increased in:
A) essential hypertension
B) malignant hypertension
C) pheochromocytoma
D) Liddle's syndrome
E) polycystic kidney disease

INT-7.177.
Which of the following decreases the production of aldosterone?
A) malignant hypertension
B) Addison's disease
C) the nephrotic syndrome
D) bilateral adrenal hyperplasia
E) liver cirrhosis
INT-7.178.
Which of the following is the most important vessel alteration developing
in elderly individuals?
A) fibrinoid necrosis
B) intima hyalinization
C) media hypertrophy
D) intima proliferation
E) cystic degeneration of the media
INT-7.179.
Which of the following methods is commonly used for the determination
of the renin concentration?
A) determination of the angiotensin I level
B) determination of the renin with radioimmunoassay (RIA)
C) determination of the renin substrate with RIA
D) via the aldosterone secretion rate
E) determination of the angiotensin II level
INT-7.180.
Which of the following is not typical of benign nephrosclerosis?
A) hyalinization of the afferent arteriole
B) interstitial fibrosis
C) shrinkage and a low cell content in the glomeruli
D) fibroblastic proliferation of the connective tissue
E) the appearance of a homogenous substance in the internal
part of Bowman's capsule
INT-7.181.
Which of the following histological changes does not occur in
nephrosclerosis?
A) thick irregular glomerular capillary wall
B) periglomerular fibrosis
C) an absence of podocytes
D) tubular atrophy
E) alterations of the elastic membrane
INT-7.182.
Which of the following factors primarily causes thrombosis which
later leads to the formation of an atheroma?
A) platelets
B) fibrin
C) lipids
D) fibroblasts
E) cholesterol
INT-7.183.
Arteriosclerosis:
A) occurs only in hypertension

B) causes hypertension
C) does not affect glomerular arterioles
D) causes severe damge to the kidney
E) damages efferent arterioles
INT-7.184.
Which of the following etiologic types of hypertension is not a sequel
of a high mineralocorticoid level?
A) an adrenal carcinoma
B) a 11-beta-hydroxylase defect
C) Liddle's syndrome
D) a 17-alpha-hydroxylase defect
E) the administration of 11-desoxicorticosterone
INT-7.185.
In which of the following diseases does the plasma renin level always
remain low?
A) essential hypertension
B) aortic coarctation
C) pheochromocytoma
D) Conn's syndrome
E) Cushing's syndrome
INT-7.186.
In a renal allotransplantation hypertension can occur in all of the
following, EXCEPT:
A) stenosis of the renal artery
B) chronic rejection
C) corticosteroid therapy
D) the synthesis of renin by the recepient's kidney
E) azathioprine therapy
INT-7.187.
What is the most common cause of cerebral hemorrhage in hypertension?
A) atheroma
B) berry aneurysm
C) Charcot-Bouchard aneurysm
D) rupture of arteriosclerotic vessels
E) a thrombosis
INT-7.188.
Which of the following renal biopsy findings is the least probable in
a patient with severe hypertension, papilla edema, and uremia?
A) glomerular proliferation
B) focal necrotizing glomerulonephritis
C) marginal leukocytosis
D) arteriolar hyaline formation
E) fibrinoid deposits and polymorphonucleocytes in the arteriolar
wall
INT-7.189.
Which of the following alterations does not occur in primary malignant
nephrosclerosis?
A) proliferative endarteritis of the afferent arterioles
B) proliferative endarteritis of the small interlobular arteries
C) diffuse proliferative glomerulonephritis

D) necrotizing arteriolitis
E) necrotizing glomerulitis
INT-7.190.
Case Study:
Select the correct diagnosis: the patient suddenly developed unilateral
blindness, acute pancreatitis and renal failure?
A) polyarteritis nodosa
B) obstructive uropathy
C) acute tubular necrosis
D) atheroembolic renal disease
E) acute glomerulonephritis
INT-7.191.
Case Study:
A 32-year-old male patient complains of various fits of hypertension
and headaches. The repeatedly determined urine and plasma
vanillylmandelic acid, catecholamine, and metanephrine values were
normal. Select a possible explanation:
A) cerebral astrocytoma
B) a non-functioning adrenal tumor
C) aortic coarctation
D) aberrant adrenal tissue
E) an inadequate sampling of the urine
INT-7.192.
Case Study:
The blood pressure of a pregnant woman (24th week of gestation)
was 170/ 100 mmHg. Select a possible explanation:
A) borderline or mild hypertension
B) pseudohypertension of pregnancy
C) hyperkinetic syndrome
D) severe hypertension
E) sustained hypertension
INT 7.193.
Which drugs or drug combinations should be avoided in the therapy
of pheochromocytoma?
A) dibenzyline
B) beta-blockade followed by alpha blockade
C) alpha-blockade followed by beta blockade
D) prazosin
E) clonidine
INT-7.194.
Which of the following studies will yield pathological values in
neurofibromatosis with hypertension?
A) the determination of the concentration of metanephrine in a
24-hour urine sample
B) the plasma renin activity
C) the serum cortisol level
D) the determination of the concentration of aldosterone in a 24hour urine sample
E) the fractional potassium excretion
INT-7.195.

In which of the following diseases, (which are accompanied with


orthostatic hypertension), does hyperglycemia, glucosuria,
hypermetabolism, and type-II multiple endocrine neoplasm occur?
A) Cushing's disease
B) essential hypertension
C) malignant hypertension
D) pheochromocytoma
E) cerebellar hemangioblastoma
INT-7.196.
Which of the following substances can erroneously alter the
vanilly1mandelic acid values ?
A) methyldopa
B) MAO-inhibitors
C) clofibrate
D) ethanol
E) nitroglycerin
INT-7.197.
Which of the following is typical of familial juvenile nephrotuberculosis?
A) medullary sponge kidney
B) medullary cystic kidney disease
C) chronic interstitial nephritis
D) chronic pyelonephritis
E) polycystic renal disease
INT-7.198.
Which of the following is typical in medullary cystic renal disease?
A) the course is usually symptom-free
B) the cysts are usually localized in the cortex
C) a progressive renal insufficiency
D) enlarged kidneys
E) diagnosis is based on the data of intravenous pyelography
INT-7.199.
Which symptom is not common in medullary sponge kidney?
A) relapses of hematuria
B) relapses of nephrolithiasis
C) relapses of urinary tract infections
D) anemia
E) anomalies which can be noted in the intravenous pyelogram
INT-7.200.
Foamy cells occur in all of the following, EXCEPT:
A) chronic glomerulonephritis
B) lipoid nephrosis
C) membranous glomerulonephritis
D) hereditary nephritis
E) nephrosclerosis
INT-7.201.
Which of the following symptoms is the most typical of unilateral
kidney agenesis?
A) an enlarged contralateral kidney
B) a unilateral absence of kidney on the urographic image
C) a decreased urine concentrating ability

D) a GFR of under 60 ml/min


E) the hemitrigonum can be observed by cystoscopy
INT-7.202.
Which of the following symptoms does not occur in nephroptosis?
A) abdominal seizures in the orthostatic position
B) postural hypertension
C) postural hyperaldosteronism
D) abdominal pain in Trendelenburg's position
E) renal insufficiency
INT-7.203.
All of the following symptoms occur in renal vein thrombosis, EXCEPT:
A) severe lumbar pain
B) ureter incisure on the intravenous pyelogram
C) hematuria
D) proteinuria
E) medial deviation of the ureter
INT-7.204. Single Choice Question
Renal vein thrombosis can be a complication of all of the following
EXCEPT:
A) hypernephroma
B) congestive cardiac failure
C) in a patient in the terminal state of papillary necrosis
D) nephrosclerosis
E) nephrosis syndrome
INT 7.206.
Primary renal impairment in disseminated intravascular thrombosis
is due to:
A) a hyaline thrombus
B) swelling of the endothelial cells
C) the formation of "half-moon" lesions
D) the rupture of basal membrane
E) the proliferation of mesangial cells
INT-7.207.
Which of the following changes is considered a late complication of
the hemolytic-uremic syndrome of infants?
A) anemia
B) thrombocytopenia
C) hypertension
D) intravascular coagulation
E) none of the above
INT-7.209.
Which of the following is a sex-dependent, dominantly inherited disease?
A) Wilson's disease
B) cystinuria
C) cystinosis
D) Hartnup disease
E) vitamin D-resistant rickets
INT-7.210.
Which of the following is an autosomic, dominantly inherited disease?

A) Lowe's syndrome
B) Wilson's disease
C) primary renal tubular acidosis
D) nephrogenic diabetes insipidus
E) Fanconi's syndrome
INT-7.211.
Which of the following diseases does not show an autosomal inheritance?
A) vitamin D-resistant rickets
B) cystinosis
C) cystinuria
D) galactosemia
E) the primary excretion of oxalate
INT-7.214.
Which of the following substances occurs in the urine in Hartnup
disease?
A) glucose
B) phosphate
C) protein
D) tryptophan
E) cysteine
INT-7.215.
Which of the following substances does not occur in the urine in
Hartnup disease?
A) histidine
B) tryptophan
C) phenylalanine
D) methionine
E) arginine
INT-7.216.
Which of the following remarks about proximal tubular acidosis is FALSE?
A) a decreased proximal maximum bicarbonate reabsorption
B) an intact distal acidifying system
C) after administration of ammonium chloride the urine pH value
becomes less than 5.4
D) an increased bicarbonate excretion in the urine
E) an impaired distal acidifying system
INT-7.217.
The urine is acidic in case of proximal tubular acidosis if there is a:
A) low bicarbonate serum level
B) low chloride serum level
C) low potassium serum level
D) low sodium serum level
E) none of the above
INT-7.218.
All of the following findings can occur in symptom-fee hay-fever
EXCEPT:
A) nasal and palatal pruritis
B) aqueous nasal discharage
C) hyperemia of the nasal mucosa and neck adenopathy
D) paroxysmal sneezing

E) increased lacrimation
INT-7.219.
Case Study:
A patient with lung carcinoma becomes weak and lethargic, his serum
calcium level is 16.4 mg/ 100 ml. What is the first therapeutic
intervention?
A) phosphate iv.
B) mithramycin iv.
C) glucocorticoids iv.
D) saline and furosemide iv.
E) calcitonin iv.
INT-7.220.
Which of the following findings strongly indicates cystic fibrosis?
A) familial occurrance of cystic fibrosis; anomalies of pulmonary
function
B) anomalies of pulmonary function; pancreatic insufficiency
C) pancreatic insufficiency; increased level of electrolytes in the
sweat
D) a high serum electrolyte level; anomalies in the chest X-ray
findings
E) anomalies in the chest X-ray findings; afamilial occurrance of
cystic fibrosis
INT-7.221.
Case Study:
A 55-year-old patient has a primary carcinoma of unknown origin.
Which of the following is totally resistant to therapy?
A) adenocarcinima of the prostate
B) adenocarcinoma of the lung
C) adenocarcinoma of the breast
D) germinal cell carcinoma
E) carcinoma of the thyroid gland
INT-7.222.
Eosinophilia primarily occurs in:
A) enterobiasis
B) diarrhea due to Giardiasis (lambliasis)
C) Schistosomiasis
D) measles
E) corticosteroid therapy
INT-7.223.
All of the following can occur as a complication of chronic obstructive
pulmonary disease EXCEPT:
A) cor pulmonarye
B) polycythemia
C) respiratory insufficiency
D) left ventricular insufficiency
E) bronchogenic carcinoma
INT-7.224.
Chronic obstructive pulmonary disease (COPD) can be either
emphysematous or bronchitic, depending on the character of the pathological
alterations in the lung. Though they rarely occur as autonomous

forms, the two diseases can be differentiated on the basis of


their clinical manifestation. Select the most common feature characteristic
of emphysematous and bronchitic COPD:
A) polycythemia
B) bronchodilators can improve the airflow
C) dyspnea
D) a chronic cough
E) hypercapnia
INT-7.225.
Which of the following statements truly reflects the physical signs of
aortic regurgitation?
A) an acute and pronounced increase of the stroke volume accompanied
with the physical symptoms typical of the disease
B) concentric left ventricular hypertrophy is the main compensatory
mechanism
C) a low frequency. blowing diastolic murmur which is better
auscultated in the recumbent position
D) the occurrance of an Austin Flint's murmur indicates sustained
insufficiency
E) the Quincke-type pulse is best observed in the femoral artery
INT-7.226.
Immune deficiency is diagnosed if:
A) the functional activity of the T lymphocytes cannot be identified
neither in vivo, nor in vitro
B) there is a decrease of antibody production
C) the T cells cannot change the yellow color of nitro-blue
tetrazolium into blue
D) neutrophil and macrophage migration is lower than the
chemotaxis stimulus
E) no hemolysis can occur if the serum of the patient is added to
an RBC-anti-RBC antibody system
INT-7.227.
Osteomalacia developing due to an increased excretion of phosphate
is typical of one of the following:
A) a vitamin D deficiency
B) an inherited vitamin D dependence
C) a familial vitamin D resistance
D) anti-seizure medication
E) severe liver pathology
INT-7.228.
Case Study:
A 50-year-old female patient complains of redness, swelling and rigidity
of the distal interphalangeal joints. She has no other
arthralgias. The most probable diagnosis is:
A) erosion osteoarthritis
B) rheumatoid arthritis
C) systemic lupus erythematosus (SLE)
D) ankylosing spondylitis
E) systemic sclerosis
INT-7.229.
Case Study:

In an elderly patient with a marked proteinuria, glomerular pathology


is assumed. A diagnostic kidney biopsy is absolutely
contraindicated if.
A) the diastolic pressure is 120 mmHg
B) the serum creatinine level is 2.5 mg/ 100 ml (the normal value
is 0.8-1.4 mg/ 100 ml)
C) the patient is over 65 years old
D) nephrosis without signs of renal casts is present
E) the patient had previously undergone a renal biopsy
INT-7.230.
Case Study:
A 23-year-old male patient is examined for hypogonadism. Which of
the following findings indicates primary testicular disease (not the
one due to hypothalamic or pituitary causes)?
A) anosmia
B) increased FSH and LH levels
C) an eunochoid appearance
D) decreased libido and potence
E) a decreased sperm cell count and motility
INT-7.231.
Which of the following findings permits a differentiation between
painless thyroiditis and Graves disease?
A) an enlarged thyroid gland
B) a low serum TSH level
C) an increased serum thyroxine level
D) a low uptake of radioactive iodine
E) pressure tenderness and pain in the thyroid gland
INT-7.232.
Which of the following statements is the most typical feature of an
amebic liver abscess?
A) that surgical drainage is necessary
B) it can be caused by one of the six ameba types
C) in the USA it commonly occurs among homosexuals and hospital
patients
D) it usually indicates an intestinal amebic infection
E) it usually occurs in patients with normal serum alkalic
phosphatase levels but elevated transaminase levels
INT-7.233.
All of the following participate in the pathomechanism of an immediate
type hypersensitivity reaction EXCEPT:
A) serotonin
B) bradykinin
C) anaphylactic eosinophilic chemotactic factor
D) basophilic chemotactic factor
E) neutrophilic chemotactic factor
INT-7.234.
All of the following clinical alterations occur in patients with pulmonary
embolism EXCEPT:
A) hypoxia
B) insufficiency of the right heart
C) cyanosis

D) deep vein thrombosis


E) bradycardia
INT-7.235.
Case Study:
A patient with chronic renal insufficiency due to severe chronic hypertension
is examined for chest pain. For the last 2 years the patient
has been undergoing hemodialysis twice a week. Hypotensive episodes
occurred several times during dialysis. The chest pain is localized above
the trapezius muscle; in an orthostatic position it decreases, and during
deep inspiration it increases. Select the most probable cause of
this chest pain:
A) pericarditis
B) coronary disease
C) diffuse esophageal spasm
D) pulmonary embolism
E) costochondritis
INT-7.236.
The cornea is involved in which of the following:
A) sarcoidosis
B) hypercholesterolemia
C) congenital syphilis
D) tuberculosis
E) pneumocystosis
INT-7.237.
All of the following occur in the "afferent loop" syndrome, EXCEPT:
A) vitamin B 12 malabsorption
B) megaloblastic anemia
C) poor absorption of glucose
D) an expansion of the colon flora
E) improvement due to wide-spectrum antibiotics
INT-7.238. Single Choice Question
The most pronounced gastrointestinal fluid absorption occurs in
the:
A) duodenum
B) jejunum
C) ileum
D) colon
E) none of the above
INT-7.239.
All of the following can occur as adverse effects of cimetidine, EXCEPT:
A) increased serum transaminase levels and hepatotoxicity
B) rebound acid secretion after discontinuation of the drug
C) mental confusion
D) agranulocytosis
E) gynecomastia
INT-7.240.
Which of the following occurs in panhypogammaglobulinemia?
A) giardiasis
B) gastric cancer
C) viral enteritis

D) gluten-sensitive enteropathy
E) all of the above
INT-7.241.
The appearance of fetoprotein in the serum of a patient with severe
viral hepatitis suggests one of the following:
A) inevitable death
B) the development of a hepatoma
C) it has no clinical relevance
D) this is a favorable sign of liver regeneration
E) it indicates the development of a chronic active liver disease
INT-7.242.
The most common hepatic lesion in cystic fibrosis is:
A) postnecrotic cirrhosis
B) fat atrophy
C) multinodular biliary cirrhosis
D) Laennec's cirrhosis
E) none of the above
INT-7.243.
Which of the following cardiac surgical interventions causes jaundice?
A) pulmonary valve repair
B) aortic valve repair
C) mitral valve repair
D) tricuspid valve repair
INT-7.244.
In which of the following does a radical cyst develop in the mouth?
A) at the root of a retained tooth
B) at the crown of a retained tooth
C) at the root of a carious tooth
D) as a residual formation of the thyroglossal duct at the root of
the tongue
E) if there is an obstruction of the salivary duct, at the root of the
tongue
INT-7.245.
All of the following can cause mechanical ileus EXCEPT:
A) cholera
B) hernia
C) carcinoma
D) volvulus
E) gallstones
INT-7.246.
All of the following can be caused by a gallstone EXCEPT:
A) biliary cirrhosis
B) acute pancreatic necrosis
C) atrophic cirrhosis
D) chronic cholecystitis
INT-7.247.
Which of the following is affected in Dupuytren's contracture?
A) the tendons
B) the muscles

C) the palmar fascia


D) the joint
E) the meniscus
INT-7.248.
Dupuytren's contracture:
A) is a congenital pathology
B) causes extension contracture of the fingers
C) occurs in pseudosarcomatous fibromatosis
D) is caused only by mechanical factors
E) is more common in women
INT-7.249.
All of the following cause osteoplastic bone metastases, EXCEPT:
A) gastric cancer
B) breast cancer
C) bronchial cancer
D) prostate cancer
E) hypernephroma
INT-7.250.
Case Study:
A middle-aged male patient is examined for back pain. The X-ray study
showed round-shaped hypodense regions in the lumbar vertebra. Laboratory
findings also include an increased sedimentation rate and high
levels of serum alkaline phosphatase. Select the most probable diagnosis:
A) prostate carcinoma metastasis
B) gastric carcinoma metastasis
C) hypernephroid carcinoma metastasis
D) osteolytic osteosarcoma
E) benign osteoblastoma
INT-7.251.
Select the correct localization of a concretion in case of hydrops
vesica felleae:
A) the hepatic duct
B) the cystic duct
C) the common bile duct
D) Vater's papilla
E) Wirsungian's duct
INT-7.252.
Combined tumors of the salivary gland:
A) usually have a submaxillary localization
B) are usually malignant
C) usually develop in the parotid gland
D) usually cause facial nerve paralysis
E) typically cause sialolithiasis
INT-7.253.
Select the correct term for transplantation of tissue-friendly foreign
materials:
A) replantation
B) alloplasty
C) homologous transplantation
D) autogenous transplantation

E) isologous transplantation
INT-7.254.
Select the single correct statement:
A) preblastomatosis is a pathological alteration preceding skin
cancer
B) preblastomatosis is a pathological alteration which leads to the
development of a tumour
C) preblastomatosis is a long-term pathological alteration which
does not lead to malignancy
D) preblastomatosis means pre-invasive cancer
E) hematologic myeloid tumors are defined as preblastomatosis
INT-7.255.
Pneumothorax can be caused by all of the following EXCEPT:
A) trauma
B) bullous emphysema
C) damage of the thoracic duct
D) intersitial emphysema
E) positive pressure artificial ventilation
INT 7.256.
Case Study:
A round-shaped shadow of 3 cm in diameterwas found on an X-ray screening
of a 65-year-old, complaint-flee, smoking male patient. 15 years ago
the patient had undergone a successful operation for colon carcinoma.
Select the most probable diagnosis:
A) tuberculosis
B) carcinoma metastasis
C) aspiration pneumonia
D) bronchial carcinoma
E) pulmonary abscess
INT-7.257.
All of the following are typical of Graves' disease, EXCEPT:
A) obesity
B) goiter
C) left ventricular hypertrophy
D) exophthalmus
INT-7.258.
The most common cause of Addison's disease is:
A) autoimuune adrenal atrophy
B) amyloid accumulation in the adrenals
C) tumor metastasis in the adrenals
D) bilateral adrenal apoplexia
E) adrenal tuberculosis
INT-7.259.
Which of the following hormones is increased in Conn's syndrome?
A) cortisol
B) adrenaline
C) noradrenaline
D) aldosterone
E) cortisone

INT-7.260.
Case Study:
Marked hypertension and its sequels, as well as polyuria and pplydipsia
were found in a middle-aged patient. serum Na: 152 mmol/L; serum
K: 2,2mmo1/L. Select the correct diagnosis:
A) Cushing's syndrome
B) diabetes mellitus
C) Addison's disease
D) Conn's syndrome
E) phaeochromocytoma
INT-7.261.
Xanthoma is common in:
A) diabetes insipidus
B) acute hemorrhagic pancreatitis
C) diabetes mellitus
D) diffuse acute glomerulonephritis
E) liver cirrhosis (Laennec's)
INT-7.262.
Case Study:
A patient's auricular cartillage has a brownish color, the urine sample
turns brown after a while, and the patient complains of
arthralgia. Select a correct diagnosis:
A) erythropoietic porphyria
B) gout
C) ochronosis
D) cystinosis
E) hemoglobinuria
INT 7.263
Which of the following statements is not typical of post-hepatic jaundice?
A) acholic stool
B) direct positive diazo-reaction
C) cholemia
D) increased bilirubin in the urine
E) increased urobilinogen in the urine
INT-7.264.
In all of the following melanine is accumulated in the skin, EXCEPT:
A) Mongolian spot
B) vitiligo
C) nevus coeruleus (blue nevus)
D) ephelis (freckle)
E) chloasma (melasma)
INT-7.265.
Define the term vitiligo:
A) a generalized lack of melanin in the skin
B) a circumscribed lack of melanin in the skin
C) lipopigment
D) reaction in the peritraumatic areas
E) a degeneration of trophoblasts
INT-7.266.
Which of the following statements is typical of albinism?

A) a,tyrosinase enzyme deficiency


B) a decreased level of glucuronide-transferase
C) a decreased level of ceruloplasmin
D) a Kayser-Fleischer circle
E) liver cirrhosis
INT-7.267.
Select the site of iron absorption:
A) the esophagus
B) the stomach
C) the intestine
D) the colon
E) the sigmoid colon-rectum area
INT-7.268.
Which of the following pigments cannot appear in the urine?
A) hemoglobin
B) myoglobin
C) hemosiderin
D) porphyrin
E) melanin
INT-7.269.
Dwarfism occurs in:
A) hypopituitarism
B) pediatric chronic renal disease
C) hypothyroidism
D) rickets
E) fetal chondrodystrophy
F) all of the above
INT-7.270.
Which of the following can occur in chronic systemic congestion?
A) thickening of the alveolar walls
B) esophagus varicosity
C) pulmonary edema
D) spot-like hemorrhages of the gastric mucosa
E) kidney micro-infarctions
INT-7.271.
In anemia:
A) the circulatory volume increases
B) the circulatory volume decreases
C) the blood viscosity increases
D) the circulatory volume is unchanged
E) none of the above
INT-7.272.
Which of the following diseases is complicated by chronic lower leg
ulcers?
A) occlusion of the femoral artery
B) occlusion of the popliteal artery
C) thrombosis of the crural veins
D) occlusion of a lymph duct
E) syphilis

INT-7.273.
In all of the following thrombosis can develop, EXCEPT:
A) leukemia
B) polycythemia
C) systemic lupus erythematosus
D) potassium chlorate intoxication
E) echidnotoxin intoxication
INT-7.274.
In all of the following thrombosis can develop, EXCEPT:
A) circumscribed vein varicosity
B) atrial septum defect
C) enlarged atria or ventricles
D) aneurysms
E) gravidity
INT-7.275.
Define the term "phlebolith":
A) an organized thrombus
B) a venous stone
C) calcification of the venous wall
D) inflammation of the vein wall
E) the inflammation of a thrombus
INT-7.276.
All of the following are sequels of thrombosis EXCEPT:
A) full resorption
B) organization
C) recanalization
D) hyalinization
E) calcification
INT-7.277.
In which of the following should generalized mycosis be suspected?
?r) hypertensive disease
B) cardiac decompensation
C) gastric ulcer
D) long-term antibiotic therapy
E) diabetes insipidus
INT-7.278.
Which of the following symptoms does not belong to the Tetralogy of
Fallot?
A) atrial septal defect
B) ventricular septal defect
C) aortic dextroposition
D) pulmonary stenosis
E) right ventricular hypertrophy
INT-7.279.
In which of the following vascular beds does Buerger's disease commonly
occur?
A) basilar artery
B) arteries of the limbs
C) coronary artery
D) renal artery

E) branches of the mesenteric artery


INT-7.280.
Pericarditis can be caused by all of the following, EXCEPT:
A) uremia
B) transmural myocardiac infarction
C) hematogenic dissemination of tuberculosis
D) rheumatic fever
E) cardiomyopathy
INT-7.281.
Hematuria is typical of all of the following diseases, EXCEPT:
A) acute glomerulonephritis
B)acute pyelonephritis
C) renal tuberculosis
D) renal calculus
E) renal cancer

INT-7.282.
In which of the following does hematuria occur?
A) acute pyelonephritis
B) chronic pyelonephritis
C) acute glomerulonephritis
D) membranous glomerulonephritis
E) amyloid nephrosis
INT-7.283.
Hypertension occurs in all of the following, EXCEPT:
A) acute pyelonephritis
B) acute glomerulonephritis
C) chronic glomerulonephritis
D) nephropathy of pregnancy
E) diabetic glomerulosclerosis
INT-7.284.
Which of the following is the most typical complication of a urinary
bladder calculus?
A) perforation of the urinary bladder
B) urocystitis
C) hydronephrosis
D) pyelonephritis
E) urethritis
INT-7.285.
Initial gonococcemia causes gonorrheal urethritis plus one of the
following:
A) myocarditis
B) endocarditis
C) encephalitis
D) meningitis
E) hepatitis
INT-7.286.
Initial gonococcemia causes gonorrheal urethritis plus one of the
following:

A) myocarditis
B) osteomyelitis
C) arthritis
D) parotitis
E) meningitis
INT-7.287.
Acute diffuse glomerulonephritis:
A) is a purulent renal inflammation caused by Streptococcus
B) is a purulent renal inflammation caused by any pathogen
C) usually occurs following Streptococcus infection
D) about 50% of the cases have a fatal outcome
E) is caused by infected urine
INT-7.288.
The nephrotic syndrome occurs in all of the following, EXCEPT:
A) membranous glomerulonephritis
B) thrombosis of the renal vein
C) paraproteinemic nephrosis
D) cholemic nephrosis
E) Kimmelstiel-Wilson's syndrome
INT-7.289.
In all of the following renal calculi can be formed, EXCEPT:
A) in urinary tract obstruction
B) in congenital metabolic disorders
C) in constant substantial fluid loss
D) along with gallstones
E) in pyelitis
INT-7.290.
The most pronounced enlargement of the lymph nodes occurs in one
of the following types of leukemia:
A) acute lymphoid leukemia
B) chronic myeloid leukemia
C) chronic lymphoid leukemia
D) acute myeloid leukemia
E) erythroleukemia
INT-7.291.
Which of the following red blood cells alterations is typical of irondeficient
anemia?
A) poikilocytosis
B) anisocytosis
C) hypochromia
D) fragility
E) polychromasia
INT-7.292.
Marked splenomegaly occurs in which of the following types of
leukemia?
A) acute lymphoid leukemia
B) acute myeloid leukemia
C) chronic lymphoid leukemia
D) chronic myeloid leukemia
E) polycythemia

INT-7.293.
Myasthenia gravis can be accompanied by a tumor of which organ?
A) adrenals
B) lung
C) thymus
D) ovaries
E) none of the above
INT-7.294.
Which of the following is typical of Hodgkin's disease stage II?
A) the involvement of lymph nodes of a single region and the spleen
B) the involvement lymph nodes of a single region on both sides
of the diaphragm
C) the involvement of lymph nodes of several regions on both
sides of the diaphragm
D) the involvement of lymph nodes of several regions on both
sides of the diaphragm, and alterations in the spleen
E) the involvement of lymph nodes of several regions on one side
of the diaphragm
INT-7.295.
Myofibrotic hepatomegaly is caused by:
A) fatty degeneration
B) absorption disorders
C) centrolobular necrosis
D) cholestasis
E) extramedullary hematopoiesis
INT-7.296
Which of the following can be a sequel of bronchiectasis?
A) left ventricular dilation
B) left atrial dilation
C) a pulmonary artery embolism
D) a pulmonary vein thrombosis
E) secondary amyloidosis
INT-7.297.
Pulmonary emboli originate from:
A) the femoral artery
B) the femoral vein.
C) one of the pulmonary veins
D) the brachiocephalic trunk
E) none of the above
INT-7.298.
Which of the following cannot be a sequel of obstructive pneumonia?
A) carnification
B) otitis media
C) gangrene
D) endocarditis
E) keratitis
INT-7.299.
Which of the following can be a sequel of chronic emphysema?
A) hemothorax

B) pulmonary abscess
C) secondary amyloidosis
D) left ventricular hypertrophy
E) pulmonary edema
INT-7.300.
Which of the following processes can cause serous pleuritis?
A) bronchopneumonia
B) lobar pneumonia
C) tuberculosis
D) lung cancer
E) pulmonary abscess
INT-7.301.
Which of the following can lead to the development of chronic cor
pulmonale?
A) lobar pneumonia
B) thromboembolism of the pulmonary vein
C) lung edema
D) chronic pulmonary emphysema
E) bronchopneumonia
INT-7.302.
All of the following occur in Goodpasture's syndrome, EXCEPT:
A) focal glomerulonephritis
B) septic spleen
C) pulmonary fibrosis; induration
D) cutaneous purpura
E) positive berlin-blue staining in the lung parenchyma
INT-7.303.
Which of the following can develop after long-term antibiotic
therapy?
A) esophageal candidiasis
B) esophageal stricture
C) corrosive esophagitis
D) achalasia
E) diffuse scleroderma
INT-7.304.
In which of the following parts of the gastrointestinal tract does
diverticulosis primarily develop?
A) in the esophagus
B) in the duodenum
C) in the jejunum and ileum
D) in the ascending colon
E) in the sigmoid colon
INT-7.305.
Select the correct name for longitudinal ruptures and bleeding developing
in the lower part of the esophagus and the cardia following
forced vomiting?
A) Waterhouse-Friderichsen syndrome
B) Mallory-Weiss syndrome
C) Hanot's syndrome
D) Stein-Leventhal syndrome

E) Sheehan's syndrome
INT 7.306.
Define Crohn's disease:
A) regional enteritis
B) tabes mesaraica (tuberculosis of the mesenteric glands)
C) stercoral abscess
D) intestinal cystoid pneumatosis
E) necrotizing enteritis
INT-7.307.
Select the correct name for a gastroduodenal ulcer developing together
with a gastrin-secreting pancreatic tumor:
A) Zollinger-Ellison's syndrome
B) Mallory-Weiss's syndrome
C) Waterhouse-Fridrichsen's syndrome
D) Stein-Leventhal's syndrome
E) Sheehan's syndrome
INT-7.308.
Which of the following substances causes coagulation necrosis of
the gastric mucosa?
A) sodium hydroxide (NaOH)
B) phosphorus
C) lead
D) mercury
E) arsenic
INT-7.309.
Case Study:
A patient complains of frequent blushing fits. Asthmatic fits and
profuse diarrhea are also frequent. A physical examination revealed
a harsh murmur above the heart. Select the correct diagnosis:
A) Basedow's disease
B) Carcinoid syndrome
C) Tetralogy of Fallot
D) Polycythemia rubra vera
E) Peutz-Jeghers syndrome
INT-7.310.
The Zollinger-Ellison syndrome is caused by:
A) an adenoma of the pancreatic beta cells
B) an adenoma of the pancreatic alpha cells
C) an adenoma of the pancreatic gamma cells
D) a carcinoma of the exocrine pancreas
E) mucoviscidosis
INT-7.311.
A dilated, rigid, painless gallbladder (Courvoisier's s sign) and jaundice
is typical of one of the following:
A) infectious hepatitis
B) hepatocellular carcinoma
C) cancer of the pancreas head
D) cancer of the pancreas tail
E) calculus in the Wirsungian's duct

INT-7.312.
Which of the following cells primarily infiltrate the portal tract in
acute viral hepatitis?
A) neutrophils
B) eosinophils
C) lymphocytes
D) plasma cells
E) giant cells
INT-7.313.
Coagulopathy developing in obstructive jaundice is due to:
A) secondary thrombocytopenia
B) thrombocytopenia due to decreased vitamin K absorption
C) increased capillary fragility
D) thrombasthenia
E) a low activity of tissue thromboplastin
INT-7.314.
Which of the following can cause priapism?
A) syphilis
B) urethritis
C) induratio penis plastica (penal prosthesis)
D) increased libido
E) epispadiasis
INT-7.315.
Define the term priapism:
A) an extrapyramidal disorder
B) a mental disease
C) pathological erection
D) penis inflammation
E) penis gangrene
INT-7.316.
The most important etiological factor of epididymitis is:
A) syphilis
B) gonorrhea
C) brucellosis
D) blastomycosis
E) listeriosis
INT-7.317.
Vinyl chloride causes cancer in which organs?
A) the liver
B) the adrenals
C) the colon
D) the thyroid
E) the thymus
INT-7.318.
Which of the following substances induces urinary bladder cancer
in humans?
A) p-aminodiphenyl (aniline dyes)
B) benzene
C) hydramine
D) aflatoxin

E) vinyl chloride
INT-7.319.
Asbestos causes cancer in which of the following organs?
A) the kidneys
B) the pleura
C) the brain
D) the adrenals
INT-7.320.
A tuberculotic infection can occur in all of the following, EXCEPT:
A) an airway infection
B) an alimentary tract infection
C) a percutaneous infection
D) a trans-placental infection
E) a sexually transmitted infection
INT-7.321.
All of the following are typical characteristics of polysystemic
autoimmune infections, EXCEPT:
A) a genetic background is important
B) extrinsic factors contribute to the development of the disease
C) an undulating course
D) there is a marked clinical heterogenicity
E) they are more common in the elderly (over 60-year-old)
INT-7.322.
All of the following drugs can elicit systemic lupus erythematosus,
EXCEPT:
A) procainamide
B) hydralazine
C) isoniazide
D) chlorpromazine
E) gold salts
INT-7.323.
All of the following laboratory findings are used for the evaluation of
systemic lupus erythematosus (SLE) activity, EXCEPT:
A) the WBC count
B) the antinuclear antibody level
C) the anti-DNA antibody level
D) the total complement level
E) the creatine-phosphokinase level
INT-7.324.
Which of the following is the most frequently occurring polysystemic
autoimmune disease?
A) systemic lupus erythematosus (SLE)
B) Mixed Connective Tissue Disease (MCTD)
C) dermatomyositis
D) progressive systemic sclerosis (scleroderma)
E) rheumatoid arthritis
INT-7.325.
In systemic lupus erythematosus (SLE) the immune complexes are
deposited in all of the following tissues, EXCEPT:

A) the skin
B) the renal glomeruli
C) the renal tubuli
D) the synovia
E) the thyroid basal membrane
INT-7.326.
All of the following comprise different histologic forms of Lupus nephritis,
EXCEPT:
A) focal glomerulonephritis
B) membranous glomerulonephritis
C) membranoproliferative glomerulonephritis
D) microscopic polyarteritis
E) mesangioproliferative nephritis
INT-7.327.
All of the following are criteria and symptoms of Mixed Connective
Tissue Disease (MCTD), EXCEPT:
A) Raynaud's symptom
B) swollen hands and fingers
C) myositis
D) esophageal dysmotility
E) xerostomia
INT-7.328.
All of the following polysystemic autoimmune diseases contribute to the
development of Mixed Connective Tissue Disease (MCTD), EXCEPT
A) systemic lupus erythematosus (SLE)
B) rheumatoid arthritis
C) progressive systemic sclerosis (scleroderma)
D) Sjrgen's syndrome
E) polymyositis/dermatomyositis
INT-7.329.
All of the following are among the glandular symptoms of Sjrgen's
syndrome, EXCEPT:
A) pharyngitis - bronchitis
B) vulvitis
C) vaginitis
D) pyelitis
E) conjunctivitis
INT-7.330.
All of the following belong to primary vasculitis diseases, EXCEPT:
A) polyarteritis nodosa
B) Chrug-Strauss syndrome
C) Henoch-Schnlein syndrome
D) necrotizing vasculitis induced by hepatitis B
E) Wegener's granulomatosis
INT-7.331.
Select a cytostatic that is most commonly used in the therapy of
vasculitis:
A) azathioprin (Imuran)
B) cyclophosphamide
C) cyclosporine A

D) chlorambucil (Leukeran)
E) methotrexate
INT-7.332.
All of the following are synonyms of proteinase 3 (c-ANCA),
EXCEPT:
A) p29
B) myeloblastin
C) Wegener's autoantigen
D) vinculin
E) azurophil-granule protein
INT-7.333.
The presence of anti SS-A or anti SS-B autoantibodies indicates that
polymyositis/dermatomyositis can be accompanied by one of the
following autoimmune diseases:
A) progressive systemic sclerosis (scleroderma)
B) Sjrgen's syndrome
C) Crohn's disease
D) rheumatoid arthritis
INT-7.334.
Which of the following side effects does not occur in cyclosporine
therapy?
A) nephrotoxicity
B) anemia
C) neurologic toxicity
D) hypertension
INT-7.335.
Which of the following cytokines stimulates the formation of IgE?
A) IL-3
B) IL-4
C) IL-2
D) IL-1
INT-7.336.
All of the following are mast-cell mediators, EXCEPT:
A) histamine
B) heparin
C) platelet-activating factor (PAF)
D) leukotriene B4
E) endothelin
INT-7.337.
All of the following are atopic diseases, EXCEPT:
A) allergic rhinitis
B) allergic bronchial asthma
C) gastrointestinal allergy
D) allergic conjunctivitis
E) hypertensive pneumonitis
INT-7.338.
Select a congenital immune deficiency disease based on a
hypofunction of the NK cells:
A) Bruton-type agammaglobulinemia

B) selective IgA deficiency


C) hyper-IgM syndrome
D) Chediak-Higashi syndrome
E) Job's syndrome
INT-7.339.
In which systemic lupus erythematosus (SLE) type do antihistone
antibodies develop?
A) neonatal SLE
B) drug-induced SLE
C) SLE of the elderly
D) SLE in pregnant women
INT-7.340.
Which of the following cytokines does not play an important role in the
pathomechanism of rheumatoid arthritis?
A) IL-1
B) IL-3
C) TNF-alpha
D) TNF-beta
INT-7.341.
Which of the following side-effects is not typical of Methotrexate?
A) tinnitus
B) hepatic dysfunction
C) myelosuppression
D) ulcerative stomatitis
INT-7.342.
Select the correct therapy of rheumatoid artritis (RA) complicated by
vasculitis:
A) gold salts
B) cyclophosphamide
C) chloroquine
D) sulfasalazine
INT-7.343.
Which study should be performed if infectious arthritis is suspected?
A) X-ray study
B) scintigraphy
C) articular puncture
D) antibiotic therapy
INT-7.344.
All of the following occur in progressive systemic sclerosis
(scleroderma), EXCEPT:
A) pulmonary fibrosis
B) the incidence of antinuclear autoantibodies in the serum is
about 90%
C) chronic active hepatitis
D) dysmotility of the lower esophageal segment
E) Raynaud's syndrome
INT-7.345.
A drug-induced autoimmune hemolytic anemia can be caused by all
of the following drugs, EXCEPT:

A) quinidine
B) alpha-methyldopa
C) penicillin
D) digitalis
INT-7.346.
All of the following diseases can occur in the neonatal period,
EXCEPT:
A) thrombopenia (ITP)
B) myasthenia gravis
C) Basedow's disease
D) pernicious anemia
E) systemic lupus erythematosus (SLE)
INT-7.347.
Which of the following statements is the most important proof that
Hashimoto's thyroiditis is an autoimmune disease?
A) anti-thyroglobulin antibodies are found in the serum
B) anti-Tg autoantibodies are found in the thyroid gland
C) lymphocyte infiltration of the thyroid gland
D) the disease can be transmitted by T cells
E) HLA/B8 and DR3 are more common
INT-7.348.
All of the following autoimmune diseases can accompany
autoimmune adrenalitis, EXCEPT:
A) immune thyroiditis
B) hypoparathyroidism
C) ovarian insufficiency
D) pernicious anemia
E) multiple sclerosis
INT-7.349.
Which hypersensitivity reactions play a role in the pathomechanism
of systemic lupus erythematosus (SLE)?
A) type I and II hypersensitivity reactions
B) type II and III hypersensitivity reactions
C) type III and IV hypersensitivity reactions
D) all of the above
INT-7.350.
The first heart sound is loud in:
A) first degree AV block
B) the Wolff-Parkinson-White (WPW) syndrome
C) mitral stenosis due to calcification
D) aortic stenosis
E) hypertrophic obstructive cardiomyopathy
INT-7.352.
Which of the following ECG changes are typical of Printzmetal's angina
during a fit?
A) ST depression in a localized area
B) ST elevation in a localized area
C) diffuse ST depression
D) diffuse ST elevation
E) none of the above

INT-7.353.
Select the most important effect of a normal dose of nitroglycerin in
angina pectoris (apart from coronary dilation!):
A) relaxation of bronchial smooth muscles
B) dilation of the peripheral vessels
C) constriction of the peripheral arterioles
D) constriction of the peripheral venules
E) none of the above
INT-7.354.
Select the drug of primary choice in supraventricular tachycardia
with narrow ventricular QRS complexes:
A) digoxin
B) procainamide
C) dopamine
D) verapamil
E) propranolol
INT-7.355.
Which of the following most probably occured in the history of a 16year-old boy with diabetes mellitus?
A) a recent viral infection
B) an insulin response to a glucose load is about 50% of the normal
C) a decreased energy uptake is required
D) an initiation of biguanide treatment before starting insulin
therapy
E) the patient can participate in physical training lessons and
can participate in other games requiring physical performance
INT-7.356.
The typical symptom of ischemic colitis in its acute phase is:
A) steatorrhea
B) a normal X-ray picture after a barium meal
C) nausea and vomiting
D) symptoms of generalized peritonitis
E) hemorrhagic diarrhea
INT-7.357.
Which of the following statements is typical of chronic
granulomatosis?
A) neutrophils cannot phagocyte the bacteria
B) chronically enlarged lymph nodes
C) recurrent candidiasis
D) a dominant inheritance pattern
E) hypogammaglobulinemia
INT-7.358.
All of the following can increase the normal insulin requirement, EXCEPT:
A) pregnancy
B) infections, fever, sepsis
C) idiopathic spontaneous exacerbations
D) hypothyroidosis
E) burns due to irradiation and ultraviolet damage of the deep
tissues

INT-7.359.
All of the following can induce systemic lupus erythematosus (SLE),
EXCEPT:
A) chlorpromazine
B) phenytoin
C) aspirin
D) procainamide
INT-7.361.
Which of the following antibiotics can form unsoluble chelates with
the aluminium of antacids, which would then impair their absorption?
A) penicillins
B) tetracyclines
C) erythromycin
D) sulfonamides
E) none of the above
INT-7.362.
The occurrence of hemolytic anemia in methyl-dopa therapy is:
A) less than 1%
B) 5%
C) 10%
D) 15%
E) 33%
INT-7.363.
Case Study:
Select the drug of choice in .a 2-year-old girl with fever and
polyarthritis (diagnosis: juvenile rheumathoid arthritis):
A) cytozan
B) prednisolone
C) aspirin
D) chloroquine
E) penicillinamine
INT-7.364.
Which of the following facilitates the antiocoagulant effect of
coumarin?
A) phenylbutazone
B) multivitamins containing vitamin K
C) high-dose salicylate
D) quinine and quinidin
INT 7.365.
All of the following increase the blood glucose level, EXCEPT:
A) corticosteroids
B) clofibrate
C) diazoxide
D) lithium carbonate
INT-7.366.
Medication recommended for the therapy of pneumococcus pneumonia is:
A) erythromycin
B) streptomycin
C) penicillin G
D) ampicillin

INT-7.367.
All of the following statements are true, EXCEPT:
A) procainamide is an effective antiarrhythmic drug
B) propranolol should not be used in congestive cardiac insufficiency
C) quinidine sulfate reaches its maximum serum value 1.5 hours
following its administration
D) verapamil can change atrial fibrillation into atrial flutter
INT-7.368.
All of the following drugs are used for the therapy of bronchial
asthma, EXCEPT:
A) terbutaline
B) theophylline
C) neostigmine
D) prednisolone
INT-7.369.
Which of the following statements is true for nephrogenic diabetes
insipidus?
A) an autosomal dominant inheritance
B) an autosomal recessive inheritance
C) a low GFR
D) a low serum ADH level
E) in heterozygous women, the urine concentrating capacity is
decreased
INT-7.370.
Where does hypernephroma develop?
A) in the glomerular epithelial cells
B) in the tubular epithelial cells
C) in the epithelial cells of the calyx
D) in the glomerular endothelial cells
E) in the juxtaglomerular apparatus
INT-7.371.
Microhematuria probably will not develop in:
A) lipoid nephrosis
B) membranous glomerulonephritis
C) proliferative glomerulonephritis
D) membranoproliferative glomerulonephritis
E) lupus nephritis
INT-7.372.
Which of the following statements about physiological protein excretion
in humans is true?
A) the daily excretion is under 150 mg
B) the urine protein content rarely exceeds 10-20 mg/ 100 ml
C) albumin comprises about 80% of the total protein content in
normal urine
D) about 25 mg of Tamm-Horsfall's protein is excreted daily
E) the urinalysis test-paper fails to reveal the light-chain proteins
INT-7.373.
Nephrosis occurs in all of the following, EXCEPT:
A) right heart insufficiency

B) constrictive pericarditis
C) obstruction of the inferior caval vein above the renal artery
D) polycystic kidney disease
E) amyloidosis
INT-7.374.
Which of the following criteria is the most important for diagnosis of
the nephrotic syndrome?
A) edema
B) the serum albumin concentration is under 3 g/ 100m1
C) the urine protein excretion rate is over 3.5 g/24 h.
D) hypercholesterolemia
E) increased coagulation
INT-7.375.
Which of the following statements about orthostatic proteinuria is
FALSE?
A) a normal protein excretion in the recumbent position
B) continuous proteinuria in a small cohort of patients
C) the renal function is impaired in most patients
D) the 24 hour urine protein level rarely exceeds 1 g
E) all of the above
INT-7.376.
Which of the following diseases does not usually lead to the development
of the nephrotic syndrome?
A) glomerulonephritis
B) polycystic kidney
C) renal vein thrombosis
D) - lupus nephritis
E) Kimmensteil-Wilson syndrome
INT-7.377.
Select the correct clinical picture of glomerulonephritis:
A) acute nephritis
B) the nephrotic syndrome
C) acute nephritis with nephrotic proteinuria
D) a persistant symptom-free proteinuria with or without hematuria
E) all of the above
F) none of the above
INT-7.378.
Which of the following statements about glomerulonephritis and its
relationship to neoplasia is FALSE?
A) glomerulonephritis is typically accompanied by lung carcinoma
B) the nephrotic syndrome can develop one year before neoplasia
C) the prognosis of the nephrotic syndrome is always favorable
D) tumor antigens are found in glomerular immune deposits
E) glomerulonephritis can occur together with lymphoma
INT-7.379.
Which infectious disease is usually followed by acute
glomerulonephritis?
A) abdominal typhus
B) varicella
C) infectious mononucleosis

D) ECHO viral infection


E) all of the above
INT-7.380.
In acute poststreptococcal glomerulonephritis:
A) streptococcus pharyngitis or impetigo always develop
B) male patients are-usually affected
C) the nephrotic syndrome is the most common form of this
glomerulonephritis
D) the AST (antistreptolysin) titer correlates well with the severity
and outcome of the disease
E) all of the above
F) none of the above
INT-7.381.
Acute nephritis occurs in all of the following, EXCEPT:
A) essential mixed cryoglobulinemia
B) Guillain-Barr's syndrome
C) acute porphyria
D) solid tumors after irradiation
E) Coxsackievirus infection
INT-7.382.
Which of the following symptoms of glomerulonephritis due to a
streptococcal infection is the most unfavorable in terms of prognosis?
A) severe oliguria
B) hypertension
C) pulmonary edema
D) proteinuria
E) a very low serum complement level
INT-7.383.
The light microscopic histologic picture, similar to that of
mesangiocapillary glomerulonephritis, is observed in all of the following
diseases, EXCEPT:
A) systemic lupus erythematosus (SLE)
B) scleroderma
C) Henoch-Sch6nlein purpura
D) poststreptococcal glomerulonephritis
E) sickle cell anemia
INT-7.384.
In which of the following cases does glomerulonephritis with a fast
clinical progress not develop?
A) systemic lupus erythematosus (SLE)
B) cryoglobulinemia
C) bacterial endocarditis
D) lipoid nephrosis
E) Henoch-Schnlein purpura
INT-7.385.
In which of the following diseases does pulmonary hemorrhage occur?
A) systemic lupus erythematosus (SLE)
B) Henoch-Sch6nlein purpura
C) hypersensitive vasculitis
D) Legionnaires' disease

E) all of the above


F) none of the above
INT-7.386.
Focal segmental glomerulonephritis accompanies all of the following
diseases, EXCEPT:
A) systemic lupus erythematosus (SLE)
B) acute poststreptococcal glomerulonephritis
C) polyarteritis nodosa
D) Henoch-Sch6nlein purpura
E) bacterial endocarditis
INT-7.387.
Case history:
A 45-year-old male patient suffers for years from rheumatoid arthritis.
Nephrotic edema has recently developed. Urine analysis revealed red
blood cells present in the urine; a 2-hour postprandial serum glucose
was 11 mmol/L. After a 6-week steroid therapy, the proteinuria remained
unchanged. Which of the following diseases is the least probable?
A) amyloidosis
B) chronic membranous nephropathy
C) normal glomeruli as evident by a light microscopic study
D) renal vein thrombosis
E) nodular glomerulonephritis
INT-7.388.
Which of the following statements about urinary tract infections is
FALSE?
A) the urine pH of pregnant women is optimal for bacterial growth
B) pyelonephritis due to E.coli is due to hematogenic dissemination
C) the occurrence of E.coli in the urethra is observed in women
with repeated infections of the urinary tract
D) a higher osmolarity inhibits bacterial growth
INT-7.389.
Which of the following statements about urinary tract infections in
female patients is FALSE?
A) the occurrence of bacteriuria increases with age
B) a shorter urethra better protects against infections
C) urethral discharge increases the occurrence of bacteriuria
D) a sedentary life-style promotes infection
E) the growing occurrence of bacteriuria with age has no relation
to sexual activity
INT-7.390.
Which of the following statements about catheter-induced urinary
tract infections is FALSE?
A) it is the most common nosocomial infection
B) a bladder lavage has a beneficial effect
C) in candida infections systemic amphotericin-B therapy is always
recommended
D) infection induced by short-term catheterization does not require
therapy
E) in candida infections oral therapy with 5-flucytozin is effective
INT-7.391.

Which of the following findings indicates analgetic-induced nephropathy?


A) red blood cells-casts
B) oval adipose bodies
C) sterile pyuria
D) phenacetine crystals
E) pigmented hyaline casts
INT-7.392.
Which of the following drugs does not cause renal disease?
A) penicillin
B) gentamycin
C) erythromycin
D) sulfonamides
E) cephalosporins
INT-7.393.
Plasma or tissue erythropoietin-like substances are accumulated in
all of the following diseases, EXCEPT:
A) hypernephroma
B) cerebellar hemangioma
C) hepatoma
D) solitary renal cyst
E) gastric carcinoma
INT-7.394.
Case Study:
A 50-year-old agricultural worker was admitted with nausea, vomiting with
a peculiar aftertaste of metal, and diarrhea. He had been working with
insecticides and disinfectants. The serum urea nitrogen value was 25 mmol/L.
The urine volume excreted in 24 h. was 300 ml. Proteinuria, red blood
cellsand epithelial cell casts were observed. Select the correct therapy:
A) monitoring of fluid and electrolyte balance with fluid and electrolyte
replacement therapy if needed
B) high dose steroid therapy
C) a dimercaptol (BAL) injection (3 mg/kg/24 h.)
D) gastric lavage
E) dimercaptol (BAL) injection 3 mg/kg followed by hemodyalisis
INT-7.395.
Nephritis with potassium loss is assumed if potassium loss is accompanied
by one of the following:
A) hyperchloremic acidosis
B) a good reponse to spironolactone
C) alkalosis
D) a high plasma aldosterone level
E) tubular proteinuria
INT-7.396.
When do the following alterations occur: calcification of basal ganglia,
subcutaneous calcium deposits, short metacarpal bones, and
mental retardation?
A) pseudohypoparathyroidism
B) idiopathic hypoparathyroidism
C) hypoparathyroidism after thyroidectomy
D) hyperparathyroidism
E) pseudohyperparathyroidism

INT-7.397.
By which mechanism does inorganic phosphate therapy decrease
the serum calcium level?
A) it decreases the intestinal calcium uptake
B) it increases urinary calcium excretion
C) it decreases bone resorption
D) stimulates bone formation
E) it increases calcium excretion in the feces
INT-7.398.
What is the incidence of renal calculi in hyperparathyroidism?
A) 10%
B) 10-25%
C) 25-50%
D) 60-70%
E) 100%
INT-7.399.
Which of the following statements is not characteristic of
urate calculi?
A) calculi producing X-ray opacity
B) the excretion of reddish sand
C) the excretion of urate crystals
D) a gouty arthritis
E) a positive family history
INT-7.400.
Hyperoxaluria occurs in:
A) ethylenglycol intoxication
B) a high intake of pyrdoxin
C) the consumption of meals with a high oxalate content
D) primary oxaluria
E) inflammatory intestinal disease
INT-7.401.
Case Study:
A 30-year-old male patient suffers from recurrent renal calculi. An
abdominal X-ray study revealed bilateral nephrocalcinosis. Which of
the following is the least probable?
A) hyperparathyroidism
B) sarcoidosis
C) primary hyperoxaluria (oxalosis)
D) renal tubular acidosis
E) cystinuria
INT-7.402.
Which of the following renal calculi are the most common?
A) urate
B) calcium-phosphate
C) mixed calcium-oxalate and calcium-phosphate
D) magnesium ammonium phosphate
E) calcium oxalate
INT-7.403.
Which of the following diseases commonly occurs in pregnant

women with renal calculi?


A) pre-eclampsia
B) urinary tract infections
C) spontaneous abortions
D) congenital anomalies
E) renal insufficiency
INT-7.404.
Which of the following interventions is not correct in the therapy of
uremic pericarditis?
A) prolonged dialysis
B) anticoagulant therapy
C) formation of a pericardial "window"
D) detachment of the pericardium
E) suction of the pericardial fluid
INT-7.405.
Which of the following drugs causes neuropathy in uremic patients?
A) vibramycin
B) chloramphenicol
C) trimethoprim
D) nitrofurantoin
E) cephalosporins
INT-7.407.
Which of the following gastrointestinal symptoms of renal insufficiency
is not improved by hemodialysis?
A) uremic gastritis
B) uremic fetor
C) peptic ulcer
D) uremic colitis
E) nausea and vomitus
INT-7.409.
Which of the following drugs should be markedly decreased in
uremia?
A) gentamycin
B) lincomycin
C) nafcillin
D) oxacillin
E) novobiocin
INT-7.410.
All of the following occur in pregnancy, EXCEPT:
A) an increased body water content
B) a decreased blood volume
C) an increased cardiac output
D) an increased renal blood flow
E) an increased GFR
INT-7.411.
In pregnancy, limb edema is due to one of the following:
A) salt retention
B) hypoalbuminemia
C) an increased capillary permeability
D) an increased venous pressure

E) an increased blood pressure


INT-7.412.
Which of the following usually causes symptom-free bacteriuria during
pregnancy?
A) anemia
B) a premature delivery
C) a congenital anomaly
D) pyelonephritis
E) toxemia
INT-7.413.
Which of the following alterations does not cause preeclampsia?
A) essential hypertension
B) aortic coarctation
C) unilateral disease of the renal a.
D) chronic glomerulonephritis
E) pyelonephritis
INT-7.414.
Which of the following agents is beneficial for hypertension in pregnancy?
A) sodium nitroprusside
B) hydralazine
C) minoxidil
D) captopril
E) reserpine
INT-7.415.
Which of the following causes cessation of toxemia after delivery?
A) delivery of the placenta
B) delivery of the fetus
C) uterus decompression
D) urether decompression
E) the loss of amniotic fluid
INT-7.416.
Which of the following tests indicates survival of the transplanted
cadaver kidney?
A) the lymphocyte toxicity test
B) a mixed lymphocyte culture
C) the WBC agglutination test
D) a complement fixation test
E) ABO blood typing test
INT-7.417.
Define the type of renal transplantation between non-monozygotic
twins:
A) allograft
B) xenograft
C) isograft
D) syngeneic graft
E) heterograft
INT-7.418.
Which of the following infections commonly occurs in patients with
renal transplants?

A) Pneumocystis carinii
B) Listeria meningitis
C) Pneumococcus pneumonia
D) Influenza
E) Cytomegalovirus infection
INT-7.419.
Which of the following intra-abdominal organs does not move with
respiration?
A) the kidney
B) the pancreas
C) the spleen
D) the transverse colon
E) the stomach
INT-7.420.
An Ophthalmologic complication of ulcerative colitis is:
A) perforating scleromalacia
B) keratitis
C) conjunctivitis
D) episcleritis
E) optic nerve neuritis
INT-7.421.
The most common gastrointestinal disease in civilized countries is:
A) duodenal ulcer
B) diverticulosis
C) colon carcinoma
D) irritable colon
E) reflux esophagitis
INT-7.422.
Which of the following substances is the most potent stimulant of
gastric acid secretion?
A) proteins
B) fats
C) carbohydrates
INT-7.423.
A 5-fold elevation of serum amylase level strongly suggests:
A) parotitis
B) pancreatitis
C) intestinal obstruction
D) pancreatic carcinoma
E) penetrating ulcer
INT-7.424.
A characteristic symptom of pyloric obstruction is:
A) bile vomiting
B) abdominal murmurs
C) resonance above Traube's space
D) succussion (splashing sound)
E) visible peristalsis
INT-7.425.
A bilocular, hour-glass stomach is caused by:

A) syphilis
B) lymphoma
C) a gastric ulcer
D) congenital anomalies
E) paraesophageal hiatus hernia
INT-7.426.
In which of the following diseases does a massive gastrointestinal
hemorrhage rarely occur?
A) esophageal varicosity
B) reflux esophagitis
C) gastric ulcer
D) erosive gastritis
INT-7.427.
The smallest absorbing unit of intestinal mucosa is the:
A) microvillus
B) goblet cell
C) cylindric cell
D) Golgi's body
E) villus
INT-7.428.
Cimetidine interacts with all of the following, EXCEPT:
A) warfarin
B) phenytoin
C) phenobarbital
D) propranolol
E) diazepam
INT-7.429.
Which of the following is the rarest complication of regional enteritis
(Crohn's disease)?
A) internal fistula
B) external fistula
C) closed perforation
D) open perforation
E) intra-abdominal abscesses
INT-7.430.
Aluminium-containing antacids decrease the absorption of:
A) atropine
B) iron
C) tetracycline
D) all of the above
E) none of the above
INT-7.431.
The most common cause of gastrointestinal obstruction by a foreign
body is:
A) bezoars
B) parasites
C) intestinal calculi
D) iron-containing calculi
E) gallstones

INT-7.432.
The diagnosis of malaria is based on:
A) a stained blood smear for the identification of the pathogenic
agent
B) a hemoculture
C) identification of the pathogenic agent on the skin
D) a fluorescent antibody study
INT-7.433.
The diagnosis of acute mesenteric vascular obstruction is based on:
A) special biochemical studies
B) angiography
C) abdominal exploration
D) none of the above
INT-7.434.
Which of the following metabolic disorders occurs if gastric evacuation
is impeded?
A) hypercalcemia
B) acidosis
C) hypochloremic alkalosis
D) hyperchloremia
E) none of the above
INT-7.435.
Liver transplantation is a new method for the therapy of fatal liver
diseases. Which of the following markedly improves the post-transplantation
survival rate?
A) a better selection of patients
B) the early recognition of malignant diseases
C) a better understanding of the pathomechanism of liver insufficiency
D) cyclosporine A
INT-7.436.
Which part of the colon is the largest feces reservoir?
A) the cecum
B) the transverse colon
C) the descending colon
D) the sigmoid colon
E) the rectum
INT-7.437.
The defecation stimulus is triggered by:
A) contraction of the external anal sphincter
B) contraction of the internal anal sphincter
C) distension of the sigmoid colon
D) distension of the rectum
E) contraction of the rectum
INT-7.438.
The anatomic anomaly which causes congenital megacolon
(Hirschsprung's disease) is:
A) hypertrophy of the descending colon
B) lack of colon peristalsis
C) absence of the autonomic plexuses in the colon
D) rectal atresia

E) lack of the internal rectal sphincter


INT-7.439.
The disease frequently misdiagnosed as regional enteritis is:
A) acute pyelonephritis
B) irritable colon
C) diverticulosis
D) appendicitis
E) gastritis
INT-7.440.
How often (% incidence) does ulcerative colitis affect the
rectosigmoidal mucosa?
A) 10-20%
B) 25-30%
C) 50-70%
D) 70-85%
E) 85-100%
INT-7.441.
Which segment of the colon most commonly shows signs of toxic
megacolon or toxic dilation due to ulcerative colitis?
A) the cecum
B) the transverse colon
C) the descending colon
D) the sigmoid colon
E) the rectum
INT-7.442.
The most common site of diverticulosis in the colon is:
A) the rectum
B) the sigmoid colon
C) the descending colon
D) the transverse colon
E) the cecum
INT-7.443.
The most common complaint in carcinoma of the rectum is:
A) diarrhea
B) anal pain
C) abdominal pain
D) constipation
E) melena
INT-7.444.
In which segment of the colon do unrevealed or misdiagnosed carcinomas
usually develop?
A) in the cecum
B) in the transverse colon
C) in the descending colon
D) in the sigmoid colon
E) in the rectum
INT-7.445.
The most common cause of chronic relapsing pancreatitis is:
A) gallstones

B) alcohol consumption
C) Whipple's disease
D) trauma
E) infection
INT-7.446.
Case Study:
A 40-year-old worker had a massive upper gastrointestinal hemorrhage
accompanied by splenomegaly. He never consumed alcohol. Which of the
following could have caused this gastrointestinal bleeding?
A) a peptic ulcer
B) gastritis
C) esophageal varices
D) gastric carcinoma
E) esophagitis
INT-7.447.
In alcoholic cirrhosis and esophageal varicosities:
A) the presence and size of the varices is related to the extent of
hypertension
B) the risk of gastrointestinal hemorrhage correlates with the severity
of portal hypertension
C) the risk of gastrointestinal hemorrhage correlates with the size
of the varices
D) all of the above
E) none of the above
INT-7.448.
Select the localization of regional enteritis relapsing after
surgery:
A) the jejunum
B) the site of the anastomosis
C) the colon
D) at any site of the gastrointestinal tract
E) the rectum
INT-7.449.
The most important organ of the gastrointestinal tract is:
A) the stomach
B) the liver
C) the gallbladder
D) the appendix
E) none of the above
INT-7.450.
Hypergastrinemia occurs in:
A) pancreatic adenoma
B) gastric carcinoma
C) pheochromocytoma
D) Zollinger-Ellison's syndrome
E) all of the above
D) none of the above
INT-7.451.
Which of the following proteins is not produced by the liver?
A) albumin

B) alphal-globulin
C) alpha 2-globulin
D) beta-globulin
E) gamma-globulin
INT-7.452.
If a patient with classic reflux esophagitis shows no reaction to
cimetidine or a therapeutic elevation of his bed then therapy must
be supplemented with metoclopramide because:
A) it stimulates gastric evacuation, which is impaired in 50% of
these patients
B) it increases the lower esophageal sphincter tone
C) it improves the acid clearance
D) it improves gastritis which occurs in 75% of these patients
E) all of the above
INT-7.453.
The enzyme metabolizing alcohol is:
A) alcohol reductase
B) alcohol oxidase
C) alcohol dehydrogenase
D) alcohol synthetase
E) glucose- 6-phosphatase
INT-7.454.
The most common endocrinopathy occurring in the ZollingerEllison Syndrome is:
A) pheochromocytoma
B) hyperthyroidism
C) hypopparathyroidism
D) hypoparathyroidism
E) Cushing's syndrome
INT-7.455.
How does the vagal tone affect the basal gastrin level?
A) it elevates the gastrin level
B) it lowers the gastrin level
C) it has no effect on the gastrin level
INT-7.461.
Which of the following statements supports the diagnosis of
hemolytic jaundice?
A) an elevated non-conjugated bilirubin level in the plasma
B) the presence of bilirubin in the urine
C) an elevated reticulocyte count
D) both (A) and (C) are true
E) none of the above
INT-7.462.
Case Study:
In a patient with primary biliary cirrhosis, polyclonal gammopathy
was established by protein-electrpphoresis. The immunoglobulin
found is predominantly:
A) IgA
B) IgM
C) IgG

D) IgD
E) IgE
INT-7.463.
Liver cirrhosis is the most common, but not the sole cause of portal
hypertension and esophageal varicosities. Which varices can be
treated with a splenectomy?
A) schistosomiasis
B) thrombosis of the splenic veins
C) nodular regenerative hyperplasia
D) none of the above
INT-7.464.
Which of the following agents inhibits vitamin B12 absorption?
A) phenytoin
B) methotrexate
C) cycloserine
D) trimethoprim
E) all of the above.,
INT-7.465.
A manifestation of Crohn s disease can occur in:
A) the oral cavity
B) the esophagus
C) the stomach
D) the ileum
E) all of the above
INT-7.466.
Which of the following bile acids can be absent in advanced cirrhosis?
A) cholic acid
B) chenodeoxicholic acid
C) deoxycholic acid
D) lithocholic acid
E) none of the above
INT-7.467.
Which of the following endocrine alterations can be accompanied by
an exudative ascites?
A) hyperparathyroidism
B) hyperthyroidism
C) hypothyroidism
D) Addison's disease
E) acromegaly
INT-7.468.
The highest secretin secretion occurs in the:
A) stomach
B) duodenum
C) jejunum
D) ileum
E) none of the above
INT-7.469.
The incidence of lymphoma is higher in which of the following?
A) Sjrgen's syndrome
B) patients receiving immunosuppresive therapy after renal

transplantation
C) chronic active hepatitis requiring immunosuppressive
therapy
D) systemic lupus erythematosus (SLE)
E) all of the above
INT-7.470.
Which of the following is the most important factor affecting
substrate absorption after an intestinal resection?
A) the extent of the resection of the intestine
B) the presence of the ileocecal valve
C) the state of the remaining intestine and related organs
D) the adaptation of the remaining intestine and stomach
E) all of the above
INT-7.471.
Case Study:
2 weeks after a coronary bypass operation the patient's SGOT/SGPT
was 30/350 with a normal liver function. 3 months later the patient
complained of fatigue and weakness, his SGPT level increased over
300. Which of the following is the most probable diagnosis?
A) delta-hepatitis
B) B-type hepatits
C) non-A non-B hepatitis
D) CMV hepatitis
E) activation of a chronic hepatitis
INT-7.472.
Which of the following is well tolerated by lactase-deficient individuals?
A) milk
B) joghurt
C) sour cream
D) none of the above
INT-7.473.
All of the following are clinico-pathological symptoms of lead intoxication,
EXCEPT:
A) anemia with basophilic granules in the red blood cells
B) membranous nephropathy with nephrotic syndrome
C) increased renal excretion of aminolevulinic acid and
coproporphyrin
D) lethargy, stupor, mental retardation and encephalopathy
INT-7.474.
Which of the following statements is not characteristic of chronicmyeloid
leukemia?
A) the Philadelphia chromosome is present in 95% of the cases
B) the alkaline phosphatase level in leukemic cells is elevated
C) splenomegaly is more pronounced in myeloid leukemia than
all the rest of the leukemias
D) deoxynucleotidyl-transferase is identified in many leukemic
cells during the "blast crisis"
E) if the liver is involved, leukemic infiltration occurs in the portal
tract and sinusoids.
INT-7.475.

From the following select the correct form of malignant lymphoma


with typical lacunal cells, which mainly occurs in young women and
predominantly affects mediastinal or supraclavicular lymph nodes:
A) Hodgkin's lymphoma - mixed cell type
B) Hodgkin's lymphoma - predominantly lymphocytic type
C) non-Hodgkin's lymphoma - non-differentiated type
D) Hodgkin's lymphoma - nodular sclerotic type
E) Hodgkin's lymphoma - lymphocyte-depleted type
INT-7.476.
Which of the following is not included in myeloproliferative syndrome?
A) chronic myeloid leukemia
B) polycythemia
C) myelofibrosis with extramedullary myeloid metaplasia
D) primary thrombocytopenia
E) acute lymphoid leukemia
INT-7.477.
Which of the following alterations is not characteristic of immunodeficient
diseases (AIDS)?
A) lymphoid depletion in the cortical and paracortical areas of
lymph nodes
B) viral inclusions in the histiocytes of lymph nodes
C) gammaglobulinemia
D) an inverted ratio of T-helper/T-inductor cells
E) common pneumocystis carni infections
INT-7.478.
Proliferative glomerular alterations occur in all of the following,
EXCEPT:
A) Fanconi's syndrome
B) bacterial endocarditis
C) Henoch-Sch6nlein purpura
D) lupus erythematosus
E) Wegener's granulomatosis
INT-7.479.
The most common histological type of thyroid carcinoma is:
A) medullary carcinoma
B) undifferentiated small-cell carcinoma
C) undifferentiated large-cell carcinoma
D) papillary carcinoma
E) follicular carcinoma
INT-7.480.
Select a correct definition of primary hepatocellular carcinoma
(HCC)?
A) in some geographical areas, a correlation exists between the
incidence of hepatocellular carcinoma and the alfatoxin content
in the diet
B) HCC occurs more frequently in alcoholics, than in cirrhosis
due to hemochromatosis
C) a chronic hepatitis infection might play a role in the development
of HCC
D) an elevated alpha-fetoprotein level is found in 2/3 of the patients
E) HCC frequently causes invasion of the hepatic vein

INT-7.481.
The most common type of an infiltrating thyroid carcinoma is:
A) medullary carcinoma
B) follicular carcinoma
C) Paget's disease
D) papillary carcinoma
E) a desmoplastic adenocarcinoma with stroma reaction
INT-7.482.
The initial lesion caused by rheumatoid arthritis:
A) remains in the articular cartilage
B) remains in the articular cavity
C) remains in the synovia
D) remains in the bones comprising the joint
E) remains in the secondary ligaments and tendons
INT-7.483.
The incubation period of rabies shows individual differences and is
related to the:
A) infecting dose of the virus
B) type of the vector
C) history of vaccination
D) distance between the wound and the brain or spinal cord
E) clinical form of the vector
INT-7.484.
The most pathognomic renal lesion caused by diabetes mellitus:
A) glomerulosclerosis
B) polycystic disease
C) necrotizing papillitis
D) acute pyelonephritis
E) chronic pyelonephritis
INT-7.485.
Which of the following cannot be a probable cause of severe
Raynaud's disease in a 50-year-old female patient?
A) cryoglobulinemia
B) thoracic outlet syndrome
C) rheumatoid arthritis
D) systemic lupus erythematosus (SLE)
E) scleroderma
INT-7.486.
Which of the following is an example of active immunization?
A) a subclinical tuberculosis infection
B) immunization with diphteria-antitoxin
C) immunization with anti-hepatitis A human serum
D) the of antibodies in maternal milk
appearance
E) the transplacental transfer of maternal IgG
INT-7.487.
Case Study:
Laboratory findings in a patient with an assumed diagnosis of
pertussis showed a 25000 leukocyte count and 75% lymphocytosis.

However nasopharyngeal cultivation did not support the diagnosis.


Which of the following diagnosis can be assumed?
A) an adenovirus infection
B) an unidentified viral infection
C) Rickettsiosis
D) none of the above
INT-7.488.
Which of the following studies should be performed as a last step to
verify the diagnosis in a patient with a neck tumor?
A) ultrasound
B) an X-ray study
C) tumor biopsy
INT-7.489.
The maximum appendicitis morbidity occurs in the following age
group:
A) 5-20-years-old
B) 20-30-years-old
C) 40-60-years-old
INT-7.490.
Hyperplastic necrotizing and fibrotic transmural inflammation in
regional enteritis usually occurs:
A) in the midportion of the intestine
B) in the proximal part of the intestine
C) in the distal part of the intestine
INT-7.491.
Case Study:
Which of the following is the most probable diagnosis in a 24-year-old
male patient suffering from arthritis, conjunctivitis and urethritis?
A) rheumatoid arthritis
B) Reiter's syndrome
C) pseudo-gout
D) gonorrhea
E) ankylosing spondylitis
INT-7.492.
Which of the following is the drug of primary choice in Salmonella
typhi infections?
A) penicillin
B) trimethoprim-sulfamethoxazole
C) erythromycin
D) chloramphenicol
E) tetracyclines
INT-7.493.
Which of the following statements is the most characteristic of
anemia of a chronic disease?
A) a high serum iron level and a high total iron binding capacity
B) deficient iron stores in the bone marrow
C) an increase of sideroblasts
D) a low serum iron level and a low total iron binding capacity
E) macrocytosis

INT-7.494.
All of the following tissues contain alkaline phosphatase, EXCEPT:
A) skin
B) bone
C) leukocytes
D) placenta
E) liver
INT-7.495.
Which of the following is the most characteristic of ulcerative proctitis?
A) it is premalignant state
B) it is a precursor of Crohn's disease
C) it is a psychiatric disease
D) it always causes diarrhea
E) it responds to a steroid enema
INT-7.496.
Which of the following is a dead vaccine?
A) pertussis immunization
B) BCG immunization
C) measles immunization
D) rubella immunization
E) yellow fever immunization
INT-7.497.
Which of the following enzymes (denoted in short form) best indicates
acute myocardial necrosis?
A) SGOT
B) SGPT
C) CK
D) CK-MB izozyme
E) LDH
INT-7.498.
Case Study:
A 48-year-old male patient is treated for Zollinger-Ellison's syndrome
with cimetidine and anticholinergic agents. The disease is well maintained
with this therapy, however gynecomastia and marked impotence
have developed. Due to these side-effects the patient asks for a
different therapy. Which of the following can be recommended?
A) a total gastrectomy
B) a reduced cimetidine dose and an elevation of the
anticholinergic dose
C) a shift from cimetidine to ranitidine
D) to reassure and comfort the patient
E) a vagotomy and a decreased cimetidine dose
INT-7.499.
The release of pancreatic polypeptide is regulated by the vagus
nerve, cholinergic stimulation, diet, and blood glucose level. Which
of the following statements about pancreatic polypeptide is true?
A) the pancreatic polypeptide level is always increased in nearly
all syndromes with watery diarrhea
B) pancreatic polypeptide release is inhibited in acute pancreatitis
C) pancreatic polypeptide level is lower in cases of duodenal ulcer
D) pancreatic polypeptide level is normal after selective gastric

vagotomy
E) pancreatic polypeptide production is higher in diabetic patients
INT-7.500.
Bacterial overgrowth following extensive intestinal resection can cause:
A) hepatic steatosis
B) arthritis
C) colon pseudo-obstruction
D) lactate acidosis
E) all of the above
INT-7.501.
Case Study:
A 30-year-old patient with chronic active hepatitis (type B) converted
from hepatitis E-antigen+ to hepatitis E antibody+. His
transaminase values became normal and a liver biopsy showed inactive
postnecrotic cirrhosis. What should be done next?
A) comfort the patient; inform him that he is in complete remission
B) inform the patient that no relationship exists between the E
antibodies and the transaminase values
C) observe the patient, since spontaneous seroconversion to Eantigen
would reactivate the disease
D) none of the above
INT-7.505.
The adequate therapy of anemia developing in the cecal loop syndrome
is:
A) surgery
B) folic acid supplementation
C) vitamin B12 supplementation
D) wide-spectrum antibiotics
E) iron supplementation

INT-7.509.
Which of the following diseases has the same sigmoidoscopic findings
as those in shigellosis?
A) amebiasis
B) salmonellosis
C) granulomatous colitis
D) idiopathic ulcerative colitis
E) diverticulitis
INT-7.512.
Case Study:
A 24-year-old female patient has a 2-year history of mild non-erosive
arthritis. The patient also suffers from severe depression, which developed
when she had started taking oral contraceptive pills. The patient
is penicillin-sensitive. At examination a pericardial friction sound and
proteinuria (over 3.5g) were found. Select the most probable diagnosis:
A) rheumatoid arthritis
B) rheumatic fever
C) mixed connective tissue disease
D) systemic lupus erythematosus (SLE)
E) polyarteritis nodosa

INT-7.513.
Which pacemaker type is the most common in Hungary?
A) asynchronous VOO
B) ready-to-use VVI
C) atria stimulated AAI
D) AV sequential (double chamber) DVI
E) automatic pacemaker DDD
INT-7.514.
In which disease is a pacemaker usually implanted?
A) total AV block
B) hyperesthesia of the carotid sinus
C) sick sinus syndrome
D) SA node block
E) none of the above
INT-7.515.
Which of the following drugs cannot be administered if atrial fibrillation
with high ventricular rate (250/min) and a wide QRS is found?
A) disopyramide
B) quinidine sulfate
C) procainamide
D) digitalis
E) lidocaine
INT-7.516.
Which of the following causes outflow obstruction in hypertrophic
cardiomyopathy?
A) valvular stenosis
B) suprava1vular stenosis
C) subaortic muscular stenosis
D) subvalvular membranous stenosis
E) none of the above
INT-7.517.
Define the term Rivero's sign:
A) a murmur of mitral insufficiency which gets louder during inspiration
B) a murmur of tricuspid insufficiency which gets softer on inspiration
C) a murmur of tricuspid insufficiency which gets louder on inspiration
D) all of the above
E) none of the above
INT-7.518.
In which of the following diseases is the delta-wave absent in a preexcitation
syndrome?
A) Wolff-Parkinson-White syndrome, type A
B) Wolff-Parkinson-White syndrome, type B
C) incomplete Wolff-Parkinson-White syndrome
D) Lown-Ganong-Levine (LGL) syndrome
E) all of the above
INT-7.519.
Which of the following ST segment alterations observed on the ECG
during an exercise tolerance test is the most characteristic of
myocardial ischemia?
A), no ST segment alteration is observed during this test .

B) an upward-sloping ST segment
C) a downward -sloping ST segment
INT-7.520.
Which of the following mitral prolapse-induced arrhythmias (usually occurring
at night during increased vagal tone) is the most dangerous?
A) ventricular extra-systole
B) non-sustained ventricular tachycardia
C) sustained ventricular tachycardia
D) torsade de points
E) atrial tachycardia
INT-7.521.
In which of the following diseases is epithelial podocyte fusion never
observed on electron microscopic pictures?
A) toxemia of pregnancy
B) aminonucleoside nephrosis
C) lipoid nephrosis
D) benign nephrosclerosis
E) diabetic glomerulonephritis
INT-7.522.
Which of the following alterations accompany focal glomerulonephritis?
A) lupus nephritis
B) reflux nephropathy
C) scarring of the focal segments; proliferative glomerulonephritis
D) massive obesity
E) all of the above
INT-7.523.
Which of the following neoplasms correlates with membranous
glomerulonephritis?
A) lung cancer
B) colon cancer
C) renal tumors
D) gastric cancer
E) all of the above
INT-7.524.
In which of the following diseases do electron dense deposits usually
occur?
A) lipoid nephrosis
B) diabetic glomerulonephritis
C) amyloidosis
D) penicillinamine therapy
E) renal tuberculosis
INT-7.525.
Which of the following statements is not typical of membranous
glomerulonephritis?
A) proteinuria but not so pronounced that it causes edema
B) microhematuria
C) a normal complement level
D) non-selective proteinuria
E) hypertonia

INT-7.526.
Which of the following statements about the recovery period after
post-streptococcal glomerulonephritis (PSGN) is FALSE?
A) electrodense deposits usually disappear after 6 weeks
B) mesangial cell proliferation will stop after 8 weeks
C) microhematuria and proteinuria can, occur for 2 years after
recovery
D) the antistreptolysin-O (ASO) level normalizes after several months
E) PSGN usually carries a good prognosis
INT-7.527.
Which of the following statements concerning Goodpasture's syndrome
is FALSE?
A) a synonym for glomerulonephritis caused by antiglomerular
basement membrane antibodies
B) pulmonary hemorrhage always disappears after a bilateral
nephrectomy
C) antiglomerular basement membrane antibodies show crossreactivity
with alveolar membrane
D) pulmonary hemorrhage is sometimes not recognized
E) agressive plasma replacement therapy can stop the pathological
process in the lung
INT-7.528.
Mesangial IgA deposits do not occur in:
A), healthy adults
B) alcoholic cirrhosis
C) Henoch-Schnnlein purpura
D) systemic lupus erythematosus (SLE)
E) amyloidosis
INT-7.529.
Which medication is contraindicated in hyperuricemia?
A) ammonium chloride
B) sodium-bicarbonate
C) penicillin
D) acetazolamide
E) calcium salts
INT-7.530.
Which of the following can occur in primary hypothyroidism?
A) TSH does not increase following intravenous TRH administration
B) a -pathologically low RBC sedimentation rate
C) a decreased binding capacity of thyroxin binding globulin
D) an elevated creatinine-phosphokinase level
E) delta waves on the EEG
INT-7.531.
Which of the following statements about poststreptococcal
glomerulonephritis (PSGN) is least probable?
A) hypertension at the onset is rare
B) pediatric encephalopathy is very common
C) atypical manifestation is common in elderly patients
D) circulatory insufficiency is a predominant sign
E) most of the symptoms are sequels of salt and water retention

INT-7.532.
Which type of renal calculi is the most common?
A) apatite concrement
B) magnesium-ammonium-phosphate concrement
C) urate concrement
D) cystine concrement
E) calcium oxalate with or without calcium phosphate
INT-7.533.
70-80% of a renal calculus is comprised of:
A) calcium
B) phosphate
C) uric acid
D) oxalic acid
E) ammonium
INT-7.534.
Renal calculus-induced hypercalciuria can be treated with all of the
following methods, EXCEPT:
A) a low salt diet
B) thiazide diuretics
C) a low calcium diet
D) a low phosphate diet
E) oral phosphate administration
INT-7.535.
Which of the following should be the drug of choice in idiopathic
hypercalciuria?
A) ascorbic acid
B) trichlormethiazide
C) oral orthophosphate
D) cortisone
E) cellulose phosphate
INT-7.536.
Which of the following pathological alterations does not affect the
hyperkalemic EGG?
A) hypernatremia
B) hypocalcemia
C) acidosis
D) hyponatremia
INT-7.537.
Cardiovascular complications comprise about 10% of the annual
mortality of dialysed patients. Which of the following is another substantial
mortality factor in this group of patients?
A) sepsis
B) neuropathy
C) cerebral complications
D) metabolic disorders
E) anemia
INT-7.538.
If a patient with ulcerative colitis develops thromboembolic complications,
which of the following statements concerning such related
coagulation disorders is true?

A) an increased platelet count is a causative factor


B) an elevated fibrinogen level is a causative factor
C) a decreased factor V is a causative factor
D) an elevated factor VII is a causative factor
INT-7.539.
Lyme disease is caused by:
A) Streptobacillus moniformis
B) Borrelia recurrentis
C) Borrelia burgdorferi
D) Bartonelle baciliformis
E) none of the above
INT-7.540.
Chronic gallbladder inflammation without gall stones comprises:
A) a substantial part of all gallbladder diseases
B) a moderate part of all gallbladder diseases
C) a very small part of all gallbladder diseases
INT-7.541.
Case Study:
A 22-year-old male patient has a painless swollen ankle. Swelling developed
during a 2 month period. The patient also reported a 9 year
history of bilateral pleural pain, and recurrent, 3-9 day-periods of a
swollen painful right knee and fever. Select the cause of the edema:
A) amyloidosis
B) phlebitis
C) cor pulmonale
D) rheumatic cardiac disease
E) chronic glomerulonephritis
INT-7.542.
Cerebral metastatic carcinomas usually originate from the:
A) prostate gland
B) esophagus
C) bones
D) lungs
E) salivatory glands
INT-7.543.
In which of the following malignancies does parathyroid hormone
overload occur?
A) oat-cell carcinoma of the lung
B) thyroid carcinoma
C) epithelial carcinoma of the lung
D) fibrosarcoma
E) lymphoma
INT-7.544.
Case Study:
In a 23-year-old patient with idiopathic ulcerative colitis (in remission
due to drug therapy) re-activation of the disease occurred. The patient
had hemorrhagic diarrhea with straining. Sigmoidscopy revealed ulceration
at 40 cm. Which examination should be performed next?
A) cglonoscopy
B) air-contrast barium enema

C) feces examination for enteric pathogens


D) eosinophilic feces examination
INT-7.545.
All of the following statements about drugs for iron-deficiency
anemia are true, EXCEPT:
A) oral iron preparations contain ferrous iron
B) iron dextran contains ferric iron
C) ferrous sulfate is the drug of choice for iron-deficiency anemia
D) ferrous sulfate contains more than 90% elementel iron
E) diarrhea or constipation can occur with oral iron preparations
MULTIPLE CHOICE QUESTIONS / TYPE I
Select the correct answers to the following questions!!!
...each question may have more than one correct answer.
INT-7.546.
Pirenzepine:
A) is a liquid antacid
B) is a new anticholinergic drug
C) elicits its action via muscarinic receptors
D) rarely causes mouth dryness
E) all of the above
INT-7.548.
The immunological characteristics of primary biliary cirrhosis include:
A) an increased serum IgA value
B) antimitochondrial antibodies
C) an increased serum IgM value
D) anti-Kuppfer-cell antibodies
E) none of the above
INT-7.549.
Hemochromatosis:
A) is more common in women than in men
B) is rare before middle age
C) results from an autosomal recessive trait
D) hepatomas occur with increased frequency in patients with
longstanding hemochromatosis
E) none of the above
INT-7.551.
H-2 receptor antagonist drugs include:
A) cyproheptadine
B) cimetidine
C) meclizine
D) ranitidine
E) cyclizine

INT-7.556.
Atrial myxoma:
A) is the most common primary tumor of the heart
B) usually occurs in the right atrium
C) can be diagnosed with echocardiography
D) mimics mitral stenosis

E) presents the most varied clinical picture of all cardiac tumors


INT-7.557.
Typical features of thyroid carcinoma include:
A) hyperthyroidism can occur during therapy with radioactive
iodine
B) an increased uptake of radioactive iodine reflects carcinomainduced
alterations
C) hypocalcemia develops in the medullary type of thyroid carcinoma
D) a normal serum thyroxine level
E) thyroxine administration has a beneficial effect in the papillary
and follicular types of thyroid carcinoma
INT-7.558.
Central cyanosis:
A) usually occurs at p02 values of under 50 mmHg
B) cyanotic but warm extremities are typical
C) right-left shunt cyanosis can be releaved by the inhalation of
100% oxygen
D) can occur in normal individuals at an altitude of 5000 feet
E) occurs in hereditary hemorrhagic telangiectasia
INT-7.559.
The fourth heart sound:
A) can be physiological if it occurs in young individuals under 30years-old
B) starts simultaneously with the "a" wave of the jugular vein
pulse curve
C) occurs on the sudden opening of the mitral valve
D) is related to ventricular distensibility
E) is a sign of systemic hypertension
INT-7.560.
The intensity of cardiac murmurs:
A) increases on the right side during expiration
B) amyl nitrate increases the murmur in aortic stenosis
C) increases during Valsalva's maNEUver in hypertrophic obstructive
cardiomyopathy
D) in mitral stenosis the intensity of the murmur depends on the
extent of the stenosis
E) increases during a physical load in aortic insufficiency
INT-7.561.
A protruding pulmonary artery on the chest X-ray indicates:
A) a ventricular septal defect
B) valvular pulmonary stenosis
C) Tetralogy of Fallot
D) severe mitral stenosis
E) a massive pulmonary embolism
INT-7.562.
Under normal conditions, the stroke volume increases:
A) during increased sympathetic activity
B) during increased parasympathetic activity
C) in an increased ventricular end-diastolic volume
D) in changing from the recumbent position to the upright position

E) in high ambient temperatures


INT-7.563.
A left ventricular aneurysm:
A) is a common sequel of rheumatic fever
B) causes recurrent ventricular tachycardia
C) causes ST depression on the ECG
D) leads to cardiac insufficiency
E) correlates with a higher incidence of systemic embolization
INT-7.564.
The typical ECG signs of an acute pulmonary embolism include:
A) sinus bradycardia
B) ST-T alterations in leads V5-V6
C) right deviation of the R axis on the ECG
D) S1-Q3 phenomenon on the ECG
E) right bundle branch block
INT-7.565.
Which of the following is characteristic of mitral stenosis?
A) an "opening snap"
B) a shifted apical beat
C) it is a complication of infective endocarditis
D) it is more common in women
E) a history of rheumatic fever is frequently found in this disorder
INT-7.566.
The first signs of mitral prolapse include:
A) mitral insufficiency
B) atypical chest pain
C) an "opening snap"
D) symptoms of infective endocarditis
E) a Q-wave and ST elevation on the ECG
INT-7.567.
Typical features of severe aortic stenosis include:
A) syncope induced by physical load
B) a louder second sound over the aorta
C) a blood pressure of 180/120 mmHg
D) left ventricular hypertrophy on the ECG/Echocardiography
E) the systolic gradient between the left ventricle and the aorta
exceeds 60 mmHg
INT-7.568.
Typical features of hypertrophic obstructive cardiomyopathy include:
A) mitral insufficiency
B) digoxin therapy is important in the early stages
C) a familial background is quite common
D) the pregnancy should not be interrupted during the course of
this disease
E) syncope is a typical feature
INT-7.569.
Typical features of Eisenmenger's syndrome occurring in atrial
septal defect include:
A) it develops under the age of 20-years-old

B) a large "a" wave is typical on the jugular pulse tracing


C) pulmonary insufficiency
D) differential cyanosis
E) surgery is the necessary treatment
INT-7.570.
A prolonged QT interval on the ECG is characteristic of
A) hypercalcemia
B) digitalis therapy
C) ischemic myocardial disease
D) hypothermia
E) quinidine therapy
INT-7.571.
Which of the, following parameters increase in left ventricular
insufficiency?
A) the pulmonary venous pressure
B) the left ventricular end diastolic pressure
C) the lung distensibility
D) the p02
E) the pCO2
INT-7.572.
The intensity of the first heart sound is altered in:
A) complete AV block
B) ventricular tachycardia
C) right bundle branch block
D) the Wolff-Parkinson-White (WPW) syndrome
E) atrial fibrillation
INT-7.573.
Select the false statements concerning congestive cardiomyopathy:
A) an increased end-diastolic and end-systolic volume
B) a low end-diastolic and end-systolic volume
C) the ventricular wall/volume ratio is shifted towards the latter
D) the ventricular wall/volume ratio is shifted towards the former
E) a high pulmonary capillary pressure
INT-7.574.
Which of the following ECG changes develop in the early stage of an
acute transmural myocardial infarction?
A) a dome-like ST-elevation
B) a pathological Q wave
C) negative T waves
D) ST depression in the infarcted area
E) none of the above
INT-7.575.
Which of the following cause vasodilation during the development of
cardiac insufficiency?
A) angiotensin II
B) aldosterone
C) atrial natriuretic hormone
D) bradykinin
E) renin

INT-7.576.
Which of the following occur in acromegaly?
A) hypertension
B) intense sweating
C) hyperprolactinemia
D) hypophosphatemia
E) decreased glucose tolerance
INT-7.577.
Drugs inducing goiter include:
A) aminosalicylic acid
B) digoxin
C) lithium
D) propylthiouracil
E) streptomycin
INT-7.578.
Hypoglycemia is typical in:
A) Gierke's disease
B) renal glycosuria
C) acute alcohol intoxication
D) phenylketonuria
E) primary hepatoma
INT-7.579.
Hyperthyroidism can occur in:
A) deafness
B) pretibial myxedema
C) ataxia
D) glycosuria
E) unilateral exophthalmus
INT-7.580.
Typical alterations in Paget's disease include:
A) increased bone resorption and bone formation
B) stronger than normal bones
C) an increased serum parathormone level
D) high-output cardiac insufficiency is the usual cause of death
E) a high level of urinary hydroxyproline excretion
INT-7.581.
Hypercalcemia is caused by:
A) pseudohypoparathyroidism .
B) sarcoidosis
C) hyperinsulinism
D) hyperthyroidism
E) vitamin D deficiency
INT-7.582.
Hypocalcemia is caused by:
A) acute pancreatitis
B) hypoproteinemia
C) hyperthyroidism
D) vitamin D intoxication
E) pseudohypoparathyroidism

INT-7.583.
Which of the following are typical of hypercholesterolemia?
A) a lipoprotein lipase enzyme deficiency
B) it is a feature of the nephrotic syndrome
C) it is characterized by a dominant familial inheritance pattern
D) hyperthyroidism
E) it is a typical sign of primary biliary cirrhosis
INT-7.584.
An increased bone density verified by an X-ray study is typical of
A) Hand-Schller-Christian disease
B) prostatic metastasis
C) multiple myeloma
D) Paget's disease
E) osteoporosis
INT-7.585.
Which of the following statements are characteristic of
pheochromocytoma?
A) the intravenous administration of beta-mimetic agents is necessary
in an acute hypertensive crisis
B) remote metastases have already developed by the time of diagnosis
C) the diagnosis is usually based on paroxysmal hypertensive fits
D) an excessive renal excretion of 5-hydroxy-indolacetic acid
E) the maximum incidence is between the third and fifth decades
INT-7.586.
Symptoms of Conn's syndrome include:
A) low potassium excretion in the urine
B) tetany
C) a high renin level
D) hyperchlorhydria
E) polyuria and polydipsia
INT-7.587.
An enhanced uptake of radioactive iodine occurs in:
A) hyperthyroidism
B) subacute thyroiditis
C) iodine deficiency
D) Graves' disease
E) mercaptoimidazole therapy
INT-7.588.
Etiologic factors of acute pancreatitis include:
A) Addison's disease
B) hyperparathyroidism
C) hypothermia
D) pancreatic carcinoma
E) hyperlipidemia
INT-7.589.
Which of the following observations indicate the development of a
hepatocellular carcinoma in a someone with a long history of liver
cirrhosis?
A) splenomegaly
B) an arterial murmur over the liver

C) a high titer of antimitochondrial antibodies


D) a sudden increase of the serum alkaline phosphatase level
E) ascites with a protein content of 40 g/L
INT-7.590.
Which of the following statements is typical of Wilson's disease?
A) an early feature is acute hemolytic anemia
B) the most common renal abnormalities are tubular
C) the treatment of choice is deferoxamine
D) trientine hydrochloride is an alternative therapeutic drug
E) it results from the progressive accumulation of iron
INT-7.591.
Which of the following is observed in severe pyloric stenosis?
A) hypokalemia
B) an elevated urea concentration
C) a low urine pH value
D) hypochloremia
E) tetany
INT-7.592.
Which of the following is observed in the Zollinger-Ellison syndrome?
A) diarrhea
B) excessive gastric acid secretion after the intravenous injection
of pentagastrin
C) cimetidine fails to improve the symptoms
D) it is considered a benign pancreatic tumor
E) it is usually accompanied with other endocrine tumors
INT-7.593.
Symptoms of vitamin C deficiency include:
A) a low urine ascorbic acid level following an oral load of vitamin C
B) light exposure induces pigmented erythematous skin alterations
C) swollen gum and friable gums
D) cardiac insufficiency
E) a peripheral neuropathy
INT-7.594.
Typical alterations in a newly diagnosed case of leukemia include:
A) thrombocytopenia
B) basophilia
C) a decreased alkaline phosphatase activity in the leukocytes
D) a low vitamin B 12 level
E) an elevated serum uric acid level
INT-7.595.
A myeloid type leukemoid reaction occurs in:
A) a subphrenic abscess
B) lung carcinoma with bone metastasis
C) infectious mononucleosis
D) acute hemolysis
E) a pertussis infection
INT-7.596.
A splenectomy is indicated in:
A) polycythemia vera rubra

B) chronic idiopathic thrombocytopenic purpura


C) autoimmune hemolytic anemia
D) Felty's syndrome
E) pernicious anemia
INT-7.597.
A markedly increased serum iron level occurs in:
A) chronic anemia caused by infection
B) thalassemia
C) sideroblastic anemia
D) hemochromatosis
E) untreated pernicious anemia
INT-7.598.
A high count of neutrophilic leukocytes is typical of.
A) influenza
B) leptospirosis
C) diphteria
D) abdominal typhus
E) pertussis
INT-7.599.
Case Study:
Which of the following laboratory findings suggest chronic myeloid
leukemia instead of myeloid fibrosis in 55-year-old male patient with
splenomegaly?
A) a WBC count of over 50,000
B) an increased NEUtrophilic activity of alkaline phosphatase
C) a higher reticulocyte and granular red blood cell count in the
peripheral blood
D) the presence of the Philadelphia chromosome
E) anemia
INT-7.600.
Characteristic findings in polycythemia vera rubra include:
A) a decreased plasma volume
B) a tendency to hemorrhage easily
C) an increased RBC sedimentation rate
D) itching and pruritis
E) an increased neutrophilic activity of alkaline phosphatase
INT-7.602.
Side effects of prolonged therapy with corticosteroids include:
A) osteoporosis
B) gastritis
C) myopathy
D) suppression of pituitary-adrenal function
E) growth retardation in children
INT-7.606.
Which of the following drugs can cause skin pigmentation as a
side-effect?
A) chlorpromazine
B) phenylbutazone
C) chlorochine
D) streptomycin

E) busulfan
INT-7.607.
Which of the following drugs can cause pulmonary fibrosis?
A) amiodarone
B) bleomycin
C) colchicine
D) phenytoin
E) nitrofurantoin
INT-7.608.
Which of the following drugs can decrease the efficiency of oral contraceptives?
A) rifampin
B) coumarin derivatives
C) phenytoin
D) furosemide
E) hydralazine
INT-7.609.
Which of the following drugs can cause glomerular damage and is
therefore nephrotoxic?
A) penicillamine
B) gentamicin
C) methysergide
D) captopril
E) amphotericin B
INT-7.610.
Which of the following drugs can cause cholestatic jaundice?
A) acetaminophen
B) methyltestosterone
C) alpha-methyldopa
D) chlorpropamide
E) primaquine
INT-7.611.
Disopyramide (Palpitin):
A) is a positive inotropic agent
B) its adverse effects relate to its anticholinergic action
C) is efficient in the treatment of ventricular arrhythmias
D) is a negative inotropic agent
E) causes urine retention
INT-7.612.
Which of the following drugs can cause a syndrome resembling systemic
lupus erythematosus?
A) procainamide
B) hydralazine
C) phenytoin
D) methotrexate
E) isoniazid
INT-7.613.
Which of the following drugs can cause hirsutism?
A) spironolactone
B) phenytoin

C) minoxidil
D) digoxin
E) clomiphene
INT-7.614.
Morphine:
A) is conjugated in the liver
B) relaxes the gastroesophageal sphincter
C) causes pupil constriction which is resistant to atropine
D) increases the pressure in the biliary ducts
E) causes urinary retention
INT-7.615.
Which of the following are mostly absorbed in the duodenum?
A) calcium
B) iron
C) vitamin B 12
D) folic acid
E) bile acid salts
INT-7.616.
Acute ulcerative disease occurs in the following cases:
A) hypothyroidism
B) cirrhosis
C) polycythemia vera
D) Cushing's syndrorpe
E) pregnancy
INT-7.617.
The most common symptoms of acute ulcer perforation are:
A)fever
B) vomiting
C) shoulder pain
D) abdominal pain
E) hyperperistalsis
INT-7.618.
More than 100 different complications of ulcerative colitis ahd
Crohn's disease are known. Most of them improve following the healing
or resection of the affected intestine. Which of the following complications
show no improvement?
A) erythema nodosum
B) pyoderma gangrenosum
C) myopericarditis
D) phlebothrombosis
E) sclerotizing cholangitis
INT-7.619.
Which of the following symptoms suggest penetration of an ulcer?
A) night pain
B) a decreased response to anti-acid therapy
C) back pain
D) increasing pain
E) changes in the rhythmicity of the pain
INT-7.620.

Which of the following symptoms are common in a subdiaphragmal


abscess?
A) fever
B) dyspnea
C) cough
D) hiccups
E) pain
INT-7.621.
Which of the following factors can change the color of stool, making
it melena-like?
A) iron
B) aluminum hydroxide
C) charcoal
D) bismuth
E) psyllium seeds
F) a postbulbar ulcer
INT-7.622.
Which of the following occur after a massive upper gastrointestinal
bleeding?
A) a decreased platelet count
B) the WBC count is shifted to the left
C) polymorphonuclear leukocytosis
D) an increased platelet count
E) leukopenia
INT-7.623.
Which pattern of regional enteritis (Crohn's disease) is most often
recognized?
A)obstruction
B) diffuse jejunoileitis
C) abscess formation
D) inflammation
E) fistulas
INT-7.624.
Mechanical intestinal obstruction is assumed if the physical examination
reveals:
A) visible peristalsis
B) a lack of intestinal sounds
C) a "tympanitic" abdomen
D) loud peristaltic sounds
E) decreased liver dullness
INT-7.625.
After peptic ulcer surgery the patient should avoid:
A) milk
B) sugar-containing beverages
C) citrus plants
D) a low carbohydrate diet
E) all of the above
INT-7.626.
Which of the following can explain a flat oral glucose tolerance curve
while the intravenous curve is normal?

A) liver disease
B) pancreatic insufficiency
C) celiac disease
D) tropical sprue
E) gastric retention
INT-7.627.
Which of the following methods are used in the differential diagnosis
of congenital megacolon?
A) sigmoidoscopy
B) study of the distal rectum
C) rectum biopsy
D) barium enema
E) stool cultivation
INT-7.628.
Which of the following enzymes normally occur in the stool?
A) pepsin ,
B) amylase
C) lipase
D) trypsin
E) maltase
INT-7.629.
A lesion of which of the following parts of the gastrointestinal tract
can cause tarry stool?
A) the esophagus
B) the stomach
C) the jejunum
D) the cecum
E) the sigmoid colon
INT-7.630.
Which of the following statements concerning the Irritable Bowel
Syndrome are true?
A) pain which rarely lasts for more than 1 hour
B) pain which usually awakes the patient
C) a transient relief from pain is usually brought on by defecation
D) the appetite is rarely affected
E) two clinical types of this syndrome are recognized
INT-7.631.
Amebic dysentery:
A) may be confused with ulcerative colitis
B) the stools contain mucus and blood
C) the stools contain large numbers of leukocytes
D) may be confused with bacillary dysentery
E) all of the above
INT-7.632.
The most common reason for a massive intestinal resection is:
A) intenstinal volvulus
B) thrombosis of the superior mesenteric artery
C) neoplasms
D) trauma
E) surgical mistakes

INT-7.633.
Which symptoms usually accompany carcinoma of the descending colon:
A) abdominal seizures
B) hematochezia
C) melena
D) back pain
E) changes in defecation habits
INT-7.634.
Which of the following should be considered with special care in the
differential diagnosis of appendicites?
A) regional enteritis
B) mesenteric lymphadenitis
C) ureter calculus
D) rupture of an ectopic pregnancy
E) Meckel's diverticulum
INT-7.635.
Prednisolone therapy of chronic active hepatitis is still indecisive. If
prednisolone is used, its side-effects depend on the presence of:
A) an elevated serum total protein level
B) an elevated serum albumin level
C) a decreased serum albumin level
D) an elevated serum bilirubin level
E) an low serum transaminase level
INT-7.636.
Case Study:
A 32-year-old patient has a duodenal ulcer which is resistant to a
daily dose of 1200 mg cimetidine. What can be the cause of this ineffective
therapy?
A) the Zollinger-Ellison syndrome
B) hyperparathyroidism
C) impaired absorption due to massive anti-acid therapy
D) recent gastric surgery
E) only (A) and (B) are true
INT-7.637.
The presence of immune complexes in chronic active hepatitis and
primary biliary cirrhosis have been the subject of extensive research.
Select the typical immune complex differences between these two
diseases:
A) IgM comprises 60% of cryoproteins in primary biliary cirrhosis
B) the serum complement level is normal in primary biliary cirrhosis
C) a decreased complement level occurs in chronic active hepatitis
D) enhanced complement utilization occurs in primary biliary cirrhosis
E) although 90% of patients with biliary cirrhosis exhibit immune
complexes, there is no proof of an immune complex disease
INT-7.638.
An elevated SGOT value can occur in which of the following cases?
A) liver necrosis
B) myocardial disease
C) opiate administration
D) muscle lesions

E) only (A) and (B) are true


INT-7.639.
Case Study:
An alcoholic patient complains of weakness and fatigue. The SGPT
value is 900 U and SGOT value is 600 U. Which of the following
studies should be performed?
A) screening for hepatitis B antigen
B) screening for hepatitis IgM antibody
C) screening for hepatitis core antigen if the hepatitis B antigen is
negative
D) identification of any giant cells in the urine
INT-7.640.
Which of the following can cause gallstones in patients taking oral
contraceptives?
A) a higher cholesterol saturation of the bile
B) chenodeoxycholic acid comprises a significantly lesser part of
the bile acid salts
C) cholic acid comprises a significantly larger part of the bile acid
salts
D) the chenodeoxycholic acid ratio increases in patients taking
oral contraceptives
E) oral contraceptives only affect the normal menstrual cycle
INT-7.641.
The serum ferritin level indicates the iron storing capacity. Which of
the following statements are true for patients with hepatic disease?
A) the circulating ferritin level depends on the extent of
hepatocellular damage
B) the serum ferritin level is not affected by hepatocellular
necrosis
C) the serum ferritin level is affected by the liver's iron stores
D) the serum ferritin level is relevant only in acute hepatic diseases
INT-7.642.
Typical endoscopic findings in a benign gastric ulcer show:
A) sharp regular edges
B) a clear, smooth base
C) a nodular base
D) rigid folds
E) none of the above
INT-7.643.
Clofibrate decreases the lipid level and:
A) elevates the cholesterol level in primary biliary cirrhosis
B) is metabolized by the liver
C) can cause liver necrosis
D) increases the frequency of bile formation
INT-7.644.
Which of the following are typical of systemic lupus erythematosus (SLE)?
A) the formation of immune complexes
B) skin and joint involvement is common
C) a follicular, scarry, butterfly-shaped skin lesion of the face
D) wire-like lesions and hemotoxyllin bodies in the kidney

E) onion-like lesions in the spleen


INT-7.645.
Which of the following induce formation of anti-nuclear and antinucleolar
antibodies in systemic lupus erythematosus (SLE)?
A) one-string DNA
B) double-string DNA
C) the ribonucleoprotein (RNP) component of .the extractable nuclear
antigen (ENA)
D) a component of the extractable nuclear antigen (ENA) - Sin
antigen
E) nucleolar RNA
INT-7.646.
Sjrgen's syndrome is suspected if the following occurs:
A) lymphocyte infiltration of the salivary glands
B) a burning sensation in the eyes
C) psychosis
D) ectopia lentis
E) alopecia
INT-7.647.
Which of the following alterations can be due to a high RF titer in
rheumatoid arthritis?
A) subcutaneous nodules
B) pulmonary involvement
C) hepatomegaly
D) vasculitis
E) only (A) and (D) are true
INT-7.648.
Which of the following are true for rheumatoid factor (RFs)?.
A) IgMs are found in 70% of all cases of RA
B) RFs are antibodies to altered gamma-globulin
C) RFs are not specific for RA
D) a high RF titer suggests a worse prognosis
E) RFs can be found in subacute bacterial endocarditis
INT-7.649.
Where does rheumatoid factor (RF) occur?
A) in systemic lupus erythematosus (SLE)
B) in scleroderma
C) in polyarteritis nodosa
D) in subacute bacterial endocarditis
E) in sarcoidosis
INT-7.650.
Select the characteristic histologic changes in the synovial membrane
in rheumatoid arthritis?
A) villous hypertrophy
B) plasmocytosis and lymphocytosis in the synovium
C) deposits of compact fibrin
D) high complement levels in the synovial fluid
E) foci of cell necrosis

INT-7.654.
Which of the following statements concerning urticaria are true?
A) edema develops only in the skin
B) some cases are type-I hypersensitivity reactions
C) it is the sole symptom of hereditary angioedema
D) it is elicited by both heat and cold
E) in 75% of cases the cause of the urticaria is known
INT-7.656.
Which of the following are typical lesions in erythema multiforme ?
A) hemorrhagic lesions of the lips
B) the lesions appear on the distal portions of the extremities
C) target or iris lesions
D) systemic symptoms include arthralgia
E) itching
INT-7.657.
In which of the following does erythema nodosum occur?
A) lymphoma
B) sarcoidosis
C) Streptococcal infections
D) ulcerative colitis
E) when oral contraceptives are taken over an extended period
INT-7.658.
Which complications can be due to the circulating immune complexes
in sarcoidosis?
A) bilateral hilar lymphadenopathy
B) cataract lens
C) granulomatous uveitis
D) chronic meningitis
E) erythema nodosum
INT-7.659.
Which of the following are characteristic of membranous
glomerulonephritis?
A) massive proteinuria
B) an acute decrease of the GFR
C) hypertension
D) granular casts and red blood cells in the urine
E) hypoalbuminemia
INT-7.662.
Which of the following cause chronic persistent hepatitis?
A) alcohol
B) acetaminophen (paracetamol)
C) allopurinol
D) sulfonamides
E) amiodarone
INT-7.663.
Which of the following are true for a 0 blood group patient?
A) the patient has anti-A antibodies
B) the patient has anti-B antibodies
C) the patient's parents have blood group A or B
D) the patient has a higher risk of developing a duodenal ulcer

E) the patient has a higher risk of developing a gastric carcinoma


INT-7.664.
Which of the following indicate hemolysis due to an incompatible
blood transfusion?
A) pulmonary edema
B) lumbar pain
C) headache
D) urticaria
E) hemoglobinuria
INT-7.665.
Which of the following statements about hemolytic anemia of,
newborns are true?
A) it occurs if the father is Rh-positive and the mother is Rhnegative
B) it is more probable if ABO incompatibility occurs
C) it never occurs in the first newborn
D) the most frequent pairing resulting in sensitivity is maternal
blood group O and fetal blood group A
E) it can be prevented by injecting a high-titer anti-Rh gammaglobulin
preparation within 72 hours after delivery
INT-7.666.
Which of the following causes Coombs-positive hemolysis?
A) dapsone
B) methyldopa
C) penicillin
D) Clostridium perfringens sepsis
E) cephalosporins
INT-7.667.
Which of the following alterations are more common in
Waldenstrm's macro-globulinemia than in multiple myeloma?
A) ostealgia
B) diffuse lymph node enlargement
C) renal lesions
D) amyloidosis
E) hyperviscosity syndrome
INT-7.668.
Which of the following vaccines cause neurologic complications?
A) variola (smallpox)
B) pertussis
C) yellow fever
D) measles
E) influenza
INT-7.669.
Which of the following vaccines are indicated in immunosuppressed
patients?
A) mumps
B) measles
C) tuberculosis (BCG)
D) pnemococcal vaccine (pneumovax)
E) poliomyelitis vaccine (Salk-polio)

INT-7.670.
When are specific human immunoglobulins used?
A) for hepatitis A
B) for hepatitis B
C) for rabies
D) for varicella zoster
E) for diphteria
INT-7.671.
Which of the following statements about measles vaccine are true?
A) arthritis is a common complication in adults
B) vaccination could be repeated in 2 years
C) contact vaccinal virus transmission is common
D) viral vaccine can cross the feto-placental barrier
INT-7.672.
Which of the following statements about the antirabies human diploid
cellular vaccine are true?
A) it is a live attenuated virus
B) after 6 injections seroconversion occurs in 40% of the cases
C) it should be administered if a serious chance of a rabies infection
exists
D) prophylaxis is recommended for individuals with a high risk of
infection
E) allergic encephalitis can occur in a 1:2000 ratio
INT-7.673.
Which of the following vaccines should be administered to a 9month infant according to the English immunization Table?
A) measles vaccine
B) BCG vaccine (for tuberculosis)
C) variolla (smallpox) vaccine
D) OPV (oral, attenuated poliovirus vaccine)
E) DTP (diphteria and tetanus toxoids and pertussis vaccine)
INT-7.674.
Case Study:
A child has high fever, pneumonia, and maculopapular petechias,
which are mostly located on the limbs; the limbs are edematous.
Which of the following would help in obtaining the diagnosis?
A) a living, attenuated measles vaccination in the history
B) an inactivated measles vaccination in the patient's history
C) a high anti-mumps antibody titer
D) a high anti-measles antibody titer
E) a high anti-rubeola antibody titer
INT-7.675.
Which of the following can cause a granulomatous tissue reaction?
A) Brucella abortus
B) Salmonella typhi
C) Streptococcus viridans
D) Treponema pallidum
E) Mycobacterium tuberculosis
INT-7.676.
Which of the following elements comprise a granuloma in infectious

diseases?
A) macrophages
B) plasmocytes
C) epitheloid cells
D) giant cells
E) fibroblasts
INT-7.677.
Which of the following are associated with pulmonary eosinophilia?
A) Ascaris lumbricoides
B) Strongyloides stercoralis
C) Ancylostoma braziliense
D) nickel carbonate
E) chlorpropamide
INT-7.678.
In which of the following cases does nodular lymphoid hyperplasia
commonly occur?
A) diarrhea
B) giardiasis
C) hypergammaglobulinemia
D) irradiation due to X-ray images
E) recurrent sinopulmonary infections
INT-7.679.
The normal D-xylose load depends on:
A) normal pancreatic function
B) normal liver function
C) normal intestinal absorption function
D) normal renal function
E) normal salivary function
INT-7.680.
D-xylose compared to glucose:
A) it's metabolism is independent of pancreatic function
B) it's metabolism is very low
C) it is a pentose
D) it's metabolism depends on the liver function
E) it is a very specific test for malabsorption
INT-7.681.
Evidence exists that the administration of amino acids with sidebranches
is beneficial in patients with liver cirrhosis. Which of the
following statements about such amino acids are true?
A) they improve survival
B) they improve digestion
C) they normalize the aminoacid profile
D) they induce a positive nitrogen balance
E) they improve cerebral function
INT-7.682.
Which of the following are true for the Hepatitis B virus (HBV)?
A) at least four antigen-antibody systems are related to the HBV
B) the HBV has an association with polyarteritis nodosa
C) the HBV is associated with hepatoma
D) the core antigen (HBcAg) is easily detectable in the serum

E) the core antigen's (HBcAg) presence in the serum usually provides


the first evidence of a HBV infection
INT-7.683.
Though endoscopic study-induced infections are rare, such a possibility
must be considered if postendoscopic fever occurs. Which of
the following infections can develop following gastroscopy?
A) Salmonella typhi
B) Mycobacterium tuberculosis
C) Hepatitis B
D) Stercoralis
E) Pseudomonas aeruginosa
INT-7.684.
Viral hepatitis still remains a main cause of jaundice of pregnancy.
Which of the following clinical and laboratory findings indicate a
fatty liver in pregnancy?
A) a marked elevation of the bilirubin level
B) a high serum uric acid level
C) neutrophilia
D) thrompocytopenia with giant platelets
E) a high gamma-globulin level
INT-7.685.
From the following, select two severe complications of celiac disease?
A) tomato allergy
B) intestinal lymphoma
C) porphyria
D) strictures and ulcerations of the intestine
E) a high incidence of gallstones
INT-7.686.
Which of the following factors can increase the esophageal sphincter
pressure?
A) the intraabdominal pressure
B) anticholinergic agents
C) oral contraceptives
D) bethanechol
E) metoclopramide
INT-7.687.
Which of the following statements about antinuclear factor (ANF) are true?
A) it can be identified by the direct immunofluorescent method
B) a spotty character of the immunofluorescent picture indicates
a mixed connective tissue disease
C) direct antibodies against the granulocyte nucleus occur in
rheumatoid arthritis
D) low titers are common and non-specific
E) more than 75% of systemic lupus erythematosus (SLE) patients
are positive for ANF
INT-7.688.
Which of the following accompany Sjrgen's syndrome?
A) renal tubular acidosis
B) hypogammaglobulinemia
C) lymphoma

D) primary biliary cirrhosis


E) myasthenia gravis
INT-7.689.
Rheumatic nodes can be easily identified at the following locations:
A) the elbow
B) the skull
C) the popliteal fossa
D) the lung
E) along the tendons
INT-7.690.
Pericarditis can develop as a complication of
A) polymyositis
B) systemic lupus erythematosus (SLE)
C) mixed connective tissue disease
D) polyarteritis nodosa
E) rheumatoid arthritis
INT-7.691.
Which cutaneous symptoms occur in scleroderma?
A) subcutaneous calcifications
B) hyperpigmentation
C) induration
D) telangiectasia
E) ulcerations
INT-7.692.
Which of the following statements about dermatitis herpetiformis are
true?
A) asymptomatic gluten-sensitive enteropathy is found in most
patients
B) there are IgE depostis in the involved skin areas
C) there are IgA deposits in the normal skin and around the lesions
D) it can be caused by the local application of iodine
E) antireticulin antibodies can usually be found
INT-7.693.
Which of the following occur in pemphigus?
A) intraepidermal bullae
B) the appearance of the anti-basement membrane IgG in the serum
C) IgG and complement C3 deposits along the basement membrane
D) IgG antibodies against intracellular matter of the keratinized
epithelial cells circulate in the serum
D) the epidermis is easily detached from the underlying skin
INT-7.694.
Which of the following statements about sarcoidosis are true?
A) histologically - noncaeseating epithelioid granulomas
B) bilateral hilar lymphadenopathy
C) leukocytosis is frequently present
D) a positive Kveim test helps to differentiate it from other
granulomatous diseases
E) chlorambucil is an alternative therapeutic drug
INT-7.695.

Which of the following are typical of Caplan's syndrome?


A) it is a pneumoconiosis
B) arthritis
C) circumscribed alterations of 5-50 mm in diameter visible on
the chest X-ray
D) infiltration is localized in the upper part of the lung
E) subcutaneous nodules
INT-7.696.
Which of the following are characteristic of minimal change
glomerulonephritis?
A) specific histological changes
B) hematuria
C) hypertension
D) selective proteinuria
E) an impaired GFR
INT-7.697.
Which of the following findings occur in Goodpasture's syndrome?
A) hematuria quickly followed by renal damage
B) hematuria followed by hemoptysis
C) electron microscopy reveals electrodense depositis on the basal
membrane of the renal capillaries
D) iron deficiecy anemia
E) a high titer of circulating anti-glomerular basement membrane
antibodies
INT-7.698.
Which of the following accompany primary billiary cirrhosis?
A) rheumatoid arthritis
B) renal tubular acidosis
C) Raynaud's disease
D) Coombs-positive hemolytic anemia
E) thyroiditis
INT-7.699.
Which of the following are characteristic of primary sclerotizing
cholangitis?
A) it is more common in male patients
B) it is usually accompanied by ulcerative colitis
C) there are high levels of acid phosphatase
D) it is more common in the tropics
E) it is usually accompanied by retroperitoneal fibrosis
INT-7.700.
Which of the following hepatic diseases are HLA-dependent?
A) chronic active HBsAg-negative hepatitis
B) chronic active HBsAg-positive hepatitis
C) idiopathic hemochromatosis
D) alcoholic liver disease in aged alcoholic patients
E) drug-induced liver pathology
INT-7.701.
Which of the following factors predispose chronic viral hepatitis due
to a hepatitis B infection?
A) a male gender

B) a very young age


C) an East-Asian descent
D) hemodialysis
E) Down's syndrome
INT-7.702.
Which of the following can be extrahepatic manifestations of hepatitis
B?
A) pulmonary fibrosis
B) uveitis
C) hypercalcemia
D) arthritis
E) hypertension
INT-7.703.
In which diseases can anti-microsomal antibodies (B glandular cells)
be directly identified by hemagglutination techniques?
A) in 55% of patients with Hashimoto's thyroiditis
B) in 25% of patients with Graves' disease
C) in 10% of healthy adults
D) the incidence in healthy individuals is somewhat higher than
in those who possess anti-thyroglobulin antibodies
E) in 90% of patients with idiopathic myxedema
INT-7.704.
Which of the following can accompany Addison's disease?
A) vitiligo
B) pigmentation
C) hypernatremia
D) orthostatic hypotension
E) hypokalemia
INT-7.705.
Which receptors can trigger anti-receptor antibody production?
A) the luteinizing hormone (LH) receptor
B) the acetylcholine (Ach) receptor
C) the follicle-stimulating hormone (FSH) receptor
D) the human chorionic gonadotropin (HCG) receptor
E) the insulin receptor
INT-7.706.
Which of the following can accompany autoimmune warm antibody
hemolytic anemia?
A) IgM auto-antibodies
B) Mycoplasma pneumonia infection
C) methyldopa
D) systemic lupus erythematosus (SLE)
E) IgG (auto-) antibodies
INT-7.707. Multiple Choice Question
Which of the following can cause antibody-induced
thrombocytopenia?
A) measles
B) systemic lupus erythematosus (SLE)
C) aspirin
D) gold-salts

E) quinine
INT-7.708.
Which diseases can accompany cryoglobulinemia?
A) chronic lymphoid leukemia
B) multiple myeloma
C) rheumatoid arthritis
D) heavy-chain disease
E) infective endocarditis
INT-7.709.
Which of the following statements about primary and secondary
atrial septum defects (ASD-I and ASD-II) are true?
A) right axis deviation and incomplete right bundle branch block
are common in ASD-II
B) left axis deviation is typical of ASD-I
C) an apical murmur with marked regurgitation is common in
ASD-I
D) an apical diastolic murmur is common in ASD-I
E) there is a splitting of the second heart sound throughout all
phases of respiration
INT-7.710.
Select the shapes of a pericarditis-induced ST elevation on the ECG:
A) upward convex
B) upward concave
C) dome-like
D) maintained J point
E) non-maintained J-point
INT-7.711.
Which of the following have a beneficial effect in the treatment of
pituitary diabetes insipidus?
A) water restriction
B) glucagon
C) lithium
D) chlorpropamide
E) chlorothiazide
INT-7.712.
Which of the following are typical of excessive ADH secretion?
A) water restriction improves the symptoms
B) the patient might have lung cancer
C) an increased plasma osmolarity
D) a low sodium level in the urine
E) limb and ankle edema
INT-7.713.
The causes of hirsutism include:
A) Addison's disease
B) adrenal carcinoma
C) hyperthyroidism
D) phenytoin (Diphedan) therapy
E) the polycystic ovarian syndrome
INT-7.714.

Which of the following features are characteristic of Bence-Jones


protein?
A) it is found in the urine of 70% of patients with myeloma
B) it precipitates at 37 0C and is redissolved at higher temperatures
C) it is comprised of both kappa and lambda chains
D) the "Albustix" test is positive
E) when it is present the risk of renal symptoms is increased
MULTIPLE CHOICE QUESTIONS WITH KEY ANSWERS / TYPE II
Every question or incomplete statement has only one answer in the
combinations specified after each and every question:
Select one of these mentioned key combinations!!!
INT-7.715.
Secondary osteoporosis can develop in:
1) steroid therapy
2) hyperthyroidism
3) acromegaly
4) postmenopause
5) Cushing's syndrome
6) starvation due to an absorption disorder
A) 1, 3, and 4 are true
B) 2 and 5 are true
C) 4 and 5 are true
D) only 3 is true
E) all of the above
INT-7.717.
Which of the following is characteristic of "shock lung"?
1) congestive lung
2) hemorrhage
3) atelectasia
4) edema
5) capillary thrombosis
A) 1 and 3 are true
B) 2, 4, and 5 are true
C) 1 and 4 are true
D) only 3 is true
E) all of the above
INT-7.719.
Schistosomiasis has the following characteristic clinical symptoms:
1) fever and allergic symptoms
2) abdominal pain and diarrhea
3) portal hypertension
4) cor pulmonale
5) myocarditis
A) 1, 3, and 4 are true
B) 2 and 4 are true
C) 1 and 5 are true
D) only 5 is true
E) all of the above

INT-7.720.
Pulmonary involvement, abdominal complaints, and eosinophilia
occur in:
1) ascariasis
2) strongyloidosis
3) hymenolepiasis
4) trichinosis
A) 1 and 2 are true
B) 3 and 4 are true
C) 1 and 4 are true
D) 1, 3, and 4 are true
E) 1, 2, and 3 are true
F) all of the above
G) none of the above
INT-7.721.
Which of the following characterizes the cerebrospinal fluid in aseptic
meningitis?
1) a turbid fluid
2) a low glucose content
3) a polymorphonuclear reaction at the onset of the disease
4) an elevated protein level
A) 1 and 4 are true
B) 2 and 4 are true
C) 1 and 3 are true
D) 3 and 4 are true
E) all of the above
F) none of the above
INT-7.722.
Hypothermia occurs in:
1) myxedema
2) high-dose sedatives
3) alcohol intoxication
4) hyperthyroidism
A) 1, 2, and 3 are true
B) 1 and 3 are true
C) 2 and 4 are true
D) only 4 is true
E) all of the above
INT-7.723.
Which of the following methods is used in the therapy of opiate
intoxication?
1) gastric lavage in case of an oral opiate intake
2) naloxone iv.
3) meperidine
A) 1 and 2 are true
B) 1, 2, and 3 are true
C) only 2 is true
D) only 1 is true

INT-7.725.
Case Study:
An elderly patient with severe airway obstruction and chronic
bronchitis is treated with digitalis for circulatory insufficiency. During
therapy he suddenly develops a supraventricular paroxysmal
tachycardia (160/min) which was terminated with physical interventions.
Which of the following medications should be used for the
therapy of this arrhythmia?
1) digoxin iv.
2) pindolol iv.
3) metoprolol (Betaloc, cardioselective beta-blocker) iv.
4) quinidine p.o.
5) verapamil iv.
A) 1, 3, and 4 are true
B) 3 and 5 are true
C) 3 and 4 are true
D) 1 and 4 are true
E) 2, 4, and 5 are true
INT-7.726.
In which of the following do pulmonary abscesses develop?
1) pneumococcus type-III pneumonia
2) mitral stenosis
3) pulmonary infarction
4) scleroderma
5) regional enteritis
A) 1, 2, and 3 are true
B) 1 and 3 are true
C) 2 and 4 are true
D) only 4 is true
E) all of the above
INT-7.728.
Which of the following disorders can accompany lung carcinoma?
1) hypercalcemia
2) gynecomastia
3) myopathy
4) Cushing's syndrome
5) leukemoid reaction
A) 1, 2, and 3 are true
B) 1 and 3 are true
C) 2 and 4 are true
D) only 4 is true
E) all of the above
INT-7.729.
Which of the following methods help in the diagnosis of lung cancer?
1) bronchoscopy
2) a scalene lymph node biopsy
3) a cytological smear from the sputum
4) angiography
5) ultrasound

A) 1, 2, and 3 are true


B) 1 and 3 are true
C) 2 and 4 are true
D) only 4 is true
E) all of the above
INT-7.730.
Which of the following indicate an inoperability of lung cancer?
1) a markedly decreased lung compliance
2) a markedly decreased carbon dioxide diffusion capacity
3) a severely altered ventilation/perfusion ratio
4) marked hypoventilation
5) severe pulmonary hypertension
A) 1, 2, and 3 are true
B) 1 and 3 are true
C) 2 and 4 are true
D) only 4 is true
E) all of the above
INT-7.731.
The following alterations occur in less than 1% of hypertensive patients?
1) Cushing's disease
2) primary hyperaldosteronemia
3) pheochromocytoma
4) renovascular hypertension
A) 1, 2, and 3 are true
B) 1 and 3 are true
C) 2 and 4 are true
D) only 4 is true
E) all of the above
INT-7.732.
Case Study:
A blood pressure value of 170/120 was found on four occasions in a
25-year-old female patient who takes no antihypertensives. A physical
study showed no alterations. What should be done next?
1) nothing, because women are less susceptible to the sequels of
hypertension
2) to prescribe a high potassium diet and recheck the patient after
6 weeks
3) measure the sodium content of the red blood cells
4) check for secondary hypertension
A) 1, 2, and 3 are true
B) 1 and 3 are true
C) 2 and 4 are true
D) only 4 is true
E) all of the above
INT-7.733.
Case Study:
In a weak male patient with sinus bradycardia one doctor repeatedly
measured blood pressure values of 170/ 100 mmHg, while an other

doctor always obtained 160/ 100 mmHg. Which of the following


might have caused this difference?
1) the cuff is too wide
2) digital devices have different accuracies ("silent" range)
3) the emotional liability of the patient
4) the two doctors released the air with different speeds
A) 1, 2, and 3 are true
B) 1 and 3 are true
C) 2 and 4 are true
D) only 3 and 4 is true
E) all of the above
INT-7.734.
Which of the following alterations can be found in the fundus in
both hypertension and arteriosclerosis?
1) blurred disk margins
2) narrowed vessels
3) crossing of the vessels
4) copper-wire arteries
A) 1, 2, and 3 are true
B) 1 and 3 are true
C) 2 and 4 are true
D) only 4 is true ,
E) all of the above
INT-7.736.
Select the characteristic features of hypertension in end-stage renal
disease:
1) the hypertension is usually volume-dependent
2) the plasma renin level is usually low
3) the hypertension can be controlled with adequate dialysis
4) the hypertension is of a low-volume type
A) 1, 2, and 3 are true
B) 1 and 3 are true
C) 2 and 4 are true
D) only 4 is true
E) all of the above
INT-7.737.
Case Study:
A 14-year-old boys blood pressure is 140/ 100 mmHg. Two weeks ago he complained
of a sore throat Which ofthe following studies should be performed
next?
1) a urine sediment study
2) a urine cultivation for the presence of any microbes
3) a urine study with sulfosalicylic acid
4) the determination of the urine pH value
A) 1, 2, and 3 are true
B) 1 and 3 are true
C) 2 and 4 are true
D) only 4 is true
E) all of the above

INT-7.741.
Which of the following pathological states is accompanied by hypertension?
1) aortic insufficiency
2) hyperthyroidism
3) beriberi disease
4) aortic calcification
A) 1, 2, and 3 are true
B) 1 and 3 are true
C) 2 and 4 are true
D) only 4 is true
E) all of the above
INT-7.742.
Which of the following findings indicates a primary aldosteronism in
a hypokalemic patient treated with diuretics?
1) the urine sodium level is 120 mmol/24 hours, the urine potassium
level is 12 mmol/24 hours
2) the urine sodium level is 200 mmol/24 hours, the urine potassium
level is 20 mmol/24 hours
3) the urine sodium level is 120 mmol/24 hours, the urine potassium
level is 20 mmol/24 hours
4) the urine sodium level is 80 mmol/24 hours, the urine potassium
level is 40 mmol/24 hours
A) 1, 2, and 3 are true
B) 1 and 3 are true
C) 2 and 4 are true
D) only 4 is true
E) all of the above
INT-7.745.
Fibrinoid necrosis:
1) commonly occurs in essential hypertension
2) occurs in systemic lupus erythematosus (SLE)
3) occurs in aortic coarctation
4) usually occurs in malignant hypertension
A) 1, 2, and 3 are true
B) 1 and 3 are true
C) 2 and 4 are true
D) only 4 is true
E) all of the above
INT-7.747.
Case Study:
A 32-year-old male patient has paroxysmal hypertension and headaches,
but the repeated values of the urine and plasma levels of
vanilinmandelic acid, catacholamines and metanephrine are always
normal. Which of the following can be assumed?
1) inadequate urine sampling techniques
2) aberrant adrenal tissue
3) an inactive adrenal tumor
4) a cerebral astrocytoma
A) 1, 2, and 3 are true

B) 1 and 3 are true


C) 2 and 4 are true
D) only 4 is true
E) all of the above
INT-7.748.
Select the differential criteria of renal and non-renal progressive hypertension:
1) proteinuria
2) red blood cells casts
3) a high serum creatinine level
4) none of the above
A) 1, 2, and 3 are true
B) 1 and 3 are true
C) 2 and 4 are true
D) only 4 is true
E) all of the above
INT-7.749.
An atheroma-induced narrowing of the renal artey:
1) usually involves the proximal one-third of the renal artery
2) can be bilateral
3) post-stenotic dilation is common
4) can have a 'beaded" appearance
A) 1, 2, and 3 are true
B) 1 and 3 are true
C) 2 and 4 are true
D) only 4 is true
E) all of the above
INT-7.750.
Which of the following statements about fibromuscular dysplasia are true?
1) it can be uni- or bilateral
2) it can have an extrarenal localization
3) the distal two-thirds of the arteries are usually involved
4) it can be accompanied by an intracranial aneurysm
A) 1, 2, and 3 are true
B) 1 and 3 are true
C) 2 and 4 are true
D) only 4 is true
E) all of the above
INT-7.751.
Which of the following statements is true?
1) red wine alters the effect of MAO inhibitors
2) naloxone can cause hypotension in patients treated with nontricyclic
antidepressants
3) non-steroid anti-inflammatory agents can interfere with diuretics
and beta-blockers
4) nasal drops can cause hypotension
A) 1, 2, and 3 are true
B) 1 and 3 are true
C) 2 and 4 are true

D) only 4 is true
E) all of the above
INT-7.752.
Case Study:
A 55-year-old patient on antihypertensive therapy complains of palpitation
and headache. He also has accompanying signs of fluid retention. Which
of the followIng antihypertensive agents can cause these symptoms?
1) hydralazine
2) phentolamine
3) minoxidil
4) bumetanide
A) 1, 2, and 3 are true
B) 1 and 3 are true
C) 2 and 4 are true
D) only 4 is true
E) all of the above
INT-7.753.
Which of the following agents can block beta-adrenergic receptors?
1) ergot alkaloids
2) nicotine
3) phentolamine
4) propranolol
A), 1, 2, and 3 are true
B) 1 and 3 are true
C) 2 and 4 are true
D) only 4 is true
E) all of the above
INT-7.754.
Which of the following statements is true for prazosin?
1) it has a beneficial effect in peochromocytoma
2) it is contraindicated in pheochromocytoma
3) it is a postsynaptic alpha-adrenergic blocking agent
4) therapy should be started in the morning with high-doses
A) 1, 2, and 3 are true
B) 1 and 3 are true
C) 2 and 4 are true
D) only 4 is true
E) all of the above
INT-7.755.
The efficiency of which of the following antihypertensive agents depends
on the plasma renin activity?
1) methyldopa
2) guanabenz
3) clonidine
4) captopril
A) 1, 2, and 3 are true
B) 1 and 3 are true
C) 2 and 4 are true

D) only 4 is true
E) all of the above
INT-7.756.
Case Study:
A 60-year-old female patient has diabetes mellitus, hypertension
(160/105 mmHg), hyporeninemia, and hypoaldosteronemia. Which
of the following diuretics should not be used in the course of
antihypertensive therapy?
1) metolazone
2) amiloride
3) chlorthalidone
4) triamterene
A) 1, 2, and 3 are true
B) 1 and 3 are true
C) 2 and 4 are true
D) only 4 is true
E) all of the above
INT-7.757.
Case Study:
A 60-year-old female patient is admitted to an intensive care unit for
dyspnea. The patient has been treated with digoxin and diuretics. A
month later she developed a syncopal episode. What should be done?
1) an ECG should be taken
2) nifedipine therapy should be initiated
3) a determination of the serum potassium level
4) a determination of the serum calcium and magnesium levels
A) 1, 2, and 3 are true
B) 1 and 3 are true
C) 2 and 4 are true
D) only 4 is true
E) all of the above
INT-7.758.
Which of the following statements about calcium channel blocking
agents is true?
1) verapamil and diltiazem block the intracellular calcium influx
2) diltiazem enhances calcium outflux from the cell
3) a combination of calcium entry blockers with beta blockers
can lead to cardiac insufficiency
4) the direct effect of sublingual nifedipine administration lasts
for 3 hours
A) 1, 2, and 3 are true
B) 1 and 3 are true
C) 2 and 4 are true
D) only 4 is true
E) all of the above
INT-7.759.
Which of the following antihypertensive drugs has the most pronounced
sedative effect?
1) methyldopa

2) hydralazine
3) clonidine
4) guanethidine
A) 1, 2, and 3 are true
B) 1 and 3 are true
C) 2 and 4 are true
D) only 4 is true
E) all of the above
INT-7.760.
Which of the following drugs can cause ejaculation disorders?
1) reserpine
2)methyldopa
3) nifedipine
4) guanethidine
A) 1,2,and 3 are true
B) l and 3 are true
C) 2 and 4 are true
D) oniy 4 is true
E) all of the above
INT-7.761.
Which of the following effects is characteristic of nitroprusside?
1) dilation of the systemic veins
2) a decreased venous return
3) a decreased afterioad due to reduced peripheral resistance
4) it increases cardiac output in non-cardiac patients
A) 1,2,and 3 are true
B) l and 3 are true
C) 2 and 4 are true
D) only 4 is true
E) all of the above
INT-7.762.
Which of the following statements about antihypertensive drugs is tunie?
1)chlorthalidone, metolazone, and triamterene are ali diuretics
and have a similar effect on the potassium balance
2) captopril and saralasin are ACE inhibitors
3) propranolol, timolol, and atenolol are all lipid-soluble betablockers
4) minoxidil, hydralazine, diazoxide, and nitroprusside are all
vasodilators and elevate the plasma renin activity
A) 1,2, and 3 are true
B) l and 3 are true
C) 2 and 4 are true
D) only 4 is true
E) all of the above
INT-7.764.
Which of the following statements about beta-blockers is true?
1) they can cause hyperkalemia
2) they can cause hypernatremia
3) they can affect the release of insulin

4) they are contraindicated in cases of hypokalemia


A) 1, 2, and 3 are true
B) 1 and 3 are true
C) 2 and 4 are true
D) only 4 is true
E) all of the above
INT-7.765.
Which of the following methods is used in the therapy of local, regional
breast cancer:
1) a radical mastectomy (according to Halstead)
2) a modified radical mastectomy
3) chest wall irradiation
4) removal of the tumor followed by regional, high-dose irradiation
A) 1, 2, and 3 are true
B) 1 and 3 are true
C) 2 and 4 are true
D) only 4 is true
E) all of the above
INT-7.766.
Splenectomy and an exploratory laparotomy should be performed in
patients with Hodgkin's disease in stage:
1) IA, involving the neck
2) IIA, involving the neck and mediastinum
3) IIB, involving the neck and mediastinum
4) IVA
A) 1, 2, and 3 are true
B) 1 and 3 are true
C) 2 and 4 are true
D) only 4 is true
E) all of the above
INT-7.767.
A ball on the foot develops in patients with:
1) a bronchogenic carcinoma
2) a lung abscess
3) bronchiectasia
4) chronic obstructive pulmonary disease
A) 1, 2, and 3 are true
B) 1 and 3 are true
C) 2 and 4 are true
D) only 4 is true
E) all of the above
INT-7.768.
The following symptoms indicate that lung cancer has spread to
extrapulmonary areas:
1) back pain
2) the changing character of a chronic cough
3) hoarseness
4) pleuritis with signs of hydrothorax

A) 1, 2, and 3 are true


B) 1 and 3 are true
C) 2 and 4 are true
D) only 4 is true
E) all of the above
INT-7.769.
In patients with pleural mesothelioma the following factors in the
history helps in obtaining the diagnosis:
1) if the patient smokes
2) any previous exposure to asbestos
3) any previous exposure to silicon
4) the presence of a pleural hydrothorax
A) 1, 2, and 3 are true
B) 1 and 3 are true
C) 2 and 4 are true
D) only 4 is true
E) all of the above
INT-7.770.
Select the complications of chronic acute cholecystitis which require
early surgical intervention:
1) gallbladder empyema
2) ileus
3) emphysematous cholecystitis
4) jaundice
A) 1, 2, and 3 are true
B) 1 and 3 are true
C) 2 and 4 are true
D) only 4 is true
E) all of the above
INT-7.771.
Dynamic ileus can be caused by the following:
1) chronic lead intoxication
2) acute diffuse peritonitis
3) biliary colic
4) renal colic
5) intermittant acute porphyria
A) 1, 2, and 4 are true
B) 2, 3, and 4 are true
C) all of the above
D) 2, 3, 4, and 5 are true
E) 1, 2, 3, and 4 are true
INT-7.772.
Early metastases of gastric cancer can develop in the following organs:
1) the axillary lymph nodes
2) the supraclavicular lymph nodes
3) the inguinal lymph nodes
4) the liver
5) the testis

A) 1, 3, and 5 are true


B) 2, 4, and 5 are true
C) 1 and 3 are true
D) 2 and 4 are true
E) 2 and 5 are true
INT-7.773.
Prostatitis can be caused by the following:
1) gonorrhea
2) syphilis
3) tularemia
4) tuberculosis
5) brucellosis
A) 1, 3, and 5 are true
B) 1 and 4 are true
C) 2, 3, and 5 are true
D) only 3 is true
E) all of the above
INT-7.774.
Which of the following can develop following an embolism or occlusion
of an artery of the lower limb:
1) dry gangrene
2) wet gangrene
3) a hemorrhagic infarction
4) anemic infarction
5) bone tuberculosis
A) 1, and 2 are true
B) 1, 2, and 5 are true
C) only 1 is true
D) only 2 is true
E) 1, 2, 3, and 4 are true
INT-7.775.
Which alterations can necrotic tissue undergo?
1) resorption
2) emptying via a fistula
3) formation of a local cyst
4) ulceration
5) scarring
A) 1, 2, and 3 are true
B) 2 and 5 are true
C) 2, 4, and 5 are true
D) 2, 3, 4, and 5 are true
E) all of the above
INT-7.776.
Which of the following calculus localizations can lead to obstructive
jaundice?
1) in the gallbladder
2) in the common bile duct
3) in the cystic duct

4) at the papilla of Vater


5) in Wirsungian's duct
A) 1, 3, and 5 are true
B) 2 and 4 are true
C) 3 and 5 are true
D) only 3 is true
E) all of the above
INT-7.777.
Fatty degeneration of the liver can occur in the following:
1) alcohol intoxication
2) diabetes mellitus
3) malnutrition
4) anemia
5) carbon tetrachloride (CCl4) intoxication
A) 1, 2, and 3 are true
B) 2 and 4 are true
C) 3 and 5 are true
D) only 4 is true
E) all of the above
INT-7.778.
Fatty degeneration of the liver can be caused by:
1) chloroform
2) tetracycline
3) phosphorus
4) a methionine-deficiency
5) diabetes mellitus
A) 1, 2, and 3 are true
B) 1, 4, and 5 are true
C) 2 and 3 are true
D) only 3 is true
E) all of the above
INT-7.779.
Pathological proteins can accumulate in the plasma in:
1) ochronosis
2) malignant melanoma
3) Waldenstrm's macroglobulinemia
4) multiple myeloma
5) myeloid leukemia
A) 2 and 5 are true
B) only 1 is true
C) 1, 2, and 5 are true
D) 3 and 4 are true
E) 4 and 5 are true
INT-7.780.
Steatorrhea can be caused by:
1) decreased lipase production
2) bile secretion disorders
3) damage of the intestinal epithelium

4) celiac disease
5) tropical sprue
A) 1, 3, 4, and 5 are true
B) 4 and 5 are true
C) 1, 2, 4, and 5 are true
D) 2 and 5 are true
E) all of the above
INT-7.781.
Hemolytic jaundice can be caused by:
1) arsenic intoxication
2) an incompatible transfusion
3) gallstones
4) pernicious anemia
5) a pancreatic tumor
A) 1, 3, and 5 are true
B) 2 and 5 are true
C) 1, 2, and 4 are true
D) only 3 is true
E) all of the above
INT-7.782.
Which of the following can stimulate the formation of renal calculi?
1) dehydration
2) diabetes insipidus
3) hyperparathyroidism
4) hypertension
5) urine retention
A) 1, 3, and 5 are true
B) 2 and 4 are true
C) 2 and 3 are true
D) only 4 is true
E) all of the above
INT-7.783.
The effects of parathyroid hormone include:
1) an increased absorption of calcium in the intestines
2) an increase of the renal excretion of calcium
3) an increase of tubular phosphate excretion
4) an increased osteoblast activity
5) an increased osteoclast activity
A) 1, 2, and 4 are true
B) 2 and 4 are true
C) 2, 3, and 4 are true
D) 3 and 5 are true
E) only 4 is true
INT-7.784.
In which of the following does hypercalcemia occur?
1) in steatorrhea
2) in an overdose of vitamin D
3) in hypoparathyroidism

4) in hyperparathyroidism
5) in multiple myeloma
A) 1, 2, 4. and 5 are true
B) all of the above
C) 2 and 4 are true
D) 1, 3, and 4 are true
E) 2, 4, and 5 are true
INT-7.785.
Which of the following hormones can elevate the serum calcium level?
1) thyroxine
2) calcitonin
3) parathyroid hormone
4) glucocorticoids
5) testosterone
A) 2, 3, 4, and 5 are true
B) only 3 is true
C) 1, 2, and 3 are true
D) 3 and 4 are true
E) all of the above
INT-7.786.
Select the sequels of a pituitary eosinophilic adenoma:
1) gigantism
2) gargoylism
3) cachexia
4) acromegaly
5) dwarfism
A) 2, 3, and 5 are true
B) 2 and 3 are true
C) 1 and 4 are true
D) only 3 is true
E) all of the above
INT-7.787.
Which of the following can cause an air embolism?
1) damage of the pulmonary veins
2) damage of the systemic veins
3) inhalation of hyperbaric oxygen
4) opened veins following the abruption of the placenta
5) damage of the lymphatic vessels
A) 1, 2, and 4 are true
B) 3 and 5 are true
C) only 3 is true
D) 1 and 5 are true
E) all of the above
INT-7.788.
Select the sequels of lower limb thrombophlebitis:
1) a "Zahn-type" infarction
2) embolism of the pulmonary arteries
3) leg ulcers

4) renal infarction
5) aneurysm formation
A) 1, 3, and 4 are true
B) 2 and 4 are true
C) 1 and 5 are true
D) 2 and 3 are true
E) all of the above
INT-7.789.
Which of the following can be termed as local edema:
1) nutritional edema
2) inflammatory edema
3) cardiac edema
4) lymphedema
5) angioneurotic edema
A) 1 and 3 are true
B) 3 and 5 are true
C) 2, 4, and 5 are true,
D) only 3 is true
E) all of the above
INT-7.790.
Select the signs of Hutchinson's triad:
1) barrel-shaped teeth with serrated edges
2) syphilitic pemphigus
3) interstitial keratitis
4) pneumonia alba
5) deafness
A) 2, 3, and 4 are true
B) 1, 3, and 5 are true
C) 2 and 4 are true
D) only 2 is true
E) all of the above
INT-7.791.
Select the dangers of immunosuppressive therapy:
1) the occurrence of intercurrent infections
2) the development of a malignant tumor
3) a decreased titer of auto-antibodies
4) anemia
5) prolonged wound healing
A) 1, 3, and 5 are true
B) 2 and 3 are true
C) 3 and 4 are true
D) 1, 2, 4, and 5 are true
E) only 3 is true
INT-7.792.
Which of the following states increase the incidence of arteriosclerosis?
1) obesity
2) gastric cancer
3) diabetes mellitus

4) hypothyroidism
5) lymphoid leukemia
A) 1, 3, and 4 are true
B) 2 and 5 are true
C) 2, 3, and 5 are true
D) only 4 is true
E) all of the above
INT-7.793.
Select the sequels of arteriosclerosis:
1) dry gangrene of the foot
2) aortic aneurysm
3) myocardial infarction
4) cerebral softening
5) hemorrhagic necrosis of the bowel
A) 1, 3, and 5 are true
B) 2 and 4 are true
C) 3 and 4 are true
D) only 5 is true
E) all of the above
INT-7.794.
Which of the following diseases can lead to the development of
chronic cor pulmonale?
1) aortic coarctation
2) mitral insufficiency
3) patent foramen ovale
4) emphysema
5) silicosis
A) 1, 3, and 5 are true
B) 2, 4, and 5 are true
C) 3 and 4 are true
D) only 3 is true
E) all of the above
INT-7.795.
Which of the following can cause right-sided heart hypertrophy?
1) emphysema
2) silicosis
3) an old fibrotic tuberculosis infection
4) mitral stenosis
5) kyphoscoliosis
A) 1, 3, and 4 are true
B) 2 and 4 are true
C) 3 and 5 are true
D) only 4 is true
E) all of the above
INT-7.796.
Secondary cardiomyopathy can be caused by:
1) hyperthyroidism
2) pheochromocytoma

3) beriberi
4) amyloidosis
5) glycogenosis
A) 1, 3, and 5 are true
B) 2 and 4 are true
C) 4 and 5 are true
D) only 3 is true
E) all of the above
INT-7.797.
Right-sided heart hypertrophy can be caused by:
1) emphysema
2) silicosis
3) mitral stenosis
4) carcinoid syndrome
5) Adams-Stokes syndrome
A) 1 and 2 are true
B) 1, 2 and 3 are true
C) 1, 2 and 4 are true
D) 1, 2, 3, and 4 are true
E) all of the above
INT-7.798.
An aortic aneurysm can develop in:
1) arteriosclerosis
2) vascular syphilis
3) Takayashu's disease
4) giant cell arteritis
A) 1, 2, and 3 are true
B) 1 and 3 are true
C) 1 and 4 are true
D) 1 and 2 are true
E) all of the above
INT-7.799.
Which of the following alterations can occur in uremia?
1) fatty degeneration of the liver
2) pulmonary edema
3) gastroenteritis
4) fibrous pericarditis
5) osteomyelitis
A) 1 and 5 are true
B) 2, 3, and 4 are true
C) only 3 is true
D) 3 and 5 are true
E) all of the above
INT-7.800.
The clinical picture of nephrosis can occur in:
1) amyloidosis
2) malignant nephrosclerosis
3) anemic renal infarction

4) diabetic nephropathy
5) membranous glomerulonephritis
A) 1, 4, and 5 are true
B) 2 and 4 are true
C) only 3 is true
D) 1 and 2 are true
E) all of the above
INT-7.801.
Hydronephrosis can occur in:
1) urethral stricture
2) prostatic hypertrophy
3) diabetes insipidus
4) spinal damage
5) the lodging of a concrement in the urinary tract
A) 1, and 2 are true
B) 1, 2, and 3 are true
C) all of the above
D) 1, 2, 4, and 5 are true
E) 2 and 5 are true
INT-7.802.
Systemic hypertension can occur in:
1) acute diffuse glomerulonephritis
2) nephropathy of pregnancy
3) polyarteritis nodosa
4) renal cortical adenoma
5) hypernephroid carcinoma
A) all of the above
B) 1, 2, and 3 are true
C) 1, 3, 4 and 5 are true
D) 1, 4, and 5 are true
E) 1, 3, and 5 are true
INT-7.803.
Pulmonary hypertension can be caused by:
1) emphysema
2) silicosis
3) kyphoscoliosis
4) hydrothorax
5) destructive lung tuberculosis
A) 1, 2, and 3 are true
B) 2 and 4 are true
C) 4 and 5 are true
D) only 3 is true
E) all of the above
INT-7.804.
Which of the following diseases can be complicated by chronic cor
pulmonale?
1) anthracosis
2) emphysema

3) silicosis
4) bronchiectasia
5) bronchial asthma
A) 1, 3, and 5 are true
B) 2 and 4 are true
C) 3 and 4 are true
D) only 4 is true
E) all of the above
INT-7.805.
Which of the following is characteristic of silicosis:
1) a diffuse fibrosis
2) left ventricular hypertrophy
3) pleural adhesions
4) sparkling crystals are visible under UV light in a histological
specimen
5) chronic cor pulmonale
A) 1, 3, and 5 are true
B) 2 and 4 are true
C) 1 and 2 are true
D) only 4 is true
E) all of the above
INT-7.806.
Where do ulcers typically occur in the Zollinger-Ellison syndrome?
1) in the limbs
2) in the soft palate
3) in the stomach
4) in the duodenum
5) in the rectum
A) 1, 2, 3, and 4 are true
B) 2, 3, and 4 are true
C) 3 and 4 are true
D) 3, 4, and 5 are true
E) 2 and 3 are true
INT-7.807.
Obstructive jaundice can be caused by:
l) carcinoma of Vater's papilla
2) a concrement in the common bile duct
3) obstruction of the Wirsungian duct
4) a carcinoma of the gallbladder
5) a carcinoma of the pancreatic tail
A) 1, 2, and 5 are true
B) 1 and 2 are true
C) 1, 2, 3, and 5 are true
D) all of the above
E) 1, 2, 4, and 5 are true
INT-7.808.
Case Study:
A young female patient complains of sweating and palpitation. An
isulinoma is suspected. Which of the following findings support the diagnosis?

1) a low serum glucose level and a non-detectable serum insulin


level during a typical fit
2) the fits occur several hours before meals
3) a low serum pro-insulin level
4) a high serum C-peptide level following an infusion of insulin
A) 1, 2, and 3 are true
B) 1 and 3 are true
C) 2 and 4 are true
D) only 4 is true
E) all of the above
INT-7.810.
Which of the following drugs are considered as disease-modifying
agents in rheumatoid arthritis:
1) D-penicillinamine
2) glucocorticoids
3) gold-containing agents
4) non-steroid anti-inflammatory agents
A) 1, 2, and 3 are true
B) 1 and 3 are true
C) 2 and 4 are true
D) only 4 is true
E) all of the above
INT-7.811.
Which of the following factors markedly contributes to the development
of head and neck cancer?
1) alcohol
2) smoking
3) the Epstein-Barr virus
4) the hepatitis B virus
A) 1, 2, and 3 are true
B) 1 and 3 are true
C) 2 and 4 are true
D) only 4 is true
E) all of the above
INT-7.812.
Case Study:
A patient's mental state has changed in the recent past; his serum
sodium level is 119 mmol/L. The serum osmolarity is 265 mOsm/L
and the urine osmolarity is 350 mOsm. Sodium excretion is 85
mOsm/24h. Select the cause of this developing syndrome:
1) lung tuberculosis
2) demeclocycline
3) chlorpropamide
4) sodium depletion
A) 1, 2, and 3 are true
B) 1 and 3 are true
C) 2 and 4 are true
D) only 4 is true
E) all of the above

INT-7.813.
Select the two drugs which should be avoided in the therapy of hypertension
in systemic lupus erythematosus (SLE) patients:
1) captopril (Tensiomin)
2) methyl-dopa (Dopegyt)
3) nifedipine (Corinfar)
4) hydralazin (Depressan)
5) oxprenolol (Trasicor)
A) 1 and 2 are true
B) 1 and 3 are true
C) 1 and 4 are true
D) 1 and 5 are true
E) 2 and 3 are true
F) 2 and 4 are true
G) 2 and 5 are true
H) 3 and 4 are true
I) 3 and 5 are true
J) 4 and 5 are true
INT-7.815.
Which of the following are considered as symptomatic criteria for
systemic lupus erythematosus (SLE):
1) photosensitivity
2) ulceration of the buccal mucosa
3) atrioventricular blockade due to anti-SS-antibody
4) uveitis
5) polymyositis
A) 1 and 2 are true
B) 2 and 3 are true
C) 3 and 4 are true
D) 4 and 5 are true
E) 1 and 4 are true
F) 1 and 5 are true
G) 2 and 4 are true
H) 3 and 5 are true
IN-7.818.
Select the relationship between physiological and pathological
autoimmunity:
1) they cannot occur simultaneously
2) they can occur simultaneously
3) they can occur simultaneously but are not related to each other
4) they show a close relationship .
5) all autoimmune events are pathological
A) 1 and 2 are true
B) 1 and 3 are true
C) 1 and 4 are true
D) 1 and 5 are true
E) 2 and 3 are true
F) 2 and 4 are true
G) 2 and 5 are true
H) 3 and 4 are true

I) 4 and 5 are true


INT-7.817.
Which cells found in a salivatory gland biopsy of a patient with
Sjrgen's syndrome indicate local alterations:
1) CD4+ lymphocytes
2) DR+ macrophages
3) natural killer cells
4) eosinophilic cells
5) plasmocytes
A) 1, 3 and 5 are true
B) 1, 2 and 4 are true
C) 1, 4, and 5 are true
D) 2, 3 and 4 are true
E) 2, 4, and 5 are true
F) 3, 4, and 5 are true
G) 1, 2, and 5 are true
H) 1, 2, and 3 are true
I) 1, 3, and 4 are true
INT-7.818.
Select the viruses and antigens related to Sjrgen's syndrome:
1) HTLV I virus
2) Epstein-Barr virus
3) Coxsackie virus
4) adenoviruses
5) p24 antigen of the HIV virus
A) 1, 3, and 5 are true
B) 1, 2 and 4 are true
C) 1, 4, and 5 are true
D) 2, 3, and 4 are true
E) 2, 4, and 5 are true
F) 3, 4, and 5 are true
G) 1, 2, and 5 are true
H) 1, 2, and 3 are true
INT-7.820.
Which organs are primarily affected in polyarteritis nodosa (PAN)?
1) the kidney
2) the thyroid gland
3) the peripheral nervous system
4) the retina
5) the testis
A) 1, 3, and 5 are true
B) 1, 2, and 3 are true
C) 1, 3, and 4 are true
D) 1, 2, and 5 are true
E) 1, 2, and 4 are true
F) 2, 3, and 4 are true
G) 2, 3, and 5 are true
H) 2, 4, and 5 are true
I) 3, 4, and 5 are true

INT-7.821.
Select the typical signs of rheumatic polymyalgia:
1) it is a disease of elderly women
2) it is a disease of middle-aged women
3) it is accompanied by increased red blood cell sedimentation
rate
4) low dose steroid therapy is beneficial
5) cytostatic therapy is required as a rule
A) 1, 2, and 3 are true
B) 1, 3, and 4 are true
C) 1, 4, and 5 are true
D) 1, 3 and 5 are true
E) 2, 3 and 4 are true
F) 2, 3, and 5 are true
G) 2, 4, and 5 are true
H) 3, 4, and 5 are true
INT-7.822.
Which of the following belong to the group of hypersensitivity
vasculitis?
1) Schnlein-Henoch's purpura
2) Wegener's granulomatosis
3) serum disease
4) temporal arteritis
5) obliterating thrombangitis
A) 1 and 2 are true
B) 1 and 3 are true
C) 1 and 4 are true
D) 2 and 5 are true
E) 2 and 4 are true
F) 3 and 4 are true
INT-7.823.
Which of the following histocompatibility antigens predisposes
to rheumatoid arthritis?
1) HLA/DRI
2) HLA/DR2
3) HLA/DR3
4) HLA/DR4
A) 1 and 2 are true
B) 1 and 3 are true
C) 1 and 4 are true
D) 2 and 3 are true
E) 2 and 4 are true
F) 3 and 4 are true
INT-7.824.
Which of the following can be used in the therapy of progressive systemic
sclerosis:
1) D-penicillamine derivatives
2) long-term corticosteroid therapy
3) long-term cytostatic therapy
4) antacids and H2-receptor antagonists for the treatment of

reflux esophagitis
5) physical therapy
A) 1, 2, and 3 are true
B) 1, 2 and 4 are true
C) 1, 4, and 5 are true
D) 2, 3 and 4 are true
E) 2, 3, and 5 are true
F) 1, 2, and 5 are true
G) 3, 4, and 5 are true
H) 2, 4, and 5 are true
INT-7.825.
Which of the following drugs is used for the prevention of repeated
spontaneous abortions?
1) high-dose gamma-globulin
2) steroids
3) immunization with paternal lymphocytes
4) cyclophosphamide
5) anti-CD3 monoclonal antibodies
A) 1, 2, and 3 are true
B) 1, 2 and 4 are true
C) 1, 2, and 5 are true
D) 1, 3, and 4 are true
E) 1, 3 and 5 are true
F) 1, 4, and 5 are true
G) 2, 3 and 4 are true
H) 2, 4, and 5 are true
I) 2, 3, and 5 are true
J) 3, 4, and 5 are true
INT-7.826.
Select the symptoms of rheumatic fever:
1) polyarthritis
2) chorea minor
3) splenomegaly
4) hematuria
5) carditis
A) 1, 2, and 3 are true
B) 1, 2 and 4 are true
C) 1, 2, and 5 are true
D) 1, 3, and 4 are true
E) 1, 3 and 5 are true
F) 2, 3 and 4 are true
G) 2, 4, and ~ are true
H) 3, 4, and 5 are true
I) 1, 4, and 5 are true
J) 2, 3, and 5 are true
INT-7.827.
Which of the following findings support the autoimmune character
of pernicious anemia?
1) the presence of anti-parietal cell antibodies
2) vitiligo

3) immune complex mediated renal lesions


4) it occurrs more frequently than multiple sclerosis
5) it occurrs more frequently than Hashimoto's thyroiditis
A) 1, 2, and 3 are true
B) 1, 3, and 4 are true
C) 1, 4, and 5 are true
D) 1, 2, and 5 are true
E) 1, 3 and 5 are true
F) 2, 3 and 4 are true
G) 2, 4, and 5 are true
H) 3, 4, and 5 are true
INT-7.828.
Which of the following interventions and diseases can activate the
complement system:
1) plasmapheresis
2) hemodialysis
3) pacemaker surgery
4) acute pancreatitis
5) surgical anesthesia
A) 1, 2, and 3 are true
B) 1, 2 and 4 are true
C) 1, 2, and 5 are true
D) 1, 3, and 4 are true
E) 1, 3 and 5 are true
F) 1, 4, and 5 are true
G) 2, 3 and 4 are true
H) 2, 4, and 5 are true
I) 3, 4, and 5 are true
J) 2, 3, and 5 are true
INT-7.829.
Select the clinical symptoms which accompany hyper-eosinophilia:
1) hypertension
2) myalgia
3) endomyocardial fibrosis
4) lung infiltration
5) cholangitis
A) 1, 2, and 3 are true
B) 1, 2 and 4 are true
C) 1, 2, and 5 are true
D) 1, 3, and 4 are true
E) 1, 3 and 5 are true
F) 1, 4, and 5 are true
G) 2, 3 and 4 are true
H) 2, 4, and 5 are true
I) 2, 3, and 5 are true
J) 3, 4, and 5 are true
INT-7.830.
Select the sites of immune complex deposition:
1) the chorionic plexus
2) the renal basement membrane

3) the retina
4) in synovial tissue
5) in subcutaneous tissue
A) 1, 2, and 3 are true
B) 1, 2 and 4 are true
C) 1, 2, and 5 are true
D) 1, 3, and 4 are true
E) 1, 4, and 5 are true
F) 1, 3 and 5 are true
G) 2, 3 and 4 are true
H) 2, 4, and 5 are true
I) 2, 3, and 5 are true
J) 3, 4, and 5 are true
INT-7.831.
Which of the following viruses play a role as initiating agents in the
development of insulin-dependent diabetes mellitus:
1) mumps virus
2) coxsackie B4 virus
3) measles virus
4) Epstein-Barr virus
5) herpes simplex virus
A) 1, 2, and 3 are true
B) 1, 2 and 4 are true
C) 1, 2, and 5 are true
D) 1, 3, and 4 are true
E) 1, 3 and 5 are true
F) 1, 4, and 5 are true
G) 2, 3 and 4 are true
H) 2, 4, and 5 are true
I) 2, 3, and 5 are true
J) 3, 4, and 5 are true
INT-7.832.
Which of the following symptoms are related to the "lupus anticoagulant":
1) hemorrhage
2) thrombocytopenia
3) anemia
4) repeated abortions
5) proneness to thrombosis
A) 1, 2, and 3 are true
B) 1, 2 and 4 are true
C) 1, 2, and 5 are true
D) 1, 3, and 4 are true
E) 1, 3 and 5 are true
F) 1, 4, and 5 are true
G) 2, 3 and 4 are true
H) 2, 3, and 5 are true
I) 2, 4, and 5 are true
J) 3, 4, and 5 are true
INT-7.833.
Select the cytokines which play a primary role in the activation of

endothelial cells:
1) IL-3
2) IL-1
3) TNF-alpha
4) IL-10
5) interferon-gamma
A) 1, 2, and 3 are true
B) 1, 2 and 4 are true
C) 1, 2, and 5 are true
D) 1, 3, and 4 are true
E) 1, 3 and 5 are true
F) 1, 4, and 5 are true
G) 2, 3 and 4 are true
H) 2, 4, and 5 are true
I) 2, 3, and 5 are true
J) 3, 4, and 5 are true
INT-7.834.
What does a liver biopsy performed in an alcoholic liver lesion reveal?
1) diffuse steatosis
2) intracytoplasmic hyaline
3) monoglobular (regular) cirrhosis
4) central sclerotizing hyaline necrosis
A) l, 2, and 3 are true
B) 1 and 3 are true
C) 2 and 4 are true
D) only 4 is true
E) all of the above
INT-7.835.
Which of the following alterations are characteristic of an early
myocardial infarction?
1) undulation of the myocardial fibrils at the borderline of the
infarction
2) a lowered myocardial succinate-dehydrogenase activity
3) a lowered glycogen content in the myocardial fibers
4) immunoperoxidase-positive antiglobulin in the necrotized
myofibrils
A) 1, 2, and 3 are true
B) 1 and 3 are true
C) 2 and 4 are true
D) only 4 is true
E) all of the above
INT-7.836.
Which of the following findings is characteristic of focal segmental
glomerulosclerosis?
1) the non-selective character of the proteinuria
2) a kidney biopsy often reveals IgM and C3 deposits
3) the progressive character of chronic glomerulonephritis in
many patients
4) the development of the nephrotic syndrome in many patients

A) 1, 2, and 3 are true


B) 1 and 3 are true
C) 2 and 4 are true
D) only 4 is true
E) all of the above
INT-7.837.
Which of the following enzymes is deficient in.hemolytic anemia?
1) pyruvate-kinase
2) glutathione-synthetase
3) glucose-6-phosphate-dehydrogenase
4) phosphofructokinase
A) 1, 2, and 3 are true
B) 1 and 3 are true
C) 2 and 4 are true
D) only 4 is true
E) all of the above
INT-7.838.
Which of the following cardiac disorders can elicit cyanosis in the
early neonatal period:
1) Tetralogy of Fallot
2) an atrial septal defect
3) transposition of the major vessels
4) endocardial fibroelastosis
A) 1, 2, and 3 are true
B) 1 and 3 are true
C) 2 and 4 are true
D) only 4 is true
E) all of the above
ASSOCIATION QUESTIONS
Associate the following terms/statements marked by the letters A, B,
C... with the corresponding statements/terms marked by and in the
order given by the figures 1, 2, 3...
...for example: 1-C, 2-B, 3-A, 4-D. Put the answer as C, B, A, D!
(Note: Different statements can be associated with the same terms!!!)
INT-7.839.
Associate the following statement(s) with their corresponding term(s)!
A) potassium excreting kidney
B) difficulties in the initiation of movements; slow relaxation
C) during such a fit the serum potassium level decreases
D) athletes
1) Familial hypokalemic paralysis
2) Thomsen's syndrome
3) Primary hyperaldosteroneism
4) Ossifying myositis
INT-7.840.
Associate the following statement(s) with their corresponding term(s)!
A) vitamin A intoxication
B) it is complicated by sarcoma

C) neurologic complications; fractures


D) skin pigmentation
E) a saddle-nose deformity
F) uneven cartilage
G) in the differential diagnosis angina pectoris
1) Paget's disease
2) Hyperostosis
3) Osteoporosis
4) Fibrotic dysplasia
5) Ollier's syndrome
6) Relapsing polychondritis
7) Tietze's syndrome
INT-7.842.
Associate the following statement(s) with their corresponding term(s)!
A) macroglobulin, vascular insufficiency, plasmapheresis
B) occurs in multiple myeloma, macroglobulinemia, systemic
lupus and peripheral vascular insufficiency
C) macroglobulin which causes hemolytic anemia
1) Increased viscosity
2) Cryoglobulins
3) Cold agglutinins
INT-7.843.
Associate the following statement(s) with their corresponding term(s)!
A) rash, pruritis, arthritis, neurologic disorders
B) hemorrhagic necrosis caused by endotoxins
C) deficiency of the inhibitor of the first complement component
1) Schwartzman's reaction
2) Serum disease
3) Inherited angio-edema
INT-7.846.
Associate the following statement(s) with their corresponding term(s)!
A) hypertension
B) hypotension
C) both of the above
D) none of the above
1) Vasopressin
2) Prostaglandins
3) Renin
4) Aldosterone
5) Glucagon
6) Bradykinin
INT-7.847.
Associate the following statement(s) with their corresponding term(s)!
A) an increased venous capacity
B) sympathetic blockade
C) an inhibition of the converting enzyme
D) a low plasma volume

E) short-term alpha blockade


1) Nitroprusside
2) Captopril
3) Methyldopa
4) Amiloride
5) Phentolamine
6) Chlorthalidone
INT-7.848.
Associate the following term(s) with their corresponding statement(s)!
A) Guanethidine
B) Propranolol
C) Pindolol
D) Saralasin
E) Clonidine
1) withdrawal is accompanied by increased catecholamine production
(rebound effect)
2) it inhibits angiotensin receptors
3) intrinsic sympathomimetic activity
4) alpha-adrenergic vasoconstriction
5) catecholamine release following iv. administration
INT-7.849.
Associate the following term(s) with their corresponding statement(s)!
A) Nephrosclerosis
B) Nephrosis
C) Chronic pyelonephritis
D) Renovascular disease
E) Conn's syndrome
1) the 24 hour protein excretion is 3.8 g; normal iv. urogram
2) the 24 hour protein excretion is 400 mg; normal iv. urogram
3) the 24 hour protein excretion is 400 mg; iv. urogram reveals
the symmetrically shortened, deformed calyx
4) the 24 hour protein excretion is 800 mg; iv. urogram shows the
left kidney to be 7cm long and the right kidney to be 10 cm long
INT-7.850.
Associate the following term(s) with their corresponding statement(s)!
A) 11-hydroxylase defect
B) 3-beta-dehydrogenase defect
C) Both of the above
D) None of the above
1) hypertension
2) hypokalemia
3) hyperkalemia
4) glucocorticoid therapy
5) metyrapone therapy
6) hypogonadism
7) virilisation
8) usually associated with fatal outcome in pediatric patients
INT-7.851.

Associate the following term(s) with their corresponding statement(s)!


A) Bilateral nodular hyperplasia
B) Aldosterone-producing adrenal adenoma
C) Both of the above
D) None of the above
1) it is accompanied by hypokalemia, hypertension, and low
plasma renin level
2) the incidence in males is higher than in females
3) the plasma aldosterone level is lowered in the upright position
4) the plasma aldosterone level is not lowered in the upright position
5) a blood sample from the adrenal vein indicates a bilateral disorder
6) it is accompanied by hypotension
INT-7.853.
Associate the following term(s) with their corresponding statement(s)!
A) X-bound agammaglobulinemia
B) Severe combined immune deficiency
C) Regular variable immune deficiency
D) Immunpoglobuline deficiency with an elevated IgM
E) Selective IgA deficiency
1) an antibody deficient syndrome, sometimes with cyclic
neutropenia, thrombocytopenia or hemolytic anemia
2) hypogammaglobulinemia with normal count of circulating B
cells
3) agammaglobulinemia and severe cellular immune deficiency
4) a mild antibody deficient syndrome sometimes accompanied
by infections, allergic disorders and autoimmunity
5) a syndrome caused by abnormal rearrangement of
immunoglobulin-coding genes; due to this the
immunoglobulin molecule cannot be assembled
INT-7.854.
Associate the following term(s) with their corresponding statement(s)!
A) Hepatitis
B) Viral hepatitis
C) Acute cholecystitis
D) Chronic active hepatitis
E) Fatty liver
1) jaundice, itching, xanthomas, hepatosplenomegaly, high
serum alkalic phosphatase and cholesterol level
2) jaundice, anorexia, nausea, tenderness of the liver, high
transaminase level, normal alkaline phosphatase
3) jaundice, hepatosplenomegaly, moderately increased
transaminase level, hypergammalbuminemia, smooth muscle
antibody positivity
4) jaundice, fever, hepatomegaly, renal insufficiency, coma, high
alkaline phosphatase level, moderately increased transaminase
level
5) jaundice, tenderness and pain in the right upper quadrant,
leukocytosis
INT-7.855.
Associate the following statement(s) with their corresponding term(s)!

A) cholestyramine has a beneficial effect


B) a good response to a gluten-free diet
C) a good response to wide-spectrum antibiotics
D) a good response to the withdrawal of milk
E) the administration of digestive enzymes decreases steatorrhea
1) Lactase deficiency
2) Celiac disease
3) Enteropathy caused by bile acid salts
4) Chronic pancreatitis
5) Whipple's disease
INT-7.856.
Associate the following term(s) with their corresponding statement(s)!
A) Carcinoma of the pancreas body
B) Mild, chronic pancreatitis
C) Pancreatitis related to alimentary and metabolic factors
D) Carcinoma of the pancreas head with complete obstruction
E) Pancreatic disease related to hemochromatosis
1) a lack or markedly decreased amount of fluid, low bicarbonate
and amylase level
2) a decreased amount of fluid, with normal bicarbonate and
amylase level
3) a normal amount of fluid, with low bicarbonate and normal or
low amylase level
4) a normal amount of fluid, with normal bicarbonate and low
amylase level
5) an increased amount of fluid, low bicarbonate and normal
amylase level
INT-7.857.
Associate the following term(s) with their corresponding statement(s)!
A) Celiac sprue
B) Tropical sprue
C) Both of the above
D) None of the above
1) it usually responds to wide-range antibiotic therapy
2) it shows a dramatic improvement to folic acid therapy
3) during exacerbation the D-xylose excretion is very low
4) it is usually accompanied by megaloblastic anemia
5) the diagnosis is based on the identification of PAS-positive
macrophages in the lamina propria
INT-7.858.
Associate the following term(s) with their corresponding statement(s)!
A) Pancreas
B) Lung
C) Kidney
D) Muscle
E) Breast
1) Zenker's waxy necrosis
2) enzymatic fatty necrosis
3) traumatic fatty necrosis

4) anemic infarction
5) hemorrhagic infarction
INT-7.859.
Associate the following term(s) with their corresponding statement(s)!
A) Gaucher's disease
B) Hunter's syndrome
C) Niemann-Pick's disease
D) Hand-Schuller-Christian's disease
E) Gierke's disease
1) cholesterol
2) phosphatide
3) cerebrozide
4) glycogen
5) mucopolysaccharide
INT-7.860.
Associate the following statement(s) with their corresponding term(s)!
A) penicillinase-producing Streptococcus
B) Enterococcus
C) Clostridium welchii and tetani
D) Klebsiella pneumoniae
E) Proteus mirabilis
1) Ampicillin
2) Gentamycin
3) Penicillin G and an Aminoglycoside
4) Methycillin
5) Penicillin G
INT-7.861.
Associate the following statement(s) with their corresponding term(s)!
A) extrapyramidal symptoms
B) acute gout
C) goiter
D) gynecomastia
E) pancreatitis
F) peripheral neuropathy
1) Propylthiouracil
2) Isoniazid
3) Spironolactone
4) Antimetabolic agents
5) Phenothiazides
6) Corticosteroids
INT-7.862.
Associate the following term(s) with their corresponding statement(s)!
A) Clonidine
B) Reserpine
C) Prazosine
D) Methyldopa
E) Captopril
1) postsynaptic alpha-adrenergic blockade

2) bradycardia, increased gastrointestinal activity and myosis


3) the inhibition of angiotensin I angiotensin II conversion
4) it's metabolite acts as a neurotransmitter in the central alphaadrenergic
receptors
5) Coombs-positivity occurs in 20% of the patients who take this
drug, though hemolytic anemia is very rare
6) an alpha-2 adrenerggrc agonist which affects the central nervous sytem,
depresses the cardiovascular system, and induces vasodilation
INT-7.863.
Associate the following term(s) with their corresponding statement(s)!
A) Ampicillin
B) Carbenicillin
C) Tetracycline
D) Second generation cephalosporins
1) it penetrates into the cerebrospinal fluid and, therefore, can be used
in the therapy of meningitis caused by gram-negative pathogens
2) it is an antibiotic Df a primary choice in Pseudomonas infection
of the urinary tract
3) it is an antibiotic of primary choice in the therapy of otitis media
caused by Hemophilus influenzae
4) a rash always occurs if the drug is used in the therapy of Infectious
Mononucleosis
5) it has a beneficial effect in Rickettsia infections
6) diarrhea is a common side-effect
INT-7.864.
Associate the following term(s) with their corresponding statement(s)!
A) Digoxin
B) Lidocaine
C) Propranolol
D) Albuterol
1) a sudden withdrawal in patients with angina pectoris can
cause angina
2) it is an efficient bronchodilator
3) it should not be given to asthmatic patients
4) it lowers the increased ventricular rate in atrial fibrillation
5) it affects only ventricular arrhythmias
INT-7.865.
Associate the following term(s) with their corresponding statement(s)!
A) Diffuse spasm
B) Achalasia
C) Scleroderma
1) due to lack of esophageal peristalsis, it cannot relax during swallowing
2) early repeated peristaltic contractions
3) the lower part of the esophagus cannot contract
INT-7.866.
Associate the following term(s) with their corresponding statement(s)!
A) Bile acid salts
B) Steatorrhea
C) Both of the above

D) None of the above


1) a decreased production of deoxycholic acid
2) the resection of a 100 cm or longer part of the ileus
3) symptomatic therapy with cholestyramine
4) the MCT (medium chain triglycerides) have a beneficial effect
5) bile acid malabsorption
6) decreased bile acid synthesis
INT-7.867.
Associate the following term(s) with their corresponding statement(s)!
A) Meckel's diverticulum
B) Hirschprung's disease
C) Pyloric stenosis
D) Cystic fibrosis
E) Enteric cyst
1) no ganglion cells are found in a rectal biopsy
2) surgical resection of the involved, non-hypertrophic segment
3) a rudimentary omphalomesenteric duct
4) inflammation elicits symptoms which cannot be differentiated
from appendicitis
5) surgical resection of the hypertrophic muscle has a beneficial effect
6) meconium ileus
INT-7.868.
Associate the following term(s) with their corresponding statement(s)!
A) Adenomatous polyp (tubular adenoma of the colon)
B) Villous adenoma (papillaryy colon)
C) Both of the above
D) None of the above
1) it consists of regular glands and crypts covered with cylindric
epithelium differentiated as absorptive, mature goblet cells
2) it is accompanied by familial multiple polyposis '
3) carcinoma in situ will develop in aboutl0% of these polyps
4) commonly occurs at the rectosigmoid border
5) histologically is similar to juvenile polyps
INT-7.869.
Associate the following term(s) with their corresponding statement(s)!
A) Tubular type colonic polyp
B) Intermediate form of colonic polyp
C) Villous adenoma
1) a 53% probability of malignancy
2) a 35% probability of malignancy
3) a 46% probability of malignancy
INT-7.870.
Associate the following term(s) with their corresponding statement(s)!
A) Tropical sprue
B) Celiac spree
C) Both of the above
1) it is more common within one family or in the individuals with

0 Rh+ blood group


2) a sensitivity to the gliadine fraction of glutene
3) uniform involvement of the intestine
4) steatorrhea
5) a good response to therapy
INT-7.871.
Associate the following statement(s) with their corresponding term(s)!
A) mucosal penetration
B) enterotoxin
1) Salmonella
2) Shigella
3) Cholera vibrio
INT-7.872.
Associate the following statement(s) with their corresponding term(s)!
A) it can cause peripheral neuropathy and can induce hemolysis
in G-6PD-deficient patients
B) if administered to the newborn or mother during the 3rd
trimester, it can cause Kernicterus in the newborns
C) it is best for the therapy of prostatitis
D) it is recommended for the therapy of Serratia- and
Pseudomonas-induced infections
E) it is not recommended during pregnancy due to danger of the
maternal hepatic lesions and fetal dental dysplasia
1) Trimethoprim-sulfamethoxazol
2) Sulfonamides
3) Tetracyclins
4) Nitrofurantoin
5) Aminoglycosides
INT-7.873.
Associate the following term(s) with their corresponding statement(s)!
A) Gastrin
B) Secretin
C) CCK-PZ
1) it increases the motillity of the antrum of the stomach
2) it postpones gastric evacuation
3) it relaxes the pyloric sphincter
RELATION ANALYSIS
In the following questions determine if the. statement in the first half
of the sentence and the explanation in the second half of the sentence
are true and if a causal relationship exists between them. Select
the single correct version from the five possible combinations:
A) both the statement and the explanation are true and a
causal relationship exists between them;
B) both the statement and the explanation are true but there
is no causal relationship between them;
C) the statement is true, but the explanation is false;
D) the statement is false, but the explanation itself is true;
E) both the statement and the explanation are false.

INT-7.874.
The femoral vein is the only site from which a pulmonary embolism
can originate, therefore in cases of femoral vein thrombosis the danger
of pulmonary embolism should be considered.
INT-7.875.
Glomerulonephritis is common in systemic lupus erythematosus
because the circulating antigen-antibody complexes precipitate in
the walls of the glomerular capillaries.
INT-7.876.
Prostate hyperplasia leads to the retention of urine, therefore a bacterial
infection can easily occur in prostatic hyperplasia.
INT-7.877.
The urethral lumen always narrows in nodular prostate hyperplasia,
therefore hypertrophy of the bladder's smooth muscles occurs.
INT-7.878.
An embolus is always solid because fluids and gases cannot cause
obstruction of the vascular lumen.
INT-7.879.
Anaerobic glycolysis is enhanced in the body's tissue during hypoxia,
therefore glycogen accumulates in the hypoxic hepatic tissue.
INT-7.880.
Melanin accumulation occurs in Addison's disease because
hydrocortisone fails to inhibit the pituitary melanocyte-stimulating
hormone (MSH) secretion.
INT-7.882.
In hepatic jaundice urobilinogen is not present in the urine because
the bile secreting function of the liver cells is impaired.
INT-7.883.
Urethra obstruction can lead to calculus formation because the retention
and concentration of the urine promotes this process.
INT-7.884.
During starvation the fat stores decrease, therefore the regression
of a pre-existing fatty liver might occur.
INT-7.885.
Aortic stenosis markedly contributes to the left ventricular load,
therefore left ventricular hypertrophy is a sequel of aortic stenosis.
INT-7.886.
At higher atmospheric pressures the blood absorbs less gases,
therefore a sudden decompression can cause an air embolism.
INT-7.887.
A large amount of proteins is excreted in the urine during the acute
phase of glomerulonephritis, therefore in acute glomerulonephritis
the subsequent hypoproteinemia causes edema.

INT-7.888.
Fluid transudation across the lymphatic vessel walls occurs in
lymphedema because in lymphedema the permeability of the lymphatic
vessels increases.
INT-7.889.
Heparin exhibits an antithrombin effect because the heparin molecule
is comprised of acidic mucopolysaccharides.
INT-7.890.
Inflammatory hyperemia is an active process because the vascular
permeability is increased in a focus of inflammation.

INT-7.891.
Anemic infarction does not always develop after an occlusion of the
mesenteric artery because an impairment of the venous outflow is another
important factor in the development of hemorrhagic infarction.
INT-7.892.
Gelatinous pneumonia is an exudative form of pulmonary tuberculosis
because a specific granuloma is the most typical tuberculotic reaction.
INT-7.893.
In Transposition Corrigata the aorta originates from the anterior part of
the heart, while the pulmonary artery originates from the posterior part of the
heart, therefore Transposition Corrigata is incompatible with life.
INT-7.894.
The caseous exudate occurring in tuberculous pericarditis can be
calcified, therefore congestion, ascites, and edema can develop in
tuberculous pericarditis.
INT-.895.
The left ventricular blood supply is inadequate in mitral insufficiency,
therefore. the left ventricle can exhibit a slight hypertrophy
in mitral insufficiency
INT-7.896.
Regurgitation of blood occurs in aortic insufficiency, therefore Zahn's
pouches can develop under the aortic valves in aortic insufficiency.
INT-7.897.
In mitral insufficiency the congestion can spread to the pulmonary
circulation, therefore the right atrium and ventricle will undergo
dilation and hypertrophy in mitral insufficiency.
INT-7.898.
The juxtaglomerular system plays an important role in the development
of essential hypertension because an increased amount of
renin interacts with angiotensin and causes hypertension.
INT-7.899.
Renal papillary necrosis mostly occurs in diabetic patients because the
Kimmelstiel- Wilson's syndrome is a typical renal disease in diabetes.

INT-7.900.
A marked cell decomposition occurs in leukemic patients following
irradiation, therefore the renal accumulation of urate salts is expected
in post-radiation therapy in leukemic patients.
INT-7.901.
A partial or intermittant urethra occlusion can lead to the development
of hydronephrosis because urine retention in the pyelon usually
leads to a bacterial infection.
INT-7.902.
Acute glomerulonephritis is accompanied by albuminuria, therefore the
edema typical of acute glomerulonephritis is due to hypoproteinemia.
INT-7.903.
Chronic pyelonephritis is usually accompanied by hypertension, therefore
renal alterations typical of hypertension occur in pyelonephritis.
INT-7.904.
Glomerular atrophy occurs during an interstitial inflammatory process
in the kidney, therefore glomerular hyalinization will sooner or
later occur in chronic pyelonephritis.
INT-7.905.
A primary glomerular lesion develops following phenacetine abuse,
therefore interstitial inflammation is due to secondary tubular alterations
caused by phenacetine abuse.
INT-7.906.
The red blood cells are more vulnerable in the spherocytic-type of
anemia because the red blood cells' osmotic resistance is decreased
in the spherocytic type of anemia.
INT-7.907.
Metaplasia of the bronchial epithelium can occur in bronchiectasis,
therefore adeno-carcinoma is common in patients with
bronchiectasis.
INT-7.908.
The lung volume increases in acute lung dilation in case of asphyxial
death, therefore these pulmonary changes are similar to
those occurring in emphysema.
INT-7.909.
Dyspnea is one of the leading symptoms of emphysema because the
surface of the emphysematous lung is decreased.
INT-7.910.
Pulmonary alterations elicited by silicosis are irreversible because
the lung cannot eliminate the absorbed silicon.
INT-7.911.
A malabsorption syndrome can develop in chronic pancreatits because
a decreased absorption surface caused by chronic
pancreatitis leads to malabsorption

INT-7.912.
A progressive narrowing of the lumen is characteristic of regional
enteritis, therefore regional enteritis is a common cause of pyloric
stenosis.
INT-7.913.
Anemia can develop in malabsorption syndrome because in malabsorption
the fat absorption is impaired.
INT-7.914.
Male patients with liver cirrhosis frequently develop gynecomastia
because the hepatic estrogen inactivation system is progressively
destroyed.
INT-7.915.
A gallstone can sometimes impact itself in the hepatic duct, therefore
acute hemorrhagic pancreatic necrosis can develop as a complication
of cholelithiasis.
INT-7.916. Relation Analysis
Portal congestion develops due to liver cirrhosis, therefore an
intermittant jaundice can occur as a complication of liver cirrhosis.
CASE STUDIES
Answer the multiple task questions (simple choice and multiple choice
with/without key answers; relation analysis etc.) as they are related to
each case study!!!
INT-7.918.
A 28-year-old female patient had a tonsilectomy at the age of 6, and an
operation due to otitis media at the age of 9. Since that time she has
had recurrent otitis media and consequently received the appropriate
drug therapy. A month before admission the patient had rhinitis and
took oxytetracycline (Tetran). A day before admission she complained
of otalgia in the right ear. Next morning the patient had nausea and
fever (39oC). By that time her husband reported that he could not
communicate with her.
At examination: occipital stiffness; Kernig-positivity; narrow pupils only
slight reactivity to light. Central facial paralysis on the left side. Increased
flexor tone in the upper limbs. Lower limbs are also flexed at
knees. Stupor. Lumbar puncture: colorless cerebrospinal fluid. Cell
count: 51 000/mm3. Pandy's test (globulin in the CSF): positive.
7.918/ 1. Single Choice Question
The most probable diagnosis is:
A) choriomeningitis
B) cerebral abscess
C) purulent meningitis
D) epidemic encephalitis
E) basilar tuberculous meningitis
7.918/2. Single Choice Question
The most typical symptom of the disease is:
A) an increased flexor tone of the upper limbs
B) a central facial paralysis
C) an impaired light reaction of the pupil

D) disorientation
E) meningeal signs
7.918/3. Single Choice Question
The typical CSF finding is:
A) dissociation of the cell proteins
B) compression CSF
C) a high cell count with a granulocyte prevalence
D) a mixed cell finding
E) an increased lymphocyte count
7.918/4. Single Choice Question
Medication and therapy include:
A) surgery
B) antibiotics + steroids
C) aimed antibiotic therapy
D) antituberculotic therapy
E) anti-inflammatory therapy
INT-7.919.
A 37-year-old female patient was urgently admitted due to abdominal
seizures. During childhood the patient had been treated for jaundice
in an infectious diseases ward. She has had 3 pregnancies; one was
terminated by abortion. During the first pregnancy the patient complained
of abdominal seizures on the right side. She took drotaverin
(No-Spa) and was on a diet for a long time. Then the patient became
symptom-free. She took oral contraceotive (norgestrol ethynil-estradiol
- Ovidon) for three years. Two months before admission the oral contraceptive
had been replaced by an IUD. The patient frequently
compained of headaches and nausea. Before admission she had fever,
vomited and developed abdominal seizures under the right costal arch,
in the area of the cardia of the stomach. Spasmolytics relieved the
pain, but not completely. The urine of the patient had a slightly darker
color than normal.
Physical status (most important findings): subicterus, enlarged (2 fingers),
tender liver. Pressure tenderness under the right costal arch.
Laboratory findings: temperature: 38.9oC. red blood cell sedimentation
rate: 39 mm/h. Serum bilirubin: 45 mol/L. SGOT: 210 IU; SGPT:
180 IU. ALP: 99 IU/L; GGT: 130 IU/L. WBC: 12900. Smear: Stab cells:6,
Segm.: 83, Lymph.: 11%. Urine: norm. Specific density:. 1024. Prot.:
(++). Ubg.: norm., Bilirubin: positive. Precipitate: several WBC/visual
field.
7.919/ 1. Single Choice Question
The most probable diagnosis is:
A) adrug-induced cholestasis
B) viral hepatitis
C) acute pyelonephritis
D) calculous cholecystitis
E) adnexitis
7.919/2. Select One Of The Key Combinations
Which of the following studies should be performed in the acute
phase of the disease to support the diagnosis?
1) an ERCP study
2) a HbsAg determination
3) a serum amylase determination

4) an ultrasound study
5) cholecystography
A) (1),( 2), and (5) are true
B) (3), (4), and (5) are true
C) (2), (3), and (4) are true
D) all of the above
E) none of the above
7.919/3. Select One Of The Key Combinations
Which of the following laboratory parameters should be regularly
monitored during the diasease?
1) serum Fe, Cu, and ferritin
2) bacteriological and parasitologic bile tests
3) WBC and serum amylase level
4) serum bilirubin, GOT, GPT, ALP
5) Weber's stool test
A) (1), ( 2). (4), and (5) are true
B) (3), (4), and (5) are true
C) (3) and (4) are true
D) all of the above
E) none of the above
7.919/4. Select One Of The Key Combinations
Which of the following complications is the most probable?
1) coagulation disorders
2) chronic active hepatitis
3) pancreatitis
4) empyema vesicae felleae
5) hydronephrosis
A) (2), (3), and (4) are true
B) (1), (4), and (5) are true
C) (3) and (4) are true
D) all of the above
E) none of the above
7.919/5. Select One Of The Key Combinations
Which of the following studies supporting the diagnosis should be
performed?
1) if a liver disease due to a long-term use of oral contraceptive is
suspected, a liver biopsy should be done
2) if the fever is further elevated an acute cholecystography
should be done
3) an ultrasonic study should always be done first, since this
non-invasive method yields important information
4) if a hydrops develops, ERCP should be done
5) palpation of hydrops is an indication for surgery
A) (1), (3), and (5) are true
B) (2), (4), and (5) are true
C) (3) and (5) are true
D) all of the above
E) none of the above

INT-7.920.
Case Study
A 42-year-old female patient complained for 3 months of sustained
headaches. Mild analgetics she subsequently received were successful
for the headaches. No other symptoms were found in the history.
The patient had three normal pregnancies, but during the third trimester
of the fourth pregnancy the patient had to take diuretics. The
only disease she had recently, was a cystitis which was treated with
antibiotics. The patient's father and uncle both died of a heart attack
at the age of 43 and 46 years old respectively. Physical examination:
the patient is slim, her blood pressure taken at rest on both arms is
180/120 mmHg. Vein "notching" was observed during examination of
the fundus. No other alterations could be observed. The initial laboratory
tests showed trace amounts of protein in the urine. The ECG, and
chest X-ray findings were normal.
7.920/ 1. Single Choice Question
The most probable cause of the hypertension is:
A) pheochromocytoma or renal artery disease
B) renal artery disease or brain tumor
C) cerebral tumor or Cushing's syndrome
D) Cushing's syndrome or primary hypertension
E) primary hypertension or chronic pyelonephritis
7.920/2. Single Choice Question
All of the following diagnostic studies should be performed, EXCEPT:
A) intravenous pyelography
B) a bone marrow puncture
C) a creatinine clearance test
D) the plasma renine activity
7.920/3. Single Choice Question
If hypokalemia is found then all of the following can be assumed, EXCEPT:
A) pheochromocytoma
B) primary aldosteronism
C) Cushing's syndrome
D) renal perenchymal disease
E) antidiuretic drug abuse
7.920/4. Single Choice Question
If a patient has primary hypertension, the primary therapeutic interventions
comprise all of the following, EXCEPT:
A) rest
B) intravenous hydralazine
C) hydrochlorothiazide
D) a low salt diet
E) sedatives
INT-7.921.
Case Study
A 28-year-old male patient was admitted to an intensive care unit for
chest pain (which had lasted for two hours), cough, and dyspnea. He
had had several similar fits in the past. Such fits were usually
elicited by emotional factors or strain. The patient(also has hay fever. Furthermore,
the members of his family also have similar symptoms and fits.
Physical examination: dyspnea, orthopnoea and cyanosis. A whistlingstridorous

sound was found on auscultation of the lungs. Heart rate:


130/min, regular. Sputum: sticky, mucous. Arterial P02: 65 mmHg.
7.921 / 1. Single Choice Question
The most probable diagnosis is:
A) right heart insufficiency
B) left heart insufficiency
C) intrinsic bronchial asthma
D) extrinsic bronchial asthma
E) pneumoconiosis
7.921/2. Single Choice Question
All of the following can cause this disease, EXCEPT:
A) antigens, allergens
B) excessive salt intake
C) emotional stress
D) infection
E) analeptics
7.921/3. Single Choice Question
The most probable respiratory functional finding during a fit is:
A) a decreased FEV 1
B) an increased vital capacity
C) a decreased residual volume
D) a decreased airway resistance
E) an increased arterial P02
7.921/4. Single Choice Question
The best therapeutic step is:
A) fluid restriction
B) cardioversion
C) beta-mimetics, bronchodilators
D) beta-adrenergic blockers
E) diuretics
INT-7.922.
Case Study
A 64-year-old male patient was admitted complaining of frequent urination
urges, anxiety, and nocturia. The symptoms have been persisting
for 6 months. Apart from the afore mentioned symptoms, the patient
also had two episodes of burning dysuria, which was treated
with antibiotics. Furthermore, the patient also has a one-year history
of angina pectoris and takes nitroglycerin when needed.
Physical examination: blood pressure: 130/90, heart rate 90/min.
Rectal examination revealed an enlarged prostate. Laboratory and ECG
findings were not contraindictory for surgery. The prostate was removed
by a transurethral resection with minimal blood loss. Six hours
following surgery the patient started shivering, his temperature increased
to 40 C and his blood pressure fell to 90/60 mmHg.
7.922/1. Single Choice Question
The most probable diagnosis is:
A) gram-negative sepsis
B) a myocardial infarction
C) a post-operative hemorrhage
D) arrhythmia
E) lobular pneumonia

7.922/2. Single Choice Question


A further study of the hypotension will reveal all of the followIng,
EXCEPT:
A) lactic acidosis
B) cold, wet skin
C) an increased level of fibrin-degradation products
D) sinus tachycardia on the ECG
E) an increased urine output
7.922/3. Single Choice Question
Bacteriemia due to urinary tract infection is caused by:
A) Salmonella typhi murium
B) Shigella sonnei
C) Vibrio cholerae
D) Hemophilus influenza
E) Escherichia coli
7.922/4. Single Choice Question
The most important intervention is:
A) cardioversion
B) the monitoring of the central venous pressure
C) an electric pacemaker
D) salt restriction
E) potassium infusion
INT-7.923.
Case Study
A 31-year-old male patient was admitted for dyspnea, ankle edema
developing in the evening, and a belt-like, tense pain over the liver
area. These complaints developed over a span of a half-a-year. The
patient has no history of rheumatic arthralgia, diabetes or hypertension.
He smokes 8-10 cigarettes per day and systematically consumes
1-2 dl of strong alcoholic beverages and about 1/3 L of wine every day.
7.923/1. Single Choice Question
The most probable diagnosis is:
A) a congenital defect
B) an alcoholic myocardial lesion
C) asymmetric septal hypertrophy
D) chronic cor pulmonarye
E) ischemic heart disease
7.923/2. Single Choice Question
All of the following physical findings support the assumed diagnosis,
EXCEPT:
A) relative dullness of the heart on the left side up to the axMary line
B) a gallop rhythm
C) hepatomegaly, ankle edema
D) a diastolic murmur at the apex
E) cyanosis of the extremities and lips
7.923/3. Single Choice Question
The diagnosis based on the case history and physical findings can
be verified by all of the following instrumental studies, EXCEPT:
A) an ECG
B) a chest X-ray plus 2-dimensional heart imaging
C) an echocardiogram

D) a stress ECG
E) apex cardiography
F) heart catheterization
7.923/4. Single Choice Question
If the assumed diagnosis is correct, all of the following alterations
can be found, EXCEPT:
A) a right bundle branch block or other conduction disorders on
the ECG
B) an enlarged heart due to a hypertrophied left ventricle ("cor
bovinum") with sustained pulmonary congestion on the X-ray image
C) left ventricular dilation, hypokinetic septum including the posterior
wall of the left atrium, and intact valves on the echocardiogram
D) a pansystolic plateau on the apical ECG
E) cardiac catheterization reveals a left-right shunt
7.923/5.
All of the following therapeutic steps should be undertaken EXCEPT:
A) compensation with, digitalis, diuretics, and potassium replacement
therapy
B) a change of habits
C) vasodilators
D) surgery
E) the prohibition of alcohol
INT-7.924.
Case Study
A 56-year-old female patient complains of weakness and fatigue that have
been persisting persisting for about half a year. The patient sweats at
night and is sometimes febrile. During this period she has lost 10 kg. A month
ago she complained of pressure under the left costal arch. The patient sometimes
has palpitations and pain which radiates to her left arm.
Physical examination: the patient's skin and mucosa are pale. Percussion
causes sternal pain. The liver is palpable. The spleen is enlarged
exceeding the left costal arch by 4 cm.
Laboratory findings: Hb: 100 g/L; Hct: 0.29; WBC: 100 g/L; platelets:
350 g/L; blood smear: myeloblasts: 0.01; promyelocytes: 0.03;
myelocytes: 0.06; juvenile: 0.08; bands: 0.10; segmented: 0.60; eo: 0.02;
ba: 0.06; ly: 0.04; red blood cells: sustained hypochromia and
anisocytosis. Normoblasts: 1 / 100 WBC; platelets: anisocytosis.
7.924/ 1. Single Choice Question
The most probable diagnosis is:
A) myelofibrosis
B) leukemoid reaction
C) chronic granulocytic leukemia
D) thrombocytemia
E) polycythemia vera
7.924/2. Single Choice Question
Keeping in mind the above noted diagnosis, select the stage of the
disease:
A) the initial stage
B) the CML accelerated stage
C) the CML inactive stage
D) the blastic stage

7.924/3. Single Choice Question


Which of the following is a characteristic laboratory finding?
A) a chromosome analysis will demonstrate the Philadelphia chromosome
B) the morphology of the blood cells is abnormal
C) monocytosis
D) the leukocyte alkaline phosphate score is high
E) a low reticulocyte count
7.924/4. Single Choice Question
Select the therapeutic drug of first choice:
A) tamoxifen (Zitazonium)
B) cyclophosphamide (cytoxan)
C) busulfan (Myleran)
D) melphalan (Alkeran)
7.924/5. Select One Of The Key Combinations
Select the most common causes of death in the above noted disease:
1) disorders of pacemaker function
2) blast crisis
3) busulfan-induced bone marrow aplasia
4) pathologic fractures
5) infections
A) (1), (2), and (5) are true
B) (2},- (3), and (5) are true
C) (1) and (4) are true
D) only (3) is true
E) all the above
INT-7.925.
Case Study
A 56-year-old male patient was urgently admitted in an unconscious
state in an otherwise good physical state with stable vital signs. The
patient has had sustained hypertension for several years. The patients
smokes constantly and consumes 4-5 cups of coffee daily. Physical
examination: a male patient in otherwise stable general condition.
Emphysematous chest and coarse inspiratory sounds with prolonged
expiration were found. The heart dullness exceeds the medial clavicular
line by 2 fingers to the left. Regular rhythmical heart rate,
bradycardia, low systolic murmur at the apex. Heart rate: 52/min,
blood pressure: 150-100 mmHg. Pedal dorsalis pulses are not palpable,
the medial and posterior tibial arteries are' palpable. Adipous abdominal
wall, palpable, with no signs of pathological resistance or tenderness.
The liver exceeds the right costal arch by 3 fingers. It has a
smooth surface, with a round-shaped edge; no tenderness is found.
The spleen is not palpable. Limb edema is absent.
Laboratory findings: urine and blood - negative. Se cholesterol: 8.6
mmol/L; Triglycerides: 5.1, SI; GOT: 16; GPT: 39; LDH: 190; gammaGT: 66 IU. ECG: PQ interval: 0.40; repolarization: normal.
7.925/1. Single Choice Question
The most probable diagnosis is:
A) epilepsy
B) hypoglycemia
C) Adams-Stokes syndrome
D) myocardial infarction

7.925/2. Single Choice Question


Select the most relevant study to support the diagnosis:
A) ECG
B) PCG
C) electromyography
D) ultrasound study
E) EEG
7.925/3. Single Choice Question
Select the most effective mode of therapy:
A) digitalis therapy
B) pacemaker implantation
C) diuretic therapy
D) sedation
E) anticoagulant therapy
7.925/4. Single Choice Question
After therapy the patient's state improved dramatically, he experienced
no malaise or states of unconsciousness. Can the patient continue
his work?
A) he can continue his full-time work
B) the patient can only take a part-time job
C) the patient cannot work at all
D) the patient should be limited to bed-rest .
INT-7.926.
Case Study
A 32-year-old female patient felt a sudden retrosternal pain, which
radiated to the shoulder and neck. The patient had an asphyxia episode
and coughed. The patient takes oral contraceptives for 2 years.
Physical examination: dyspnea, cyanosis, tachycardia (130 /min). The
left ankle is slightly swollen and tender. No other pathological signs
were found. Blood pressure: 110/80 mmHg, red blood cell sedimentation
rate: 8 mm/h. WBC: 6200; Ht: 41%. Urine: negative findings.
7.926/ 1. Select One Of The Key Combinations
Which of the following studies are the most important in supporting
the diagnosis:
1) ECG
2) blood smear
3) lung scintigraphy
4) echocardiography
5) heart catheterization
6) coronary angiography
7) urinalysis
A) (2), (4), (6), and (7) are true
B) (4), (6), and (7) are true
C) (1) and (3) are true
D) (2) and (5) are true
E) all of the above
7.926/2. Single Choice Question
Select the most probable correct diagnosis:
A) myocardial infarction
B) pulmonary embolism
C) pericarditis

D) lung tuberculosis
E) acute pancreatitis
7.926/3. Single Choice Question
Select the correct therapy:
A) antibiotics
B) immune suppression
C) anticoagulants
D) rest and diet
E) antituberculotics
INT-7.928.
Case Study
A 55-year-old male patient suffers for years from chronic bronchitis
which causes obstructive ventilation disorders. He also has diabetes
mellitus. The patient was urgently admitted with fever, cough, substantial
amount of purulent sputum, dyspnea, and cyanosis. The arterial
blood gas analysis was as follows: P02: 51 mmHg; PC02: 54
mmHg; pH: 7.28; St. bicarbonate: 31 mmol/L.
7.928/ 1. Single Choice Question
Select the correct acid-base disorder:
A) metabolic acidosis
B) metabolic alkalosis
C) respiratory acidosis
D) respiratory alkalosis
7.928/2. Single Choice Question
The patient required oxygen therapy and received 4 L 02/min via a
nasal catheter. After 24 hours the blood gas analysis showed the following
changes: P02: 65 mmHg; PC02: 81 mmHg; pH: 7.20; St. bicarbonate:
36 mmol/L. Select the correct therapy:
A) increase the oxygen via the nasal catheter
B) decrease the oxygen via the nasal catheter
C) artificial respiration is required
D) inhalation of 100% (pure) oxygen
INT-7.929.
Case Study
A 30-year-old female patient had fever and observed red, coin-size,
tender nodes on the extensor surface of both hindlimbs 3 months after
delivery. The family doctor recommended further examination. A
chest X-ray showed lobular opacity in the hilus of both lungs. Blood
analysis, ESR, and other laboratory studies were negative.
7.929/1. Single Choice Question
Select the correct diagnosis:
A) Hodgkin's disease
B) tuberculosis
C) sarcoidosis
D) leukemia
E) infectious mononucleosis
7.929/2. Select One Of The Key Combinations
Which of the following endoscopic studies should be performed to
verify the diagnosis?
1) bronchoscopy
2) Klassen-biopsy

3) pleural biopsy
4) mediastinoscopy
5) thoracotomy
A) (1) and (3) are true
B) (1) and (4) are true
C) (2) and (4) are true
D) (1) and (5) are true
E) (3) and (5) are true
7.929/3. Single Choice Question
Select the correct therapy:
A) combined antibiotic
B) combined antituberculotic
C) cytostatic
D) steroid
E) antibiotics+steroids
F) antituberculotic+steroid
INT-7.930. Case Study
A 50-year-old alcoholic male patient has complained for several months
of fatigue, weakness, and abdominal swelling. The swelling was at first
intermittant, later it became constant. The day before, his hindlimbs
became swollen too. The patient lost his appetite and his body weight
decreased.
Physical examination: jaundice, free abdominal fluid, liver: a four -fin-ger enlargement, uneven surface.
Laboratory findings: Se-bilirubin: 50; direct bilirubin 40 mol/1; GOT:
50 IU; GPT: 22 IU; gamma-GT: 580 IU; ALP: 40 IU/L. Urine: bilirubin:
positive, UBG: increased.
7.930/ 1. Single Choice Question
Select the correct diagnosis:
A) alcoholic hepatitis
B) liver metastases
C) hepatic cirrhosis
D) right heart insufficiency
7.930/2. Select One Of The Key Combinations
Which of the following studies should be done in the acute phase of
the disease to verify the diagnosis?
1) duodenal tube
2) liver scintigraphy
3) cholecystography
4) liver biopsy
A) (1, (2), and (3) are true
B) (1) and (3) are true
C) (2) and (4) are true
D) only (4) is true
E) all of the above
7.930/3. Select One Of The Key Combinations
Select the possible complications of the disease:
1) esophageal varicous bleeding
2) hepatoma
3) coagulation disorders

4) empyema vesicae felleae


A) (1, (2), and (3) are true
B) (1) and (3) are true
C) (2) and (4) are true
D) only (4) is true
E) all of the above
INT-7.931.
Case Study
A 31-year-old female patient consults her family doctor. She complains
of a strong pain under the right costal arch and in the lumbar area
which has persisted for several weeks. The pain lasts from several minutes
to several hours and is not related to meals or the quality of the
consumed food. Sometimes she feels pain when travelling by bus. When
the pain is very intense, it is accompanied by nausea and swelling.
The patient was afebrile at all times. Sometimes she felt a more frequent
urge to urinate, but urination was never accompanied by a burning
sensation. Her menstruation cycle is normal. The patient takes no
medications.
7.931 / 1. Single Choice Question
Which of the following questions should be asked on the basis of the
above complaints?
A) Have you experienced intense sweating?
B) Have you had dark stools?
C) Have you had dark urine?
D) Do you have an appetite?
E) Do you feel disgusted by certain meals?
7.931/2. Select One Of The Key Combinations
Which of the following laboratory tests should be performed to verify
the diagnosis?
1) the red blood cell sedimentation rate
2) a complete WBC count
3) the urine amylase concentration
4) any urine precipitate
5) the urine bilirubin concentration
A) (1, (2), and (3) are true
B) (4) and (5) are true
C) (2), (3), and (4) are true
D) all of the above
E) none of the above
7.931/3. Single Choice Question
After having done these laboratory tests, which of the following studies
should be performed?
A) liver scintigraphy
B) an X-ray of the stomach
C) a native abdominal X-ray
D) abdominal ultrasonography
E) a CT
7.931/4. Single Choice Question
Select the appropriate therapy:
A) antibiotics

B) sedation
C) spasmolytics
D) diuretics
E) antacids
7.931/5. Select One Of The Key Combinations
Which of the following recommendations would you give your patient?
1) Check your body weight regularly!
2) Check your stools regularly!
3) Check your urine regularlyl
4) Check your menstruations regularly!
5) Check your body temperature regularly!
A) (1), (2), (3), and (4) are true
B) (2), (4), and (5) are true
C) (3) and (5) are true
D) all of the above
E) none of the above
7.931/6. Single Choice Question
Which specialist should the patient consult if the complaints persist?
A) a gynecologist
B) a surgeon
C) a urologist
D) an internist
E) a rheumatologist
INT-7.932. Case Study
A diagnosis of Graves' disease was established. Other diseases could
not be identified. Read the following case study carefully! Don't forget
that some data can be misleading!
K.J. is a 28 -year-old female patient.
History: the patient observed
frequent defecations with soft stool accompanied by itching of the rectal
area. She has a good appetite but still lost 8 kg. The patient is
always sweating, her hands are shaking, and she feels a stinging pain
in her eyes. The patient has never been pregnant, but she has experienced
lactorrhea.
Status: height: 172 cm; body weight: 55 kg. Heart rate: 116/min, fast
and rhythmic; blood pressure: 150/60 mmHg. The, thyroid gland shows
unhomogenous enlargement. Intention tremor. Divergent strabismus.
The sclera is seen during upward-downward gaze. Frequent blinking.
A diastolic murmur over the aorta. The liver edge is 5 cm beneath the
right costal arch.
Laboratory finding's: sedimentation rate: 4 mm/h. Hb: 13,4 g/ 100 ml.
Ht: 42%. Cholesterol: 3.l,mmol/L. Bi: 22 mol/L. SGOT: 40 U/L, alkaline
phosphatase: 56 U/L. UN: 5.6 mmol/L. SeNa: 139, K: 4.6 mmol/
L. Glucose tolerance test after oral glucose load. shows a "tower"-type
blood glucose curve. ECG: sinus rhythm with several supraventricular
extrasystoles. Biventricular load, "P pulmonale".
Chest X-ray: a 2mm narrowing of the trachea and a narrowed
esophagus at the level between cervical V- dorsal II vertebra. The apex
of the lungs is covered by a shade of soft tissues. A marked enlargement
of the right ventricle.
Radionuclide diagnostics of the thyroid gland: T3 uptake: 1.61; T3
4.2 nmol/L. T4 228 nmol/L. These values suggest hyperthyroidism.

The m99Tc-scintigraphy revealed a hot area of 3 cm in diameter in the


right lobe of the gland. This area corresponds to a palpable node. The
other areas of the thyroid are not contoured.
7.932/1. Select One Of The Key Combinations
All of the following findings are typical of Graves' disease, EXCEPT:
1) frequent defecation, soft stool
2) rectal itching
3) loss of body weight despite good appetite
4) sweating
5) tremor, nervousness
6) ophthalmologic disorders
7) galactorrhea
A) (1), (2), (5), and (6) are not typical findings
B) (2) and (7) are not typical findings
C) (5), (6), and (7) are not typical findings
D) (4), (5), and (7) are not typical findings
E) (2), (3), (5), and (6) are not typical findings
7.932/2. Relation Analysis
It is difficult to differentiate Graves' disease from the nephrotic syndrome
because in both cases the serum cholesterol value is abnormal.
A) both the statement and the explanation are true and show a
causal relationship exists between them
B) both the statement and the explanation are true, but no causal
relationship exists between them
C) the statement is true but the explanation is false
D) the statement is false but the explanation itself is true
E) both the statement and the explanation are false
7.932/3. Select One Of The Key Combinations
All of the following are typical ECG and chest X-ray findings occurring
in Graves' disease, EXCEPT:
1) sinus tachycardia
2) supraventricular extrasystole
3) a biventricular load .
4) 'P pulmonale"
5) a narrowed trachea and esophagus
6) an enlarged right ventricle
7) a soft tissue shade on the apex of the lungs
A) (3), (5), and (7) are not typical findings
B) (1), (4), and (5) are not typical findings
C) (4) and (6) are not typical findings
D) (1), (3), (5), and (6) are not typical findings
E) (3), (4), (5), and (7) are not typical findings
7.932/4. Relation Analysis
Toxic adenoma of the thyroid gland can cause Graves' disease because
it elicits hyperthyroidism with exophthalmus
A) both the statement and the explanation are true and a causal
relationship exists between them
B) both the statement and the explanation are true, but there is
no causal relationship between them
C) the statement is true but the explanation is false
D) the statement is false but the explanation itself is true

E) both the statement and the explanation are false


7.932/5 Select One Of The Key Combinations
All of the following drugs or drug combinations can be used in the
therapy of Graves' disease, EXCEPT:
1) beta-blockers
2) tranquilizers
3) bromocriptine (Parlodel)
4) methimazole
5) propylthiouracil
6) clofibrate (Miscleron)
7) hydrocortisone
A) (2), (6) and (7) cannot be used
B) (1), (3), and (5) cannot be used
C) (4), (6), and (7) cannot be used
D) (2), (3), (4), and (5) cannot be used
E) (3) and (6) cannot be used
INT-7.933.
Case Study
A 55-year-old hypertensive, smoking male patient was treated half a
year ago for a sudden intensive retrosternal pain radiating to his left
arm and chin accompanied by sweating. After discharge the patient
remained complaint-free for a short period. Later the symptoms re-,
turned. The patient also complained of weakness and fatigue,
retrosternal pain, dyspnea, a mild limb edema, and a sensation of pressure
in the hepatic area occurred even after minor physical exercise.
The patient has been treated with digitalis and diuretics.
Physical examination: lip- and acrocyanosis, ankle edema. Enlarged
(the size of a palm) liver. The relative dullness of the left heart is
shifted to the lateral axillary line and that of the right heart is about 1 finger
to the right. Tachycardia, gallop rhythm. Congestive murmur (rhonchus)
above the diaphragm. Harsh respiration.
Laboratory findings: cholesterol: 9.6 mmol/L. Triglyceride: 5.4 mmol/L.
Total lipids: 14 G/L. We: 21 mm/h. Blood glucose: 5.0 mmol/L. Ht: 48%.
ECG: sinus rhythm; tachycardia; deviation to the left; I-II, aVL, V1-4:
low QS complexes; V5-6: low R wave with ST elevation. Several ventricular
extrasystoles from the same focus. Chest X-ray + 2-D imaging
of the heart: marked enlargement of the left heart; paradoxical, slow
pulsation along the left contour of the heart.
7.933/ 1. Single Choice Question
The most probable diagnosis is:
A) decompensated aortic defect
B) left ventricular aneurysm after an extensive anterior
myocardial infarction
C) congestive cardiomyopathy
D) tricuspid insufficiency
E) subacute bacterial endocarditis
F) Epstein's anomaly
G) left atrial myxoma
H) subendocardial ischemia
7.933/2. Select One Of The Key Combinations
Which of the following studies should be performed to support this
diagnosis?

1) 2D-echocardiography
2) phono-mechanocardiography
3) radionuclide left ventriculography
4) a thallium-scintigraphy exercise test
5) left ventricular angiocardiography; coronary angiography
6) thallium-scintigraphy at rest
7) an electrophysiologic study
8) X-ray chymography
A) (2), (5), and (8) are true
B) (1), (2), (4), (6), and (7) are true
C) (1), (3), (5), (6), and (8) are true
D) (1), (2), (3), and (7) are true
E) (4), (6), and (8) are true
7.933/3. Select One Of The Key Combinations
Select the correct therapy:
1) implantation of an artificial valve
2) compensation with digitalis, diuretics, potassium, and coronary
dilators with a long-term effect
3) a left ventricular aneurysm section combined with coronary
by-pass surgery if necessary
4) fibrinolytic therapy
5) steroids; salicylates
6) antibiotic therapy
A) (2) and (3) are true
B) (3), (4), and (6) are true
C) (5) and (6) are true
D) (1) and (6) are true
E) (2), (4), and (5) are true
INT-7.934.
Case Study
A month ago a patient became subfebrile. He also complained of a sore
throat, poor general condition, pain in the limbs and in the chest, and
cough. At present he complains of dyspnea, sensation of pressure in
the liver, tachycardia. He can sleep only with an elevated pillow.
Physical examination: mild lip cyanosis. Respiration is accompanied
by flaring of the nostrils. Bilateral protruding jugular veins, tachycardia.
A third heart sound at the apex was noted; decreased heart sounds.
The relative dullness of the heart is shifted toward the lateral chest
wall. Harsh respiration. The liver is enlarged by 4 flngers. The spleen
is not palpable. Adequate pulsation of the peripheral arteries. Blood
pressure: 120/70 mmHg.
ECG: sinus tachycardia, low-voltage. Left axis deviation. Diffuse, flattened
T waves.
Chest X-ray (+ 2-D imaging of the heart): "cor bovinum". Low pulsation
along the heart contoure.
Laboratory findings: We: 30 mm/h. AST: 120 U; SGOT, GPT, and alkaline
phosphatase - normal. Pharynx: bacteria + . A high titer of the
anti-Coxsackie virus antibodies.
WBC count: Se: 59%; Ly: 40%; Mo: 1%.
7.934/ 1. Single Choice Question
The most probable diagnosis is:
A) a pulmonary embolus

B) a left ventricular aneurysm following a myocardial infarction


C) an exudative pericarditis with a substantial amount of fluid
after a viral infection
D) a combined mitral defect
E) rheumatic carditis
F) viral hepatitis
7.934/2. Single Choice Question
Which of the following non-invasive studies should be done to support
this diagnosis?
A) echocardiography
B) lung scintigraphy
C) phono-mechanocardiography
D) bronchoscopy
7.934/3. Select One Of The Key Combinations
Select the correct therapy:
1) salicylates, steroids and rest (if necessary)
2) implantation of an artificial valve
3) digitalis
4) hepatoprotective therapy; diet; bed-rest
5) anticoagulant therapy
6) diuretics; pericardial puncture if necessary
A) (1) and (6) are true
B) (1), (2), and (5) are true
C) (3), (5), and (6) are true
D) (1) and (5) are true
E) (3) and (6) are true
INT-7.936.
Case Study
A 26-year-old female patient complained for 3 years of hypogastric,
spastic pain accompanied by abdominal swelling. Every year the patient
has 5-6 bouts of diarrhea, each lasting for 2-3 months. The stool
contains fresh red blood and mucus. The patient also complains of
frequent occasions of tenesmus. She has lost weight and complains of
arthralgia. The symptoms worsen after consumption of milk. Physical
examination: slightly anemic mucosa; hypogastric tenderness. Other
findings negative.
7.936/1. Single Choice Question
The most probable diagnosis is:
A) a rectal tumor
B) Crohn's disease
C) the irratable bowel syndrome
D) a severe form of ulcerative colitis
E) salmonellosis
7.936/2. Select One Of The Key Combinations
Which of the following should be done to support this diagnosis:
1) a stool cultivation for amebiasis and dysenteria
2) rectoscopy; biopsy; stool cultivation
3) only irrigoscopy
4) a digital rectal exploration
5) stool cultivation; colonoscopy; biopsy

A) (2) and (5) are true


B) (1), (2), (3), and (4) are true
C) (1), (3), and (5) are true
D) (2), (3), (4), and (5) are true
E) (1), (2), (3), and (5) are true
7.936/3. Single Choice Question
Select the etiology and pathogenesis of this disease:
A) a viral infection
B) a bacterial infection
C) can be an autoimmune disorder
D) unknown
E) an allergy to milk and other food products
7.936/4. Single Choice Question
Select the most characteristic complication of this disease:
A) liver and pulmonary metastases
B) intestinal perforation
C) fistula formation
D) sooner or later ileus will develop
E) since the patient is in a precancerous state, cancer might develop
in the future
7.936/5. Single Choice Question .
Select the prognosis of this disease:
A) uncurable, with a fatal outcome
B) it does not affect the life-style of the patient
C) cancer will develop
D) it can be controlled with conservative therapy
E) secondary infections will markedly reduce the life-span of the
Patient
7.936/6. Single Choice Question
Select the most characteristic form of the disease:
A) continuous deterioration
B) exacerbations and remissions are typical
C) it has a sudden onset with severe toxic state
D) spontaneously heals in several weeks
E) lasts for years, with constant activity
7.936/7. Single Choice Question
Select the most relevant therapy for this disease:
A) diet; spasmolytics
B) surgery
C) surgery; X-ray therapy
D) sulfosalazine; steroids
E) antibacterial and symptomatic therapy
INT-7.937.
Case Study
An 18-year-old female patient complains for about a month of fatigue,
dyspnea after exercise, and swollen limbs. Three days before admission
the patient had an upper respiratory infection. The patient is pale.
A palm-size dullness was detected over the left side of the diaphragm.
Chest X-ray: fluid (a 5-finger sized area) over the left diaphragm. Edema
of both lower limbs. Clear heart sounds, rhythmic contractions. Pleural

puncture: straw-yellow color fluid: 1.5% protein; specific density:


1012. BP: 130/80; sedimentation: 65 mm/h; cholesterol: 6.6 mmol/L;
se-creatinine: 96 mmol/L; urine protein excretion: 2.8 g/24h; se total
protein: 52 g%.
7.937/1. Single Choice Question
The most probable diagnosis is:
A) cardiac insufficiency
B) lymphedema
C) the nephrotic syndrome
D) acute glomerulonephritis
E) diabetic glomerulosclerosis
7.937/2. Select One Of The Key Combinations
Which of the following data support the assumed diagnosis?
1) hypercholesterolemia
2) anuria
3) proteinuria
4) leukemic cells
5) edema
A) (1), (2) and (5) are true
B) (2) and (4) are true
C) (1), (4), and (5) are true
D) (2), (3), and (5) are true
E) (1) and (5) are true
7.937/3. Select One Of The Key Combinations
Which of the following studies should be done to support the diagnosis?
1) ECG
2) a kidney concentrating ability test
3) abdominal lymphography
4) renal ultrasonography
5) renal biopsy
A) (1), (3) and (5) are true
B) (2) and (4) are true
C) (1), (4), and (5) are true
D) (1) and (4) are true
E) (4) and (5) are true
7.937/4. Select One Of The Key Combinations
Select the typical findings of this disease:
1) a decreased glomerular filtration rate
2) sinus arrhythmia
3) metabolic acidosis
4) hypoproteinemia
5) an increased red blood cell sedimentation rate
A) (1), (3), and (5) are true
B) (2) and (4) are true
C) (1), (4), and (5) are true
D) (2), (4), and (5) are true
E) (1), (2), (4), and (5) are true
7.937/5. Select One Of The Key Combinations
Select the correct therapy?

1) cardiac therapy
2) immunosuppressive therapy
3) steroids
4) the patient does not require drug therapy
5) peritoneal dialysis
A) (1), (3), and (5) are true `
B) (2) and (5) are true
C) (2) and (3) are true
D) only (4) is true
E) (1), (2), and (3) are true
7.937/6. Select One Of The Key Combinations
Which of the following parameters should be checked during therapy?
1) the serum transaminase levels
2) urine protein excretion
3) the red blood cell sedimentation rate
4) the total se protein
5) Weber's test (feces)
A) (1), (3), and (4) are true
B) (1), (2), (3), and (4) are true
C) (2), (3), and (5) are true
D) (2), (3), and (4) are true
E) (1), (3), and (5) are true
7.937/7. Single Choice Question
Select the most probable prognosis of this disease:
A) a total cure within two years
B) a progression of the process
C) the development of diabetic retinopathy
D) cor pulmonale
INT-7.839.
Case Study
A 32-year-old obese female patient was admitted due to an epigastric
pain on the right side. After a 5-hour observation the symptoms indicated
appendicitis and appendectomy was subsequently performed.
The postoperative period and the next day were uneventful. When the
patient tried to stand up she complained of pain in the right lower
limb accompanied by a period of dyspnea. After rest however, the symptoms
disappeared. Physical examination: sustained tachycardia. On
the 3rd day the patient had a severe stabbing pain in the chest during
walking with marked dyspnea, cough, and hemorrhagic sputum. The
most important observations in the status were: dyspnea, marked
cyanosis, protruding neck veins, a circumscribed pleural friction sound
above the left diaphragm, subfebrility, several cm difference in the diameter
of the lowe limbs, and hypotension.
X-ray: lung opacity over the left diaphragm. ECG: P pulmonale and
signs of right heart strain. The patient had tuberculosis in the past.
The patient never took any medications other than oral contraceptives
(for three years).
7.939/1. Single Choice Question
The most probable diagnosis is:
A) pneumonia
B) pulmonary tuberculosis

C) superficial thrombophlebitis
D) thrombosis of the deep veins with subsequent pulmonary embolism
E) cardiac insufficiency
7.939/2. Single Choice Question
All of the following studies can verify the diagnosis EXCEPT:
A) a chest X-ray
B) ECG
C) venography
D) laboratory studies (hemostasis, enzymes)
E) a lung-scan
F) lymphography
7.939/3. Single Choice Question
Select the most helpful therapy:
A) combined antibiotic therapy
B) cardiac support
C) low-dose heparin therapy
D) high-dose intravenous heparin therapy
E) diuretics
7.939/4. Single Choice Question
How long should the patient be treated after disappearance of the
acute symptoms?
A) until improvement is noted
B) until the patient is complaint-free
C) for 2-3 weeks after disappearance of the complaints
D) for half a year or a year after disappearance of the complaints
7.939/5. Single Choice Question
Select the drug of choice for prolonged therapy:
A) aspirin (Colfarit) tabl.
B) heparin
C) rutoside (Venoruton)
D) coumarin (Syncumar)
E) antibiotics
7.939/6. Single Choice Question
Select the laboratory studies indicated in controlling this patient's
state:
A) a platelet count
B) the bleeding time
C) the prothrombin time
D) the coagulation time
E) the partial thromboplastin time
7.939/7. Single Choice Question
Which of the following does not contribute to the development of
this disease?
A) immobilization
B) obesity
C) varicosity
D) surgery
E) hypertension
F) oral contraceptives

INT-7.940.
Case Study
A 40-year-old female patient lost 6 kg in 3 months. She complains of
fatigue, vertigo and headaches. In one month the patient had 4 occasions
of orthostatic vertigo and fainted twice. The patient also complains
of a constant abdominal pain. She has lost her appetite, frequently vomits,
and has had diarrhea for several days. The patient observed an
axillary and pubic hair loss and complains of decreased libido despite a normal
menstruation cycle. Six weeks ago she sunbathed for several hours and
got a tan of a surprisingly intensive color; she still has the tan of the
same and even darker color. The patient is dysthymic and excitable.
Physical examination: the patient is adynamic and has increased skin
pigmentation without cyanosis. Blood pressure is 100/70 mmHg (lower
than the usual value mentioned by the patient). Heart and lungs negative; free abdomen; ptosis of the liver (one finger beyond the costal
arch). The liver is smooth, not edgy. The spleen is not palpable. No.
tenderness in the renal region. No edema. Loss of axillary hair.
7.940/ 1. Single Choice Question
The most probable diagnosis is:
A) a prolactin-secreting adenoma of the anterior pituitary
B) hypothyroidism
C) primary adrenal insufficiency
D) the first signs of menopause
E) post-partum pituitary damage (Sheehan's syndrome)
F) anorexia nervosa
G) porphyria cutanea tarda (urocoproporphyria)
7.940/2. Select One Of The Key Combinations
Select the most relevant clinical study for this diagnosis:
1) "Cold-pressor" test
2) the presence or absence of orthostatic hypotension
3) palpation of the lymphatic nodes
4) a simple equilibrium test (Romberg's)
5) examination of the buccal mucosa
6) examination of the palms
A) (1), (2), and (3) are true
B) (2), (5), and (6) are true
C) (3, (4), and (6) are true
D) (1), (3), 5), and (6) are true
E) all of the above
7.940/3. Select One Of The Key Combinations
Select the relevant questions which can support the diagnosis:
1) Did the patient have sarcoidosis?
2) When was thelast pregnancy, if any?
3) Did the patient have antituberculotic therapy?
4) Did the patient have a cranial injury?
5) Did the patient take hormonal contraceptives?
A) (1), (3), and (5) are true
B) (2), (3), and (4) are true
C) (1), (2), (3), and (4) are true
D) (1) and (3) are true
E) all of the above

7.940/4. Select One Of The Key Combinations


Considering the diagnosis, select the most important accompanying
diseases or symptoms:
1) chronic thyroiditis
2) fatty degeneration of the liver
3) polyglobulinemia; an elevated red blood cell count
4) increased susceptibility to vein thrombosis
5) chronic atrophic gastritis; hypoacidity
A) (1), (3), (4), and (5) are true
B) (1) and (5) are true
C) (2), (3), and (5) are true
D) (3), (4), and (5) are true
E) all of the above
7.940/5. Select One Of The Key Combinations
Which of the following should be done:
1) Ask for admittance to an internal ward
2) Define the patient as unfit to work; a control examination
should be done after a 6-8 week rest. The following steps
depend on the state of the patient (improved - deteriorated)
3) Ask for a consultation with a gynecologist
4) Ask for admittance to a special endocrinologic ward
5) Explain to the patient that her problem is not so serious.
Prescribe oxedrine (Sympathomin) drops and appetite improving
therapy - cyproheptadine (Peritol). The patient should
avoid sunlight, because photosensitivity indicates proneness
to autoimmune diseases.
A) (1) and (4) are true
B) (2), (3), and (4) are true
C) (1), (2), and (5) are true
D) (2), (4), and (5) are true
E) all of the above
7.940/6. Select One Of The Key Combinations
If you have a chance to perform certain simple and quick tests,
which of the following would confirm your diagnosis?
1) an X-ray of the sella turcica
2) the blood glucose level
3) the red blood cell sedimentation rate
4) a cholesterol and triglycerin test (information on the thyroid
function)
5) determination of the Se Na+, K-, and Cl- levels
6) an ECG
A) (1), (4), and (6) are true
B) (2), (3), and (5) are true
C) (2) and (5) are true
D) (1), (2), (3), and (4) are true
E) all of the above
INT-7.941.
Case Study
A 67-year-old female patient complains of arthralgia which started
about 3 months ago. She has swollen wrists, spindle-like

interphalangeal joints, swollen knees (first left, then right) and ankles.
In the morning she complains of joint rigidity. The patient has no appetite,
fever, and has lost 10 kg. She coughs, and the sputum is sometimes
hemorrhagic. Chest X-ray: on the left side a round-shaped opacity
with blurred edges and hypodense area in the middle is observed.
7.941 / 1. Select One Of The Key Combinations
Select the correct diagnosis:
1) uric arthritis
2) rheumatoid arthritis
3) Caplan's-syndrome (pneumoconiosis)
4) bronchial cancer
5) paraneoplastic syndrome
A) (1), (3), and (5) are true
B) (4), and (5) are true
C) (1), (3), and (4) are true
D) (2) and (5) are true
E) all of the above
7.941/2. Select One Of The Key Combinations
Which of the following methods support the diagnosis:
1) the red blood cell sedimentation rate
2) the complement level
3) Waaler-Rose's latex test
4) determination of the antinuclear factor
5) bronchoscopy-cytology-biopsy
6) the serum uric acid level
A) (1), (3), and (4) are true
B) (2), (4), (5), and (6) are true
C) (1), (3), (5), and (6) are true
D) (3), (4), and (5) are true
E) all of the above
7.941/3.
Select the adequate primary therapy:
A) steroids
B) non-steroid antiinflammatory agents
C) cytostatics in an immunosuppressive dose
D) combined cytostatic therapy in tumor dose
E) surgery
INT-7.942.
Case Study
A 58-year-old male patient complains for 3-4 weeks of heartburn, belching,
a burning epigastric pain relieved by sodium bicarbonate or milk;
the pain never radiates elsewhere. The patient has no fever. He has
lost his appetite and 3 kg. He has stool only every second day. The
patient is normacidic. Gastric X-ray: filling excess is observed at the
lesser curvature and irregular folds in the adjacent area.
Laboratory findings: We: 26 mm/h; WBC: 8.4 G/L; Hb: 8.81 mmol/L; Ht: 0.40.
7.942/1. Single Choice Question
Which of the following steps would you recommend:
A) immediate admission to an internal ward, because the disease
requires hospitalization
B) immediate admission to a surgical ward, because the patient

must soon undergo immediate surgery


C) prescribe an immediate diet for patients with ulcer, as well as a
therapy aimed at neutralization and decrease of acid production
D) outpatient gastrofiberoscopy, because the clinical signs and
the X-ray findings suggest the possibility of a ventricular
malignant ulcer. Until the histological data are ready, prescribe
a diet for patients with ulcer, as well as a therapy aimed at
neutralization and decrease of acid production
E) same as in (C); 4-5 weeks later repeat control X-ray study. If
the ulcer is healed the therapy can be terminated. If no healing
tendency is found perform a gastrofiberoscopy and/or refer the
patient to a surgical ward.
7.942/2. Single Choice Question
Select the adequate diet if the patient is treated conservatively:
A) hunger diet: tea or milk with 1-2 cakes in the morning and
evening; some unseasoned, lean soup and mashed potatoes
B) the patient should avoid alcohol consumption, seasoned
meals, and meat, and should eat three times a day
C) the patient should avoid seasoned, fatty, smoked and other
heavy meals, alcohol, and coffee. Otherwise the diet should
guarantee a normal daily energy requirement, and adequate
vitamin, and fat consumption. Between the three main meals
recommend to the patient to have milk and dry cakes.
D) avoid seasoned food and alcohol, restrict the amount of consumed
food; the diet should be low in fiber and fat. Recommend
to the patient to eat every hour
E) let the patient keep his own diet (it is the recent trend in
dietology); recommend him to avoid food which causes him
complaints and to consume everything that does not cause
him symptoms of any kind.
INT-7.943.
Case Study
A 27-year-old female patient was urgently admitted due to complaints
of dyspnea and pleuritic chest pain. Four days before the patient observed
swelling and pressure tenderness in the right leg. Clinical signs
suggest deep vein thrombosis which might have caused pulmonary
embolism.
7.943/ 1. Select One Of The Key Combinations
Select data in the history which support the diagnosis of deep vein
thrombosis:
1) oral contraceptives
2) long-term immobilization of the lower limbs
3) injury of the lower extremity
4) hypertension
A) (1), (2), and (3) are true
B) (1) and (3) are true
C) (2) and (4) are true
D) only (4) is true
E) all of the above
7.943/2. Select One Of The Key Combinations
Which of the following methods help in the verification of deep vein
thrombosis:

1) Doppler's ultrasonography
2) contrast venography
3) impedance plethysmography
4) nuclear scanning with 125-iodine labelled fibrinogen
A) (1), (2), and (3) are true
B) (1) and (3) are true
C) (2) and (4) are true
D) only (4) is true
E) all of the above
INT-7.944.
Case Study
A 65-year-old female patient complains of progressive abdominal distension.
lasting for two weeks. In a period of a half-a-year the patient
lost 5.5 kg, but regained 3 kg during the past two weeks.
Physical examination: a thin woman with visible ascites and pale mucosa. Ht: 0.30; WBC:
9.8 g/L; left inguinal lymph nodes are enlarged but not tender.
7.944/ 1. Single Choice Question
Select the most relevant method of examination:
A) a liver biopsy
B) a lymph node biopsy
C) a diagnostic ascites puncture
D) intravenous pyelography
E) an X-ray of the upper part of the GI tract
7.944/2. Multiple Choice Question
Which of the following can cause a milk-like ascites?
A) pus
B) chylus
C) trauma
D) all of the above
E) none of the above
7.944/3. Multiple Choice Question
Chylus ascites was found. Select the most probable diagnosis:
A) lymphoma
B) pancreatic carcinoma
C) syphilis
D) tuberculosis
E) none of the above
7.944/4. Single Choice Question
Which of the following methods help in establishing the final diagnosis:
A) gastrointestinal studies
B) a lymph node biopsy
C) a rectal biopsy
D) intravenous pyelography
E) arteriography
7.944/5. Single Choice Question
The most probable diagnosis is:
A) colon carcinoma
B) lymphoma
C) syphilis
D) gastric carcinoma

E) pneumococcal peritonitis
INT-7.945.
Case Study
A 47-year-old female patient presented with a complaint of arthralgia
which started about 6 months ago. The joints of her fingers are red
and swollen; in the morning she complains of joint rigidity in the fingers,
toes, wrists and knees. The pain decreases after an hour. Physical
examination: red, swollen interphalangeal proximal joints and
multiple small nodes on the extensor surface of the elbow.
7.945/1. Single Choice Question
The most probable diagnosis is:
A) osteoarthritis
B) gout
C) scleroderma
D) psoriatic arthritis
E) rheumatoid arthritis
7.945/2. Single Choice Question
All of the following parameters are pathologically altered, EXCEPT.
A) bone marrow iron stores
B) red blood cell sedimentation rate
C) latex agglutination test
D) immunoglobulin levels
E) X-ray of the hands
INT-7.946.
Case Study
A 60-year-old female patient complains of progressive weakness in
both hindlimbs which started three days ago. Several hours before the
examination the patient became incontinent. Physical examination:
bilateral plantar reflex (Babinski positive), bilateral weakness in the
legs and decreased sensory function up to the middle of the abdomen.
7.946/ 1. Single Choice Question
Select the most important urgent study:
A) myelography
B) CT study of the brain
C) cystoscopy
D) CT scan of the whole body
E) EEG
7.946/2.
All of the following diagnoses are possible, EXCEPT:
A) cerebral tumor
B) spinal meningeoma
C) multiple sclerosis
D) transverse myelitis
E) spinal epidural abscess
ANSWER KEY (INT)
1.T
101.2.F
102.3.T
103.A
4.T
104.5.F
105.-

201.E
202.E
203.E
204.D
205.-

301.D
302.B
303.A
304.E
305.B

401.E
402.C
403.B
404.B
405.B

501.C
502.503.504.505.D

601.ABCDE
602.ACDE
603.ABCE
604.BD
605.BCE

701.ABCDE
702.DE
703.CE
704.ABD
705.BE

801.D
802.B
803.E
804.E
805.A

6.T
7.T
8.9.10.11.F
12.C
13.14.15.F
16.B
17.E
18.E
19.E
20.E
21.E
22.23.E
24.F
25.E
26.C
27.A
28.E
29.E
30.C
31.C
32.D
33.A
34.F
35.E
36.C
37.E
38.E
39.E
40.D
41.F
42.C
43.E
44.B
45.D
46.D
47.E
48.E
49.E
50.E
51.52.53.D
54.C

106.D
107.108.D
109.110.E
111.A
112.B
113.C
114.D
115.D
116.D
117.D
118.C
119.A
120.A
121.C
122.B
123.C
124.E
125.A
126.A
127.B
128.C
129.A
130.C
131.E
132.E
133.A
134.D
135.B
136.E
137.A
138.D
139.D
140.C
141.A
142.E
143.C
144.B
145.D
146.E
147.C
148.B
149.A
150.A
151.A
152.E
153.154.E

206.B
207.C
208.209.E
210.C
211.A
212.213.214.D
215.E
216.E
217.A
218.C
219.D
220.C
221.B
222.C
223.D
224.C
225.D
226.A
227.C
228.A
229.A
230.B
231.D
232.C
233.D
234.E
235.A
236.A
237.C
238.B
239.B
240.E
241.D
242.C
243.D
244.C
245.A
246.C
247.C
248.C
249.E
250.C
251.B
252.C
253.B
254.B

306.A
307.A
308.D
309.B
310.C
311.C
312.C
313.B
314.B
315.C
316.B
317.B
318.A
319.B
320.C
321.E
322.E
323.E
324.E
325.E
326.D
327.E
328.D
329.D
330.D
331.B
332.D
333.A
334.B
335.B
336.E
337.E
338.D
339.B
340.B
341.A
342.B
343.C
344.C
345.D
346.D
347.D
348.E
349.B
350.B
351.352.B
353.B
354.D

406.407.C
408.409.A
410.B
411.D
412.D
413.B
414.B
415.A
416.A
417.A
418.E
419.B
420.D
421.D
422.A
423.B
424.D
425.C
426.B
427.A
428.C
429.D
430.D
431.E
432.A
433.C
434.C
435.D
436.D
437.D
438.C
439.D
440.E
441.B
442.B
443.E
444.A
445.B
446.C
447.C
448.B
449.B
450.E
451.E
452.E
453.C
454.C

506.507.508.509.D
510.511.512.D
513.B
514.C
515.D
516.C
517.C
518.D
519.C
520.D
521.D
522.E
523.E
524.D
525.A
526.B
527.B
528.E
529.A
530.D
531.A
532.E
533.A
534.D
535.B
536.A
537.C
538.C
539.C
540.C
541.A
542.D
543.C
544.C
545.D
546.BCD
547.548.BC
549.BCD
550.551.BD
552.553.ABC
554.ACD

606.ACE
607.ABE
608.AC
609.AD
610.BD
611.BCDE
612.ABCE
613.BC
614.ADE
615.AB
616.BCD
617.BCD
618.BE
619.ABCDE
620.ABCDE
621.ACD
622.BCD
623.ABCDE
624.ACD
625.ABC
626.CDE
627.CD
628.BCDE
629.ABCD
630.CDE
631.ABD
632.AB
633.ABDE
634.ABCDE
635.CD
636.ABCD
637.ABCDE
638.ABCD
639.ABCD
640.ABC
641.AC
642.AB
643.AB
644.ABCDE
645.BD
646.AB
647.ABD
648.ABCDE
649.ABCDE
650.ABCE
651.BD
652.BD
653.654.BD

706.CDE
707.ABDE
708.ABCE
709.ABCE
710.BD
711.DE
712.AB
713.BDE
714.AE
715.E
716.717.E
718.719.E
720.E
721.D
722.A
723.A
724.725.B
726.B
727.728.E
729.A
730.E
731.A
732.D
733.D
734.E
735.736.A
737.B
738.739.740.741.E
742.D
743.744.745.C
746.747.D
748.D
749.A
750.E
751.B
752.B
753.D
754.B

806.C
807.B
808.D
809.810.B
811.A
812.B
813.F
814.815.A
816.F
817.G
818.G
819.E
820.A
821.B
822.B
823.C
824.C
825.A
826.C
827.D
828.B
829.G
830.B
831.A
832.I
833.H
834.E
835.A
836.E
837.A
838.B
839.CBAD
840.BACDFE
841.842.ABC
843.BAC
844.845.846.ACDADB
847.ACBDED
848.EDCBA
849.BACD
850.CCDCDA
851.CBBAAD
852.853.DCBEA
854.ABDBC

55.B
56.A
57.58.59.60.61.C
62.A
63.B
64.65.E
66.B
67.A
68.D
69.B
70.E
71.C
72.A
73.A
74.C
75.B
76.A
77.A
78.C
79.B.
80.E
81.D
82.B
83.B
84.C
85.E
86.D
87.A
88.A
89.A
90.C
91.C
92.D
93.A
94.D
95.A
96.B
97.D
98.A
99.E
100.B

155.156.157.D
158.159.E
160.161.A
162.D
163.C
164.E
165.B
166.A
167.A
168.C
169.E
170.B
171.D
172.D
173.C
174.E
175.A
176.B
177.B
178.B
179.A
180.D
181.B
182.A
183.D
184.C
185.D
186.E
187.C
188.C
189.C
190.D
191.A
192.D
193.B
194.A
195.D
196.E
197.B
198.C
199.D
200.E

255.C
256.D
257.A
258.A
259.D
260.D
261.C
262.C
263.E
264.B
265.B
266.A
267.C
268.C
269.F
270.D
271.D
272.C
273.C
274.B
275.B
276.A
277.D
278.A
279.B
280.E
281.B
282.C
283.A
284.B
285.B
286.C
287.C
288.D
289.D
290.B
291.C
292.D
293.C
294.E
295.E
296.E
297.B
298.E
299.C
300.C

355.E
356.E
357.B
358.D
359.C
360.361.B
362.A
363.C
364.B
365.B
366.C
367.D
368.C
369.E
370.B
371.A
372.C
373.D
374.C
375.C
376.B
377.E
378.C
379.E
380.F
381.C
382.A
383.B
384.D
385.E
386.B
387.C
388.B
389.B
390.C
391.C
392.C
393.E
394.E
395.A
396.A
397.C
398.D
399.A
400.B

455.A
456.457.458.459.460.461.D
462.B
463.B
464.E
465.E
466.C
467.C
468.B
469.E
470.E
471.C
472.D
473.B
474.B
475.D
476.E
477.C
478.A
479.D
480.B
481.E
482.C
483.D
484.A
485.A
486.A
487.A
488.C
489.B
490.C
491.B
492.D
493.D
494.A
495.E
496.A
497.D
498.B
499.A
500.E

901.B
902.C

911.B
912.C

921.DBAC
922.AEEB

931.CBDCCC
932.BDCEE

941.BEE
942.DC

555.AD
556.ACDE
557.DE
558.ABE
559.BDE
560.BCE
561.ABDE
562.ACE
563.BDE
564.DE
565.ADE
566.ABD
567.ADE
568.CDE
569.BC
570.CDE
571.ABE
572.ABE
573.BD
574.AB
575.CD
576.ABCE
577.ACD
578.ACE
579.BDE
580.AE
581.BD
582.ABE
583.BCE
584.BD
585.CE
586.BDE
587.CD
588.BCDE
589.BDE
590.ABD
591.ABCDE
592.AE
593.AC
594.BCE
595.ABD
596.BCD
597.BCDE
598.BC
599.AD
600.BDE

655.656.ABCD
657.BCDE
658.ACE
659.ADE
660.661.662.ABCDE
663.ABCD
664.BCE
665.ABDE
666.BCE
667.BE
668.ABE
669.ABC
670.BCD
671.AD
672.CD
673.DE
674.BD
675.ABDE
676.ACDE
677.ABCDE
678.ABE
679.CDE
680.ABCDE
681.CD
682.ABC
683.ACDE
684.BCD
685.BD
686.DE
687.BCD
688.ACDE
689.ABDE
690.BDE
691.ABCD
692.ACE
693.ADE
694.ABDE
695.ABCE
696.AD
697.ABDE
698.ABCE
699.ABE
700.ACD

755.D
756.C
757.B
758.E
759.B
760.D
761.A
762.D
763.764.B
765.C
766.A
767.A
768.B
769.C
770.B
771.C
772.D
773.B
774.A
775.C
776.B
777.E
778.E
779.D
780.E
781.C
782.A
783.D
784.E
785.C
786.C
787.A
788.D
789.C
790.B
791.D
792.A
793.E
794.B
795.E
796.E
797.D
798.D
799.B
800.A

855.DBAEC
856.DABCE
857.BBCBD
858.DAECB
859.DAECB
860.ADBAC
861.CFDBAE
862.CBEDDA
863.DBAACA
864.CDCAB
865.BAC
866.ABABCD
867.BBAACD
868.DABCD
869.CAB
870.BBACC
871.AAB
872.CBEAD
873.AABCC
874.D
875.D
876.A
877.A
878.E
879.C
880.A
881.882.D
883.D
884.D
885.A
886.D
887.E
888.C
889.B
890.B
891.E
892.B
893.C
894.A
895.E
896.A
897.A
898.D
899.B
900.A

903.A
904.A
905.E
906.A
907.C
908.C
909.A
910.A

913.B
914.A
915.B
916.B
917.918.CECC
919.DCCCC
920.EBAB

923.BDDED
924.CCACB
925.CABA
926.CBC
927.928.CC
929.CBF
930.CCA

933.BCA
934.CAA
935.936.DACEDBB
937.CAECCDA
938.939.DFDDDCE
940.CBDBAC

943.AE
944.CAB
ABCDBB
945.AE
946.AA

SURGERY (SUR-8)
SINGLE CHOICE QUESTIONS
Select the single best response to each of the following questions!!!
SUR-8.1.
The most appropriate method for maintaining patent airways in a
patient with multiple trauma resulting from a car accident due to
drunken driving is:
A) oropharyngeal intubation
B) endotracheal intubation
C) thrusting the jaw and lifting the chin
D) intubation with a cuffed endotracheal tube
E) tracheostomy
SUR-8.5.
Which of the following is the most common site of aortic injury in
the thorax?
A) at the origin of the aortic arch
B) just above the origin of the innominate artery
C) at the fusion ofthe thoracal and abdominal segments of the aorta
D) just distal to the ligamentum arteriosum
E) between the origin of the left common carotid artery and that
of the left subclavian artery
SUR-8.6.
The most appropriate therapy for clean, fresh lacerations of peripheral
nerves is
A) debridement of the wound only
B) the administration of antibiotics only
C) immediate approximation of separated nerve endings
D) delayed suture
E) immobilization of the limb only
SUR-8.7.
The triad of abdominal trauma followed by gastrointestinal bleeding
and the development of colicky abdominal pain indicates:
A) a duodenal perforation
B) rupture of the pancreas complicated by pancreatitis
C) hemobilia
D) a gall-bladder rupture
E) the development of a stress-ulcer
SUR-8.8.
Which of the following drugs could cause acute adrenal insufficiency
in the postoperative period?

A) vitamin C
B) aspirin
C) heparin
D) meperidine / pethidine (Dolargan)
E) cephalothin (Keflin)
SUR-8.9.
The energy source for the brain during protracted fasting in the period
following severe trauma is:
A) glucose
B) amino acids
C) lipids
D) ketones
E) glycerol
SUR-8.10.
Hemorrhagic diathesis developing after multiple transfusions is predominantly
the consequence of:
A) thrombocytopenia
B) a reduced fibrinogen level
C) a reduced prothrombin level
D) an increased fibrinolytic activity
E) a reduced serum calcium level
SUR-8.14.
When treating fractures:
A) immobilization of adjacent joints is unnecessary
B) adjacent joints are occasionally immobilized
C) immobilization of the joints results in the development of contractures
D) only the proximal joint is immobilized
E) both the joint proximal and the one distal to the fracture site
should be immobilized
SUR-8.15.
Case Study:
Two hours following the application of a plaster splint for a
supracondylar fracture, of the radius, a patient returns to the clinic
complaining of severe pain in his injured hand. The fingers are
swollen and cyanotic on examination. The most appropriate solution
would be to:
A) monitor the patient
B) administer vasodilators
C) administer analgesics
D) cut and loosen the splint at the fingers
E) cut the splint in its whole length immediately
SUR-8.16.
All of the following statements are valid regarding the process of
fracture healing, EXCEPT:
A) there is a hematoma at the fracture site initially
B) aseptic inflammation develops
C) the acidic (low pH) local environment promotes bone
resorption at the fractured bone ends
D) fibroblasts enter the hematoma and fibrous tissue develops at
the fracture site
E) the fibrous tissue is remodeled directly into bone tissue

SUR-8.17.
Clavicular fractures of children under 10 years of age should be
treated as follows:
A) no therapy is necessary in most cases
B) reduction and the application of a "figure-of-eight" splint
dressing
C) open reduction is appropriate
D) the medial third of the clavicle should be removed
E) open fixation by a medullary rod is necessary
SUR-8.18.
Numbness and paresthesias developing in the region between the
index finger and thumb in a patient having sustained a fracture of
the radius at the typical site (Colles' fracture) indicates:
A) displacement of fractured bone ends
B) excessive callus formation
C) compression of the median nerve
D) compression of the radial nerve
E) compression of the ulnar nerve
SUR-8.19.
The principle of treatment for radial and ulnar fractures:
A) the angulation of the ulna must be reduced
B) the angulation of the radius must be reduced
C) the overriding of fractured ends must be reduced
D) the proper relationship of the radius and the ulna must be restored
E) only the elbow should be immobilized
SUR-8.20.
The displacement occurring in Monteggia's fracture is characterized
by the:
A) angulation of the ulna
B) angluation of both bones of the forearm
C) angulation of the radius and the luxation of the ulna
D) the luxation of the head of the radius
E) angulation of the ulna and the luxation of the head of the radius
SUR-8.21.
The most helpful information in the diagnosis of acute osteomyelitis
of 1 week duration in infants is provided by:
A) a bone x-ray
B) a hemoculture
C) a radiologic examination of the adjacent joint
D) local tenderness of the bone
E) general systemic dysfunction
SUR-8.22.
Falling on the soles from a considerable height most often results in a:
A) medial malleolar fracture
B) fracture of the talus
C) lateral malleolar fracture
D) scaphoid fracture
E) fracture of the calcaneus
SUR-8.23.

A non-impacted fracture of the femoral neck causes a characteristic


deformity of the lower extremity, that is:
A) shortening and external rotation
B) shortening only
C) external rotation only
D) flexion, adduction and internal rotation
E) flexion, adduction and external rotation
SUR-8.24.
All of the following statements are valid regarding intertrochanteric
fractures of the femur, EXCEPT:
A) this fracture is sustained by elderly people
B) open reduction and internal fixation is required
C) immobilization for 3-4 months is necessary
D) the ends of the fragments can be either displaced or properly
aligned
E) remodeling of the callus to bone tissue is often absent
SUR-8.25.
What is the usual position of the lower extremity after habitual
luxation of the hip?
A) the hip is in flexion and adduction
B) the hip and the knee are both extended
C) the hip is flexed, the knee is extended
D) in the neutral position
E) the hip is rotated externally
SUR-8.26.
The therapy of osteogenic sarcoma is:
A) irradiation
B) administration of antimetabolites
C) amputation
D) curettage
E) excision
SUR-8.27.
Case Study:
A 20-year-old football player presents with a "locked up" knee after a
torsion injury to his lower extremity. The most likely diagnosis is:
A) avulsion of the tibial tuberosity
B) fracture of the distal portion of the femur
C) meniscus injury
D) disruption of the tibial collateral ligament
E) disruption of the anterior cruciate ligament
SUR-8.28.
What is the pathomechanism of the most common type of luxation of
the elbow, i.e. posterior luxation?
A) an abrupt lifting of a heavy object
B) a mighty blow on the humerus
C) knocking the elbow against the top of the table
D) falling on extended arms
E) elevation of the arm above the head
SUR-8.29.
The most severe complication of an open fracture is usually:

A) hemorrhage
B) limb shortening
C) infection
D) comminuted fracture
E) muscle contracture
SUR-8.30.
All of the following conditions are associated with parathyroid
hyperfunction, EXCEPT:
A) brown tumours
B) osteitis fibrosa cystica generalisata
C) general demineralization
D) cystic lesions in the bones
E) sclerosis

SUR-8.31.
Genu valgum can develop in all the following conditions, EXCEPT:
A) vitamin D deficiency
B) Fanconi's syndrome
C) central paresis
D) rheumatoid arthritis
E) postmenopausal osteoporosis
SUR-8.32.
Dupuytren's contracture is associated with all the following conditions,
EXCEPT:
A) familial trait
B) epilepsy
C) alcoholism
D) chronic pulmonary tuberculosis
E) excessive manual activity
SUR-8.33.
All of the following conditions are characteristic of lumbar disc rupture,
EXCEPT:
A) urgency or pollakiuria (frequency)
B) weakness of the toe on extension
C) decreased/ sluggish Achilles' reflex
D) sole (plantar extensor) reflex
E) Lasgue's sign
SUR-8.34.
The primary source of metastatic spread of malignancies affecting
the spine is most often:
A) the kidney
B) the prostate
C) the breast
D) the lung
E) some other bone
SUR-8.35.
All of the following statements about cervical spondylosis are true,
EXCEPT:
A) it has a predilection for cervical segments C4-C6
B) bulging is more frequently lateral than medial

C) extension /traction of the neck aggravates pain


D) an insufficient blood supply is the most likely cause of spinal
cord damage
E) when signs are bilateral, radiography shows anterior fusion of
cervical vertebrae
SUR-8.36.
All of the following conditions belong to bone development disorders
due to altered cartilage proliferation and calcification, EXCEPT:
A) achondroplasia
B) osteogenesis imperfecta
C) metaphyseal aclasis
D) polyostotic fibrous dysplasia
E) dyschondroplasia
SUR-8.37.
All of the following features are characteristic of achondroplasia,
EXCEPT:
A) 'trident' (3 fingered) hand
B) mental disorders
C) ball shaped/globular skull
D) dwarfism
E) prognathism
SUR-8.38.
A characteristic feature of bone disorders associated with scurvy is:
A) abnormal cartilage growth
B) a lack of cartilage calcification
C) a bone remodeling defect
D) an insufficient production of intercellular matrix
E) the production of defective osteoid
SUR-8.39.
All of the following features are characteristic of bone pathology in
rickets, EXCEPT:
A) a widened epiphyseal zone
B) a wide and abnormal metaphysis
C) blood vessel fragility
D) cartilage proliferation
E) deformed costochondral syndesmoses ('rickety rosary')
SUR-8.40.
Which form of osteosarcoma carries the worst prognosis?
A) parosteal sarcoma
B) the radiant- spicular type developing in puberty
C) sarcoma associated with Paget's disease
D) the sclerosing variant of osteosarcoma
E) the metaphyseal osteolytic form
SUR-8.41.
Pyogenic arthritis is common in all the following conditions, EXCEPT:
A) in patients with debilitating disease
B) in patients treated with cortisol
C) in rheumatoid arthritis
D) in youngsters pursuing sports actively
E) in premature infants

SUR-8.42.
Osteoarticular tuberculosis frequently involves the:
A) vertebrae
B) hip
C) knee
D) shoulder
E) wrist
SUR-8.43.
The most frequent cause of lung a abscess is:
A) the aspiration of infective material from the oral cavity or pharynx
B) a blood-borne infection
C) lymphatic spread from an infective focus
D) a penetrating chest injury
E) bronchogenic carcinoma
SUR-8.44.
A lung abscess develops most frequently in the:
A) left-upper lobe
B) left-lower lobe
C) lingular segment
D) upper-dorsal segments of the right lung
E) right-middle lobe
SUR-8.45.
The optimal treatment of uncomplicated rib fractures is:
A) immobilization
B) emergency surgery and fixation of fragments with wire sutures
C) external fixation with adhesive tape
D) the perifocal infiltration of a local anesthetic around the injured
intercostal nerve
E) to allow the fractured ribs to heal naturally
SUR-8.46.
Case Study:
The chest x-ray of a 30-year-old patient with a simple rib fracture
shows a 50% pneumothorax. The treatment of choice is:
A) external fixation with adhesive tape
B) the infiltration of the injured intercostal nerve with a local
anesthetic
C) coughing and respiratory exercises
D) the administration of analgesics
E) tube thoracostomy and underwater (Bulow) drainage
SUR-8.47.
hen the lung is covered by a rigid, fibrinopurulent crust, the most
appropriate therapy is:
A) decortication
B) pneumonectomy
C) rib resection
D) antibiotic therapy
E) the injection of streptokinase into the pleural cavity
SUR-8.48.
A solitary mass in the lung of a middle-aged patient is most likely a:

A) granuloma
B) malignancy
C) adenoma
D) tuberculotic lesion
E) secondary neoplasm
SUR-8.49.
The treatment of choice for hemothorax with an effusion of 500 cm3
or more is by:
A) needle aspiration
B) intercostal tube thoracostomy
C) thoracotomy and ligation of the ruptured blood vessel
D) supportive therapy with monitoring
E) transfusion of fresh blood
SUR-8.50.
The most frequent cause of thoracic empyema is:
A) pneumonia of the ipsilateral lung
B) the rupture of an emphysematous bleb
C) a penetrating chest injury
D) a subphrenic abscess
E) ruptures of the thoracic segments of the esophagus
SUR-8.51.
The most useful procedure for the diagnosis of bronchiectasis is:
A) a chest x-ray
B) pulmonary angiography
C) bronchography
D) a pulmonary CT-scan
E) an arterial blood-gases analysis
SUR-8.52.
All of the following represent an indication for resection in pulmonary
tuberculosis, EXCEPT:
A) an open cavity with positive sputum despite 3-6 months of
chemotherapy
B) a negative sputum with destroyed segment residue
C) a local infection caused by atypical acid-fast bacteria
D) a new lesion with positive sputum
E) tuberculotic bronchiectasis of the middle and lower lobes
SUR 8.54.
All of the following statements are valid regarding lung cancer, EXCEPT:
A) lung cancer is never symptom-free
B) on roentgenograms, it can occur as a coin lesion (round
shadow) on the periphery of the lung
C) hemoptysis is common in lung cancer
D) a dry, distressing, unproductive cough is often the only symptom
of lung cancer
E) a partial or complete airway obstruction may predispose to
lung infection
SUR-8.55.
All of the following are manifestations of the superior vena cava
syndrome,
EXCEPT:

A) increased venous pressure


B) edema of the head and neck
C) enlarged veins which are visible on the anterior chest wall
D) cyanosis
E) dyspnea
SUR-8.56.
All of the following are diagnostic signs of cardiac tamponade,
EXCEPT:
A) increased venous pressure
B) shock
C) a reduced cardiac output
D) increased filling volume of the heart during diastole
E) a reduced blood pressure
SUR-8.57.
An important sign of chronic constrictive pericarditis visible on a
chest roentgenogram is:
A) a diminished heart size
B) displacement of the right main bronchus
C) a prominent hilar pattern in the lung
D) pericardiac calcification
E) an elevation of the diaphragm
SUR-8.58.
The main contraindication to surgery in patent ductus arteriosus is:
A) a left-to-right shunt
B) shunting of blood from the aorta to the pulmonary artery
C) shunting of blood from the pulmonary artery to the aorta
D) cardiomegaly
E) hilar pulsation on fluoroscopy
SUR-8.59.
The typical sign of coarctation of the aorta visible on a chest
roentgenogram is:
A) a notching of the necks of the 3rd-9th ribs
B) a notchingof the neck of the first rib
C) an eroded sternum
D) cardiomegaly
E) thoracic hemivertebra
SUR-8.60.
All of the following features are characteristic of Tetralogy of Fallot,
EXCEPT:
A) growth retardation
B) long, slender forgers
C) a bruit in the precordial region
D) a rough systolic rumble along the left sternal margin
E) visible cyanosis of the nailbeds
SUR-8.61.
All of the following can be detected on a roentgenogram of a patient
with Tetralogy of Fallot, EXCEPT:
A) a prominent vascular pattern all over the lung
B) a "boot-shaped" heart
C) decreased pulsation of the pulmonary artery
D) an enlarged right ventricle

E) narrow shadows of the large arteries in the upper region of the


mediastinum
SUR-8.62.
Which patients with Tetralogy of Fallot are candidates for surgical
correction?
A) all patients
B) patients with right ventricle hypertrophy
C) patients with dominating pulmonary stenosis
D) patients without cyanosis
E) patients over 15 years of age
SUR-8.63.
If a neonate with Tetralogy of Fallot is not cyanotic at birth, then:
A) the diagnosis is incorrect
B) the patient has polycythemia vera
C) the patient has anemia
D) the patient has a patent ductus arteriosus
E) right ventricular function is deteriorating
SUR-8.64.
The 5-year survival of all lung cancer cases is not higher than:
A) 10%
B) 20%
C) 30%
D) 40%
E) 50%
SUR-8.65.
The most common congenital heart abnormality is:
A) a ventricular septal defect
B) an atrial septal defect
C) a patent ductus arteriosus
D) transposition of the great vessels
E) Tetralogy of Fallot
SUR-8.66.
A small volume/easily compressible radial pulse is characteristic of
A) aortic valve stenosis
B) coarctation of the aorta
C) a patent ductus arteriosus
D) stenosis of the mitral valve
E) Tetralogy of Fallot
SUR-8.67.
All of the following are associated with the formation of aneurysms
of the descending aorta, EXCEPT:
A) Marfan's syndrome
B) injury
C) syphilis
D) atherosclerosis
E) cystic median necrosis (Erdheim's cystic median necrosis)
SUR-8.68.
The most common and significant sequel of an aneurysm of the descending
aorta is:

A) rupture
B) aortic insufficiency
C) sternal erosion
D) compression of the trachea
E) compression of the vena cava
SUR-8.69.
What is the most common cause of dissecting aneurysms of the thoracic
aorta?
A) atherosclerosis
B) syphilis
C) degeneration of the tunica media
D) injury
E) coarctation of the aorta
SUR-8.70.
The most common malignancy of the bony chest wall is:
A) solitary bone metastasis
B) osteogenic sarcoma
C) multiple myeloma
D) Ewing's sarcoma
E) chondrosarcoma
SUR-8.71.
A possible etiologic factor in the development of pleural
mesothelioma is:
A) pneumoconiosis
B) asbestosis
C) anthracosis
D) hereditary disposition
E) peritoneal mesothelioma
SUR-8.72.
Hemoptysis can result from all of the following, EXCEPT:
A) bronchial carcinoma
B) bronchial asthma
C) bronchiectasia
D) pulmonary infarction
E) pulmonary tuberculosis
SUR-8.73.
Which of the following cardiac disorders causes hemoptysis:
A) mitral valve stenosis
B) tricuspid valve incompetency
C) Tetralogy of Fallot
D) aortic stenosis
E) anomaly of the pulmonary veins
SUR-8.75.
All of the following are consequences of mitral valve stenosis,
EXCEPT:
A) a reduced cardiac output
B) atrial fibrillation
C) embolism in the systemic circulation
D) increased peripheral vascular resistance
E) pulmonary edema

SUR-8.76.
All of the following are radiologic signs of mural valve stenosis,
EXCEPT..
A) a calcified mitral valve
B) an enlarged heart
C) straightening of the left margin of the cardiac silhoulette
D) the displacement of the esophagus on lateral films
E) a double-contour at the right parasternal border
SUR 8.77.
What percent of the small intestine can be removed without severe
consequences on digestive capacity and the subsequent risk of
metabolic disorders?
A) 5%
B) 10%
C) 30%
D) 70%
E) 90%
SUR-8.78.
All of the following statements are valid regarding gastric polyps,
EXCEPT.
A) gastric polyps have a propensity for multiple occurrence
B) gastric polyps develop on atrophic gastric mucosa
C) gastric polyps may develop first after puberty
D) gastric polyps can develop as a feature of familial disease
E) gastric polyps are not expected to undergo malignant transformation
SUR-8.79.
Intestinal diverticula develop most frequently in the:
A) duodenum
B) jejunum
C) transverse colon
D) descending colon
E) sigmoid colon
SUR 8.80.
All of the following signs are present in mesenteric artery obstruction,
EXCEPT:
A) a sudden pain around the umbilicus
B) that the pain is disproportionally intense compared to the severity
of physical signs
C) an urge to defecate
D) bloody stool
E) bowel sounds reflecting permanently hyperactive peristalsis
SUR-8.81.
All of the following are true for the pain associated with
pancreatitis,
EXCEPT:
A) an abrupt onset
B) diffuse epigastric pain
C) unremitting (generalized) pain
D) that the pain radiates through to the back
E) that it is not associated with nausea or vomiting

SUR-8.82.
The single most important sign of appendicitis is:
A) vomiting
B) an elevated body temperature
C) leukocytosis
D) tenderness in the right lower abdominal quadrant
E) hyperesthesia in the right lower abdominal quadrant
SUR-8.83.
Chronic respiratory insufficiency predisposes the patient to:
A) chronic cholecystitis
B) diverticulitis
C) a duodenal ulcer
D) a hiatal hernia
E) pancreatitis
SUR-8.84.
Stress ulcers have been observed in all of the following conditions,
EXCEPT:
A) after severe burns
B) in central nervous system lesions
C) after alcohol ingestion
D) after salicylate ingestion
E) during penicillin administration
SUR-8.85.
The landmark delineating the junction of the antral and fundic regions
on the lesser curvature of the stomach is the:
A) incisura angularis
B) Mayo-vein
C) left gastric artery
D) gastroduodenal artery
E) gastroepiploic artery
SUR-8.86.
The most severe complication of gastric resection is:
A) thrombophlebitis
B) dehiscence of the duodenal stump
C) hemorrhage
D) sepsis
E) atelectasis
SUR-8.87.
All of the following are extraintestinal manifestations of Crohn's disease,
EXCEPT:
A) erythema nodosum
B) arthritis
C) uveitis
D) portal fibrosis
E) an increased risk of malignant transformation
SUR-8.88.
All of the following are intestinal complications of Crohn's disease,
EXCEPT:
A) perforation

B) intestinal fistula
C) fibrosis
D) intestinal obstruction
E) polyposis
SUR-8.89.
In which disease is Kantor's "string sign" a characteristic feature?
A) ulcerative colitis
B) mesenteric obstruction
C) volvulus of the sigmoid colon
D) regional enteritis
E) carcinoma of the rectum
SUR-8.90.
In Crohn's disease, the most frequent indication for surgery is:
A) an intra-abdominal fistula
B) an external fistula
C) an intra-abdominal mass (intestinal conglomerate)
D) stagnant bowel syndrome
E) intestinal obstruction
SUR-8.91.
All of the following substances are produced by functional carcinoid
neoplasms, EXCEPT:
A) serotonin
B) 5-hydroxy-tryptophan
C) kallikrein and histamine
D) ACTH
E) parathyroid hormone (parathormone)
SUR-8.92.
The therapy of intestinal carcinoid tumours is comprised of:
A) irradiation
B) chemotherapy
C) administration of serotonin antagonists
D) surgery followed by irradiation
E) intestinal resection
SUR-8.93.
Which of the following is the anatomic border between the left and
right lobes of the liver?
A) the line connecting the falciform ligament and the inferior vena
cava
B) the line connecting the falciform ligament and the "bare area"
C) the line connecting the fossa of the gall bladder and the fossa
of the inferior vena cava
D) the line connecting the fossa of the gall bladder and the "bare
area"
E) the line connecting the "ligamentum teres" and the inferior vena
cava
SUR-8.94.
Which of the following disorders is associated with an elevated serum
a-fetoprotein level?
A) hepatoma
B) carcinoma of the colon

C) Crohn's disease
D) carcinoma of the pancreas
E) carcinoma of the gall bladder
SUR-8.95.
The pathologic lesion characteristic of the Mallory-Weiss syndrome is:
A) rupture of the esophagus
B) rupture of the gastric mucosa
C) laceration of the mucosa at the gastroesophageal junction
D) hemorrhage from a gastric polyp
E) acute gastritis
SUR-8.96.
All of the following procedures are appropriate for the treatment of
verified choledocholithiasis, EXCEPT:
A) choledochotomy and T-tube drainage
B) choledochoduodenostomy
C) endoscopic papillotomy and extraction of biliary calculi
D) transduodenal sphincteroplasty
E) percutaneous extraction of biliary calculi using a Dormia-basket
SUR-8.97.
Gastric carcinoma develops most frequently in the region of the:
A) fundus
B) cardia
B) corpus
D) pyloric and antral region
E) gastroesophageal junction
SUR-8.98.
What is the percentage of biliary calculi with a calcium content sufficient
to make the stone visible on plain abdominal roentgenograms?
A) 90%
B) 80%
C) 50%
D) 40%
E) 15%
SUR-8.99.
In persons over the age of 50, the incidence of colonic diverticula is
approximately:
A) 0-5%
B) 5-10%
C) 10-20%
D) 20-30%
E) 30-40%
SUR-8.100.
Which of the following disorders causes adynamic or paralytic
ileus?
A) bezoars
B) annular pancreas
C) peritonitis
D) volvulus
E) intussusception/invagination

SUR-8.102.
The principal factor in the etiology of acute appendicitis is:
A) bacterial infection
B) mechanical obstruction
C) local circulatory insufficiency
D) the role of chemical substances
E) the lymphatic tissue of the processus vermiformis
SUR-8.103.
What is the least prevalent site for the development of
intraperitoneal abscesses?
A) the pelvis
B) the right subhepatic compartment
C) the left subphrenic compartment
D) the lesser peritoneal sac
E) the right subphrenic compartment
SUR-8.105.
All of the following statements are valid regarding the destruction of
erythrocytes in the spleen, EXCEPT:
A) a reduction of enzymatic and metabolic activity can be observed
in association with the ageing of erythrocytes
B) elderly erythrocytes travel through the spleen slowly
C) local hypoxia and an acidic environment contribute to the degradation
of erythrocytes in the spleen
D) the ATP-content of erythrocytes is reduced to an extremely low level
E) the ageing of erythrocytes is influenced by the overall functioning
of the spleen
SUR-8.106.
Spontaneous rupture of the spleen may occur in all of the following
conditions, EXCEPT:
A) infectious mononucleosis
B) malaria
C) leukemia
D) polycythemia vera
E) liver cirrhosis / portal hypertension
SUR-8.107.
A splenectomy has a beneficial effect in all the following hemolytic
conditions, EXCEPT:
A) hereditary spherocytosis
B) hereditary elliptocytosis
C) hemolytic anemia due to glucose-6-phosphate deficiency
D) thalassemia
E) sickle-cell disease
SUR-8.108.
Characteristic features of thrombocytopenic purpura include all of
the following, EXCEPT:
A) it is more prevalent in females
B) petechias, ecchymoses or hemorrhages occur
C) splenomegaly is present
D) subnormal thrombocyte count in peripheral blood
E) the megakaryocyte count in the bone marrow is normal or increased

SUR-8.112.
All of the following are present in long-standing ulcerative colitis,
EXCEPT:
A) a shortening of the gut
B) shrinkage and thickening of the mesentery
C) enlarged masses comprised of lymph node conglomerates
D) a dull, greyish mucosal surface
E) perforation and abscesses along the mesenteric margin
SUR-8.115.
Characteristic features of ischemic colitis are influenced by all of the
following factors, EXCEPT:
A) the extent of vascular obstruction
B) the duration of obstruction
C) the patency of the collateral circulation
D) the extent of bacterial invasion
E) the intensity of the inflammation
SUR-8.116.
All of the following are appropriate in the treatment of
pseudomembranous colitis, EXCEPT:
A) discontinuing the causative antibiotic(s)
B) the correction of electrolyte- and fluid losses
C) metronidazole therapy
D) vancomycin therapy
E) corticosteroid therapy
SUR-8.117.
Which of the following is the most appropriate therapeutic measure
following the resection of a carcinoma of the descending colon?
A) colonoscopy which should be repeated every 6 months
B) a barium enema repeated every 6 months
C) monitoring of the serum CEA (carcinoembryonic antigen) level
every 3 months
D) sulfasalazine (Salazopyrine) and prednisone therapy
E) checking the stool for occult bleeding regularly
SUR-8.118.
All of the following drugs can induce gastrointestinal bleeding, EXCEPT:
A) salicylates
B) corticosteroids
C) alcohol
D) phenothiazines
E) anticoagulants
SUR-8.119.
The most common cause of a massive hemorrhage from the colon is:
A) a carcinoma of the descending colon
B) colonic diverticulosis
C) ulcerative colitis
D) a carcinoma of the ascending colon
E) multiple polyposis
SUR-8.120.
All of the following are characteristic features of the PlummerVinson syndrome, EXCEPT:

.A) atrophy of the oral mucosa


B) koilonychia (concave fingernails)
C) anemia
D) dysphagia
E) subacute complex degeneration of the spinal cord
SUR-8.125.
Which of the following compounds is contraindicated following a
liver resection?
A) carbohydrates
B) albumin
C) vitamin K
D) hypnotic analgesics
E) antibiotics
SUR-8.126.
The incidence of biliary calculi increases after which of the following
operations?
A) the resection of a liver lobe
B) a subtotal gastrectomy
C) a splenectomy
D) a resection of the jejunum
E) a resection of the ileum
SUR 8.127.
Mechanical ileus due to obstruction by a biliary stone develops most
frequently in the:
A) duodenum
B) jejunum
C) ileum
D) sigmoid colon
E) anorectal junction
SUR-8.128.
All of the following are characteristic features of acute suppurative
cholangitis, EXCEPT:
A) jaundice
B) fever and chills
C) shock
D) hemorrhagic diathesis
E) CNS depression
SUR-8.129.
Which of the following features reflect a poor prognosis in acute
pancreatitis?
A) high surges of serum amylase levels
B) hyperglycemia and glycosuria
C) a high urinary amylase level
D) a prolonged coagulation time
E) a reduced serum calcium level
SUR-8.130
All of the following factors cause acute pancreatitis, EXCEPT:
A) alcohol
B) hypercalcemia associated with parathyroid dysfunction
C) biliary calculi

D) hyperlipidemia
E) hemochromatosis
SUR-8.131.
The most appropriate surgical intervention for a rectal carcinoma
located 4 centimetres above the anal orifice is:
A) an anterior resection
B) a rectal resection with the "pull-through" procedure
C) an abdominoperineal amputation of the rectum
D) a posterior resection
E) a two-stage resection
SUR-8.132.
In which of the following conditions is a "drain-pipe" colon a
characteristic feature visible on a roentgenogram taken after a
barium enema?
A) amebiasis
B) ulcerative colitis
C) tuberculosis of the colon
D) granulomatous enteritis
E) familial polyposis
SUR-8.133.
Which of the following "polyps" is considered a precancerous lesion?
A) pedunculated polyps
B) villous adenomas
C) polypoid adenomas
D) juvenile polyps
E) pseudopolyps
SUR-8.134.
All of the following statements are valid regarding thrombosed
hemorrhoids,. EXCEPT:
A) the abrupt appearance of a painful external bulge in the
anal region
B) the appearance of a tense, bluish, tender induration on the
anal border
C) an untreated lesion has a tendency to ulcerate and bleed
D) hemorrhoids usually resolve within 24 hours
E) prompt surgical decompression brings symptomatic relief
SUR-8.135.
Which of the following procedures is appropriate for the definitive
diagnosis of congenital megacolon?
A) a stool culture and parasitology
B) a rectal biopsy
C) a radiographic examination
D) pancreatic enzyme activity measurements in stool specimens
E) a sweat test
SUR-8.136.
The operation of choice for congenital megacolon is:
A) a colostomy
B) an enterostomy
C) a splanchnicectomy
D) a total colectomy

E) none of the above


SUR-8.137.
The most prevalent type of colorectal polyps are:
A) villous polyps
B) postinflammatory polyps
C) tubular adenomas
D) juvenile polyps
E) hereditary multiple polyps
SUR-8.138.
The anatomical border between the anus and the rectum is the:
A) lateral hemorrhoidal groove
B) interhemorrhoidal groove
C) dentate line
D) pecten
E) anorectal ring
SUR-8.139.
Colonic obstruction due to a carcinoma of the recto-sigmoidal junction
is best treated by:
A) a primary abdomino-perineal amputation
B) decompression by colostomy only
C) a primary anterior resection
D) resection and a colostomy
E) decompression by the insertion of a tube above the level of obstruction
and delayed elective surgery
SUR-8.140.
Juvenile polyps of the colon and rectum can produce all of the following
symptoms, EXCEPT:
A) blood-streaked stool
B) anemia
C) intussusception (invagination)
D) malignant transformation
E) prolapse from the anus
SUR-8.141.
Biochemical alterations induced by villous adenomas of the rectum
comprise all of the following, EXCEPT:
A) hypokalemia
B) hyponatremia
C) hypocalcemia
D) hypochloremia
E) uremia
SUR-8.142.
All of the following statements are valid regarding villous adenomas,
EXCEPT:
A) villous adenomas are most prevalent in the rectum
B) villous adenomas seldom develop in individuals under 45
years of age
C) villous adenomas occur as multiple lesions
D) villous adenomas cause electrolyte distrubances
E) pathological signs of malignant transformation are common

SUR-8.143.
Elevated CEA (carcinoembryonic antigen) levels have been observed
in all of the following conditions, EXCEPT:
A) carcinoma of the pancreas
B) breast cancer
C) uremia
D) lung cancer
E) myeloma
SUR-8.144.
Anal fissures can develop on the lateral wall in all of the following
conditions, EXCEPT:
A) Crohn's disease
B) ulcerative colitis
C) tuberculosis
D) lymphogranuloma venereum
E) leukemia
SUR-8.145.
The treatment of choice for a pancreatic abscess is:
A) a cystogastrostomy
B) a cystoenterostomy
C) external drainage
D) antibiotic therapy without drainage
E) aspiration
SUR-8.146.
Which of the following conditions is characterized by the proliferation
of mesenteric fat through the intestinal wall?
A) ulcerative colitis
B) Crohn's disease
C) tropical sprue
D) ischemic colitis
E) radiation colitis
SUR-8.147.
The most frequently occurring gynecologic lesion is:
A) a cervical infection
B) a vaginal infection
C) endometrial cancer
D) cervical cancer
E) Bartholini-abscess
SUR-8.148.
The most prevalent malignant vaginal tumor in children is:
A) melanocarcinoma
B) epidermoid carcinoma
C) squamous cell carcinoma
D) botryoid carcinoma
E) leiomyosarcoma
SUR-8.149.
All of the following statements are valid regarding an in situ'carcinoma
of the cervix, EXCEPT:
A) cellular changes in the squamous epithelial lining of the cervix
represent the development of cancer

B) this type of carcinoma is asymptomatic


C) the diagnosis can be established by the histologic examination
of biopsy specimens
D) this carcinoma is best treated by a radical hysterectomy
E) the 5-year survival rate equals 100%
SUR-8.150.
All of the following statements are valid regarding a carcinoma of
the cervix, EXCEPT:
A) 95% of the lesions are squamous cell carcinomas
B) these lesions usually develop at the squamocolumnar junction
C) early carcinoma of the cervix is asymptomatic
D) pain is the earliest symptom
E) exophytic changes can be observed in 80% of the patients
SUR-8.151.
The most prevalent malignant neoplasm of the uterine body is:
A) adenoacanthoma
B) adenocarcinoma
C) squamous cell carcinoma
D) melanoma
E) leiomyosarcoma
SUR-8.152.
Acute mastitis is prevalent at/in:
A) birth
B) puberty
C) pregnancy
D) breast feeding
E) menopause
SUR-8.153.
The risk of bilateral breast cancer is high when the primary lesion
(having developed in the breast affected first) is a(n):
A) comedo carcinoma
B) inflammatory carcinoma
C) lobular carcinoma
D) Paget's disease
E) carcinoma of the papillary duct
SUR-8.154.
Breast cancer develops most freqently in which of the following regions
of the breast?
A) the upper-medial quadrant
B) the upper-lateral quadrant
C) the subareolar region
D) the lower-medial quadrant
E) the lower-lateral quadrant
SUR-8.156.
The most prevalent histologic type of breast cancer is the:
A) invasive papillary carcinoma
B) invasive ductal carcinoma
C) medullary carcinoma
D) colloidal carcinoma
E) lobular carcinoma

SUR-8.157.
The most prevalent type of parathyroid cancer is a:
A) follicular carcinoma
B) papillary carcinoma
C) anaplastic carcinoma
D). medullary carcinoma
E) secondary carcinoma
SUR .158.
All of the following belong to skeletal changes occurring in
hyperparathyroidism, EXCEPT:
A) osteitis fibrosa cystica
B) osteopetrosis
C) solitary or multilocular bone cysts
D) pathologic fractures
E) osteoporosis
SUR-8.159.
The most common type of congenital adrenal hyperplasia develops
as a result of:
A) a 21-hydroxylase enzyme deficiency
B) an 11-hydroxylase enzyme deficiency
C) a 17-hydroxylase enzyme deficiency
D) a 3-b-hydroxysteroid-dehydrogenase enzyme deficiency
E) excessive ACTH release
SUR-8.160.
The treatment of choice for Cushing's disease is:
A) a hypophysectomy
B) irradiation of the pituitary gland
C) a unilateral adrenalectomy
D) a subtotal adrenalectomy
E) a total adrenalectomy
SUR-8.161.
None of the following general anesthetics should be administered
during the narcosis of patients with pheochromocytoma, EXCEPT:
A) ether
B) cyclopropane
C) trichloro-ethylene
D) nitrous oxide
E) halothane
SUR-8.162.
In which of the following forms of primary hyperaldosteronism is the
value of surgical therapy controversial?
A) a solitary aldosterone producing adenoma
B) carcinoma of the adrenal cortex
C) primary hyperaldosteronism due to adrenal hyperplasia
D) hyperaldosteronism sensitive to glucose administration
E) primary hyperaldosteronism of unknown origin
SUR-8.163.
The most reliable test in diagnosing a pheochromocytoma is:
A) the measurement of the 24-hour urinary metanephrine excretion rate

B) the urinary catecholamines concentration


C) a urinary vanillylmandelic acid determination
D) the determination of basal plasma catecholamine levels
E) the determination of plasma catecholamine levels following the
injection of histamine
SUR-8.164.
Type II multiplex endocrine neoplasia (MEN) consits of all the following,
EXCEPT:
A) medullary carcinoma of the thyroid gland
B) pituitary neoplasms
C) pheochromocytoma
D) parathyroid adenoma or hyperplasia
E) multiple neuromas
SUR-8.165.
The least prevalent symptom(s) of uterine myomas is(are):
A) irregular bleeding
B) pain
C) tenderness of the urinary bladder
D) malignant transformation
E) infertility and miscarriage
SUR-8.166.
All of the following are examples of APUD-cell (amine precursor uptake
and decarboxylation) tumors, EXCEPT:
A) medullary carcinoma of the thyroid gland
B) the Zollinger-Ellison syndrome
C) parathyroid adenoma
D) pheochromocytoma
E) Cushing's syndrome associated with lung cancer
SUR-8.167.
Daughters of mothers treated with DES (diethylstilboestrol) during
their pregnancies developed:
A) uterine cancer
B) breast cancer
C) ovarian serous cystadenon)as
D) carcinoma of the cervix and vagina
E) ovarian fibromatas
SUR-8.168.
The most appropriate test for the diagnosis of a urinary bladder rupture
after pelvic trauma is:
A) cystoscopy
B) intravenous urography (IVU)
C) cystography
D) a measurement of the difference between the unused and the
retrieved fluid volume during a test-lavage of the bladder
E) the detection of blood in the urine
SUR-8.169.
Which of the following procedures carries the highest risk of injury
to the ureter?
A) a radical hysterectomy
B) a penetrating or blunt trauma

C) an abdomino-perineal amputation of the rectum


D) an anterior resection of the rectum
E) irradiation of a cancer of the uterine cervix
SUR-8.170.
Which of the following elements of the genitourinary tract is removed
during a routine prostatectomy?
A) the urethral mucosa
B) the enlarged, adenomatous portion of the prostate
C) the sphincter musculature
D) the prostate itself
E) the verumontanum (colliculus seminalis)
SUR-8.173.
All of the following are characteristic features of polycystic kidney
disease, EXCEPT:
A) the diagnosis is made in infancy
B) hematuria
C) hypertension
D) renal damage
E) erythrocytosis
SUR-8.174.
Torsion of the spermatic cord:
A) occurs in youngsters
B) is a benign condition resolving without therapy
C) can be caused by epididymitis
D) is often bilateral
E) is associated with hematuria
SUR-8.175.
What is the usual therapy for papillary neoplasms occurring at the
base of the urinary bladder?
A) a transurethral resection
B) a segmental cystectomy
C) a total cystectomy
D) irradiation
E) a suprapubic cystectomy and cauterization
SUR-8.176.
Which of the following statements is true regarding renal-cell carcinoma?
A) its incidence is higher in males
B) it is not encapsulated
C) metastatic spread occurs predominantly via the lymphatic vessels
D) it can cause fever in the case of secondary infection
E) it is often bilateral
SUR-8.177.
Which of the following statements is true regarding penile cancer?
A) metastatic spread occurs through the deep pelvic veins
B) it is rare in adult males who have underwent circumcision in
infancy
C) it invades the prepuce but spares the glans
D) the para-aortic lymph nodes are usually invaded
E) irradiation is the therapy of choice

SUR-8.178.
Which of the following is a characteristic feature of retroperitoneal
fibrosis?
A) it is more prevalent in females
B) it is asymptomatic
C) it often causes obstruction of the ureter
D) it affects the lumbar region only
E) it is enclosed by a well-defined capsule
SUR-8.179.
The development of urinary calculi is associated with all of the following
factors, EXCEPT:
A) the metabolism of vitamin D
B) the presence of urea-splitting bacteria
C) urinary stasis
D) stenosis of the pyeloureteral junction
E) resorption disorders
SUR-8.180.
Which of the following urinary stones is radiolucent?
A) calcium oxalate stones
B) uric acid stones
C) cystine stones
D) triple-phosphate (magnesium ammonium phosphate) stones
E) mixed stones
SUR-8.181.
The best therapy for a circumscribed carcinoma of the prostate is:
A) a bilateral orchidectomy
B) estrogen therapy
C) an orchidectomy followed by estrogen therapy
D) a radical prostatectomy
E) a transurethral resection of the prostate
SUR-8.182.
An infection by which of the following microorganisms can predispose
someone to the formation of urinary calculi?
A) Escherichia coli
B) Proteus vulgaris and Pseudomonas aeruginosa
C) Streptotoccus faecalis
D) Mycobacterium tuberculosis
E) Bacillus typhosus
SUR-8.183.
The most frequent manifestation of a Wilms' tumour is:
A) an abdominal mass
B) weight loss
C) hematuria
D) permanent crying
C) pathologic fractures
SUR-8.184.
The most frequent symptom of a carcinoma of the renal parenchyma
in adults is:
A) an abdominal mass
B) hematuria

C) a fever of unknown origin


D) anemia
E) hemoptysis resulting from lung complications
SUR-8.185.
Which of the following is the most dominant route for the metastatic
spread of prostatic cancer to the pelvis and the lumbar vertebrae?
A) through the lymphatic vessels
B) through the Batsonian venous plexus
C) through the plexus pampiniformis
D) through the internal iliac vein
E) through the Santorinian venous plexus
SUR-8.186.
Which of the following statements regarding the venous return from
the testes (through the spermatic vein) is true?
A) the left and right spermatic veins join and drain into the inferior
vena cava
B) the right spermatic vein drains into the inferior vena cava,
whereas the left spermatic vein drains into the left renal vein
C) the right spermatic vein drains into the right renal vein, whereas
the left spermatic vein drains into the inferior vena cava
D) both spermatic veins drain into the ipsilateral renal vein
E) both spermatic veins drain into the inferior vena cava separately
SUR-8.187.
The most frequent cause of acute urinary retention in males is:
A) prostatic cancer
B) chronic prostatitis
C) benign prostatic hypertrophy
D) structural anomalies of the urethra
E) prostatic abscesses
SUR-8.188.
Necrosis of the renal papillae can occur in all of the following conditions,
EXCEPT:
A) diabetes mellitus
B) sickle-cell disease
C) phenacetin-induced renal damage
D) hydronephrosis
E) tuberculosis
SUR-8.189.
Which of the following radiologic procedures is the most valuable for
the differentation of renal cysts from renal neoplasms?
A) renal angiography
B) retrograde pyelography
C) intravenous urography
D) renal computer tomography
E) radionuclide renography
SUR-8.190.
All of the following statements are valid regarding cancer of the renal
pelvis, EXCEPT:
A) most cancers are associated with chronic infection and urinary
calculi

B) patients do not experience gross hematuria or abdominal mass


C) the histology usually denotes squamous cell or transitional
cell tumors
D) the prognosis is good after surgery
E) neither irradiation nor chemotherapy is effective
SUR-8.191.
All of the following statements are valid regarding neoplasms of the
renal pelvis and ureter, EXCEPT:
A) hematuria and colicky pain are the presenting signs
B) these neoplasms are often multiple
C) these neoplasms develop more often in males
D) the therapy of choice is nephroureterectomy with partial resection
of the bladder wall
E) postoperative irradiation and chemotherapy improves the
survival rate
SUR-8.192.
All of the following are radiological characteristics of benign
prostatic hyperplasia, EXCEPT:
A) a filling defect at the base of the urinary bladder
B) calcification of the prostate
C) urinary bladder diverticula
D) urinary calculi in the bladder
E) dilation of the ureters
SUR-8.193.
All of the following are indications for a prostatectomy in benign
prostatic hyperplasia, EXCEPT:
A) a weak urinary stream
B) distressing pollakiuria and nocturia
C) a considerable volume of infected residual urine
D) an enlarged prostate
E) secondary diverticula or urinary calculi
SUR-8.194.
The advantages of a transurethral resection of the prostate over an
open prostatectomy include all of the following, EXCEPT:
A) lower morbidity and mortality
B) urinary diverticula can be removed
C) a lower risk of wound complications and the development of
urinary fistulas
D) an indwelling urinary catheters can be removed earlier
E) a lower incidence of hemorrhagic complications
SUR-8.195.
Exploration of the kidney through the transabdominal route is indicated
in:
A) pyelonephritis
B) tuberculosis
C) neoplastic disease
D) cystic disease
E) hydronephrosis
SUR-8.196.
The keloid:

A) is less prevalent in blacks


B) is an agglomeration of fibrous tissue
C) has a tendency for malignant transformation
D) is sensitive to antibiotic therapy
E) develops in clean wounds
SUR-8.197.
The successful transplantation of "full-thickness" or "half-thickness"
skin grafts depends on all of the following factors, EXCEPT:
A) complete fixation
B) permanent immobilization
C) permanent and constant but not excessive pressure
D) the presence of denuded, white fascia under the graft
E) the lack of hematoma formation under the graft
SUR-8.198.
Homografts provide a biological dressing material for the coverage of
large, severely burnt areas with all of the following advantages, EXCEPT:
A) avoiding excessive fluid loss
B) reducing the size of the bacterial flora
C) reducing heat loss
D) restoring a positive nitrogen balance
E) protecting neurovascular structures
SUR-8.199.
Which of the following tissues regenerates by mitosis of the same
cells?
A) fat
B) bone
C) peripheral nerves
D) skin
E) skeletal muscle
SUR-8.200.
A severe burn injury is associated with all of the following factors,
EXCEPT:
A) a significant loss of body mass
B) a hypermetabolic reaction
C) hemoconcentration
D) potassium retention
E) a negative nitrogen balance
SUR-8.201.
Non-obstructive parotitis is associated with all of the following factors,
EXCEPT:
A) poor oral hygiene
B) advanced age
C) dehydration
D) debilitating illness
E) endotracheal intubation
SUR-8.203.
The most valuable information for the assessment of fluid replacement
in burned patients is:
A) diuresis is maintained at 30-50 ml/hour
B) the patient regains a normal level of consciousness

C) an adequate capillary filling time (tested under the fingernails)


D) physiologic arterial blood gases measurements
E) the development of mild pulmonary edema
SUR-8.206.
All of the following features belong to the signs and symptoms of dislocated
maxillary fractures, EXCEPT:
A) sensory loss of the face and upper lip
B) liquorrhea from the nose
C) enophthalmus
D) subcutaneous emphysema
E) malocclusion of the teeth
SUR-8.207.
All of the following statements are valid regarding mandibular fractures,
EXCEPT:
A) mandibular fractures are often complicated
B) mandibular fractures cause numbness in the lip
C) malocclusion of the teeth is an ordinary sign
D) the masseter muscles prevent further displacement of fracture
fragments
E) dental fixation is appropriate in most cases
SUR-8.208.
All of the following statements are valid regarding syndactylia,
EXCEPT:
A) males are afflicted more often. than females
B) this condition shows an autosomal dominant inheritance in
many families
C) syndactylia always affects the bones
D) a correct surgical plan is a prerequisite to the prevention of
contractures
E) the operation must be performed at 1-2 years of age
SUR-8.209.
All of of the following statements are valid regarding tenosynovitis,
EXCEPT:
A) poor natural resistance of the region is characteristic
B) starting from the wound of the thumb, the infection spreads to
the radial bursa and the interinterstitial spaces of the thenar
C) the infection of the other four fingers spreads to the ulnar
bursa
D) early surgical drainage is essential
E) postoperative physiotherapy must be started early to preserve
tendon functions
SUR-8.210.
All of the following are LeFort I type fracture fragments, EXCEPT:
A) the upper teeth and the palate
B) the lower fragments of the styloid processes
C) a portion of the walls of both maxillary sinuses (Higmoresinuses)
D) the nose tip
E) the saddle of the nose
SUR-8.211.
What is the percentage of the head and neck in total body surface

area according to the "rule of 9"?


A) 1%
B) 9%
C) 18%
D) 27%
E) 36%
SUR-8.212.
All of the following statements are valid regarding half-thickness
skin grafts, EXCEPT:
A) the healing of the donor region starts from the epithelium of
sweat glands exclusively
B) these grafts are readily available from flat body surfaces
C) the graft must cover the recipient area adequately.
D) half-thickness grafts are readily applicable for the coverage of
bones and tendons
E) the close connection of the graft with the donor region must be
preserved
SUR-8.213.
What is the treatment of choice for internal malignancies of the larynx?
A) irradiation
B) a median laryngectomy
C) a total laryngectomy
D) a total laryngectomy and radical cervical block-dissection
E) the local application of radium
SUR-8.214.
During an examination of the scalenus anticus syndrome:
A) all movements resulting in the relaxation of the anterior
scalenus muscle aggravate pain
B) pain can be elicited by pressing the shoulder passively down
while turning the head excessively to the contralateral side
C) pain can be elicited by pressing the shoulder passively down
while turning the head excessively to the ipsilateral side
D) passive flexion of the biceps muscle evokes severe pain
E) pain is aggravated by lifting the shoulder
SUR-8.215.
Salivary stones (sialoliths):
A) develop most freqently in the parotid gland
B) can be detected by palpation, using a lacrimal probe
C) can develop in association with urinary calculi
D) are easily palpable
E) are readily detected on roentgenograms
SUR-8.216.
Mixed tumors of the salivary glands:
A) develop most freqently in the submaxillary gland
B) are predominantly malignant
C) are found most frequently in the parotid gland
D) usually cause facial nerve paralysis
E) are associated with sialolithiasis
SUR-8.217.
Carcinoma of the lip

A) readily gives metastases


B) occurs more frequently in females
C) develops more frequently in the lower lip
D) is usually a basal-cell carcinoma
E) propagates via the blood stream
SUR-8.218.
All of the following are characteristic features of carcinoma of the
tongue, EXCEPT:
A) tongue carcinoma is associated with syphilis
B) tongue carcinoma is associated with poor oral hygiene
C) tongue carcinoma is usually painless
D) tongue carcinoma is frequently a chronic, torpid ulcer
E) tongue carcinoma bleeds easily on trauma
SUR-8.219.
Which of the following statements is false regarding laryngeal cancer?
A) hoarseness is an early sign.
B) the involved lymph nodes are not palpable in 35% of cases.
C) distant metastases develop at an early stage.
D) a biopsy is feasible for diagnostic purposes.
E) the five-year survival rate of laryngeal carcinomas affecting
only one of the vocal cords is 90%.
SUR-8.220.
Supraclavicular lymph nodes are often involved in:
A) the metastatic spread of primary malignancies
B) gastric and lung cancer
C) following injury to the anterior two-thirds of the tongue
D) a neuroblastoma
E) carcinoma of the lip
SUR-8.221.
Cystic hygroma:
A) is an unilocular cyst
B) is found most frequently in the mediastinum
C) is most prevalent in infants and small children
D) often undergoes malignant transformation
E) is best treated by irradiation
SUR-8.222.
All of the following statements are valid regarding neoplasms of the
carotid body, EXCEPT:
A) they are located beyond the bifurcation of the common carotid
artery
B) glomus tumors can cause paralysis of the tongue
C) these neoplasms do not secrete epinephrine
D) surgery carries a high risk due to the close proximity of the
carotid artery
E) these lesions are often malignant
SUR-8.223.
Asymptomatic enlargement of the salivary glands occurs in all of the
following conditions, EXCEPT:
A) in liver cirrhosis
B) in diabetes mellitus

C) in vitamin D deficiency
D) in kwashiorkor
E) in states of malnutrition
SUR-8.224.
All of the following are removed a during radical cervical block-dissection,
EXCEPT:
A) the sternocleidomastoid muscle
B) the external carotid artery
C) the internal jugular vein
D) the accessory spinal nerve
E) the submaxillary gland
SUR-8.225.
All of the following statements are valid regarding an ameloblastoma
of the mandible, EXCEPT:
A) this is the most common solid tumor of the mandible
B) it develops at the junction of the ramus and the body of the
mandible
C) it grows slowly but may invade soft tissues eventually
D) it is treated by a segmental resection of the mandible
E) recurrence occurs when adjacent soft-tissues have not been
removed
SUR-8.226.
All of the following statements are valid regarding lip cancer, EXCEPT:
A) most of the lesions are well-differentiated carcinomas
B) these cancers often develop at the outer margin of the lip
C) these cancers develop more often on the lower lip than on the
upper lip
D) exposure to sunlight is a predisposing factor
E) early lymphatic spread is characteristic
SUR-8.227.
All of the following statements are valid regarding carcinoma of the
hypopharynx, EXCEPT:
A) this tumor is more prevalent in males
B) smoking is an etiologic factor
C) Plummer-Vinson syndrome is an etiologic factor
D) hoarseness and dyspnea are early signs
E) distant metastases develop frequently in the lung and liver
SUR-8.228.
Which of the following neoplasms of neural origin does not belong
to the group of gliomas?
A) ependymoblastoma
B) neurinoma
C) oligodendroglioma
D) spongioblastoma
E) medulloblastoma
SUR-8.229.
The rupture of an intervertebral disk in the cervical spine:
A) causes pain radiating to the medial surface of the forearm
B) immobilization of the head alleviates the pain
C) it occurs most frequently between the 5th-6th and 6th-7th

vertebrae
D) pain is a more prevalent symptom than numbness and
paraesthesias
E) pain in the fourth finger of the hand is a useful localizing sign
SUR-8.230.
The most frequent cause of an epileptic seizure in a middle-aged patient
with an otherwise fair condition is:
A) an idiopathic seizure
B) arteriosclerosis
C) a brain tumor
D) Parkinson's disease
E) previous cerebral trauma
SUR-8.231.
The coincidence of headache, vomiting and papilledema in the same
patient is suggestive of.
A) a brain concussion
B) bulbar paralysis
C) a brain tumor
D) a migraine headache
E) a subarachnoid hemorrhage
SUR-8.232.
Leakage of cerebrospinal fluid from the nose is best treated by:
A) the insertion of a sterile oily gauze swab into the nostril
B) blowing the nose frequently
C) an emergency craniotomy
D) conservative antibiotic therapy and monitoring the patient for
7-10 days
E) the permanent suctioning of the liquor from the nose
SUR-8.233.
What is the cause of CS leaking from the ear?
A) a fracture of the sphenoid bone
B) a rupture of the tympanic membrane
C) a fracture of the lamina cribrosa
D) a fracture of the mastoid cells
E) a fracture of the temporal bone
SUR-8.234.
The estimated rate of axon regeneration following a peripheral nerve
injury is:
A) 1 millimetre per day
B) 1 millimetre per week
C) 3 millimetres per week
D) 1 centimetre per week
E) 10 centimetres per week
SUR-8.235.
The most malignant neoplasm of the brain is:
A) glioblastoma multiforme
B) astrocytoma
C) oligodendroglioma
D) spongioblastoma
E) ependymoma

SUR-8.236.
All of the following congenital abnormalities cause hydrocephalus in
infancy, EXCEPT:
A) Chiari type II malformation
B) meningocele
C) myelomeningocele
D) Dandy-Walker syndrome
E) malformation of the aqueduct
SUR-8.237.
In which diseases are stereotactic neurosurgical procedures the
most valuable?
A) brain tumors
B) cerebral aneurysms
C) parkinsonism
D) mental disorders
E) epilepsy
SUR-8.238.
Postthrombotic varicose veins develop as a result of.
A) the insufficiency of the perforant veins
B) the destruction of deep veins
C) the destruction of superficial veins
D) ileofemoral insufficiency
E) obstruction of the great saphenous vein
SUR-8.239.
All of the following are appropriate for the treatment of stasis ulceration
after venous thrombosis, EXCEPT:
A) elevation of the leg
B) compression dressing of varicose veins
C) skin grafting
D) correction of any underlying varicose veins
E) venous bypass surgery
SUR-8.240.
All of the following factors predispose to pulmonary embolism, EXCEPT:
A) a sedentary lifestyle
B) the presence of an underlying cardiac disease
C) the presence of a neoplasm
D) oral contraceptives
E) a male gender
SUR-8.241.
The primary therapy for pulmonary embolism is:
A) anticoagulant therapy
B) ligation of the inferior vena cava
C) thrombectomy
D) pulmonary embolectomy
SUR-8.242.
All of the following statements are valid regarding an aneurysm of
the splenic artery, EXCEPT:
A) this is the least prevalent of the aneurysms developing on visceral
arteries

B) it is usually the consequence of degenerative changes in the


tunica media of the arterial wall
C) it causes pain felt in the left hypochondrium and radiating to
the left shoulder
D) the calcified ring of the aneurysm may be visible on plain abdominal
roentgenograms
E) the risk of rupture is high during pregnancy
SUR-8.243.
The treatment of choice for stenosis of the internal carotid artery is:
A) thrombendarterectomy
B) excision with end-to-end anastomosis
C) bypass grafting
D) ligation of the internal carotid artery
E) administration of drugs inhibiting platelet aggregation
SUR-8.244.
The therapy of an arteriovenous fistula includes:
A) excision of the fistula and restoration of the contiguity of the
blood vessel
B) ligation of the artery distal to the fistule
C) amputation of the extremity
D) ligation of the vein distal to the fistule
E) drug therapy only
SUR-8.245.
All of the following are trophic changes resulting from chronic
ischemia, EXCEPT:
A) hair loss
B) friable, opaque fingernails
C) skin atrophy
D) muscle atrophy
E) osteoporosis
SUR-8.246.
Cervical lymphadenitis of bacterial origin is:
A) common in childhood
B) uncommon in childhood
C) relatively uncommon
D) none of the above
SUR-8.247.
Complications of hypoparathyroidism include:
A) cataract formation and cerebral calcification
B) diabetes
C) adrenal insufficiency
D) hydronephrosis
SUR-8.248.
The carpal-tunnel syndrome predominantly develops in which of the
following age groups:
A) in females about the onset of menopause
B) in males about the onset of climacterium
C) in middle-aged males and females
D) in elderly individuals

SUR-8.249.
Which of he following therapies is recommended for breast cancer
patients with extensive systemic metastases?
A) a simple mastectomy
B) a radical mastectomy
C) none of the above
SUR-8.250.
Pilonidal sinuses frequently develop in:
A) young adult males
B) young adult females
C) middle-aged patients
D) elderly patients
SUR-8.251.
The most frequent cause of arterial occlusion is:
A) embolism
B) arteriosclerosis
C) varicose veins
D) all of the above
SUR-8.252.
The leading cause of death during the first three decades of life is:
A) inadvertent poisoning
B) heart disease
C) drug addiction
D) trauma
SUR-8.253.
The first and most important step in the care of a severely injured
patient is:
A) the maintenance of patent airways
B) antishock therapy by parenteral volume replacement
C) the immediate immobilization of fractures
SUR-8.256.
Which of the following age groups is at a particular risk for post-surgical
psychiatric complications:
A) neonates
B) patients from 30 to 45 years of age
C) very young and extremely old patients
D) none of the above
SUR-8.257.
Fluid loss through the skin is extremely intense in which of the following
conditions?
A) hypothyroidism
B) hyperthyroidism
C) hypopituitarism
D) none of the above
SUR-8.258.
The incidence of congenital heart disease is:
A) 1:100 live births
B) 1:110 live births
C) 1:120 live births
D) 1:130 live births

SUR-8.259.
Which of the following heart diseases most often leads to
hemoptysis?
A) mitral regurgitation
B) tricuspid regurgitation
C) patent ductus arteriosus
D) mitral stenosis with pulmonary infarction
SUR-8.260.
Which of the following components of the fetal skeleton is the first to
undergo ossification during intrauterine life?
A) the occipital bone
B) the vertebrae
C) the ribs
D) the clavicle
E) the. Femur
SUR-8.261.
When two cartilaginous surfaces slide over each-other during articular
motion:
A) synovial fluid precludes the destruction of the cartilage
B) articular surfaces are always incongruent in all joints
C) due to its hyaluronic acid content, synovial fluid functions as
a lubricant
D) hyaluronidase does not influence the viscosity of synovial fluid
SUR-8.262.
Which of the following hormones can most likely influence growth?
A) thyroid hormone
B) hGH
C) adrenocorticotrophic hormone
D) insulin
E) all of the above
SUR-8.264.
Talipes is best treated at the age of
A) 1 month
B) 6 months
C) 1 day
D) from the date of discharge from the hospital
E) 3 months
SUR-8.265.
Case Study:
A child has suffered a "torsion" injury by putting his hand into the
washing-machine. The therapy of choice is:
A) elevation of the injured limb; hot or warm compression and
observation
B) watching out for the signs of swelling and the application of an
Ace-type elastic bandage on the swollen extremity
C) early elevation and an Ace-bandage; plus observation
D) elevation of the hand
E) elevation; ice packs and observation; a surgical elastic bandage
SUR-8.266.

The most striking feature of hematogenous osteomyelitis developing


in infants is:
A) resistance of the affected limb to passive movement
B) high-grade fever and toxicosis
C) low-grade fever and the lack of toxicosis
D) the mortality rate
E) radiologic changes
SUR-8.267.
A blood-borne metastatic infection of the vertebrae originates from
which of the following?
A) from the skin
B) from the lung
C) from the lower extremities
D) from the urogenital tract
E) from other bones
SUR-8.268.
Paronychia of the finger is best treated by which of the following
types of incisons?
A) halfway between the unguicular margin and the pulp of the
finger
B) at a considerable distance from the distal unguicular margin
C) any incision that ensures adequate drainage from the phalanx
D) a "J-shaped" incision at a 3 mm distance from the distal
unguicular margin
E) a wide "fishmouth" shaped incision
SUR-8.269.
Which of the following lesions is radiographically indistinguishable
from the articular changes induced by cortisol:
A) chondromalatia
B) hemophilic arthritis
C) gouty arthritis
D) acromegaly
E) neuropathic (Charcot's) joint
SUR-8.270.
The principal promoting factor of bone growth is:
A) cortisol
B) androgens
C) citrate
D) parathyroid hormone
E) thyroxine
SUR-8.273.
In which of the following conditions is the presence of SternbergReed cells a characteristic feature?
A) leukemia
B) multiple myeloma
C) liposarcoma
D) Hodgkin's disease
E) none of the above
SUR-8.274.
Metastatic tumours rarely develop at which of the following sites?

A) the long bones


B) the bones distal to the knee or elbow
C) the skull
D) the vertebrae
E) the pelvis
SUR-8.276.
In the case of clean, closed wounds, the most common source of a
Staphylococcus infection is:
A) the ambient air of the operating theatre
B) bedside instruments
C) dust
D) bedclothes
E) bacterial flora of the patient or personnel
SUR 8.277.
The most efficient method for the prevention of a superficial infection
after an appendectomy and suspected contamination of the
wound is:
A) sprinkling the wound with sulphonamide powder
B) drainage of the peritoneal cavity
C) flushing the wound with antibiotic solutions
D) delayed wound closure
E) none of the above
SUR-8.278.
Mammography is a particularly valuable method for the reduction of
breast cancer mortality, because:
A) it does not frighten female patients away from being examined
B) it detects most of the cases with stage I disease
C) there are no false positive results
D). it is a painless procedure
E) it is a particularly specific test in young females
SUR-8.279.
What is the percentage of breast cancer among the malignancies occurring
in females?
A) 5%
B) 10%
C) 25%
D) 20%
E) 30%
SUR-8.280.
The most prevalent cause of death after open heart surgery is:
A) hypoxemic acidosis
B) coagulation disorders
C) hypovolemia
D) hemolysis of red blood cells
E) microemboli
SUR-8.281.
The most prevalent cause of mortality due to a duodenal ulcer can be:
A) hemorrhage
B) peritonitis resulting from acute perforation
C) pyloric obstruction and inanition

D) an untreatable disease
E) an esophageal rupture associated with the regurgitation of the
acidic gastric contents
SUR-8.282.
The joints most freqently afflicted by bursitis and tendinitis are the:
A) shoulder, elbow, hip
B) shoulder, knee, ankle
C) elbow, ankle, wrist
SUR-8.283.
Most cases of acute subdeltoid bursitis are associated with which of
the following?
A) slight subluxation of the joint
B) periarticular calcification of the tendons of the rotator muscles
C) none of the above
SUR-8.284.
The greatest diagnostic value of radiography in arthritis is, that:
A) it denotes the abnormalities of bone structure as well as the
symptoms of the disease
B) it excludes factors inducing wear if arthritic changes are absent
C) both of the above
D) none of the above
SUR-8.285.
Mild infections of the hand can progress to potentially severe
infections, as:
A) the skin of the hand is a particularly heavily contaminated
area of the body
B) the hand often sustains injury when it is infected
C) there are numerous tendons with a poor blood supply and the
maintenance of any required immobilization is difficult
SUR-8.286.
The commonly occurring paronychia of the fingers is caused by:
A) frequent injury to the delicate skin of the fingers
B) extremely heavy contamination of the skin of the hand
C) excessive use and exposure of the hand and fingers during work
SUR-8.287.
The most important finger of the hand is the:
A) index finger
B) middle finger
C) thumb
D) ring finger
SUR-8.288.
Which of the following must be given a priority in the treatment of
head and neck injuries?
A) any airway obstruction and hemorrhage
B) periorbital injuries with the potential of visual loss
C) any head trauma inducing intracranial hemorrhage
D) any injuries to the cervical spine
SUR-8.289.

Flat T-waves on the ECG are suggestive of:


A) hypokalemia
B) hyperkalemia
C) a normal serum potassium level
SUR-8.290.
Cardiac arrest due to atrial fibrillation is more likely in:
A) hypokalemia
B) an excessively fast intravenous administration of potassium
SUR-8.291.
In which of the following cases does incomplete conjoining of fracture
fragments frequently occur?
A) in comminuted fractures
B) in infected fractures
C) in insufficient immobilizations
D) all of the above
E) none of the above
SUR-8.292.
Case Study:
You are summoned to an emergency at a nearby caf. Arriving on the scene,
you find a middle-aged, obese man sagging in his chair. Bystanders inform
you that the patient had ordered a coffee and two croissants. After he had
been served, he began to make forceful and wild gestures, swept
everything off from his table, howled incoherently, then collapsed back into his
chair.
On physical examination he is still uttering incoherent sentences. The patient resists physical examination and beats about with his arms. Ifis
skin and mucous membranes are of normal colour, but the patient is streaming
- with perspiration. Me chest and abdomen are normal on physical examination.
A Babinski-tendency is observed on the left side and patellar clonus
can be elicited on the right. Breath: no characteristic odour is scented.
Pulse: 88/min with sporadic extrasystoles. Blood pressure: 190/ 100 mmHg.
The most likely diagnosis is:
A) acute psychosis
B) alcoholic psychosis
C) a cerebral embolism
D) hypoglycemia
E) a transient cerebral ischemic attack
SUR-8.293.
All of the following are characteristic features of full-blown traumatic
(hypovolemic) shock, EXCEPT:
A) oliguria
B) peripheral vasoconstriction
C) an increased blood viscosity
D) stupor
E) bradycardia
SUR-8.294.
Which of the following diseases can be most likely transmitted by a
blood transfusion?
A) serum hepatitis
B) lymphoblastic leukemia
C) myeloblastic leukemia

D) erythrocytosis
E) Hodgkin's lymphoma
SUR-8.295.
Among all the severe complications of blood transfusions, the most
common problem is:
A) hemolytic reactions
B) disease transmission
C) circulatory overload
D) hypokalemia
E) alkalosis
SUR-8.296.
The incidence of which of the following cancer types has decreased
over the last 40 years?
A) breast cancer
B) lung cancer
C) colon cancer
D) stomach cancer
E) bone marrow cancer
SUR-8.297.
The most frequent severe complication of a Billroth 11 type gastric
resection is:
A) thrombophlebitis
B) dehiscence of the duodenal stump (anastomotic leakage)
C) hemorrhage
D) sepsis
E) atelectasis
SUR-8.298.
Which of the following is the characteristic feature of the MalloryWeiss syndrome?
A) mucosal rupture in the esophagus
B) rupture of the gastric mucosa
C) rupture of the esophageal mucosa and the gastric mucosa in
the region of the cardia
D) bleeding from a gastric polyp
E) portwine stains (naevus flammeus) of the stomach
SUR-8.299.
All of the following statements are valid regarding perforated appendicitis,
EXCEPT:
A) the incidence of this condition is higher in very young or old
patients.
B) it is more prevalent among the poor.
C) in acute appendicitis, early antibiotic therapy prevents the
perforation of the processus vermiformi.
D) an appendectomy must be performed if perforation has occurred.
SUR-8.300.
All of the following statements are valid regarding the development
of cancer in patients with ulcerative colitis, EXCEPT:
A) the incidence of cancer increases proportionally to the duration
of the disease
B) the age of onset of ulcerative colitis determines the incidence

of any associated cancer


C) the malignant neoplasms develop from pseudopolyps
D) the neoplasms are multiple, circumscribed and invasive
E) the prognosis is poor
SUR-8.301.
Which of the following is a sign of a poor prognosis in acute
pancreatitis?
A) the maximal value of the serum amylase activity
B) hyperglycemia and glycosuria
C) an elevated urinary amylase activity
D) a prolonged coagulation time
E) a reduced serum calcium level
SUR-8.302.
Villous adenomas (hairy polyps) of the rectum are best treated by:
A) repeated rectosigmoidoscopies
B) abdominoperineal resection
C) local excision of the lesion
D) infusion of electrolyte solutions
E) drug therapy
SUR-8.303.
Which of the following components is removed during prostatic resection?
A) the urethral mucosa
B) the hypertrophic adenomatous regions of the gland
C) the sphincter
D) the prostate gland itself
E) the verumontanum (colliculus seminalis)
SUR-8.304.
The most common indication for renal transplantation is:
A) hydronephrosis
B) end-stage glomerulonephritis or pyelonephritis
C) renal tuberculosis
D) Wilms' tumor
E) "staghorn stones"
SUR-8.306.
All of the following statements are valid regarding an adamantinoma
of the mandible, EXCEPT:
A) this is the most prevalent solid, hard neoplasm of the mandible
B) it develops at the junction of the mandibular body and the ramus
C) tt grows slowly but may invade soft-tissues eventually
D) the therapy of choice is a segmental resection of the mandible
E) recurrence is likely if the adjacent soft-tissues have not been
removed
SUR-8.307.
A carcinoma of the thyroid gland:
A) often causes hyperthyroidism
B) is usually associated with hypothyroidism
C) is generally characterized by normal thyroid function
D) only the metastases produce hormones
E) develops in toxic adenomas

SUR-8.308.
The most common nerve injury associated with trauma to the humerus
is damage to the:
A) radial nerve
B) median nerve
C) ulnar nerve
D) axillary nerve
E) musculocutaneous nerve
SUR-8.309.
The most common complication of a pelvic fracture is:
A) injury to the pars cavernosa of the urethra
B) injury to the pars membranacea of the urethra
C) injury to the scrotum
D) injury to the bladder
E) injury to the rectum
SUR-8.310.
The primary treatment for osteogenic sarcoma is:
A) irradiation
B) antimetabolites
C) radical amputation
D) abrasion
E) immunotherapy
SUR-8.311.
The most common cause of the superior vena cava syndrome is:
A) a bronchial carcinoma arising from the upper-right lobe
B) a neoplasm of the thymus
C) thyroid cancer
D) mediastinal fibrosis
E) multinodular goiter
SUR 8.312.
The therapy of choice for patients with tension pNEUmothorax associated
with dyspnea is:
A) intravenous volume replacement
B) administration of oxygen by mask
C) immediate aspiration of the air from the pleural cavity
D) administration of analeptic drugs
E) intubation
SUR-8.313.
All of the following statements are valid regarding lung cancer, EXCEPT:
A) lung cancer is never asymptomatic
B) lung cancer may be detected on roentgenograms as a coinsized
shadow on the periphery of the lung
C) hemoptysis is a common sign of lung cancer
D) a dry, distressing, improductive cough may be the only symptom
of lung cancer
E) partial or complete bronchial obstruction due to lung cancer
may predispose the patient to lung infections
SUR-8.314.
All of the following features may be visible on the roentgenogram of a
patient with Tetralogy of Fallot, EXCEPT:

A) a more pronounced vascular pattern over the lung


B) a boot-shaped heart
C) decreased pulsation of the pulmonary artery on fluoroscopy
D) enlargement of the right ventricle
E) narrowing of the shadow of the great vessels in the upper
mediastinum
SUR-8.315.
The most frequent sequel of a dissecting aneurysm of the thoracic aorta
is:
A) atherosclerosis
B) syphilis
C) degeneration of the tunica media
D) trauma
E) aortic stenosis
SUR-8.316.
An arteriovenous fistula is best treated by:
A) excision of the fistula and restoration of the contiguity of the
vessel
B) ligation of the artery distal to the fistula
C) amputation of the affected extremity
D) ligation of the vein distal to the fistula
E) conservative treatment only
SUR-8.317.
All of the following conditions are complications of empyema, EXCEPT:
A) bronchopleural fistula
B) empyema necessitatis
C) pericarditis
D) osteomyelitis
E) circumscribed pneumonia
SUR-8.318.
The primary treatment of a pulmonary embolism includes:
A) anticoagulant therapy
B) ligation of the inferior vena cava
C) thrombectomy
D) embolectomy
E) antibiotic therapy
SUR-8.319.
All of the following statements are valid regarding abdominal aortic
aneurysms, EXCEPT:
A) most patients are asymptomatic; however the lesion can be
detected by a routine physical examination
B) a pulsatile mass is palpated in the abdomen
C) ninety-five per cent of aneurysms develop beyond the level of
the renal arteries
D) calcification of the aorta and large vessels is often visible on
plain abdominal roentgenograms
E) the abrupt onset of severe pain, radiating to the back and the
hip indicates dissection or rupture of the aneurysm
SUR-8.320.
All of the following statements are valid regarding Raynaud's syndrome,

EXCEPT:
A) Raynaud's syndrome is prevalent among young neujrotic females
B) Raynaud's syndrome is a bilateral and symmetrical condition
affecting both hands
C) Raynaud's syndrome is characterized by vasospasm
D) exposure to cold results in blanching of the hands, then
cyanosis ensues, followed by hyperemia
E) the brachial pulse is always absent
SUR-8.321.
Acute mastitis is common in:
A) neonates
B) adolescents
C) pregnancy
D) lactation
E) menopause
SUR-8.322.
Metabolic sequelae of a gastric resection include:
A) macrocytic anemia
B) iron-deficiency anemia
C) metabolic alkalosis
D) calcium deficiency
E) steatorrhea
SUR-8.323.
Case Study:
A 45-year-old female patient has undergone a cholecystectomy.
Three days following the proceedure, she developed a deep-vein
thrombosis. The treatment of choice is:
A) elevation of the lower extremities
B) active exercises
C) anticoagulant therapy
D) thrombolytic therapy with streptokinase or urokinase
E) ligation of the great saphenous vein
SUR-8.324.
The immunosuppressive drug that has improved the results of organ
transplantation significantly is:
A) azathioprine (Imuran)
B) pyrmidine derivatives
C) corticosteroids
D) 0 cyclosporine A
E) actinomycin D
SUR-8.325.
What is the cause of a hyperacute rejection reaction ensuing after
renal transplantation?
A) circulating antibodies
B) cellular immunity
C) poor tissue perfusion
D) postoperative hypovolemia
E) intravascular thrombin deposition
SUR-8.326.
The anticoagulant substance used most frequently for blood-banking

purposes is:
A) 5% dextrose
B) acidic citrate-dextrose
C) citrate-phosphate-dextrose (CPD)
D) heparin
E) glycerine
SUR 8.327.
Which of the following facilitates collagen deposition during the
process of wound healing?
A) epithelial cells
B) endothelial cells
C) fibroblasts
D) capillary network
E) none of the above
SUR-8.328.
Which of the following are the precursors of fibroblasts active during
the process of wound healing?
A) epithelial cells
B) endothelial cells
C) fibroblasts from the systemic circulation
D) local mesenchymal cells
E) local basophil cells
SUR-8.329.
All of the following statements are valid regarding the hypotension
associated with spinal anesthesia, EXCEPT:
A) this hypotension is caused by the paralysis of
preganglionary symphatic fibers emerging from the Th12-L2
spinal segments
B) placing the patient in Trendelenburg's position is beneficial
C) the lower extremities must be elevated while avoiding the lowering
of the patient's head
D) vasoactive drugs are effective for this hypotension
E) this hypotension is the consequence of cerebrospinal fluid
leaking from the puncture site
SUR-8.330.
Case Study:
A 23-year-old girl developed shaking chills, high-grade fever, and
chest pain after obtaining a 75 ml blood transfusion. Select the most
likely cause of these symptoms:
A) a hemolytic transfusion reaction
B) a pyrogenic reaction
C) an infected blood product
D) the presence of cold agglutinins
E) citrate toxicity
SUR-8.331.
The treatment of hyperkalemia and its sequelae include all of the
following, EXCEPT:
A) full restriction of potassium administration
B) administration of glucose with insulin
C) administration of ion-exchange resins
D) magnesium administration

E) dialysis
SUR-8.332.
Feeding through a nasogastric tube is indicated in all of the following
conditions, EXCEPT:
A) an oropharyngeal neoplasm
B) complete dysphagia
C) for patients who refuse to eat
D) for unconscious patients unable to feed normally
E) a high gastrointestinal fistula
SUR-8.333.
All of the following factors contribute to the disruption of abdominal
surgical wounds, EXCEPT:
A) advane,ed age
B) cough
C) hypoproteinemia
D) anemia
E) malignancy
SUR-8.334.
The immediate physiologic reaction which reduces a great loss of
blood following vascular trauma is:
A) the adherence of platelets to the site of injury
B) vasoconstriction
C) serotonin release
D) the formation of a thrombus
E) the formation of a platelet plug
SUR-8.335.
An appropriate diagnostic test for the prompt verification of a suspected
coagulopathy is:
A) the bleeding time
B) the coagulation time
C) the platelet count
D) the prothrombin time
E) the partial thromboplastin time
SUR-8.336.
Generalized (diffuse) bleeding following surgical procedures may result
from all of the following, EXCEPT:
A) a massive transfusion
B) an undiagnosed coagulopathy
C) sepsis
D) insufficient hemostasis in the wound
E) fibrinolysis or defibrination
SUR-8.337.
Which of the following operations is associated with bleeding due to
increased fibrinolytic activity at the site of surgery?
A) operations on the pancreas
B) operations on the prostate
C) operations on the brain
D) open heart surgery
E) operations on the kidneys

SUR-8.338.
Classic hemophilia is characterized by all of the following features,
EXCEPT:
A) an X-linked recessive inheritance
B) a factor VIII deficiency
C) only males are affected
D) the lack of a spontaneous hemorrhage when factor VIII levels
are normal or higher than 25%
E) the excessive degradation of factor VIII
SUR-8.339.
Indications for a liver transplantation in humans include all of the
following, EXCEPT:
A) congenital biliary atresia
B) primary neoplasms confined to the liver
C) massive liver cirrhosis
D) biliary cirrhosis
E) acute liver damage
SUR-8.340.
All of the following belong to the causes of metabolic acidosis, EXCEPT:
A) diabetes
B) fasting
C) renal insufficiency
D) vomiting
E) intestinal fistulas
SUR-8.341.
Case Study:
A 45-year-old male patient develops multiple subcutaneous suffusions
following a two-week course of parenteral alimentation for a
severe gastric outlet obstruction. There is no evidence of a
gastrointestinal hemorrhage and the circulation of the patient is stable.
The most likely cause of these suffusions is:
A) disseminated intravascular coagulation (DIC)
B) sepsis
C) thrombocytopenia
D) a vitamin K deficiency
E) a calcium deficiency
SUR-8.342.
The normal osmotic pressure of the extracellular fluid is:
A) 100 mOsm/1
B) 200 mOsm/1
C) 300 mOsm/1
D) 400 mOsm/1
E) 500 mOsm/1
SUR-8.343.
All of the following can cause hypercalcemia, EXCEPT:
A) cancer metastases
B) sarcoidosis
C) multiple myeloma
D) vitamin D intoxication (hypervitaminosis D)
E) medullary carcinoma of the thyroid gland

SUR-8.344.
The rationale of dopamine administration (5-25 pg/kg/min) in circulatory
shock is that dopamine increases:
A) the release of adrenal steroids and catecholamines
B) systemic vasoconstriction
C) splanchnic perfusion
D) the cardiac volume
E) the pulse rate
SUR-8.345.
Which of the following microorganisms is considered the pathogen of
the pseudomembranous colitis associated with antibiotic therapy?
A) Bacteroides fragilis
B) Staphylococcus
C) Clostridium perfringens
D) Clostridium difficile
E) Clostridium tertium
SUR-8.347.
Hypomagnesemia develops in all of the following conditions, EXCEPT:
A) chronic alcoholism
B) hypoparathyroidism
C) pancreatitis
D) the early phase of burns
E) diabetic ketosis
SUR-8.349.
All of the following are characteristic features of Buerger's disease,
EXCEPT:
A) it is more prevalent in males
B) the peak incidence occurs between 20 and 40 years of age
C) it is rare in blacks
D) it is closely associated with smoking
E) it is an unusual form of atherosclerosis
SUR 8.350.
All of the following may lead to the thoracic outlet syndrome, EXCEPT:
A) a cervical rib
B) the scalenic anticus syndrome
C) costoclavicular compression
D) Raynaud's disease
E) a hyperabduction syndrome
SUR-8.351.
All of the following belong to the clinical features of the thoracic
outlet syndrome, EXCEPT:
A) paresthesias along the ulnar edge of the forearm
B) atrophy of the thenar muscles
C) cervical and shoulder pain
D) disappearance of the radial pulse on elevating the arm
E) ischemic gangrene of the fingers
SUR-8.352.
The most prevalent site of the pathologic lesion in occlusive
cerebrovascular disease is:
A) the middle cerebral artery

B) the carotid bifurcation


C) the basilar artery
D) the origin of the vertebral artery
E) the origin of the common carotid artery
SUR-8.353.
The most common site of peripheral arteriosclerotic aneurysms is:
A) the carotid artery
B) the subclavian artery
C) the femoral artery
D) the popliteal artery
C) the ulnar artery
SUR-8.354.
Aneurysms of the popliteal artery often present with/as:
A) rupture
B) pain and tenderness
C) a palpable mass
D) ischemic symptoms due to emboli
E) hemarthrosis in the knee
SUR-8.355.
All of the following can cause Raynauds' phenomenon, EXCEPT:
A) Buerger's disease
B) scleroderma
C) the thoracic outlet syndrome
D) vibration damage (due to the excessive use of pneumatic tools)
E) diabetic angiopathy
SUR-8.356.
Case Study:
A 65-year-old male patient passes tarry and loose stools (melena)
three days after aortobifemoral bypass surgery. The most likely
cause of this condition is:
A) a bleeding stress ulcer
B) ulcerative colitis
C) ischemic colitis
D) an aortoduodenal fistula
E) DIC (disseminated intravascular coagulation) related to a
Transfusion
SUR-8.357.
Case Study:
A 70-year-old male patient presents with a pulsating inguinal mass
and intermittent fever 6 months following aortobifermoral bypass
surgery and Dacron graft implantation. The peripheral pulse is normal
in both feet. The anastomotic site is not dilated on sonography.
The most likely cause of the lesion is:
A) a graft infection
B) a venous aneurysm resulting from the surgical trauma
C) a diverticular perforation and abscess
D) a pesudoaneurysm of the bypassed vessel
E) that the "hood" of the graft is unusually large
SUR-8.358.
Case Study:

A patient underwent reconstructive surgery 18 hours following an


acute traumatic occlusion of the femoral artery. In the recovery
room, the calf of the affected extremity is found hard and tender. The
most likely cause of this condition is:
A) undiagnosed venous trauma
B) a reperfusion reaction
C) an undiagnosed fracture of the leg
D) reperfusion syndrome
E) a muscle spasm due to nerve injury
SUR-8.359.
All of the following methods are appropriate for the diagnosis of
deep-vein thrombosis, EXCEPT:
A) phlebography
B) a radioisotope labelled fibrinogen uptake test
C) Doppler -ultrasonography
D) measurement of the venous flow-rate
E) impedance plethysmography
SUR-8.360.
Which of the following malignant neoplasms is prone to spontaneous
regression:
A) neuroblastoma
B) bronchogenic carcinoma
C) hypernephroma
D) Wilms' tumor
E) carcinoma of the urinary bladder
SUR-8.361.
The most important cause of the mortality of patients with a
tracheoesophageal fistula is/are:
A) pulmonary complications
B) diarrhea
C) vomiting
D) malnutrition, feeding incapability
E) other congenital abnormalities
SUR-8.362.
Which of the following is an etiologic factor of intestinal atresia?
A) maternal diabetes
B) maternal drug addiction
C) intrauterine vascular disorders of the intestine
D) birth trauma
E) hydramnios
SUR 8.363.
Appendicitis developing in small children is different from that occurring
in adults, because:
A) the symptoms are often caused by mesenteric lymphadenitis
B) perforation is a more prevalent complication
C) it usually follows a parotid flare
D) Meckel's diverticulum is more prevalent
E) interval appendectomy' is often feasible (in the d froid phase)
SUR-8.364.
All of the following statements regarding inguinal hernia in infants

and small children are valid, EXCEPT:


A) it is a familial disease
B) it is often bilateral
C) it develops more often in boys than girls
D) it is more prevalent in premature infants than in neonates delivered
at term
E) it is frequently associated with an anal stenosis or urethral web
SUR-8.365.
All of the following statements regarding teratomas having developed
in infants and small children are valid, EXCEPT:
A) most of these lesions develop in the sacrococcygeal region
B) they are more often external than internal in localization
C) the presenting symptom is the development of a palpable mass
D) these are highly vascularized tumors
E) most of these lesions are malignant; the prognosis is poor
SUR-8.366.
All of the following statements regarding meconium ileus are valid,
EXCEPT:
A) it is a hereditary disorder
B) it affects the lung, the pancreas, as well as the skin
C) spotty calcification can be detected in the abdominal wall
D) exocrine functions of the pancreas are decreased or absent
E) there is an aganglionary segment in the terminal ileum
SUR-8.367.
The most valuable diagnostic aid in the therapy of an imperforate
anus is:
A) serial roentgenogranis of the upper gastrointestinal tract
B) an X-ray of the chest
C) serial roentgenograms the colon
D) a plain abdominal X-ray taken with the infant hanging upside-down
E) a plain adominal X-ray taken in the erect position
SUR-8.368.
All of the following statements regarding malrotation of the gut are
valid, EXCEPT:
A) it is a hereditary disorder
B) it is the conseqence of a malrotation of the midgut
C) it develops during the 5th week of intrauterine life
D) it causes high intestinal obstruction
E) the cecum is located in the epigastrium or in the right upper
quadrant of the abdomen
SUR 8.369.
Where does Wilms' tumor originate from?
A) the renal capsule
B) the renal tubules
C) the epithelium of the renal pelvis
D) the renal cortex
E) the embryonal tissues of the kidney
SUR-8.370.
Intestinal polyps in childhood are most commonly located in the:
A) cecum

B) transverse colon
C) ileum
D) rectum and sigmoid colon
E) jejunum
SUR-8.371.
Which of the following causes the intermittent passage of variable
volumes of urine from the umbilicus?
A) exstrophy of the urinary bladder
B) persistent urachus
C) epispadiasis
D) omphalocele
E) reflux
SUR-8.372.
The most frequent cause of rectal bleeding in infants is:
A) ulcerative colitis
B) an anal fissure
C) intussusception (invagination)
D) a Meckel's diverticulum
E) a rectal polyp
SUR-8.373.
A cystic lesion developing in the midline of the neck in a 6-year-old
child is most probably a(n):
A) sebaceous cyst
B) branchiogenic cyst
C) thyrolingual cyst
D) cystic hygroma
E) lymph node
SUR-8.374.
The optimal age for orchidopexy performed for partially descended
testes is:
A) at birth
B) in the fist year of life
C) between 2 to 3 years of age
D) between 15 to 16 years of age
E) before marriage
SUR-8.375.
Cystic hygromas are:
A) congenital malformations
B) may undergo malignant transformation
C) do not develop before the second year of life
D) can develop as multifocal lesions
E) are benign neoplasms of the lymphatic system
SUR-8.376.
The most common type of esophageal atresia is:
A) esophageal atresia without fistula formation
B) a proximal esophageal web with a tracheal fistula and a blind
esophageal segment
C) a blind proximal segment and a distal esophageal stump with
a tracheal fistula
D) atresia with tracheal fistulas emerging from both esophageal

webs
E) tracheoesophageal fistula without atresia
SUR-8.377.
The optimal treatment of a neonate delivered at term with "C type"
esophageal atresia includes:
A) gastrotomy, cervical esophagostomy and delayed reconstruction
(secondary anastomosis)
B) immediate definitive reconstruction with primary anastomosis
C) sequential reconstruction
D) reconstruction by the interposition of a colon autograft
E) feeding gastrostomy and reconstruction when the infant has
reached a bodyweight of 10 kilograms
SUR-8.378.
All of the following belong to the manifestations of congenital
anomalies of the omphalomesenteric duct, EXCEPT:
A) a fecal fistula in the umbilicus
B) Meckel's diverticulum
C) an enterogenic cyst
D) a urinary fistula
E) a mucous fistula
SUR-8.379.
A divertuculum has been discovered on the terminal ileum during a
laparotomy. The complications of this lesion include all of the following,
EXCEPT:
A) diverticulitis
B) hemorrhage
C) stricture
D) volvulus
E) intussusception/invagination
TRUE-FALSE TYPE QUESTIONS
Put T for true statements and F for false statements!!!
SUR-8.380.
The omentum has a significant role in defending the abdominal cavity
against infections.
SUR-8.381.
Several hours can elapse before the characteristic symptoms of peritonitis,
resulting from intestinal perforation, due to a gunshot injury
develop.
SUR-8.382
Thoracic empyema is a relatively common disorder.
SUR-8.383.
The therapy of choice for ganglion of the wrist is conquassation
(squashing it by blunt impact).
SUR-8.384.
Bleeding from the nipple in premenopausal women is usually a
manifestation of breast cancer.

SUR-8.385.
Nausea, vomiting and malaise experienced by patients with viral
hepatitis are usually less distressing than in patients with
choledocholithiasis.
SUR-8.386.
Hepato-splenomegaly is an ordinary symptom in viral hepatitis,
whereas it is rarely present in uncomplicated choledocholithiasis.
SUR-8.387.
In small children, the surgical reinforcement of the Hesselbach-triangle
should always be attempted during the repair of an inguinal
hernia.
SUR-8.388.
In general, the treatment of fractures is less complicated in pediatric
patients than in adults.
SUR-8.389.
Establishing the diagnosis of an articular "sprain" exclusively by
clinical examination imposes a certain risk for the patient.
SUR-8.390.
The symptomatic treatment of chronic or recurrent knee complaints
is appropriate for a period sufficiently long enough so that spontaneous
improvement can occur.
MULTIPLE CHOICE QUESTIONS WITH KEY ANSWERS / TYPE II
Every question or incomplete statement has only one answer in the
following combinations:
A) if the answers 1, 2, and 3 are true
B) if the answers 1 and 3 are true
C) if the answers 2 and 4 are true
D) if only the answer 4 is true
E) if all the four answers are true
Select one of these key combinations!!!
SUR-8.391.
Which of the following parameters is useful for the recognition of
mounting intracranial pressure following head trauma?
1) a reduction in the level of consciousness
2) an increasing bradycardia
3) an elevation of the blood pressure
4) cardiac arrhythmia
SUR-8.392.
Which of the following suggests the development of a subdural
hematoma?
1) a common head injury
2) a permanent headache
3) progressively deteriorating neurologic signs
4) the abrupt onset of hemiplegia
SUR-8.395.
Which of the following methods is suitable for surgical analgesia and the
reduction of sensitivity to pain in patients with malignant pelvic

neoplasms?
1) a transection of peripheral nerves
2) an anterolateral cordotomy
3) a thalamotomy or lobotomy
4) a sympathectomy
SUR-8.396.
Signs suggesting the herniation of a vertebral disc between the L5S1 segments jeopardizing the S1 nerve root include:
1) loss of bladder function
2) weakness of the plantar flexors
3) weakness of the quadriceps muscle
4) absence of the Achilles-reflex
SUR-8.397.
Brain tumors in children:
1) develop in the posterior cranial fossa (scala posterior)
2) are extremely malignant in character
3) are located in the midline
4) have a tendency to bleed
SUR-8.398.
Precautionary measures necessary for abdominal paracentesis include:
1) emptying of the urinary blader
2) the puncture should be performed in the midline only
3) the scars of previous surgeries should be avoided
4) the puncture should not be performed for the diagnosis of intestinal
rupture
SUR-8.399.
The most valuable proceedures indicated for the diagnosis of an expanding
intracranial hematoma include:
1) a spinal tap
2) encephalography
3) electroencephalography (EEG)
4) a CT-scan
SUR-8.400.
Rupture of one of the two main bronchi is manifested by which of
the following?
1) tension pneumothorax
2) progressive mediastinal emphysema
3) subcutaneous emphysema
4) hemothorax
SUR-8.401.
The treatment of esophageal injuries includes the following:
1) immediate exploration
2) repair of the laceration
3) drainage of the operative site
4) resection of the damaged segment followed by reconstruction
SUR-8.402.
The treatment of. choice for penetrating injuries of the pancreas is:
1) suture of the pancreatic wound
2) conservative therapy

3) drainage of the damaged region


4) administration of corticosteroids
SUR-8.403.
On which of the following is the diagnosis of acute renal trauma
based on?
1) the enlargement of the shadow of the kidney visible on a plain
roentgenogram
2) the extravasation of the contrast material during intravenous
urography
3) hematuria
4) hypertension
SUR-8.404.
The therapy of adult respiratory distress syndrome includes:
1) PEEP (positive end-expiratory pressure) ventilation
2) diuretics
3) albumin
4) corticosteroids
SUR-8.405.
Which of the following statements regarding a duodenal rupture is valid?
1) blunt abdominal trauma often results in retroperitoneal rupture
2) bacterial peritonitis is a late sequel
3) the pain may radiate to the testes or to the back
4) the chemically induced inflammation is of minimal intensity
SUR-8.406. Select One Of The Key Combinations
Appropriate procedures for the treatment of colon injury include:
1) a primary suture
2) a primary suture and a proximal colostomy
3) an exteriorizing colostomy (anteposition of the damaged colon
segment)
4) a proximal colostomy
SUR-8.407.
Protein stores available for the mobilization of proteins in the catabolic
state essentially include:
1) the liver
2) the kidney
3) plasma proteins
4) skeletal muscle
SUR-8.408.
Which of the following statements regarding the WaterhouseFriderichsen syndrome is valid?
1) an acute bilateral adrenal hemorrhage occurs
2) it is a complication of a meningococcal infection
3) death is caused by infection
4) death is caused by adrenal insufficiency
SUR-8.409.
What is the cause of an increased aldosterone level measured in a
post-traumatic patient?
1) ACTH
2) potassium

3) renin-angiotensin
4) epinephrine (adrenalin)
SUR-8.410.
The indications for a total hip endoprosthesis implantation include:
1) osteoarthritis
2) septic arthritis
3) rheumatoid arthritis
4) gonococcal arthritis
SUR-8.411.
The characteristic radiologic changes visible in the knee of a
hemophiliac patient after recurrent hemorrhages include:
1) widening of the femoral condyles
2) signs of osteoporosis
3) enlargement of the intercondyler incisure
4) narrowing of the joint space
SUR-8.412.
A synovectomy is indicated in rheumatoid arthritis, because:
1) it is more feasible than joint replacement
2) it effectively alleviates articular pain
3) it increases the stability of the joint
4) it protects the joint from destruction
SUR-8.413.
Contraindications to hip arthrodesis include:
1) jobs involving permanent standing
2) vertebral osteoarthritis
3) secondary osteoarthritis
4) bilateral hip disease
SUR-8.414.
In polyomyelitis:
1) paralysis ensues on the 3rd or 4th day following the onset of
symptoms
2) some patients do not become paralyzed
3) the regeneration of muscle power may take as long as 2 years
4) sensory loss always resolves
SUR-8.415.
Indications for the surgical therapy of scoliosis include:
1) deformities uncorrectable by the application of braces
2) progression of the deformity despite bracing
3) the completion of bone growth after the correction of the deformity
4) cosmetic correction of the deformity in postural scoliosis
SUR-8.416.
Surgical treatment of congenital hip dysplasia:
1) is unnecessary under 1 year of age
2) is indicated in all forms of dysplasia
3) is mandatory in cases with soft-tissue interposition in the joint
space
4) often consists of open reduction in older children
SUR-8.417.

Diagnostic signs of congenital hip dysplasia at birth are:


1) restricted abduction during flexion of the hip
2) Ortolani's sign
3) a visible shortening of the thigh
4) Trendelenburg's sign
SUR-8.418.
In spondylolisthesis:
1) genetic heredity and an increased prevalence are observed in
inbred populations
2) radicular pain always accompanies backache
3) the "beheaded Scottish terrier" phenomenon is visible on
roentgenograms
4) ideally, young patients should not be operated upon
SUR-8.419.
Pathologic fractures may occur in:
1) malignancies
2) bone development disorders
3) metabolic disorders
4) congenital abnormalities
SUR-8.420.
The avulsion of the anterior cruciate ligament of the knee is associated
with the laceration of the:
1) lateral collateral ligament
2) medial meniscus
3) patellar ligament
4) medial collateral ligament
SUR-8.421.
The Pancoast tumor:
1) is a form of lung cancer
2) causes brachial neuralgia
3) causes Horner's syndrome
4) is usually bilateral
SUR-8.422.
When multiple ribs sustain double fractures simultaneously, a large
region of the chest wall is destabilized, that is, "flail chest" develops.
The significance of this condition is that:
1) it is difficult to diagnose
2) the affected side of the chest wall moves paradoxically with
ventilation
3) it is often associated with pericardiac tamponade
4) it leads to respiratory insufficiency
SUR-8.423.
The primary treatment of pyogenic lung abscesses includes:
1) permanent antibiotic therapy based on cultures and susceptibility
testing
2) postural drainage
3) bronchoscopic aspiration of the pus
4) surgical excision
SUR-8.424.

Efficient operations for Tetralogy of Fallot in patients aged from 1 to


5 years include:
1) an anastomosis between the superior vena cava and the right
pulmonary artery
2) an anastomosis between the left subclavian artery and the
innominate artery
3) an anastomosis between the descending aorta and the left pulmonary
artery (Potts' operation)
4) an anastomosis between the left subclavian artery and the pulmonary
artery (Blalock-Taussig operation)
SUR-8.425.
Significant branches of the left coronary artery are:
1) the circumflex artery
2) the sinoauricular nodal artery
3) the left anterior descending branch
4) the atrioventricular nodal artery
SUR-8.426.
Contraindications to the surgical treatment of ventral septal defects
include:
1) an advanced age associated with pulmonary vascular disease
2) an age over 10 years
3) the ratio of pulmonary and systemic vascular resistance is
greater than 0.9
4) an associated aortic regurgitation
SUR-8.427.
Factors contributing to an increased risk of developing lung cancer
include:
1) smoking
2) exposure to radioactive substances
3) exposure to chromium compounds
4) exposure to asbestos
SUR-8.429.
The therapy of a dissecting aortic aneurysm includes:
1) blood pressure reduction (hypotensive therapy)
2) surgical fenestration
3) excision of the aneurysm and reconstruction with a Dacron
prosthesis
4) excision of the aneurysm and suture of the dissected segment
SUR-8.430.
Indications for coronary bypass surgery include:
1) severe angina resisting drug therapy
2) a progressively deteriorating angina
3) a previous myocardial infarction and coronary disease
4) a previous myocardial infarction and coronary disease confined
to a single vessel
SUR-8.431.
Candidates for the surgical treatment of pulmonary stenosis include:
1) all patients with this condition
2) patients with cardiomegaly
3) adults with mild pulmonary stenosis

4) patients with a greater than 50 mmHg systolic-diastolic pressure


gradient at the pulmonary valve
SUR-8.432.
Indications for the surgical treatment of aortic stenosis include:
1) a valvular pressure gradient >50 mmHg
2) stenosis associated with the angina
3) any associated cardiac defects
4) all patients with aortic stenosis
SUR-8.433.
Congenital heart defects are associated with:
1) maternal rubella withstood in the first trimester of pregnancy
2) maternal drug abuse
3) mongolism (Down's syndrome)
4) maternal eclampsia
SUR-8.434.
Neural damage in patients with a right-to-left shunt may result
from:
1) cyanotic periods
2) a cerebral abscess
3) a paradox embolism
4) abnormal anatomy of the cerebral vasculature
SUR-8.435.
Hormones influencing calcium metabolism include:
1) adrenal steroids
2) thyroxine
3) growth hormone
4) parathormone (parathyroid hormone)
SUR-8.436.
Common complications of enteral and parenteral hyperalimentation
include:
1) hyperglycemia
2) glycosuria
3) hyperosmolar coma
4) septicemia
SUR-8.437.
Cyclosporine is a new drug with a potential to increase the successrate
of organ transplantations. The characteristic features of this
drug include:
1) cyclosporine is a fungal metabolite
2) cyclosporine has a propensity to induce the development of
lymphomas
3) renal toxicity
4) bone marrow toxicity
SUR-8.438.
Which of the following is not contraindicated in a patient with a
suspect tetanus wound?
1) toxoid
2) antitoxin
3) debridement of the wound

4) penicillin G
SUR-8.439.
The effect of a vagotomy on the stomach is, that it reduces:
1) the capacity of the stomach
2) gastric motility
3) intrinsic factor secretion
4) acid production
SUR-8.440.
The right branches of the vagal nerve include:
1) the celiac plexus
2) Grassi's nerve
3) the hypogastric plexus
4) the hepatic plexus
SUR-8.441.
The treatment of paralytic ileus includes:
1) intravenous fluid replacement
2) prostigmine administration
3) aspiration of the gastric contents through a nasogastric tube
4) early surgery
SUR-8.442.
The diagnosis of a hydatid cyst (Echinococcal cyst) of the liver is
suggested by:
1) hepatomegaly
2) the calcified cyst is visible on the plain abdominal x-ray
3) the results of serologic tests
4) CT-scanning
SUR-8.443.
The presence of air in the biliary tract is a diagnostic feature of:
1) a choledochoduodenal fistula
2) intestinal obstruction
3) emphysematous cholecystitis
4) viral hepatitis
SUR-8.446.
Signs and symptoms of a perforated duodenal ulcer include:
1) an abrupt onset
2) severe abdominal pain
3) muscular rigidity (guarding)
4) pneumoperitoneum
SUR-8.447.
Characteristic x-ray features of gallstone ileus include:
1) the presence of air in the biliary tract
2) dilation of the small bowel
3) concentric calcification visible in the lower-left abdominal
quadrant
4) loss of the psoas muscle shadow
SUR-8.448.
The therapy of the initial phase of an acute diverticulitis includes:
1) insertion of a nasogastric tube

2) intravenous fluid replacement


3) systemic antibiotics
4) sigmoidectomy
SUR-8.449.
The principal symptoms of a ruptured Fallopian tube pregnancy include:
1) a missed menstrual period
2) an abrupt onset of abdominal and pelvic pain
3) a unilateral adnexal mass
4) shock
SUR-8.450.
The most important features of the diagnosis of acute pancreatitis
include:
1) alcoholism in the history
2) diffuse epigastric pain and tenderness
3) elevated serum amylase levels
4) jaundice
SUR-8.451.
Laboratory parameters showing characteristically elevated values in
acute pancreatitis include:
1) the serum amylase activity
2) the urinary amylase activity
3) the serum lipase activity
4) the serum calcium level
SUR-8.452.
The characteristic features of intestinal obstruction include:
1) colicky abdominal pain
2) vomiting
3) no flatus is passed
4) a distended, "meteoristic" abdomen
SUR-8.453.
Performed in intestinal obstruction, radiography may detect:
1) abnormal gas content in the small bowel
2) multiple fluid levels
3) the absence of gas in the large bowel
4) free air on the right side, under the diaphragm
SUR-8.454.
Indications for the surgical treatment of a duodenal ulcer include:
1) perforation
2) hemorrhage
3) obstruction
4) resistance to adequate and permanent medical (conservative)
therapy
SUR-8.455.
What is the emergency treatment for a bleeding duodenal ulcer?
1) transfusion
2) nasogastric suction
3)K gastroscopy
4) gastrointestinal radiologic examinations

SUR-8.456.
The surgical treatment of perforated duodenal ulcer includes:
1) simple closure with an omental patch
2) pyloroplasty
3) pyloroplasty and vagotomy
4) selective vagotomy
SUR-8.457.
The cause of hypokalemia developing in vomiting due to a duodenal
obstruction is:
1) a loss of saliva
2) a loss of gastric juice
3) a loss of pancreatic juice
4) an exchange of H+ for K+ with a K+ loss in the kidney
SUR-8.458.
The characteristic features of Zollinger-Ellison's syndrome include:
1) massive gastric hypersecretion
2) untreatable and recurring peptic ulcer(s)
3) non-beta islet cell neoplasm of the pancreas
4) diarrhea
SUR-8.459.
A diagnosis of the Zollinger-Ellison syndrome is established by
which of the following?
1) hypergastrinemia
2) excessive acid secretion
3) a dilated stomach with an edematous mucosa
4) identification of the pancreatic tumor by selective angiography
SUR-8.460.
Clinical manifestations of the carcinoid syndrome include:
1) flushing of the upper half of the body
2) diarrhea with cramping abdominal pain
3) bronchospasm with wheezing
4) right sided valvular heart disease
SUR-8.462.
The characteristic features of stress ulcers include:
1) stress ulcers developing most freqently in the stomach
2) flat, shallow lesions
3) multiple lesions
4) perforation
SUR-8.463.
The clinical features of choledocholithiasis include:
1) jaundice
2) gallstone
3) fever
4) ascites
SUR-8.464.
Late complications related to feeding disorders following gastric resection
include:
1) megaloblastic anemia
2) steatorrhea

3) iron deficiency anemia


4) calcium deficiency
SUR-8.465.
What is the mechanism of the metastatic spread of malignancies to
the liver?
1) through the portal vein
2) through the hepatic artery
3) through the portal lymphatic vessels
4) by direct propagation
SUR-8.466.
Cholecystokinin-pancreozymin:
1) is produced in the intestinal mucosa
2) stimulates the emptying of the gall bladder
3) relaxes the sphincter of Oddi
4) stimulates the secretion of pancreatic enzymes
SUR-8.467.
The effects of secretin include:
1) stimulation of the water and bicarbonate secretion of the pancreas
2) a choleretic effect
3) the inhibition of gastric secretion
4) greater intestinal motility
SUR-8.468.
Causes of extracellular hypovolemia developing in intestinal obstruction
include:
1) fluid sequestration in the lumen of the obstructed intestinal loops
2) fluid sequestration in the intestinal wall
3) vomiting and the suction of gastric content through a
nasogastric tube
4) fluid loss into the abdominal cavity as peritoneal fluid
SUR-8.469.
Surgical decompression of a dilated and distended intestine is beneficial
because:
1) it improves the circulation of the intestinal wall
2) it removes toxins from the intestinal lumen
3) it facilitates closure of the abdominal wound
4) it reduces the risk of a wound infection
SUR-8.470.
The optimal treatment of esophageal achalasia includes:
1) bypass surgery
2) dilation
3) a vagotomy and drainage
4) an esophagomyotomy
SUR-8.471.
Tuberculotic enteritis is often manifested by:
1) intestinal obstruction
2) an intraabdominal mass of conglomerate intestinal loops
3) an alternation of constipation and diarrhea
4) massive bleeding

SUR-8.472.
The carcinoid syndrome causes primary valvular disease on which of
the following valves?
1) the mitral valve
2) the tricuspid valve
3) the aortic valve
4) the pulmonary valve
SUR-8.473
Which of the following drugs would you administer to reduce the
serum ammonia level of a patient in a hepatic coma?
1) vasopressin
2) kanamycin
3) laxatives
4) lactulose
SUR-8.474.
Carcinoids of the processus vermiformis:
1) comprise 50% of all carcinoids developing in the
gastrointestinal tract
2) about 50% of these lesions are malignant
3) the carcinoid syndrome is uncommon
4) a right-sided hemicolectomy is indicated and beneficial
SUR-8.476.
In primary sclerosing cholangitis:
1) an associated ulcerative colitis is common
2) the passage of stones causes fibrosis
3) differentiation from sclerosing carcinoma of the biliary tract is
difficult
4) a choledochoenterostomy is the therapy of choice
SUR-8.477.
In carcinoma of the gall bladder:
1) most of the patients are females
2) the presenting symptoms are characteristic and of diagnostic
value
3) gallstones are detected in 90% of the cases
4) the five-year survival rate is 50%
SUR-8.478.
In which of the following carcinomas is a pancreaticoduodenostomy
indicated?
1) carcinoma of the papilla of Vater
2) carcinoma of the duodenum
3) carcinoma of the distal common bile duct
4) carcinoma of the head of the pancreas
SUR-8.479.
The diagnostic features of an insulinoma include:
1) fasting and exertion aggravate the symptoms
2) the fasting blood glucose level is below 50 mg/dl (2,7 mmol/1)
3) the symptoms are alleviated by an intake of glucose
4) the administration of corticosteroids precludes the development
of attacks

SUR-8.480.
Desmoid tumors:
1) are benign
2) contain gonadotrophic substances in great quantity
3) have a tendency of local recurrence
4) tend to metastatize to the lungs and bones
SUR-8.481.
The clinical features of intestinal angina include:
1) postprandial pain
2) weight loss
3) steatorrhea
4) recurrent midline abdominal pain
SUR-8.482.
Case Study:
A 2.5 cm node was detected in the left lobe of the thyroid gland of a
female patient and a partial thyroidectomy was performed. A histological
examination demonstrated a papillary carcinoma of the thyroid.
The subsequent therapy includes:
1) periodic follow-up examinations
2) a total thyroidectomy
3) external irradiation
4) a modified radical cervical block-dissection
SUR-8.483.
Which of the following nerves is most prone to injury during a total
thyroidectomy?
1) the superior laryngeal nerve
2) the external laryngeal nerve
3) the internal laryngeal nerve
4) the recurrent laryngeal nerve
SUR-8.484.
Which of the following ligaments holds the uterus in place?
1) the round ligament
2) the broad ligament
3) the uterosacral ligament
4) the pectineal ligament
SUR-8.485.
Mycotic vulvovaginitis is a common problem:
1) in diabetic patients
2) in females taking oral contraceptives
3) during systemic antibiotic therapy
4) in hypertensive patients
SUR-8.486.
The classic signs of a cystocele include:
1) constipation
2) recurrenct cystitis
3) cervical bulging
4) vaginal bulging
SUR-8.487.
The important diagnostic features of chronic pelvic inflammatory

disease are:
1) tenderness of the cervix
2) bloody discharge from the cervix
3) a tender mass palpable in the pelvis
4) a soft, mobile cervix
SUR-8.488.
The clinical features of Meigs' syndrome include:
1) an ovarian fibroma
2) ascites
3) pleural effusion
4) hydrocephalus
SUR-8.489.
Bloody discharge from the nipple:
1) is associated with malignant neoplasms in 10-30% of the cases
2) may result from a ductal papilloma or carcinoma
3) may result from multiple injury
4) can be treated with a total mastectomy
SUR-8.490.
Paget's disease of the areola:
1) is extremely rare and comprises 1% of all breast cancers
2) is a primary carcinoma of the ducts
3) is an eczematous lesion
4) has a worse prognosis than breast cancer in general
SUR-8.491.
A mammography is indicated:
1) for monitoring the other breast in females who have undergone
unilateral mastectomy due to cancer
2) for the examination of voluminous, obese breasts where palpation
is unreliable
3) for the assessment of a vague mass detected in patients with
multiple cysts
4) for the annual screening of all females over 25 years of age
SUR-8.493.
After a partial mastectomy or focal excision, the therapy of breast
cancer should include:
1) chemotherapy
2) adrenalectomy
3) oophorectomy
4) irradiation
SUR-8.494
Trophoblastic disease:
1) develops from fetal annexes
2) gives metastases most frequently to the ovaries
3) responds to chemotherapy remarkably well
4) it is essential to schedule primary surgical therapy before
chemotherapy
SUR-8.495.
The histopathologic features of the thyroid gland in Graves' disease
include:

1) papillary processes in the acini


2) the accumulation of colloid in large quantities within the gland
3) an increased vascularization of the gland
4) the reduction of lymphoid conglomerates
SUR-8.496.
A medullary carcinoma of the thyroid may be associated with:
1) pheochromocytoma
2) Cushing's syndrome
3) hyperparathyroidism
4) pituitary adenomas
SUR-8.497.
A papillary carcinoma of the thyroid
1) features the slowest growth-rate among the malignancies of
the thyroid gland
2) can transform into the anaplastic form
3) is dependent on the stimulatory effect of TSH
4) has a tendency for malignant trasformation with increasing age
SUR-8.498.
A thyroid storm:
1) can be induced by trauma, infection or acidosis
2) is usually treated by considerable doses of iodine
3) cortisone is a valuable drug in its therapy
4) propranolol administration is benefical as it antagonizes
sympathic dominance
SUR-8.499.
What are the effects of parathyroid hormone (PTH)?
1) it has a direct effect on the bones, resulting in bone resorption
2) it increases the absorption of calcium from the small intestine
3) it reduces the tubular resorption of phosphates
4) it increases the tubular resorption of calcium
SUR-8.500.
The components of the pseudohyperparathyroidism syndrome include:
1) hypercalcemia
2) hypophosphatemia
3) non-parathyroid malignancy
4) renal damage
SUR-8.501.
The bone lesion characteristic of secondary hyperparathyroidism
due to chronic renal disease is:
1) hyperphosphatemia
2) hypocalcemia
3) reduced calcium absorption from the intestines (resulting from
vitamin D resistance)
4) a consequence of systemic metabolic acidosis
SUR-8.502.
Which of the following statements regarding hematuria is valid?
1) 75% of the patients are older than 40 years of age
2) the ratio of males/females is 2:1
3) the source of bleeding is often located in the urinary bladder

4) infection is a more frequent cause of hematuria than


neoplastic disease
SUR-8.503.
Acidic urine facilitates the production of which type of stones?
1) oxalate stones
2) uric acid stones
3) calcium phosphate stones
4) cystme stones
SUR-8.504.
Which of the following types of stones is dissolved by acidic urine?
1) xanthine stones
2) calcium phosphate stones
3) calcium oxalate stones
4) ammonium magnesium phosphate stones
SUR-8.505.
Which of the following statements regarding urethral webs is valid?
1) they are prevalent in boys
2) the diagnosis is established by cystourethrography
3) the prognosis is determined by the extent of the damage to the
proximal segment
4) these patients present with uremia and hypertension
SUR-8.506.
Scrotal hypospadiasis is often associated with:
1) cryptorchism
2) exstrophy of the urinary bladder
3) infertility
4) imperforate anus
SUR-8.507.
Which of the following statements regarding the treatment of renal
cell carcinoma is valid?
1) early ligation of the renal vein reduces metastatic spread
2) the kidney should be removed en bloc with the perirenal fat
3) the resection of solitary pulmonary or bone metastases prolongs
survival
4) preoperative irradiation and chemotherapy are beneficial
SUR-8.508.
Which of the following procedures is indicated for a suspected testicular
neoplasm?
1) aspiration of the hydrocele for cytology
2) measurement of the urinary gonadotropin levels
3) needle biopsy of the testis
4) surgical exploration and orchidectomy
SUR-8.509.
Which of the following statements regarding a carcinoma of the alveolar
crest is valid?
1) it develops as a deep. ulcer in the alveolar cavity
2) it is always treated by irradiation
3) a resection of the mandible is the therapy of choice
4) a radical cervical block-dissection is seldom indicated

SUR-8.510.
Cysts of the embryonal branchial arch are:
1) usually located beyond the thyroid cartilage
2) carcinomatous foci
3) treated by excision
4) always treated by irradiation
SUR-8.511.
Malignant neoplasms of the salivary glands:
1) are more prevalent in males
2) 60% of such lesions develop in the parotid gland
3) recurrent parotitis predisposes to malignant transformation
4) the prevalence of breast cancer is high in females with malignant
salivary neoplasms
SUR-8.512.
In leukoplakia of the oral cavity:
1) 5% of the lesions eventually undergo malignant transformation
2) the thin, early lesion requires surgical excision
3) patients with early hyperplasia and hyperkeratosis must abstain
from smoking and the consumption of alcoholic beverages
4) irradiation is indicated in cases where the size of the lesions
precludes total extirpation of the mucosa
SUR-8.513.
In cancer of the mandible:
1) the neoplasm is most frequently an epidermoid carcinoma
2) sore throat is a common presenting symptom
3) the five-year survival rate is about 25%
4) the clinical results of surgical therapy or irradiation are poor
SUR-8.514.
Prognathism:
1) is a hypertrophy of the mandible
2) is the anteposition of the chin and lips
3) results in malocclusion of the teeth and the early loss of denture
4) does not require correction
SUR-8.515.
Dislocation of the temporomanidbular joint:
1) occurs whent the condylar head is displaced forward through
the anterior laceration of the articular capsule
2) fixes the jaw in the open position
3) can be reduced by pushing the molar teeth downwards
4) results in a permanent facial deformity
SUR-8.516.
Second-degree burns:
1) damage the epidermis and corium
2) are characterized by blister formation
3) heal by epithelial regeneration starting from the sweat glands
4) heal by scarring
SUR-8.517.
In third-degree burns:

1) the skin and subcutaneous tissues are destroyed by coagulation


necrosis
2) flushing is a characteristic feature
3) early escharotomy and skin grafting precludes the development
of contractures
4) associated physiological abnormalities are minimal
SUR-8.518.
Which of the following factors increase the mortality of a burn injury?
1) an advanced age
2) the grade of initial shock
3) the severity and degree of burn injury
4) the volume of fluid loss through the damaged skin surface
SUR-8.519.
Which of the following tumors belong to intracranial neoplasms
showing visible calcification on the plain skull x-ray?
1) craniopharyngioma
2) oligodendroglioma
3) meningioma
4) glioblastoma multiforme
SUR-8.520.
Which of the following is an important diagnostic aid in the treatment
of a spontaneous subarachnoid hemorrhage?
1) spinal tap
2) emergency craniotomy
3) CT-scan
4) ventriculography
SUR-8.521.
Stress-fractures may occur in the:
1) pelvic bones
2) tibia
3) metatarsal bones
4) femur
SUR-8.523.
The common surgical treatment for bleeding esophageal varices includes:
1) a portocaval shunt
2) a gastric resection
3) a transection of the esophagus
4) a splenectomy
SUR-8.524.
Torsion of the testis is suspected:
1) in young patients
2) when sudden pain strikes in the inguinal region
3) if the testis is enlarged and soft
4) if hematuria is present
SUR-8.525.
The complications of prostatic resection include:
1) recurrent obstruction within 5 years in 10-25% of patients
2) urethral stricture in 10% of patients

3) prostatic cancer
4) inguinal hernia
SUR-8.526.
The causes of diarrhea associated with the Zollinger-Ellison syndrome
include:
1) the passage of gastric juice of high ionic content to the small
intestine
2) an inactivation of pancreatic enzymes resulting in steatorrhea
3) an acidic intraluminal pH stimulates intestinal motility
4) hypogastrinemia
SUR-8.527.
Diagnostic procedures indicated in testicular tumors include:
1) aspiration of the hydrocele for cytology
2) determination of tumor markers (serum a-protein and b-hCG)
3) needle-biopsy of the testis
4) surgical exploration and biopsy
SUR-8.528.
The characteristic features of a coarctation of the aorta include:
1) it develops most freqently at the junction of the aortic arch and
descending aorta, below the origin of the left subclavian artery
2) blood pressures which are different in the upper and lower extremities
3) a 'notching' of the ribs on the chest x-ray
4) left ventricular hypertrophy
SUR-8.529.
Indications for coronary bypass surgery include:
1) severe angina unresponsive to medical treatment
2) occlusion of the left main coronary artery
3) strictures localized in three main coronary branches
4) previous myocardial infarction and congestive heart failure
SUR-8.530.
Venous circulatory failure of the lower extremity can be corrected by
which of the following methods?
1) bed rest, elevation of the lower extremity and the application of
elastic bandages
2) ligation and extirpation of the saphenous vein (by stripping)
3) ligation of incompetent perforator veins
4) ligation of the saphenous vein in the femoral region and the
popliteal fossa
SUR-8.531.
Which of the following statements regarding bloody disharge from
the nipple is valid?
1) it is associated with malignancy in 20-30% of cases
2) it may result from ductal papilloma or carcinoma
3) a multifocal lesion can develop
4) a radical mastectomy is the treatment of choice
SUR-8.532.
Paget's disease of the nipple:
1) is an extremely uncommon condition comprising 1% of all
breast cancers

2) is a primary carcinoma of the duct


3) resembles eczematous dermatitis
4) is associated with a worse prognosis than breast cancers in
general
SUR-8.533.
Signs suggesting the development of a transfusion reaction in an
anesthetised patient include:
1) hypotension
2) tachycardia
3) the sudden increase of bleeding at the operative site
4) cyanosis
SUR-8.534.
Valid statements about the potassium requirements of postoperative
patients include:
1) the serum potassium level must be measured before supplementation
2) normal renal function is a prerequisite to parenteral potassium
supplementation
3) the daily potassium requirement of a normal adult is 30-40
mEq/litre
4) the urinary potassium level must be measured before the administration
of any potassium containing preparation
SUR-8.535.
Complications of immunosuppressive therapy include:
1) infection
2) hypotension
3) malignant transformation
4) a predisposition to infertility
SUR-8.536.
Respiratory acidosis is associated with:
1) elevated arterial PaCO2 values
2) reduced plasma bicarbonate values
3) an insufficient gas exhange
4) an increased arteriovenous oxygen difference (A-VO2)
SUR-8.537.
The risk of the development of severe respiratory acidosis is increased
by:
1) severe compensatory metabolic acidosis
2) hypokalemia
3) a shift to the right of the oxygen dissociation curve
4) tetany and seizures
SUR-8.538.
A massive blood transfusion may result in:
1) hypocalcemia
2) hyperkalemia
3) thrombocytopenia
4) acidosis due to unmetabolized citrate
SUR-8.539.
Which of the following pathogens cause an infection with crepitation
in the vicinity of the wound?

1) streptococci
2) Clostridium tetani
3) Clostridium perfringens
4) this feature is characteristic of Meleney's ulcer
SUR-8.540.
The effects of radiation on tissues include:
1) radiation destroys both normal and neoplastic cells, however,
neoplastic cells lack physiologic mechanisms required for regeneration
2) radiation penetrates tissues and causes ionization by collision
with tissue atoms
3) peroxide residues cause DNA and chromosomal fragmentation
4) the essential effects radiation on neoplasms result from vascular
obliteration
SUR-8.541.
Histocompatibility antigens are:
1) expressed at an extremely early phase of life
2) located on cellular membranes
3) of different avidity in various tissues
4) polypeptide chains
SUR-8.542.
Calcium has an essential role in:
1) the regulation of the stimultory threshold of neural function
2) the contractility of skeletal and cardiac muscle
3) the function of cellular membranes and intracellular components
4) the release of hormones
SUR-8.543.
The clinical features of aortoiliac vascular disease include:
1) symptoms develop only when the occlusion of the vascular lumen
reaches 90%
2) the formation of aneurysms is common
3) the incidence of coronary or cerebral vascular disease is 30-50%
4) complete occlusion of the aorta has become prevalent during
the nineties
SUR-8.545.
Therapeutic principles of segmental femoropopliteal occlusive artery
disease include:
1) vigorous exercise programs
2) sympathectomy for severe claudication
3) saphenous vein grafting
4) endarterectomy
SUR-8.546.
Factors contributing to the successful reconstruction of blood vessels
in infected wounds include:
1) extensive debridement
2) adequate antibiotic therapy
3) delayed wound closure
4) early vascular reconstruction
SUR-8.547.
Which of the following pathologic features are characteristic of stabilized

arteriovenous fistulas?
1) an increased cardiac volume
2) a widened pulse pressure
3) the development of collateral circulation
4) a reduced cardiac volume
SUR-8.548.
The clinical features of arteriovenous fistulas include:
1) dilated varicose veins
2) tachycardia ensuing on the obliteration of the fistula
3) permanent rumbles
4) perfusion which is sufficient for tissue nutrition beyond the
fistula
SUR-8.549.
Congenital arteriovenous fistulas:
1) are common in the lower extremities
2) are often multifocal
3) cause local gigantism
4) cause heart failure
SUR-8.550.
Rectal prolapse in children:
1) is common under 3 years of age
2) is usually the consequence of chronic constipation or diarrhea
3) resolves spontaneously
4) is treated by proctopexy
SUR-8.551.
What is the diagnosis of a foramen of Bochdalek hernia based on?
1) a scaphoid (concave) abdomen
2) a barrel-chest
3) cyanosis and respiratory distress
4) heart sounds which are audible over above the right axillary
line in most cases
SUR-8.552.
Which of the following is a consequence of neonatal duodenal obstruction
due to an annular pancreas?
1) bilious vomiting
2) abdominal distension/meteorism
3) "double bubble" sign visible on plain abdominal
roentgenograms
4) bloody vomiting
SUR-8.553.
Cysts of the common bile duct:
1) are manifested in the first months of life by jaundice
2) are represented by a palpable mass in the right-upper abdominal
quadrant
3) represent the best treatable causes of jaundice in infancy
4) are often multiple
SUR-8.554.
In congenital biliary atresia:
1) the abnormality is possibly caused by intrauterine viral infection

2) the extrahepatic segment of the biliary tract is replaced by fibrous


bands
3) surgical therapy is not recommended after 12 weeks of age
4) mortality associated with this condition often results from
biliary cirrhosis, portal hypertension and liver damage
SUR-8.556.
Case Study:
A needle aspiration of the knee yields a relatively thick, clear and
viscous fluid. The patient has had vague complaints for several
weeks. What is the proper action?
1) the fluid should be cultured
2) the fluid should be inspected and discarded
3) the viscosity of the fluid should be measured to decide
whether it is a transudate or an exudate
4) protein electrophoresis should be performed
SUR-8.557.
Which of the following factors contribute to the development of
shock associated with a burn injury?
1) blood loss
2) plasma loss
3) toxicosis
4) infection
5) azotemia
A) answers (1), (3), and (5) are correct
B) answers (2), (3), and (4) are correct
C) answers (3) and (5) are correct
D) only answer (5) is correct
E) all of the answers are correct
SUR-8.558.
Which of the following conditions are rapidly fatal if left untreated?
1) edema of the glottis
2) generalized edema (anasarca)
3) cerebral edema
4) pulmonary edema
5) hydrothorax
A) answers (1), (3), and (5) are correct
B) answers (2), (3), and (4) are correct
C) answers (3) and (5) are correct
D) only answer (5) is correct
E) all of the answers are correct
SUR-8.559.
Case Study:
A 12-year-old boy has undergone an appendectomy for acute appendicitis.
Four days later, signs of an advanced intestinal obstruction
and peritonitis have developed. In adddition to supportive therapy,
the treatment of choice is:
1) the administration of neostigmine in high doses
2) an emergency laparotomy for the location and drainage of any
intraabdominal abscesses
3) no intervention is required

4) the insertion of a Miller -Abott or Kantor tube and permanent


suction of the intestinal contents
A) answers (1), (3), and (5) are correct
B) answers (2), (3), and (4) are correct
C) answers (3) and (5) are correct
D) only answer (5) is correct
E) all of the answers are correct
ASSOCIATION QUESTIONS
Associate the following terms/statements marked by the letters
A, B, C... with the corresponding statements/terms marked by and in
the order given by the figures 1, 2, 3...
...for example: 1-C, 2-B, 3-A, 4-D. Put the answer as C, B, A, D!
(Note: Different statements can be associated with the same terms!!!)
SUR-8.560.
Associate the following term(s) with their corresponding statement(s)!
A) Atrophy of the muscles of the hand
B) Wrist paralysis
C) Boutonniere deformity
D) Hammer toes
1) injury to the ulnar nerve at the level of the wrist
2) injury to the extensor tendon at the terminal phalanx
3) injury to the extensor tendon near its insertion at the middle
phalanx
4) injury to the radial nerve in the upper arm
SUR-8.561.
Associate the following term(s) with their corresponding statement(s)!
A) Ulcerative colitis
B) Granulomatous colitis
C) Both of the above
D) None of the above
1) the small intestine is often involved
2) the risk of cancer is high
3) intestinal fistulas are a complication
4) a proctocolectomy is required
SUR-8.562.
Associate the following term(s) with their corresponding statement(s)!
A) Thyroid gland
B) Upper segment of the parathyroid gland
C) Lower segment of the parathyroid gland
D) Breast
1) a modified sweat gland
2) the third pharyngeal pouch (branchial arch)
3) the fourth pharyngeal pouch (branchial arch)
4) it develops from the foramen cecum as an external pouch
SUR-8.563.
Associate the following term(s) with their corresponding statement(s)!
A) Isoflurane

B) Enflurane (Enthrane)
C) Halothane (Fluothane)
D) Nitrous oxide
1) it depresses myocardial contractility
2) hepatotoxicity and jaundice
3) the least potent agent
4) undergoes only minimal metabolism in the body
SUR-8.564.
Associate the following term(s) with their corresponding statement(s)!
A) Peutz-Jeghers' syndrome
B) Familial polyposis
C) Gardner's syndrome
D) Multiple polyps
1) hamartomas
2) malignant transformation in all patients
3) osteomas and fibromas
4) pigmentation of the mucosa is absent
SUR-8.565.
Associate the following term(s) with their corresponding statement(s)!
A) Gonadotrophins
B) CEA (carcinoembryonic antigen)
C) Alpha-fetoprotein
D) 5-hydroxy-indolacetate
E) Vanillylmandelic acid
1) ovarian neoplasms G
2) carcinoid syndrome
3) desmoid tumors
4) Sipple's syndrome
5) there are elevated levels in cirrhosis
SUR-8.566.
Associate the following term(s) with their corresponding statement(s)!
A) Ulcerative colitis
B) Crohn's disease
C) Both of the above
D) None of the above
1) bleeding is common
2) malignant transformation is possible
3) it causes intestinal stricture
4) it is caused by an infection
5) it has extraintestinal manifestations
6) it involves both the small and large intestines
SUR-8.567.
Associate the following term(s) with their corresponding statement(s)!
A) Zollinger-Ellison syndrome
B) Verner-Morrison syndrome
C) Both of the above
D) None of the above

1) diarrhea is common
2) a pancreatic neoplasm
3) associated with colitis
4) a total gastrectomy is necessary
SUR-8.568.
Associate the following term(s) with their corresponding statement(s)!
A) Pseudomucinous cyst of the ovary
B) Chocolate cyst of the ovary
C) Dermoid cyst of the ovary
D) Cyst of the corpus luteum
1) it is a non-neoplastic cyst
2) it can reach an extreme size
3) associated with pelvic endometriosis
4) a benign neoplasm with a tendency for bilateral development
SUR-8.569.
Associate the following term(s) with their corresponding statement(s)!
A) Propylthiouracil
B) Antithyroid drug
C) Radioiodine therapy
D) Subtotal thyroidectomy
1) risk of agranulocytosis
2) a prothrombin deficiency
3) recurrent laryngeal nerve paralysis
4) potential hypothyroidism in all treated patients
5) an increased formation of nodules
SUR-8.570.
Associate the following term(s) with their corresponding statement(s)!
A) Primary hypothyroidism
B) Secondary hypothyroidism
C) Both of the above
D) None of the above
1) hypercalcemia and increased urinary calcium excretion
2) hypocalcemia
3) renal damage
4) normal or elevated serum alkaline phosphatase activity
5) medullary carcinoma of the thyroid
SUR-8.571.
Associate the following term(s) with their corresponding statement(s)!
A) Varicocele
B) Hydrocele
C) Spermatocele
D) Ureterocele
1) cobra-head deformity
2) it is usually left-sided
3) a cyst of the epididymis
4) it is prevalent in children
SUR-8.572.

Associate the following term(s) with their corresponding statement(s)!


A) Embryonal carcinoma
B) Seminoma
C) Interstitial cell tumor
D) Chorioepithelioma.
E) Teratoma
1) elevated urinary gonadotropin levels
2) precocious puberty in boys
3) the most prevalent testicular tumor in childhood
4) it may contain a cartilage component
5) the least malignant testicular neoplasm
SUR-8.573.
Associate the following term(s) with their corresponding statement(s)!
A) Dislocated shoulder
B) Valgus deformity of the elbow
C) Laceration of the wrist
D) Fracture of the humerus
1) the ulnar nerve is damaged
2) the median nerve is damaged
3) the axillary nerve is damaged
4) the radial nerve is damaged
SUR-8.574.
Associate the following term(s) with their corresponding statement(s)!
A) Meningocele
B) Myelocele
C) Encephalocele
D) Syringomyelia
1) a fluid-filled sac in the substance of the spinal cord
2) a sac containg cerebrospinal fluid
3) a swelling of the spinal nerves into the meningeal sac
4) cerebral tissue in the meningeal sac
SUR-8.575.
Associate the following term(s) with their corresponding statement(s)!
A) A brain tumor in adults
B) A brain tumor in children
C) Both of the above
D) None of the above
1) it is often located supratentorially
2) it resides in the posterior fossa (scala posterior)
3) a malignant glioma
4) it usually develops in the midline
5) a low-grade astrocytoma
SUR-8.576.
Associate the following term(s) with their corresponding statement(s)!
A) Extradural neoplasm of the spinal cord
B) Intradural neoplasm of the spinal cord
C) Both of the above
D) None of the above

1) C it causes paraplegia
2) it is often malignant
3) it is often benign
4) it often gives metastases
5) it shows a slow clinical course
SUR-8.578.
Associate the following term(s) with their corresponding statement(s)!
A) Gradually increasing intracranial pressure
B) Paralysis of the oculomotor nerve
C) Both of the above
D) None of the above
1) dilated and fixed pupils
2) a loss of consciousness
3) external deviation of the eye (lateral gaze)
4) bradycardia
SUR-8.579.
Associate the following term(s) with their corresponding statement(s)!
A) Spondylosis
B) Spondylolysis
C) Spondylolysthesis
D) Spondylitis
1) a bone defect of the vertebral arch
2) inflammation of the vertebra
3) the slippage of one vertebral body over the body of the adjacent
lower vertebra
4) hypertrophic bone lesions where exophytes may extend into
the intervertebral foramen and exert pressure on the nerve radix
SUR-8.580.
Associate the following term(s) with their corresponding statement(s)!
A) Histoplasmosis
B) Coccidioidomycosis
C) North-American blastomycosis
D) Aspergillosis

1) suppuration and cavities with thin walls in the lung


2) skin lesions
3) fungi balls in the lung
4) a solitary nodule in the lung
SUR-8.581.
Associate the following term(s) with their corresponding statement(s)!
A) squamous cell carcinoma of the lung
B) Undifferentiated cell carcinoma of the lung
C) adenocarcinoma of the lung
D) Alveolar cell carcinoma of the lung
1) a fast propagation
2) a peripheral location
3) the best prognosis

4) the most prevalent neoplasm of the lung


SUR-8.582.
Associate the following term(s) with their corresponding statement(s)!
A) Omphalocele
B) Gastroschisis
C) Both of the above
D) None of the above
1) a congenital abnormality of abdominal wall closure
2) a normally positioned umbilical cord
3) the defect is covered by amnionic epithelium
4) there are associated congenital abnormalities
5) spina bifida
RELATION ANALYSIS
In the following questions determine if the statement in the first half
of the sentence and the explanation in the second half of the sentence
are true and if a causal drelationship exists between them. Select
the single correct version from the five possible combinations:
A) both the statement and the explanation are true and a
causal relationship exists between them;
B) both the statement and the explanation are true but there
is no causal relationship between them;
C) the statement is true, but the explanation is false;
D) the statement is false, but the explanation itself is true;
E) both the statement and the explanation are false.
SUR-8.583.
Certain aromatic compounds, the so-called carcinogens may be also
considered as procarcinogens, because these compounds are transformed
into truly carcinogenic substances in the organism only.
CASE STUDIES
Answer the multiple task questions (simple choice and multiple choice
with/without key answers; relation analysis etc.) as they are related
to each case study!!!
SUR-8.585.
Case Study
A 22-year-old man is admitted to the hospital with injuries sustained
in an automobile accident. A lefet-sided hemothorax is detected
on the chest x-ray.
8.585/ l.
The emergency treatment is:
A) aspiration of the left pleural cavity (thoracocentesis)
B) intercostal blockade and fixation with adhesive tape
C) bilateral tube thoracostomy
D) pericardiocentesis
E) endotracheal intubation, mechanical ventilation and drainage
of the pleural cavity
8.585/2.
An x-ray taken three weeks after the injury denotes no improvement.
The next therapeutic step is:

A) a thoracotomy and evacuation of the hematoma


B) the intrapleural administration of tetracycline
C) the intrapleural injection of streptokinase
D) an exploration of the mediastinum
E) the maintenance of mechanical ventilation for another 2 weeks
SUR-8.586.
Case Study
There are three victims following a road accident; two of them have
sustained moderately severe injuries and one is severely injuried.
8.586/ l.
As the first doctor on the scene, what would you do first?
A) patients with the least severe injuries are treated first as they
have the best chance for survival
B) it is best not to'do anything until an ambulance arrives
C) you should make a report of what happened
D) you transport all three victims to the hospital with the help of
bystanders
E) provided that neither of the patients with moderate injuries is
in life-threatening condition, you start treating the patient
with the most severe injury immediately
8.586/2.
The skin of the severly injured patient is unusually pale, his breathing
is gasping and shallow, the peripheral and carotid pulses are
unpalpable. The right pupil is dilated in comparison to the left one.
Does this condition satisfy the criteria of clinical death?
A) yes, because peripheral pulses are not palpable
B) no, beacause the diameter of the pupils is different
C) yes, because muscle rigidity is also present
D) no, because respiration and circulation, though insufficient,
are detectable
E) yes, because the patient is hyperventilating
8.586/3.
Which of the following combinations represent the optimal sequence
of cardiopulmonary resuscitation?
A) the maintenance of patent airways, the administration of 100%
oxygen through a mask, external cardiac compression, drug
therapy, electric defibrillation, volume replacement
B) external cardiac compression, the maintenance of patent airways,
the administration of oxygen, drug therapy, volume replacement
C) volume replacement, external cardiac compression, drug
therapy, the maintenance of patent airways
D) the administration of 100% oxygen through a mask, the maintenance
of patent airways, external cardiac compression, drug
therapy, volume replacement
E) the maintenance of patent airways, simultaneous mechanical
ventilation and external cardiac compression, volume replacement
through a large-bore canule, the intravenous administration
of 8.4% NaHCO3, drug therapy, defibrillation depending
on the ECG findings.
8.586/4.
Which of the following agents is unsuitable in influencing the cardiac

and circulatory functions?


A) atropine
B) calcium
C) epinephrine
D) isoproterenol
E) calcium gluconate
8.586/5.
Two minutes later the heart of the patient stops functioning, the pupils
dilate progressively. Cardiopulmonary resuscitation must be
started immediately, because this progressively deteriorating condition
leads to death if left untreated.
A) both the statement and the explanation are true and a causal
relationship exists between them
B) both the statement and the explanation are correct but there
is no causal relationship between them
C) the statement is true, but the explanation is false
D) the statement is false, but the explanation itself is correct
E) both the statement and the explanation are false
8.586/6.
The detection of biologic death renders cardiopulmonary resuscitation
unnecessary, because there is no chance of restarting circulatory
and respiratory functions 4 minutes after the commencement of
clinical death.
A) both the statement and the explanation are true and a causal
relationship exists between them
B) both the statement and the explanation are true but there is
no causal relationship between them
C) the statement is true, but the explanation is false
D) the statement is false, but the explanation itself is true
E) both the statement and the explanation are false
8.586/7.
Before starting cardiopulmonary resuscitation, it must be decided
whether respiratory or cardiac damage has ensued first, because
this determines further actions as well as the sequence of measures
to be taken.
A) both the statement and the explanation are true and a causal
relationship exists between them
B) both the statement and the explanation are true but there is
no causal relationship between them
C) the statement is true, but the explanation is false
D) the statement is false, but the explanation itself is true
E) both the statement and the explanation are false
8.586/8.
The patient regains consciousness as a result of undelayed, effective
resuscitation, because unconsciousness persisting after
successful resuscitation can only be the consequence of brain
damage.
A) both the statement and the explanation are true and a causal
relationship exists between them
B) both the statement and the explanation are true but there is
no causal relationship between them
C) the statement is true, but the explanation is false

D) the statement is false, but the explanation itself is true


E) both the statement and the explanation are false
SUR-8.588.
Case Study
A victim of a road accident is brought to the emergency room. On admission,
the patient is exhausted, pale, sweating and lip cyanosis is
visible. The abdomen is distended; percussion sounds are shortened
on the left side; hepatic dullness is intact, bowel sounds are hypoactive.
Heart rate: 140/min, the pulse is thready. Blood pressure: 60/0 mmHg.
8.588/1.
Which of the following signs contradicts the possibility of circulatory
shock?
A) tachycardia
B) bradycardia
C) the abrupt onset of hypotension
D) emtpy and collapsed veins
8.588/2.
The patient in shock should be transferred to the:
A) operating theatre
B) patients' ward
C) radiology department
D) intensive care unit
8.588/3. Single Choice Question
What is the essential pathophysiology of a shock syndrome? Mark
the incorrect one among the following definitionsl
A) an acute disparity between the circulating blood volume and
the capacity of the vascular bed
B) the fall of the blood pressure and the central venous pressure
due to the reduction of the cardiac volume
C) the discrepancy between the oxygen supply and the demand of
peripheral tissues
D) an acute microcirculatory failure
8.588/4.
Select the emergency measures required for the hospital treatment
of a patient in shock!
1) the administration of laxatives and stimulants of
gastrointestinal motility
2) an analysis of the blood gases
3) a measurement of the body temperature
4) the emptying of the stomach
5) monitoring of the patient's NEUrologic status
6) monitoring of the patient's blood pressure and heart rate
7) monitoring of the patient's central venous pressure
8) securing a patent vein for volume replacement
9) measurement of the patient's height and weight
10) monitoring the diuresis
A) answers (1), (3), (4), (5) and (9) are correct
B) answers (2), (4) and (9) are correct
C) answers (3), (4) and (5) are correct
D) only answer (4) is correct

E) answers (2); (6), (7), (8) and (10) are correct


F) all the above answers are correct
8.588/5.
Which of the following findings is characteristic of circulatory
shock?
A) metabolic alkalosis
B) metabolic acidosis
C) respiratory alkalosis
D) respiratory acidosis
8.588/6.
Which of the following solutions is not used for the treatment of
shock?
A) Ringer's solution
B) dextran
C) human plasma
D) mannitol
8.588/7.
Is circulatory shock a contraindication for surgical intervention?
Mark the incorrect answer!
A) yes, because anesthetic agents would cause the collapse of the
centralized circulation of the patient
B) yes, with the exception of cases where the precipitating cause
of shock can be eliminated by surgery only
C) surgery is appropriate for all patients in shock if volume replacement
is adequate
D) all patients in shock must be operated upon if the operation is
a component of antishock therapy
SUR-8.590.
Case Study
You are called to attend to a middle-aged male patient found sitting
in the driver seat of his pickup van (6-7 minutes before your arrival).
The patient is unconscious, his skin is pale, there are no signs of
respiration or circulation, the carotid pulse is unpalpable, his pupils
are dilated. There are food remnants left in his mouth. Witnesses
inform you that the patient has had a seizure, groaned for a while,
then fell silent just before your arrival.
8.590/ 1.
The tentative diagnosis is:
A) the patient has epilepsy and he is in a postconvulsion coma
B) a cerebrovascular crisis
C) a syndrome associated with recent brain damage
D) clinical death
E) anaphylactic shock
F) the aspiration of a foreign object
8.590/2.
The immediate action to be taken is:
A) an EEG examination
B) to start volume replacement
C) to perform coniotomy
D) to perform proper cardiopulmonary resuscitation

8.590/3.
Determine the correct sequence of the phases of cardiopulmonary
resuscitation)
1) endotracheal intubation
2) external cardiac coompression
3) rescue breathing (mouth-to-mouth)
4) the removal of foreign bodies from the pharynx
5) notifying the rescue service
6) to remove the patient from the vehicle
7) a precordial thump on the chest
8) mechanical ventilation with an appropriate device
9) to set up an intravenous line for infusion
A) the correct sequence is (1), (7), and (9)
B) the correct sequence is (1), (8), (2), and (5)
C) the correct sequence is (6), (4), (3), (2), (5), (1), (8), (9), and
D) the correct sequence is (2), (3), (4), and (5)
E) the correct sequence is (7), (8), and (9)
F) only step (6) should be taken
SUR-8.591.
Case Study
A 57-year-old male patient has been experiencing low-grade fever
and an unproductive cough for the last 3 days. He presents at a
family practice complaining of circumscribed pain of several hours
duration on the left side of his chest. The pain is significantly aggravated
by breathing. Physical status: Well-built, burly male. Skin,
mucous membranes are normal, moderate hyperemia in the pharynx,
scars of a previous tonsillectomy. Auscultation is unreliable
due to the shallow respiration, however, a pleural friction rub is suspected
at the middle axillary line between the 5th and 8th ribs. Abdomen:
negative. Musculoskeletal system: intact. neurologic status:
without organic abnormalities. Heart rate: 88/min, bounding,
rhythmical. Blood pressure: 130/70 mmHg. Body temperature: 37.8
C. Therapy: 1 ampulle aminopyrine (Demalgonil) im. Thirty seconds
after the injection the patient begans to complain about weakness,
turns pale and collapses to the floor. His skin is extremely pale, lips
are cyanotic, respiration is shallow, dyspneic. The radial pulse is
unpalpable and the carotid pulse is vaguely palpated, HR: 100/min.
Somnolence ensues.
8.591/ 1.
What is your tentative diagnosis?
A) the injection was inadvertently given intravenously
B) judged by the history, a pulmonary embolism is likely
C) anaphylactic shock
D) myocardial infarction
E) the circulatory collapse is due to anemia resulting from an unknown
internal hemorrhage
8.591/2.
What are the immediate actions to be taken?
1) call the rescue service immediately then start drug therapy
2) start assisted ventilation
3) secure a vein for parenteral volume replacement

4) an ECG elevate the lower extremities of the


5) supine patient
A) the correct sequence is (5), (3), and (2)
B) the correct sequence is (1) and (4)
C) only step (1) should be taken
D) the correct sequence is (1), (2), (3), (4) and (5)
8.591/3.
Which of the following medications should be administered to the
patient?
1) epinephrine
2) pethidine
3) theophylline
4) corticosteroids
5) digitalis
6) antihistamines
A) answers (1), (4), and (6) are correct
B) answers (2) and (5) are correct
C) answers (5) and (6) are correct
D) only answer (5) is correct
E) all of the answers are correct
SUR-8.592.
Case Study
A 67-year-old male patient is found unconscious by his neighbour,
who calls the family practitioner immediately. The history of the patient
includes an ulcer disease. The skin of the patient is extremely
pale, cold and cyanotic. Skin turgor is poor, the tongue is dry. The
capillary refill, assessed by pressing the fingernails, is slow. Auscultation
reveals bronchial breath sounds. The abdomen is slightly distended,
tender. Blood pressure: 6G/40 mmHg. Heart rate: 140/min.
Respiratory rate: 40/min.
8.592/1.
What would be your tentative diagnosis?
A) pneumonia
B) hyperglycemic coma
C) shock syndrome
D) acute heart failure
E) hypoglycemic coma
8.592/2.
What is the leading sign that your diagnosis can be based on?
A) pale skin; tachypnea; bronchial breath sounds
B) bronchial breath sounds; tachycardia
C) the lack of signs indicating any external trauma
D) hypotension; tachycardia; signs of poor peripheral perfusion
E) poor skin turgor and a tender abdomen
8.592/3.
Which of the following drugs is contraindicated for the adjunctive
therapy of the diagnosed disease?
A) digitalis
B) calcium gluconate

C) 8.4% sodium bicarbonate


D). methylprednisolone
E) insulin
8.592/4. Relation Analysis
The insertion of an indwelling bladder catheter is recommended by
all means, because the measurement of diuresis is required for the
establishment of a more accurate diagnosis.
A) both the statement and the explanation are true and a causal
relationship exists between them
B) both the statement and the explanation are true but there is
no causal relationship between them
C) the statement is true, but the explanation is false
D) the statement is false, but the explanation itself is true
E) both the statement and the explanation are false
SUR-8.593.
Case Study
You are called to a road accident that occured 50 meters from your
office. A bus full of people has overturned at high-speed and landed
on its side. Fetching your emergency bag you hurry to the site and
find the following 5 victims:
1) a 50-year-old male with periorbital suffusion (raccoon's eyes);
cerebrospinal fluid leaking from his left ear; respiration is absent;
carotid pulse is not palpable
2) a 45-year-old female with an open skull fracture in the frontoparietal
region; bradypnea and stupor are present, the patient
is tossing about, touches her head freqently with her hands
3) a 30-year-old male with an open fracture of the left femur and
a greyish-pale, sweating face
4) a 50-year-old male with a large, lacerated, bone-deep wound
on the left upper arm, with profuse venous bleeding
5) a 38-year-old male with superficial abrasions all over his body
8.593/ 1.
What is your first action?
A) transport all the victims to your office with the help of bystanders
B) start emergency therapy on the site and call the rescue service
C) organize the transportation of the victims to your office
D) hurry to notify the rescue service
8.593/2.
What would be the optimal sequence of your actions?
1) victim 1: complex resuscitation
2) victim 2: assisted ventilation
3) victim 2: oro-pharyngeal toilette; suctioning of respiratory secretions
4) victim 4: control of bleeding
5) victim 3: wound care; immobilization
A) the correct sequence is (1), (2), and (3)
B) the correct sequence is (2) and (4)
C) the correct sequence is (3), (4) and (5)
D) the correct sequence is (3) and (5)
8.593/3.
Which victim needs volume replacement before transportation in an
ambulance car?

A) victim 1
B) victim 2
C) victim 5
D) victim 3
E) victims 3 and 4
8.593/4.
How would you transport victim 2?
A) in the stable lateral recumbent position
B) in semi-Fowler's position
C) in the sitting position
D) in the supine recumbent position, with the lower extremities
Elevated
SUR-.594.
Case Study
A 45-year-old female patient is suspected of having colon cancer.
8.594/ 1.
Which of the following sets of signs and symptoms is characteristic
of a neoplasm of the descending colon?
A) occult bleeding; lower abdominal pain
B) constipation alternating with diarrhea, blood and mucus containing
stools, abdominal cramps
C) fever; recurrent diarrhea accompanied by vomiting, colicky abdominal
pain
D) recurrent tenesmus; blood and mucus containing stools; the
patient complains of incomplete emptying of the rectum even
by repeated attempts
8.594/2.
Which of the following procedures is hazardous in mechanical intestinal
obstruction due to a suspected neoplasm of the colon?
A) percussion of the abdomen
B) colonoscopy
C) a GI series of x-rays
D) rectoscopy
E) a digital rectal examination
F) a barium enema
SUR-8.595.
Case Study
An 18-year-old female was involved in a head-on collision with another
vehicle while driving her car. She was using her safety belt at
the time of the accident. An 8 cm x 5 cm abrasion is visible in the
lower region of the chest, at the level of the costal margin, slightly
right to the mamillary line. Intensive care: finds the patient with pale
skin and running cold sweat. Her pulse is thready; blood pressure is
100/60 mmHg. The abdomen is soft, nontender. The patient complains
of pain in the right hypochondrium on breathing; the pain
radiates to the right shoulder. Bradycardia is present and blood
pressure falls despite parenteral volume replacement.
8.595/1.
What is your tentative diagnosis?
A) contusion of the abdominal wall

B) a retroperitoneal hematoma associated with contusion of the


right kidney
C) rupture of the liver
D) rupture of the diaphragm
8.595/2.
Which of the following is the most objective procedure for the verification
of the diagnosis?
A) a plain chest x-ray
B) a plain abdominal x-ray
C) an upper GI series performed with water-soluble contrast material
D) urinalysis
E) diagnostic peritoneal lavage
8.595/3.
What is the therapy of choice?
A) a transfusion
B) close observation with a CBC, urinalysis and radiography repeated
hourly; BP, HR and WBC count checked every 15 minutes
C) a laparotomy; surgical control of any bleeding
SUR-8.596.
Case Study
A 35-year-old female presents with rectal bleeding and mucous discharge
from the anus associated with tenesmus.
Rectosigmoidoscopy reveals a neoplasm that bleeds easily, located
14 centimetres from the dentate line. A biopsy is performed.
8.596/1.
Histology would suggest the following:
A) a juvenile polyp
B) an adenomatous polyp
C) a villous adenoma
D) a pseudopolyp associated with ulcerative colitis
E) endometriosis
8.596/2.
These neopolasms are extremely prevalent in the:
A) appendix
B) cecum
C) transverse colon
D) rectum and sigmoid colon
E) anal canal
8.596/3.
The treatment of choice if the lesion proves to be malignant is:
A) fulguration
B) local excision
C) anterior resection
D) abdominoperineal amputation of the rectum
E) excision by colonoscopy
SUR-8.597.
Case Study
A 26-year-old male experiences a sudden stabbing pain in the left
side of his chest, associated with a hacking cough and dyspnea. The

patient presents at the emergency room of the nearby hospital. Plain


chest roentgenograms reveal the collapse of the left lung to the size
of a fist and some pleural fluid in the sinus. The right lung is normal.
The midline position of the mediastinum is retained. ECG: HR:
110/min, sinus rhythm with occasional supraventricular
extrasystoles.
8.597/ 1.
The most likely diagnosis is:
A) exudative pleurisy
B) a small anteroseptal myocardial infarction
C) severe pneumonia involving the whole left lung with concomitant
inflammatory effusion
D) a pulmonary embolism in the left lung
E) pneumothorax
8.597/2.
The treatment of choice, based on the correct diagnosis is:
A) administration of antibiotic combinations; antitussives; water
or steam inhalation; bed rest
B) strict bed rest; continuous monitoring of cardiovascular functions
in the intensive care unit; digitalis therapy; administration
of antiarrhythmic drugs
C) tube thoracostomy and permanent suctioning
D) intermittent positive end-pressure ventilation (IPPB) with a
pressure-controlled ventilator until the resolution of the
radiologic and ECG abnormalities
E) oxygen administration and prompt preparation for an emergency
thoracotomy
F) acute selective coronary angiography and intracoronary
thrombolysis with streptokinase
G) intravenous thrombolysis in order to dissolve any pulmonary
Emboli
8.597/3.
The condition recurred 2 months later with similar symptoms and
was treated succesfully. Another recurrence ensued in the 5th
month following the first admission of the patient. Similar radiographic
and ECG changes are detected on the present admission.
The treatment of choice is:
A) tube thoracostomy to achieve the expansion of the lung then
thoracotomy and definitive surgical therapy
B) prompt introduction of intravenous heparin therapy, then the
gradual transfer of the patient to permanent warfarin therapy
to prevent the occurrence of
C) puncture of the pleural cavity at the typical site using a smallbore
needle, further relapses aspiration of the air; administration
of antitussives; bed rest
D) emergency admission to the coronary care unit and prompt
initiation of intracoronary thrombolysic therapy (within 2
hours)
E) antibiotic therapy guided by cultures and susceptibility testing
from the sputum and pleural fluid, until bacteriology tests
yield negative results
8.597/4.

Which of the following would you recommend to the patient on discharge?


1) a pulmonary follow-up is necessary because this condition is
associated with an increased incidence of lung cancer
2) cardiology care and follow-up is necessary, because relapses
may occur and left ventricular aneurysm may develop
3) considering the frequent recurrences of his condition, the patient
should apply for a disability pension
4) a pulmonary follow-up is required, because this condition may
develop in the other lung. as well
5) the patient should stop smoking, because this was the precipitating
factor of his condition
6) the patient may return to his previous job if follow-up studies
performed 6 weeks later are negative
7) the patient is discharged in full health and is fit for work immediately
8) it is advisable to stop smoking because it predisposes to further
recurrences of this condition, although smoking is not a
direct etiologic factor
A) answers (1), (2), and (3) are correct
B) answers (2), (4), and (7) are correct
C) answers (3), (5), and (8) are correct
D) answers (4), (6), and (8) are correct
E) answers (5) and (7) are correct
F) only answer (5) is correct
G) all the answers are correct
SUR-8.598.
Case Study
A 68-year-old male patient has been treated in the hospital recently
for a myocardial infarction. He starts complaining about a sudden,
colicky pain felt in the periumbilical and epigastric region. His skin
is greyish pale and is covered with sweat; the patient is in apparent
distress. An hour later the pain subsides. Then the abdominal pain
returns in the right lower quadrant and distension ensues, bowel
sounds are absent. The condition culminates in the development of
shock.
8.598/ 1.
The most likely diagnosis is:
A) acute cholecystitis
B) occlusion of the superior mesenteric artery due to embolism
C) diverticulosis
D) carcinoma of the transverse colon
E) none of the above
8.598/2.
Based on the correct diagnosis, the characteristic finding in the peripheral
blood count is:
A) early leukopenia
B) leukocytosis
C) a significant rise of the eosinophil cell count
D) lymphocytosis
E) none of the above
8.598/3.
Which of the following diagnostic procedures is appropriate for the

verification of the tentative diagnosis?


A) plain abdominal radiography
B) barium enema
C) selective angiography of the mesenteric artery
D) abdominal ultra-sound
E) none of the above
8.598/4.
Therapy may include:
A) early exploration and embolectomy
B) an emergency cholecystectomy
C) colostomy of the transverse colon
D) observation for at least 6 hours before deciding about the indication
for surgery
E) none of the above
SUR-8.599.
Case Study
A 36-year-old male patient was found in his home in an extremely
prostrated condition by his relatives. The patient had been lying on
the floor conscious and complained that he felt weak. Half an hour
earlier he had vomited about 3 decilitres of fresh blood. He was
transported to the medical emergency room of the local hospital by
an ambulance. Medical history: the patient disclosed that he had
been consuming ardent spirits regularly for 20 years. He had endured
an infectious disease associated with jaundice. He had been
experiencing heartburn, epigastric pain regularly in the autumn and
in the spring the last several years, but did not seek medical advice.
Physical status: Pale, clammy skin, a slight jaundice is visible in the
sclera. Blood pressure: 90 mmHg, heart-rate: 120/min; the peripheral
veins are empty. The patient passed liquid stool with large volumes
of fresh blood.
8.599/ 1.
What is your first action in the emergency room of the hospital?
A) to secure a blood sample for blood-grouping
B) to administer plasma expanders
C) to administer pethidine
D) to perform an emergency gastroscopy
E) to order a barium meal study
8.599/2.
Provided that the hemorrhage is intense and shock develops, is it
mandatory to wait for the official blood-grouping results reported by
the local blood bank?
A) yes
B) no
8.599/3.
How can a blood transfusion be performed without the results of the
official blood-grouping?
A) the attending physician determines the ABO group of the patient
himself
B) the attending physician determines the ABO and Rh group of
the patient himself
C) type 0 Rh-negative blood is transfused immediately

D) two properly qualified and authorized persons determine the


ABO and Rh-group of the patient independently
8.599/4.
Determine the correct sequence of measures to be taken during the
emergency therapy of the patient with upper gastrointestinal bleeding!
1) angipgraphy
2) gastroscopy
3) insertion of a nasogastric tube
4) antishock therapy
5) blood-grouping
6) barium meal
A) the correct sequence is (1), (2), (3), (4), (5), and (6)
B) the correct sequence is (5), (4), (3), and (2)
C) the correct sequence is (4), (6), (3), and (1)
D) the correct sequence is (6), (5), (3), and (4)
8.599/5.
All of the following physical signs help the examiner to establish or
verify the correct diagnosis, EXCEPT:
A) low-grade jaundice
B) spider angiomas
C) caput medusae
D) ascites
E) palmar erythema
8.599/6.
Which of the following is the most likely source of the patient's
hemorrhage?
1) Barrett's ulcer
2) Mallory-Weiss syndrome
3) Peutz-Jeghers syndrome
4) nasal polyp
5) portal hypertension
6) duodenal ulcer
7) jejunal angiodysplasia
A) answers (2) and (7) are correct
B) answers (1), (3), and (4) are correct
C) answers (2) and (5) are correct
D) answers (1), (2), (4), and (7) are correct
E) answers (5) and (6) are correct
F) answers (6) and (7) are correct
G) answers (1) and (7) are correct
H) answers (3) and (4) are correct
8.599/7.
Is the rupture of esophageal varices obligatory in all patients with
liver cirrhosis?
A) yes
B) no
8.599/8.
Considering the history of the patient, which type of endoscopy
would you prefer?

A) esophagoscopy
B) esophago-gastroscopy
C) colonoscopy
D) bronchoscopy
8.599/9.
Supposing that the source of the hemorrhage is identified as a ruptured
subcardial varix, which of the following procedures would you consider?
1)tamponade by inserting a Sengstaken-Blakemore tube
2) an infusion of vasopressin
3) a transfusion
4) sclerotherapy through a rigid endoscope
5) laser-coagulation
6) percutaneous transhepatic embolization
7) transection of the esophagus
8) emergency bypass-surgery
9) the administration of B-blockers
10) total gastrectomy
A) answers (5), (8), and (10) are correct
B) answers (3), (5), (7), and (8) are correct
C) answers (2), (4), (6), and (10) are correct
D) answers (1) and (3) are correct
E) answers (2), (3), (8), and (9) are correct
F) answers (4), (7), and (10) are correct
G) answers (1), (2), and (3) are correct
H) answers (3), (5), and (9) are correct
8.599/10.
Which therapeutic modalities could be considered if the combined
conservative treatment is unsuccesful?
1) percutaneous transhepatic embolization
2) emergency bypass surgery
3) a total gastrectomy
4) sclerotherapy of the esophageal varices
5) ligation of the vena cava
6) axillofemoral bypass surgery
7) none of the above
A) answers (1), (2), and (4) are correct
B) answers (2), (3), (5), and (6) are correct
C) answers (1), (3) and (5) are correct
D) answers (3) and (4) are correct
E) answers (5) and (6) are correct
F) only answer (7) is correct
8.599/11.
Which classification is used to categorize patients with liver cirrhosis?
A) Dukes' classification
B) Child's classification
C) TNM-classification
8.599/12.
How do the operations performed for portal hypertension influence
the spontaneous clinical course of the disease?
A) bleeding is controlled but disease progression is enhanced

B) bleeding is controlled and disease progression is left unchanged


C) the progress of the disease is retarded
SUR-8.600.
Case Study
A 52-year-old obese female patient presents with excruciating pain
in her left lower extremity, which started about 6 hours earlier. The
limb has become swollen, cold and of a bluish-white color. Initial
numbness has been replaced by virtual total immobility. Physical
examination: peripheral pulses are barely palpable on the significantly
swollen lower extremity. The history is negative for heart disease.
The patient has sustained a fracture of the contralateral leg in
an automobile accident; she is still wearing the plaster cast. The latter
made her mobilization extremely difficult. She has varicose veins
on both lower extremities and has had recurrent phlebitis previously.
8.600/ 1.
The initial physical examination would include all of the following,
EXCEPT:
A) an inspection
B) measurement of the blood pressure
C) palpation of the peripheral pulses
D) measurement of the limb circumference
E) a bimanual pelvic examination
F) measurement of leg temperature
G) auscultation of the abdomen
H) auscultation of the heart
8.600/2.
Considering the above, the tentative diagnosis is:
A) occlusion of the femoral artery due to an embolism
B) superficial phlebitis
C) phlegmasia cerulea dolens
D) aortic dissection
E) arterial thrombosis
8.600/3.
What is the most important factor for the establishment of the diagnosis?
A) arteriography
B) phlebography
C) isotope studies
D) plethysmo-dynamometry
E) Doppler-examination
F) inspection + physical examination
8.600/4.
What is the first action to be taken after the establishment of the
diagnosis?
A) ultrasonography
B) intravenous heparin therapy
C) phlebography
D) arteriography
E) urgent referral to the hospital
8.600/5.
All the following are essential principles of conservative therapy,

EXCEPT:
A) bed rest
B) correction of fluid imbalances
C) early mobilization using elastic bandages
D) heparin administration or fibrinolytic therapy
8.600/6.
Inadequate treatment of phlegmasia alba dolens or cerulea dolens
results in which of the following early complications?
A) heart failure
B) gangrene of the limb
C) renal thrombosis
D) pulmonary embolism
E) the development of a postthrombotic syndrome
8.600/7.
Is surgical treatment recommended in thrombosis of the iliofemoral vein?
A) by all means, if conservative therapy is unsuccesful
B) no, this condition should be treated conservatively only
C) all cases of this condition should be operated upon
8.600/8.
The principle of the surgical treatment for this noted conditions is:
A) insertion of a Mobin-Uddin filter into the inferior vena cava
B) ligation of the vena cava
C) ligation of the saphenous vein
D) venous thrombectomy
E) thrombendarterectomy
8.600/9.
The principle of long-term treatment following a successful initial
therapy is:
A) physical therapy
B) antibiotic therapy
C) permanent anticoagulant therapy
D) bedrest and abstinence from exercise
E) the use of elastic bandages
SUR-8.601.
Case Study
A 64-year-old female patient complains about colicky abdominal
pain experienced for the last two days. Nausea and vomiting have
also occurred. Her history reveals constipation alternating with
diarrhea. The patient has passed bloody-mucoid stool on one occasion.
8.601 / 1
Which of the following questions facilitates the establishment of the
diagnosis?
1) did the patient undergo a hemorrhoidectomy?
2) is flatus passed?
3) did the patient receive anticoagulant therapy?
4) in which region of the abdomen do the cramps develop?
5) what kind of food precipitates the occurrence of the abdominal
cramps?
A) answers (1), (3), and (5) are correct

B) answers (3) and (4) are correct


C) answers (1), (3), and (4) are correct
D) answers (2), (4), and (5) are correct
E) all of the answers are correct
8.601/2.
Which of the following abnormal findings are present on a physical
examination of the abdomen?
1) Blumberg's sign
2) hyperactive bowel sounds
3) silent abdomen
4) a succusion splash is elicited
5) "frame meteorism" is present
8.601/3.
All the following studies aid in establishing the diagnosis, EXCEPT:
A) rectal digital examination
B) emergency rectoscopy and barium enema
C) plain abdominal radiography
D) insertion of a nasogastric tube and the examination of
gastrointestinal secretions
E) selective angiography
8.601/4.
What kind of radiologic abnormality is present in this case?
A) free air under the diaphragm
B) a reduced intestinal gas content; fluid levels are absent
C) the distended colon is visible with wide fluid levels in its lumen,
accompanied by several fluid levels in the small intestine
D) the colonic gas content is reduced and several fluid levels are
visible in the small intestine, in the right half of the abdomen
8.601/5.
Which of the following therapeutic mesaures do you consider adequate?
1) insertion of a nasogastric tube; parenteral volume replacement
2) the administration of an enema to restart intestinal passage
3) per oral administration of two tablespoonfuls of magnesium
sulphate
4) pharmacologic stimulation of intestinal motility (trifluperidol,
ubretide, caffeine)
5) per oral administration of liquid paraffine to soften any colonic
contents
A) answers (1), (3), and (4) are correct
B) answers (2), (3), and (5) are correct
C) answers (1) and (2) are correct
D) answers (4) and (5) are correct
E) all of the answers are correct
8.601/6.
What is the most prevalent site of colorectal carcinomas?
A) 73% of these neoplasms develop in the sigmoid colon or the rectum
B) colon tumors develop most frequently in the cecum
C) these tumors develop in right colon more often than in the left
colon
D) rectal localization is the least frequent occurrence

8.601/7.
In terms of oncology, which of the following operations is the most appropriate
for the treatment of colonic obstruction due to sigmoid
cancer?
A) cecostomy
B) colostomy of the transverse colon
C) Hartman's operation
D) Paul-Mikulitz operation
E) Ferguson's operation
8.601/8.
Which antibiotic combination is optimal for the therapy of peritonitis
discovered at the time of surgical exploration for colonic obstruction?
A) penicillin+streptomycin
B) cephalothin+gentamycin+metronidazole
C) chloramphenicol+tetracycline+sulfonamide
D) neomycin+5-fluorocytosin
SUR-8.603.
Case Study
A 56-year-old obese female patient is referred to the surgical emergency
room with intense colicky pain in the upper region of the abdomen. The
complaints had started 2 days ago following a fatty meal. Circumferentially
radiating epigastric pain started first, followed by nausea and
vomiting.
The patient experienced chills but did not measure her body temperature.
Her complaints became permanent, abdominal distension ensued
and neither flatus nor stools were passed. She experienced similar
though less severe complaints earlier, relieved by diet and spasmolytics.
Permanent constipation is an associated symptom. The patient had surgery
for an ectopic pregnancy 20 years ago. Physical status: the face is
strikingly flushed, the tongue is dry and covered by white coating. The
sclerae are slightly jaundiced. The heart is normal, slightly reduced
breath sounds are audible over the base of the lung. The abdomen is distended,
moderately meteoristic. Bowel sounds are absent, a succusion splash
cannot be elicited. Definite tenderness is detected under the right costal
margin and in the epigastrium, above the umbilicus. A vaguely palpable
mass is suspected in the right mamillary line. Rectal examination: dilated
rectal ampule containing some feces; the cul-de-sac is normal.
8.603/ 1.
Considering the history and the results of physical examination,
which of the following conditions is associated with similar signs?
1) acute cholecystitis
2) hydrops of the gall bladder
3) mechanical intestinal obstruction
4) duodenal perforation
5) acute pancreatitis
A) answers (1), (2), and (5) are correct
B) answers (2) and (3) are correct
C) answers (2), (3), and (4) are correct
D) answers (1) and (5) are correct
E) only answer (4) is correct
F) all the answers are correct

8.603/2.
Which of the following pathognomonic radiologic alterations could
be detected in this case?
A) left lobar pneumonia
B) Fleischner-atelectasis in the left lower lobe
C) elevation of the diaphragm
D) a small pleural effusion in the left costodiaphragmatic sinus
E) fibrous thickening of the pleura
8.603/3.
What is the treatment of choice?
A) closure of the duodenal perforation
B) a cholecystostomy
C) abolishing the cause of mechanical intestinal obstruction
D) removal of the inflamed, calculous gall bladder
E) active conservative therapy for pancreatitis
8.603/4.
Supposing that the complaints of the patient are due to acute
cholecystitits and concomitant pancreatitis, which of the following
complications is unlikely to occur?
A) obstructive jaundice
B) pancreatic pseudocyst
C) purulent cholangiohepatitis
D) hemobilia
E) acute, hemorrhagic pancreatitis
SUR-8.605.
Case Study
A 66-year-old male patient has been experiencing periodic abdominal
pain for 10 months. He has lost 10 kg of bodyweight, sought
medical advice on several occasions. The tentative diagnosis was
always intestinal obstruction (incomplete ileus) that resolved spontaneously.
Previous laboratory results: moderate anemia, ESR: 80
mm/hour. Chest radiography: normal. Barium enema: normal on
two occasions. Oral cholecystography: the gall bladder could not be
visualized. Ultrasonography: echodense shadow in the region of the
gall bladder. (The original roentgenograms are unavailable, you have
to rely on the descriptions.)
8.605/1.
You are the attending surgeon of the patient. What action do you
take?
1) perform a cholecystectomy for cholelithiasis
2) transfer the patient to the department of internal medicine for
further diagnostic testing
3) keep the patient under observation and arrange a barium enema
4) repeat the cholecystography with the intravenous administration
of contrast material
5) arrange an abdominal angiography
A) answers (1), 3), and (5) are correct
B) answers (3) and (4) are correct
C) answers (4) and (5) are correct
D) only answer (4) is correct

E) all of the answers are correct


8.605/2.
The patient defers colonoscopy. You subsequently witness the development
of abdominal cramps and peristaltic waves visible through the
abdominal wall. On inquiry, the patient discloses that he has been
experiencing periodic constipation lasting for days. This condition
sometimes resolves spontaneously. However, on most occasions he
has had to take a laxative to pass a stool.
What is the most likely diagnosis (in addition to verified
cholelithiasis)?
1) Crohn's disease
2) porphyria
3) megacolon
4) neoplasm of the colon
5) diverticulitis of the sigmoid colon
A) answers (1), (2), and (3) are correct
B) answers (2), (3), and (5) are correct
C) answers (1) and (4) are correct
D) only answer (5) is correct
E) all of the answers are correct
8.605/3.
A barium enema is repeated and does not disclose any abnormal
conditions. Which of the following tests is mandatory before performing
a cholecystectomy?
A) a determination of the urinary and serum porphyrin levels
B) liver scintigraphy
C) rectoscopy
D) intravenous urography
E) thyroid function tests
8.605/4.
No abnormality is found on rectoscopy. What is your next action?
A) recommend conservative therapy and discharge the patient
B) try to establish a more accurate diagnosis by ordering a GI series
C) perform cholecystectomy
D) continue observation
8.605/5.
A shrunken gall bladder filled with stones is found at surgery. What
is your next action?
A) to remove the gall bladder
B) to explore the abdominal cavity thoroughly in addition to performing
a cholecystectomy
C) to take full thickness biopsies from the small and large intestines
D) to perform a cholecystectomy with an appendectomy
E) to extract the stones and perform a cholecystoduodenostomy
8.605/6.
In addition to cholelithiasis, a small colon neoplasm is detected at
the hepatic flexure. The colon is undilated proximal to the lesion.
What is your next action?
A) to perform an ileotransversostomy

B) to perform a hemicolectomy
C) to perform a cholecystectomy and prepare the patient for interval
colon resection
D) to do nothing apart from closing the abdominal incision
E) to perform a cholecystectomy and double-barrel
sigmoidostomy
8.605/7.
In addition to cholelithiasis the above neoplasm is detected. However,
the small intestines - particularly the ileum - are dilated to 6-8
cm diameter. The cecum and the ascending colon are extremely distended
and dilated to 12 cm. The colonic segment distal to the lesion
is empty and collapsed. No metastases are detected. What is the procedure
of choice?
A) cholecystectomy
B) cholecystectomy +cecostomy
C) ileotransversostomy
D) right hemicolectomy
E) cholecystectomy + ileotransversostomy
SUR-8.606.
Case Study
A 28-year-old female patient sustained a corrosive injury to the
esophagus as a child after the ingestion of an alkalic substance, subsequently,
the resulting stricture was dilated on several occasions.
She had giblets soup for supper and felt that a piece of gizzard got
stuck in her esophagus. The patients presents at 9 pm to the ENT
department. A barium meal revealed complete obstruction of the lower
third of the esophagus. The physician on duty performs esophagoscopy
with a rigid instrument. The foodpiece causing the obstruction is visualized
in the strictured esophagus, at 32 cm distance from the upper
teeth. Removal of the blockage is attempted using a foreign body forceps,
but brisk bleeding results. The procedure is abandoned immediately
and the patient is admitted to the department for observation.
Two hours later she begins to complain about a retrosternal pain, then
high-grade fever (38.2 C) develops.
8.606/ 1.
What should be done?
A) the patient should be transferred to a surgical department
B) endoscopy should be repeated using a rigid esophagoscope
C) endoscopy should be repeated using a flexible esophagoscope
D) antibiotic therapy should be started and close observation
continued
E) fluoroscopy should be performed with a water-soluble contrast
Material
8.606/2.
What is the most appropriate action in the case of verified
esophageal perforation?
A) the insertion of a nasogastric tube; antibiotic therapy; surgical
observation
B) antibiotic therapy; observation and surgery if the condition of
the patient deteriorates
C) emergency surgery at a surgical department

D) diagnostic thoracocentesis
8.606/3.
What is the surgical therapy of an esophageal perforation with the
features detailed above?
A) tube thoracostomy of the right pleural cavity + gastrostomy
B) right thoracotomy; resection of the espohagus;
esophagogastrostomy
C) right collar mediastinotomy + drainage
D) gastrostomy or feeding enterostomy
E) right thoracotomy; repair of the esophagus; drainage; feeding
enterostomy
8.606/4.
How could the risk of perforation be reduced?
1) by preventive antibiotic therapy
2) by performing esophagoscopy under general anesthesia and
neuromuscular blockade
3) by using a flexible endoscope
4) by postponing the procedure to the next day
5) by the insertion of a Boas tube before esophagoscopy
A) answers (1), (4), and (5) are correct
B) answers (2) and (3) are correct
C) only answer (4) is correct
D) answers (4) and (5) are correct
E) all the answers are correct
SUR-8.607.
Case Study
A 67-year-old male patient is referred to the hospital with abdominal
pain, (predominantly lower abdominal pain) and fever of several days'
duration. On the day before his admission, as well as the preceding
day, he had shaking chills followed by 38.7 C and 39.2 C fever respectively.
The patient is anorexic and has not passed stools for 3
days. Physical examination: a tender mass is detected in the left lower
abdominal quadrant with slight regional rigidity of the abdominal wall.
8.607/1.
Which of the following questions is relevant?
1) Did the patient ever have pulmonary tuberculosis?
2) Did the patient pass bloody stools?
3) Did the patient have appendicitis?
4) Is the patient a diabetic?
5) Has the patient ever been treated in a hospital?
A) answers (2), (4), and (5) are correct
B) answers (2) and (3) are correct
C) answers (3) and (5) are correct
D) only answer (4) is correct
E) all of the answers are correct
8.607/2.
Supposing that the patient answered the above questions as follows,
that is 1: no, 2: no, 3: no, 4: yes, 5: no. Which of the following is the
most likely diagnosis?
1) acute appendicitis

2) acute suppurative nephritis


3) perforated diverticulitis of the sigmoid colon
4) left tuboovarian abscess
5) tuberculous abscess
A) answers (2), (4), and (5) are correct
B) only answer (3) is correct
C) answers (3) and (5) are correct
D) only answer (4) is correct
E) all of the answers are correct
8.607/3.
Which of the following diagnostic tests is appropriate to verify the
diagnosis?
1) intravenous urography
2) plain abdominal radiography
3) a complete blood count
4) abdominal ultrasonography
5) colonoscopy
A) answers (1), (3), and (5) are correct
B) answers (1), (2), and (4) are correct
C) answers (2) and (4) are correct
D) answers (2) and (5) are correct
E) answers (2) and (3) are correct
SUR-8.608.
Case Study
A 65-year-old male patient has a 2-month history of slight perianal
pain associated with the passage of bloody stools, periodic constipation
and diarrhea. He has not had fever; his appetite and bodyweight
are unchanged.
8.608/1.
Considering the history, which of the following conditions is most
likely?
A) internal and external hemorrhoids
B) Crohn's disease
C) neoplasm of the rectum
D) anal fissure
E) anorectal abscess
8.608/2.
Which of the following can be found on a digital rectal examination?
A) hyperemic, inflamed hemorrhoids
B) an inflamed, fluctuating mass adjacent to the anal orifice
C) chronic outer fistulas
D) dehiscence of the anal mucosa in the posterior commissure
E) a mass bulging into the rectal lumen can be palpated
8.608/3.
Which of the following diagnostic studies is indicated?
A) a GI series of x-rays
B) rectoscopy and barium enema
C) fistulography
D) ultrasonography

E) diagnostic puncture
8.608/4.
Laboratory tests reveal:
A) no significant abnormalities
B) eosinophilia
C) leukocytosis with a "shift to the left"
D) anemia
E) an elevated serum creatinine level
8.608/5.
The surgical therapy of choice is:
A) a right hemicolectomy with ileotransversostomy
B) a hemorrhoidectomy according to Milligan-Morgan or Parks
C) a Rcamier -dilation
D) incision and drainage
E) an abdominoperineal resection of the rectum
SUR-8.610.
Case Study
A 40-year-old male patient has been experiencing epigastric pain for
4 months,which has been associated with anorexia and disgust of
meat. Subsequently, he has lost 5 kg of weight. Further, a progressive
dysphagia has been developing over the last 2 weeks. On admission:
the patient is capable to swallow liquids only. Collar lymph
nodes are not palpable and his abdominal status is normal.
8.610/1.
What is the most likely diagnosis?
A) pyloric stenosis
B) carcinoma of the antral region
C) carcinoma of the cardia
8.610/2.
Which of the following diagnostic studies need to be performed?
1) barium meal
2) gastroscopy
3) biopsy
4) intravenous urography
5) diagnostic thoracocentesis
6) laparoscopy
7) selective angiography of the celiac axis
A) answers (1), (2), and (3) are correct
B) answers (2), (4), and (7) are correct
C) answers (4), (5), (6), and (7) are correct
D) only answer (6) is correct
E) all of the answers are correct
8.610/3.
Which of the following procedures would decide the operability of
the lesion?
A) liver scintigraphy
B) laparoscopy
C) gastroscopy
D) barium meal

E) a general assessment of the patient's condition


F) surgical exploration
8.610/4.
Is there any difference between technical and oncologic inoperability?
A) yes
B) no
C) there is no relevance
8.610/5.
Which of the following operations should be performed if the lesion
is operable?
1) total gastrectomy
2) proximal subtotal gastrectomy
3) gastrostomy
4) Billroth II resection
5) truncal vagotomy
A) answers (3), (4), and (5) are correct
B) answers (2) and (4) are correct
C) answers (1) and (2) are correct
D) only answer (3) is correct
E) all of the answers are correct
SUR-8.611.
Case Study
A 63-year-old female patient presents at the emergency room during
duty hours. She complains of an inability to pass stools for the past 3
days and significant abdominal distension and pain in the left half of
the abdomen. Her bowel habits have changed considerably over the
last 1-2 months plus hiccups have started 4 hours before her admission;
at present she is feeling feeble. Physical status: soft, palpable
abdomen, regional tenderness to the right of the umbilicus with local
muscular guarding. Rectal digital examination: the right wall of the
cul-de-sac is tender on palpation. Body temperature: 38.5 C, WBC:
15 G/1, urinalysis: negative.
8.611/1.
Which of the following is the most likely tentative diagnosis?
A) acute appendicitis
B) acute pancreatitis
C) perforation of the ascending colon
D) mesenteric lymphadenitis
8.611/2.
Which of the following procedures aids in establishing the correct
diagnosis?
A) plain abdominal radiography (detection of free air)
B) a gynecologic examination
C) an emergency barium enema
D) a CT-scan
8.611/3.
What kind of treatment would you recommend?
A) closer observation
B) vigorous parenteral volume replacement and the application of

ice packs on the abdomen


C) emergency surgical exploration
D) intensive antibiotic therapy
SUR-8.613.
The patient is a 66-year-old female in good general condition. She has
never been treated in a hospital, does not smoke and drinks alcohol
only occasionally. She has had no diseases other than common colds.
About a year ago, she has begun to experience moderate, colicky, right
subcostal pain related to fatty meals. These colicky periods were associated
with bilious vomiting but resolved spontaneously. For the last 2
months, she felt her bowels more distended. Constipation, a permanent
feature of her bowel habits has been alternating with mucous,
odoriferous, occassionally bloody diarrhea. Fever or subfebrility did
not occur. Despite her good appetite and unchanged bodyweight she
feels weak. Diagnostic studies: performed at the outpatient clinic revealed
cholelithiasis with several stones in the gall bladder. The results
of the laboratory studies were normal.
8.613/ 1.
Are the diagnostic studies performed so far sufficient for the establishment
of the diagnosis?
A) yes, because the studies verified suspected cholelithiasis
B) no, stool culturing should have been done because there is
mucous-bloody diarrhea in the history
C) no, because there is a strong suspicion of colorectal malignancy
and this can be verified by further investigations only
8.613/2.
Analyzing the information available, the most likely diagnosis is:
A) cholelithiasis + chronic colitis
B) cholelithiasis + amebic dysentery
C) cholelithiasis + chronic pancreatitis
D) cholelithiasis + colorectal neoplasm causing partial obstruction
8.613/3.
Which of the following diagnostic studies should be performed (in
addition to physical examination)?
1) ERCP
2) barium meal
3) barium enema
4) ultrasonography
5) rectoscopy, colonoscopy
A) answers (1), (3), and (4) are correct
B) answers (2) and (4) are correct
C) answers (3), (4), and (5) are correct
D) only answer (2) is correct
E) all of the answers are correct
8.613/4.
In addition to cholelithiasis, the diagnostic studies detected a neoplasm
in the rectosigmoidal junction. What is the treatment of choice?
A) surgical treatment of the symptomatic cholelithiasis; decompression
colostomy and interval resection of the rectosigmoidal
tumor 1-2 months later

B) surgical treatment of the rectosigmoidal neoplasm;


cholecystectomy should be postponed
C) simple complete excision of the rectosigmoidal neoplasm with
cholecystectomy if the latter is not contraindicated by the patient's
limited tolerance to surgical stress
SUR-8.614.
Case Study
A 65-year-old male patient presents with a small white spot detected
in the anterolateral region of his tongue. He is a pipe smoker since
early adulthood. The lesion was painless for a month, than gradually
became tender due to exulceration.
8.614/1.
What is the most likely diagnosis?
A) benign aspecific ulcer
B) benign leukoplakia
C) epulis
D) lingual cancer
E) lingual ranula
8.614/2.
Which of the following diagnostic tests is appropriate for the verification
of the tentative diagnosis?
A) a positive Wassermann-test
B) a positive LE phenomenon
C) a biopsy of the lesion
D) radiography of the mandible and the maxilla
E) close observation for any progression
8.614/3.
The most common sequel of this condition is:
A) metastatic spread to the supraclavicular lymph nodes
B) propagation to the collar lymph nodes
C) metastatic spread to the internal mammary nodes .
D) metastatic spread occurs in the terminal phase only
E) metastatic spread never occurs
8.614/4.
The treatment of this patient includes:
A) local radioisotope implantation
B) high-voltage radiotherapy
C) radical oral and cervical dissection
D) therapy is futile if propagation has reached the cervical lymph
nodes
E) surgical therapy and irradiation
SUR-8.615.
Case Study
A 45-year-old male patient complains about aching pains in his left
upper arm which are aggravated by active movements of the extremity.
Any prolonged usage of her arm result in dizziness and
fainting. On examination: the volume and amplitude of the peripheral
pulses are different in the upper extremities and the systemic
blood pressure is lower in the right arm.

8.615/1.
The most likely diagnosis is:
A) angina pectoris
B) subclavian steal syndrome
C) congenital abnormality of the radial artery
D) thoracic outlet syndrome
E) brachial neuralgia
8.615/2.
The pathology of this condition includes:
A) coronary arteriosclerosis
B) occlusion of the left subclavian artery proximal to the origin of
the vertebral artery
C) hypoplasia of the left radial artery
D) cervical rib
E) prolapse of a cervical intervertebral disc
8.615/3.
The most valuable procedure for the establishment of the diagnosis is:
A) carotid angiography
B) angiography of the aortic arch
C) angiography of the brachial artery
D) a CT-scan of the brain
E) radiography of the cervical spine
SUR-8.616.
Case Study
A middle-aged patient with known hypertension presents with sudden,
excruciating back pain, felt between the scapulae. The pain
does not radiate to the collar region or to the arms. Physical examination:
reveals a systolic rumble over the aorta, radiating towards
the large vessels. The blood pressures are different in the upper extremities.
8.616/ 1.
The tentative diagnosis is:
A) myocardial infarction
B) angina pectoris
C) coarctation of the aorta
D) acute prolapse of an intervertebral disc
E) dissecting aneurysm
8.616/2.
The most valuable diagnostic test would be:
A) ECG
B) exercise tolerance testing
C) esophagogram
D) radiography of the thoracic vertebrae
E) plain chest x-ray
8.616/3.
The most characteristic diagnostic sign would be:
A) Q-waves on the ECG
B) ST-depression in the stress ECG
C) stricture of the lower esophagus on the esophagogram
D) narrowing of the intervertebral space on the x-ray
E) widening of the mediastinum, calcification of the aortic wall

and pleural fluid on the left side in the chest x-ray


8.616/4.
The therapy of choice would be:
A) bed rest; administration of oxygen and digoxin
B) aortocoronary bypass grafting
C) administration of antacids
D) laminectomy
E) reduction of blood pressure; surgery of the dissecting aneurysm
if blood pressure reduction is unsuccessful
SUR-8.617.
Case Study
A 3 week-old infant is admitted because of recurrent projectile vomiting
occurring after feeding since the age of 2 weeks. The weight of
the neonate decreased despite his good appetite.
8.617/ 1.
The most likely diagnosis is:
A) neonatal peptic ulcer
B) tracheoesophageal fistula
C) hypertrophic pyloric stenosis
D) leukemia
E) esophageal atresia
8.617/2.
A common physical symptom of this condition is:
A) a scaphoid (concave) abdomen
B) visible peristalsis
C) a "tumor" with a size of an olive is palpable through the abdominal
wall
D) rales are audible over the lung
E) a distended, meteoristic abdomen
8.617/3.
Which of the following diagnostic signs is important to verify the diagnosis?
A) bilious vomiting
B) gastric dilation detected by radiography
C) Faber-test for the detection of meconium
D) alkalosis
E) a full-thickness biopsy of the gastric wall
SUR-8.618.
Case Study
A 3-year-old child is investigated for constipation present since
birth. The child experiences frequent attacks associated with abdominal
distension. On examination: the abdomen is distended and
the colon contains feces and gas in large volumes. The rectum is
empty.
8.618/ 1.
The most likely diagnosis is:
A) enterocolitis
B) massive intestinal worm infestation
C) congenital megacolon
D) idiopathic sprue

E) duodenal atresia
8.618/2.
This condition is freqently associated with:
A) Meckel's diverticulum
B) familial polyposis
C) abdominal distension
D) rectal hemorrhage
E) intestinal atresia
8.618/3.
The characteristic pathology of this condition includes:
A) infarction of the large bowel
B) multiple diverticula of the colon
C) hypertrophy of the intramural neural plexuses of the colon
D) absence of the neural plexuses of the colon
E) hypertrophy of the longitudinal muscle layer of the colonic wall
SUR-8.619.
Case Study
A 2-year-old infant is admitted for colicky abdominal pain of abrupt
onset associated with crying and frequent vomiting. The patient is
asymptomatic between the painful attacks. The mother informs you
that the child has passed bloody-mucous stools.
8.619/1.
The most likely diagnosis is:
A) intussusception/invagination
B) gastroenteritis
C) congenital pyloric stenosis
D) gastritis
E) renal stone
8.619/2.
The characteristic finding detected on physical examination is:
A) a sausage-like mass in the right upper abdominal quadrant
B) general abdominal tenderness
C) virtually no abnormality is found
D) epigastric tenderness
E) costovertebral angle tenderness

8.619/3.
A likely finding on the plain abdominal x-ray is:
A) the absence of the cecal "gas cap" in the right iliac region
B) a patchy small intestine distended with gas
C) nothing in particular can be noted
D) a dilated stomach
E) radiolucent stone in the kidney
8.619/4. Single Choice Question
The most valuable supplemental diagnostic study is:
A) a barium enema
B) an upper GI series
C) intravenous urography
D) a CT-scan

E) serial radiography of the small intestine


SUR-8.621.
Case Study
A 27-year-old female patient is brought to the surgery department of
the outpatient clinic. According to the rescue personnel, she has fallen
down some slippery stairs and knocked her head to the ground. Even
though the patient denies the loss of her consciousness she does not
remember the preceding events accurately. She has had nausea but
no vomiting. Further, none of the acute conditions associated with
unconsciousness nor any other type of trauma are present in her history.
Her blood pressure has been always normal. Physical status:
vital organ functions are normal. Blood pressure: 130/80 mmHg, heartrate:
86/min. A 5 cm subcutaneous hematoma is visible in the middle
of the patient's forehead with a 2 cm excoriation at the centre of the
injury. The left nostril is covered by crusted blood; thin bloody discharge
is leaking from the right nostril. Neurologic examination: reveals
intact mental status, nuchal rigidity is absent. Cranial nerves
are normal. Olfactory tests are not feasible. Tendon reflexes are normal,
with no other abnormal reflexes. Motor functions are intact, sensory
loss is absent. Signs of cerebral lobe injury are absent. Normal
psychomotor function. Coordination is intact, tests of cerebellar function
reveal instability during the Romberg test. No fractures are seen
on bidirectional skull films and roentgenograms of the base of the skull.
Air is detected on the 'face-up' film. One day following admission
periorbital suffusion (racoon's eyes) develops.
8.621 / 1.
What is the most likely diagnosis?
1) cerebral concussion
2) contusion of the head and excoriations of the forehead
3) cerebral contusion
4) basal skull fracture
5) intracerebral hemorrhage
6) suturolysis
A) answers (1), (4), and (5) are correct
B) answers (4), (5), and (6) are correct
C) answers (3), (5), and (6) are correct
D) answers (1), (2), and (4) are correct
E) none of the answers are correct
8.621/2.
What kind of roentgenograms would you order?
A) bidirectional skull film
B) basal skull film
C) face up film
D) all of the above
8.621/3.
Which of the following tests are suitable for the identification of the
source of the discharge leaking from the nostril?
1) pneumoencephalography
2) the appearance of a ring developing around a droplet of the
discharge dropped on filter-paper
3) measuring the glucose content of the discharge

4) the determination of the difference between radiation activity


of nasal and pharyngeal swabs during isotope cysternography
5) carotid angiography
6) injection of contrast material into the cisterna magna and the
observation of the route of elimination by serial radiography
A) answers (2), (3), (4), and (6) are correct
B) answers (1), (2), and (3) are correct
C) answers (3), (4), and (5) are correct
D) answers (4), (5), and (6) are correct
E) none of the answers are correct
8.621/4.
What is the appropriate action to be taken in the case of
liquorrhea?
A) discharge of the patient after proper wound care has been delivered,
strict bed rest, follow-up examination 1 week later
B) observation, antibiotic therapy
C) observation, antibiotic therapy, repeated spinal taps until the
cessation of cerebrospinal fluid leakage
D) observation, antibiotic therapy, repeated spinal taps for 8-10
days; surgery if leakage is permanent
8.621/5.
What are the possible risks and complications associated with this
condition?
1) headaches resulting from the permanent leakage of
cerebrospinal fluid
2) meningitis
3) damage to the olfactory nerve
4) brain abscess
5) intracranial hemorrhage associated with increased intracranial
pressure
SUR-8.622.
Case Study
A 58-year-old male patient has felt a sudden crack in his right shoulder
while lifting his coat from the rack. He could not move his right
arm anymore, the limb was fixed in the elevated position. Passive reposition
was unsuccessful. The right shoulder assumed a sloped contour
and the axis of the right humerus pointed to the axillary fossa
instead of the humeroscapular joint. Peripheral pulses are readily palpable,
no signs of damaged innervation to the arm or hand are detected.
8.622/1.
What is the most likely diagnosis?
A) fracture of the surgical neck of the humerus
B) pathologic fracture of the humerus
C) dislocation of the shoulder
D) acromioclavicular dislocation
8.622/2.
Which of the following studies is the most objective in verifying the
diagnosis?
A) bidirectional roentgenograms of the right humerus
B) bidirectional roentgenograms of the right shoulder joint

C) physical examination and inspection


D) laboratory tests including the determination of the alkaline
phosphatase activity
8.622/3.
Which of the following is the appropriate therapy for the above diagnosis?
A) plaster cast
B) reposition and plaster cast
C) surgical reduction, stable osteosynthesis
D) surgical treatment involving the resection of the proximal end
of the humerus
SUR-8.623.
Case Study
A 58-year-old male patient has been kicked by a horse. The hoof of the
animal has hit the patient at the level of the left costal margin. On
admission: the patient is complaining about severe abdominal pain in
the upper-left quadrant. The hematocrit and the WBC are normal;
the circulation is stable. Continuous monitoring of circulatory parameters
are uncharacteristic of internal bleeding. The Hct is stable, however,
the WBC count is slightly elevated. On the third day following
admission shock develops abruptly during defecation. Blood pressure
falls and can not be corrected by infusion or transfusion.
8.623/1.
The most likely diagnosis is:
A) perforated gastric ulcer
B) mechanical intestinal obstruction
C) rupture of the spleen
D) fractue of the 9-10th ribs associated with injury to the
subcostal artery
8.623/2.
Which of the following studies inappropriate in verifying the diagnosis?
A) a chest x-ray
B) plain abdominal radiography
C) emergency surgical exploration
D) laboratory tests for the monitoring of the Hct and the WBC
Count
8.623/3.
What is the therapy of choice?
A) a massive transfusion
B) a laparotomy and closure of the perforation
C) a laparotomy and the control of any bleeding
D) a gastric resection
ANSWER KEY SUR
1.D
2.3.4.5.D
6.C

101.102.B
103.D
104.105.E
106.E

201.E
202.203.A
204.205.206.B

301.E
302.C
303.B
304.B
305.306.E

401.A
402.B
403.A
404.E
405.A
406.A

501.E
502.E
503.C
504.C
505.E
506.B

601.DDECCACB
602.603.A CB ED
604.605.BCCCBCD
606.EDEB

7.C
8.C
9.D
10.A
11.12.13.14.E
15.E
16.E
17.A
18.C
19.D
20.E
21.D
22.E
23.A
24.E
25.A
26.C
27.C
28.D
29.C
30.E
31.C
32.E
33.D
34.C
35.C
36.B
37.B
38.D
39.C
40.C
41.D
42.A
43.A
44.D
45.D
46.E
47.A
48.A
49.B
50.A
51.C
52.D
53.54.A
55.E

107.C
108.C
109.110.111.112.C
113.114.115.E
116.E
117.C
118.D
119.B
120.E
121.122.123.124.125.D
126.E
127.C
128.D
129.E
130.E
131.C
132.B
133.B
134.D
135.B
136.E
137.C
138.C
139.D
140.D
141.C
142.C
143.E
144.D
145.C
146.B
147.A
148.D
149.D
150.D
151.B
152.D
153.C
154.B
155.-

207.D
208.C
209.C
210.E
211.B
212.D
213.A
214.C
215.B
216.C
217.C
218.E
219.C
220.B
221.C
222.E
223.C
224.B
225.E
226.E
227.D
228.B
229.C
230.C
231.C
232.D
233.A
234.A
235.A
236.B
237.C
238.A
239.E
240.E
241.A
242.A
243.A
244.A
245.E
246.A
247.A
248.A
249.A
250.A
251.B
252.D
253.A
254.255.-

307.C
308.A
309.B
310.C
311.A
312.C
313.A
314.A
315.C
316.A
317.D
318.A
319.C
320.E
321.D
322.A
323.C
324.D
325.A
326.C
327.C
328.D
329.E
330.A
331.D
332.E
333.E
334.B
335.B
336.D
337.B
338.E
339.C
340.D
341.D
342.C
343.E
344.D
345.D
346.347.D
348.349.E
350.D
351.B
352.B
353.D
354.D
355.E

407.D
408.A
409.E
410.B
411.E
412.E
413.C
414.A
415.A
416.B
417.A
418.B
419.A
420.D
421.A
422.C
423.A
424.D
425.B
426.B
427.A
428.429.B
430.A
431.C
432.A
433.B
434.A
435.E
436.A
437.A
438.E
439.C
440.E
441.A
442.B
443.B
444.445.446.E
447.A
448.A
449.E
450.A
451.A
452.E
453.A
454.E
455.A

507.A
508.C
509.B
510.B
511.C
512.B
513.E
514.A
515.A
516.E
517.B
518.B
519.A
520.B
521.E
522.523.B
524.A
525.A
526.B
527.B
528.E
529.A
530.A
531.A
532.A
533.A
534.A
535.B
536.B
537.C
538.E
539.B
540.A
541.E
542.A
543.A
544.545.E
546.E
547.A
548.B
549.A
550.A
551.E
552.B
553.A
554.E
555.-

607.BBC
608.CEBDE
609.610.CAFAC
611.CAC
612.613.CDCC
614.DCBE
615.BBB
616.EEEE
617.CCB
618.CCD
619.AAAA
620.621.DDADB
622.CBB
623.CCC

56.D
57.D
58.C
59.A
60.B
61.A
62.A
63.D
64.A
65.A
66.A
67.B
68.B
69.C
70.E
71.B
72.B
73.A
74.75.D
76.B
77.D
78.E
79.E
80.E
81.E
82.D
83.C
84.E
85.A
86.B
87.E
88.E
89.D
90.E
91.E
92.E
93.C
94.A
95.C
96.E
97.D
98.E
99.E
100.C

156.B
157.B
158.B
159.A
160.B
161.D
162.D
163.A
164.B
165.D
166.V
167.D
168.C
169.B
170.B
171.172.173.E
174.A
175.A
176.A
177.B
178.C
179.E
180.B
181.D
182.B
183.A
184.B
185.B
186.B
187.C
188.D
189.D
190.D
191.E
192.B
193.D
194.B
195.C
196.B
197.D
198.D
199.D
200.D

256.C
257.B
258.C
259.D
260.D
261.A
262.E
263.264.C
265.E
266.C
267.A
268.D
269.E
270.E
271.272.273.D
274.B
275.276.E
277.D
278.B
279.C
280.A
281.A
282.A
283.B
284.C
285.C
286.A
287.C
288.A
289.A
290.B
291.D
292.D
293.E
294.A
295.A
296.D
297.B
298.C
299.C
300.C

356.C
357.A
358.D
359.D
360.A
361.A
362.C
363.B
364.E
365.E
366.E
367.D
368.A
369.E
370.D
371.B
372.B
373.C
374.C
375.A
376.C
377.B
378.D
379.C
380.T
381.T
382.F
383.F
384.F
385.F
386.T
387.F
388.T
389.T
390.T
391.A
392.A
393.394.395.A
396.C
397.B
398.B
399.D
400.E

456.B
457.C
458.E
459.E
460.E
461.462.A
463.A
464.E
465.A
466.E
467.A
468.E
469.A
470.C
471.A
472.C
473.E
474.B
475.476.B
477.B
478.E
479.A
480.A
481.A
482.C
483.C
484.A
485.A
486.C
487.B
488.A
489.A
490.A
491.A
492.493.D
494.B
495.B
496.A
497.E
498.E
499.E
500.A

556.B
557.B
558.B
559.C
560.ADCB
561.BABA
562.DCBA
563.BCDA
564.ABCD
565.CDAEB
566.AABDCB
567.CCDA
568.DABC
569.BADCC
570.ABBCD
571.DACB
572.DCAEB
573.BCAD
574.DABC
575.ABABB
576.CABAB
577.578.CAAA
579.BDCA
580.BCDA
581.BCDA
582.CBAAD
583.A
584.585.AA
586.EDEBACEE
587.588.BDDEBDA
589.590.DDC
591.CAA
592.CDEA
593.BCEB
594.BC
595.CEC
596.CDC
597.ECAD
598.BBAA
599.ABDBDEBBGABB
600.ECFECDADC

S-ar putea să vă placă și